Final minus Midterm review.... still super long.

Lakukan tugas rumah & ujian kamu dengan baik sekarang menggunakan Quizwiz!

A nurse teaches a client who is prescribed nicotine replacement therapy. Which statement should the nurse include in this clients teaching? a. Make a list of reasons why smoking is a bad habit. b. Rise slowly when getting out of bed in the morning. c. Smoking while taking this medication will increase your risk of a stroke. d. Stopping this medication suddenly increases your risk for a heart attack.

Smoking while taking this medication will increase your risk of a stroke. Clients who smoke while using drugs for nicotine replacement therapy increase the risk of stroke and heart attack. Nurses should teach clients not to smoke while taking this drug. The other responses are inappropriate.

A client with colorectal cancer was started on 5-fluorouracil (5-FU) and is experiencing fatigue, diarrhea, and mouth ulcers. A relatively new chemotherapeutic agent, oxaliplatin (Eloxatin), has been added to the treatment regimen. What does the nurse tell the client about the diarrhea and mouth ulcers? A. "A combination of chemotherapeutic agents has caused them." B. "GI problems are symptoms of the advanced stage of your disease." C. "5-FU cannot discriminate between your cancer and your healthy cells." D. "You have these as a result of the radiation treatment."

"5-FU cannot discriminate between your cancer and your healthy cells." 5-FU cannot discriminate between cancer and healthy cells; therefore, the side effects are diarrhea, mucositis, leukopenia, mouth ulcers, and skin ulcers. The 5-FU treatment, not a combination of chemotherapy drugs, radiation, or the stage of the disease, is what is causing the client's GI problems.

Which precaution is most important for the nurse to teach a client who has cardiovascular autonomic neuropathy (CAN) from diabetes to prevent harm? "Check your hands and feet weekly for chronic excessive sweating." "Change positions slowly when moving from sitting to standing." "Avoid drinking caffeine or caffeinated beverages." "Be sure to take your blood pressure drug daily."

"Change positions slowly when moving from sitting to standing." Cardiovascular autonomic neuropathy (CAN) affects sympathetic and parasympathetic nerves of the heart and blood vessels. This problem contributes to left ventricular dysfunction, painless myocardial infarction, and exercise intolerance. Most often, CAN leads to orthostatic (postural) hypotension and syncope (brief loss of consciousness on standing). These problems are from failure of the heart and arteries to respond to position changes by increasing heart rate and vascular tone. As a result, blood flow to the brain is interrupted briefly. Orthostatic hypotension and syncope increase the risk for falls, especially among older adults.Although taking blood pressure medication daily is important, it does not prevent orthostatic hypotension and in fact, may make orthostatic hypotension worse. Sensation changes are associated with peripheral neuropathy, not cardiovascular autonomic neuropathy. Avoiding caffeine is no longer a recommended action.

The nurse is caring for a client who is to be discharged after a bowel resection and the creation of a colostomy. Which client statement demonstrates that additional instruction from the nurse is needed? A. "I can drive my car in about 2 weeks." B. "I should avoid drinking carbonated sodas." C. "It may take 6 weeks to see the effects of some foods on my bowel patterns." D. "Stool softeners will help me avoid straining."

"I can drive my car in about 2 weeks." The client who has had a bowel resection and colostomy should avoid driving for 4 to 6 weeks. The client should avoid drinking sodas and other carbonated drinks because of the gas they produce. He or she may not be able to see the effects of certain foods on bowel patterns for several weeks. The client should avoid straining at stool.

Which physiological processes directly prevent sever hypoglycemia in a healthy adult without diabetes who is NPO for 12 hours? (SATA) A. Gluconeogenesis B. Glycogenesis C. Glycogenolysis D. Ketogenesis E. Lipogenesis F. Lipolysis

A. Gluconeogenesis C. Glycogenolysis

After receiving education on the correct use of emergency drug therapy for asthma, which statement by the client indicates a correct understanding of the nurse's instructions? "All asthma drugs help everybody breathe better." "I must carry my emergency inhaler when activity is anticipated." "I must have my emergency inhaler with me at all times." "Preventive drugs can stop an attack."

"I must have my emergency inhaler with me at all times." The statement by the client that indicates a correct understanding of the instructions is that the emergency inhaler must be with the client at all times. Because asthma attacks cannot always be predicted, clients with asthma must always carry a rescue inhaler such as a short-acting beta agonist (e.g., albuterol).Asthma medications are specific to the disease and to the client and should never be shared or used by anyone other than the person for whom they are prescribed. They are not always good for everyone and, in fact, may do harm. An emergency inhaler should be carried all the time and not just when activity is anticipated. Preventive drugs are those that are taken every day to help prevent an attack from occurring, and do not stop an attack once it begins.

The home health nurse is teaching a client about the care of a new colostomy. Which client statement demonstrates a correct understanding of the instructions? A. "A dark or purplish-looking stoma is normal and should not concern me." B. "If the skin around the stoma is red or scratched, it will heal soon." C. "I need to check for leakage underneath my colostomy." D. "I should strive for a very tight fit when applying the barrier around the stoma."

"I need to check for leakage underneath my colostomy." The pouch system should be checked frequently for evidence of leakage to prevent excoriation. A purplish stoma is indicative of ischemia and necrosis. Redness or scratched skin around the stoma should be reported to prevent it from beginning to break down. An overly tight fit may lead to necrosis of the stoma.

A client with irritable bowel syndrome (IBS) is constipated. The nurse instructs the client about a management plan. Which client statement shows an accurate understanding of the nurse's teaching? A. "A drink of diet soda with dinner is OK for me." B. "I need to go for a walk every evening." C. "Maintaining a low-fiber diet will manage my constipation." D. "Watching the amount of fluid that I drink with meals is very important."

"I need to go for a walk every evening." Increased ambulation is part of the management plan for IBS, along with increased fluids and fiber and avoiding caffeinated beverages. Caffeinated beverages can cause bloating or diarrhea and should be avoided in clients with IBS. Fiber is encouraged in clients with IBS because it produces a bulky soft stool and aids in establishing regular bowel habits. At least 8 to 10 cups of fluid should be consumed daily to promote normal bowel function.

5. Which client statement should a nurse identify as a typical response to stress most often experienced in the working phase of the nurse-client relationship? 1. "I can't bear the thought of leaving here and failing." 2. "I might have a hard time working with you, because you remind me of my mother." 3. "I really don't want to talk any more about my childhood abuse." 4. "I'm not sure that I can count on you to protect my confidentiality."

"I really don't want to talk any more about my childhood abuse." Rationale: The nurse should identify that the client statement, "I really don't want to talk any more about my childhood abuse," reflects that the client is in the working phase of the nurse-client relationship. The working phase includes overcoming resistance behaviors on the part of the client as the level of anxiety rises in response to discussion of painful issues.

Which statement by a client with chronic obstructive pulmonary disease (COPD) and a 10 pound (4.5 kg) weight loss indicates the need for additional follow-up instruction? "I should consume plenty of fluids with my meal." "I will try eating smaller more frequent meals." "I will try to eat more protein." "I will perform mouth care prior to eating."

"I should consume plenty of fluids with my meal." The need for additional follow-up instruction is noted when the client states that he or she will drink more fluids before and during meals. This action will cause a sensation of fullness and limit adequate nourishment.Eating smaller, more frequent meals, trying to eat more protein, and performing mouth care before eating are all appropriate and positive client comments.

Which statement by a client indicates to the nurse correct understanding of what to do when the sensations of hunger and shakiness occur? "I will eat three graham crackers." "I will drink a glass of water." "I will sit down and rest." "I will give myself a dose of glucagon."

"I will eat three graham crackers." Feeling hungry and shaky are symptoms of mild hypoglycemia. Correct understanding of what the client needs to do when these symptoms occur is to eat three graham crackers. This is the correct management strategy for mild hypoglycemia.Drinking a glass of water or sitting down and resting does not remedy hypoglycemia. Glucagon is generally administered for episodes of severe not mild hypoglycemia.

The Certified Wound, Ostomy, and Continence Nurse is teaching a client with colorectal cancer how to care for a newly created colostomy. Which client statement reflects a correct understanding of the necessary self-management skills? A. "I will have my spouse change the bag for me." B. "If I have any leakage, I'll put a towel over it." C. "I need to call my home health nurse to come out if I have any problems." D. "I will make certain that I always have an extra bag available."

"I will make certain that I always have an extra bag available." The statement that the client will be certain to bring an extra bag is the only statement illustrating that the client is taking responsibility to care for the colostomy. Using a towel is not an acceptable or effective way to cope with leakage. It is not realistic that the home health nurse can make frequent visits for the purpose of colostomy care.

The nurse is teaching a client who has undergone a hemorrhoidectomy about a follow-up plan of care. Which client statement demonstrates a correct understanding of the nurse's instructions? A. "I should take Ex-Lax after the surgery to 'keep things moving'." B. "I will need to eat a diet high in fiber." C. "Limiting my fluids will help me with constipation." D. "To help with the pain, I'll apply ice to the surgical area."

"I will need to eat a diet high in fiber." A diet high in fiber serves as a natural stool softener and will prevent irritation to hemorrhoids caused by painful bowel movements. Stimulant laxatives are discouraged because they are habit-forming. Increased amounts of fluids are needed to prevent constipation. Moist heat (sitz baths) will be more effective with postoperative discomfort than cold applications; cold therapy is sometimes recommended and useful before surgery for inflamed hemorrhoids.

A 24-year-old male is scheduled for a minimally invasive inguinal hernia repair (MIIHR). Which client statement indicates a need for further teaching about this procedure? A. "I may have trouble urinating immediately after the surgery." B. "I will need to stay in the hospital overnight." C. "I should not eat after midnight the day of the surgery." D. "My chances of having complications after this procedure are slim."

"I will need to stay in the hospital overnight." Usually, the client is discharged 3 to 5 hours after MIIHR surgery. Male clients who have difficulty urinating after the procedure should be encouraged to force fluids and to assume a natural position when voiding. Clients undergoing MIIHR surgery must be NPO after midnight before the surgery. Most clients who have MIIHR surgery have an uneventful recovery.

The nurse is evaluating understanding of the treatment regimen for a client newly diagnosed with asthma. Which of these statements by the client indicates understanding of the regimen? "I will take albuterol when I go to sleep." "I will keep the rescue medication readily accessible on the first floor of my home." "I will take the long acting beta agonist even when my breathing seems OK." "I will immediately take the anti-inflammatory medication for an acute asthma attack."

"I will take the long acting beta agonist even when my breathing seems OK." The client indicates understanding of the dosing regimen when stating, "I will take the long-acting beta agonist even when my breathing seems OK." Long-acting medications are useful in preventing an asthma attack but cannot stop an acute attack.Short-acting beta2 agonists (SABAs) provide rapid, short-term relief. These "rescue" type inhaled drugs are most useful when an attack begins (as relief) or as premedication when the client is about to begin an activity that is likely to induce an attack. They are not used on a regular schedule. The client must always carry the relief drug inhaler with him or her and ensure that they do not run out of this medication. Anti-inflammatory medications decrease airway inflammation and are considered controller medications. They are not used for acute attacks.

A client has been discharged to home after being hospitalized with an acute episode of pancreatitis. The client, who is an alcoholic, is unwilling to participate in Alcoholics Anonymous (AA), and the client's spouse expresses frustration to the home health nurse regarding the client's refusal. What is the nurse's best response? A. "Your spouse will sign up for the meetings only when he is ready to deal with his problem." B. "Keep mentioning the AA meetings to your spouse on a regular basis." C. "I'll get you some information on the support group Al-Anon." D. "Tell me more about your frustration with your spouse's refusal to participate in AA."

"I'll get you some information on the support group Al-Anon." Putting the client's spouse in contact with an Al-Anon support group assists with the spouse's frustration. Telling the spouse that the client will sign up for AA meetings when the client is ready and telling the spouse to keep mentioning AA do not address the spouse's frustration with the client's refusal to participate in AA. Encouraging the spouse to say more about his or her frustration may allow the spouse to vent frustration, but it does not offer any options or solutions.

A client recently diagnosed with asthma has a prescription to use an inhaled medication with a spacer. The nurse evaluates the client has correct understanding of the use of an inhaler with a spacer when the client states which of these? Select all that apply. "I don't have to wait a minute between the two puffs if I use a spacer." "If the spacer makes a whistling sound, I am breathing in too rapidly." "I should rinse my mouth and then swallow the water to get all of the medicine." "I should shake the canister when I want to see whether it is empty." "I should hold my breath for at least ten seconds after inhaling the medication."

"If the spacer makes a whistling sound, I am breathing in too rapidly." "I should hold my breath for at least ten seconds after inhaling the medication." Slow and deep breaths ensure that the medication is reaching deeply into the lungs. The whistling noise serves as a reminder to the client of which technique needs to be used. The client should hold the breath for at least 10 seconds, however attempting to hold the breath for a minute is unnecessary and could pose a threat to oxygenation.The client must wait 1 minute between puffs regardless of the method of delivery of the medication. The client should rinse the mouth but not swallow the water. The mouth needs to be rinsed after using an inhaler with or without a spacer. This is especially important if the inhaled medication is a corticosteroid; rinsing will help prevent the development of an oral fungal infection. An empty inhaler will float on its side in water while a full inhaler will sink. Shaking an inhaler helps ensure that the medication is dispersed and the same dose is delivered in each puff.

A client suspected of having irritable bowel syndrome (IBS) is scheduled for a hydrogen breath test. What does the nurse tell the client about this test? A. "During the test, you will drink small amounts of an antacid as directed by the technician." B. "If you have IBS, hydrogen levels may be increased in your breath samples." C. "The test will take between 30 and 45 minutes to complete." D. "You must have nothing to drink (except water) for 24 hours before the test."

"If you have IBS, hydrogen levels may be increased in your breath samples." Excess hydrogen levels are produced in clients with IBS. This is due to bacterial overgrowth in the small intestine that accompanies the disease. The hydrogen travels to the lungs to be excreted. The client will ingest small amounts of sugar during the test, not an antacid. The test takes longer than 45 minutes. The client has breath samples taken every 15 minutes for 1 to 2 hours. The client needs to be NPO (except for water) for 12 hours before the test.

What is the nurse's best response to a client newly diagnosed with type 1 diabetes who asks why insulin is only given by injection and not as an oral drug? "Injected insulin works faster than oral drugs to lower blood glucose levels." "Oral insulin is so weak that it would require very high dosages to be effective." "Insulin is a small protein that is destroyed in by stomach acids and intestinal enzymes." "Insulin is a "high alert drug" and could more easily be abused if it were available as an oral agent."

"Insulin is a small protein that is destroyed in by stomach acids and intestinal enzymes." Because insulin is a small protein that is easily destroyed by stomach acids and intestinal enzymes, it cannot be used as an oral drug. Most commonly, it is injected subcutaneously.

6. A mother who is notified that her child was killed in a tragic car accident states, "I can't bear to go on with my life." Which nursing statement conveys empathy? 1. "This situation is very sad, but time is a great healer." 2. "You are sad, but you must be strong for your other children." 3. "Once you cry it all out, things will seem so much better." 4. "It must be horrible to lose a child, and I'll stay with you until your husband arrives."

"It must be horrible to lose a child, and I'll stay with you until your husband arrives." Rationale: The nurse's response, "It must be horrible to lose a child, and I'll stay with you until your husband arrives," conveys empathy to the client. Empathy is the ability to see the situation from the client's point of view. Empathy is considered to be one of the most important characteristics of the therapeutic relationship.

17. A client exhibiting dependent behaviors says, "Do you think I should move from my parent's house and get a job?" Which nursing response is most appropriate? 1. "It would be best to do that in order to increase independence." 2. "Why would you want to leave a secure home?" 3. "Let's discuss and explore all of your options." 4. "I'm afraid you would feel very guilty leaving your parents."

"Let's discuss and explore all of your options." Rationale: The most appropriate response by the nurse is, "Let's discuss and explore all of your options." In this example, the nurse is encouraging the client to formulate ideas and decide independently the appropriate course of action.

19. A newly admitted client, diagnosed with obsessive-compulsive disorder (OCD), washes his hands continually. This behavior prevents unit activity attendance. Which nursing statement best addresses this situation? 1. "Everyone diagnosed with OCD needs to control their ritualistic behaviors." 2. "It is important for you to discontinue these ritualistic behaviors." 3. "Why are you asking for help, if you won't participate in unit therapy?" 4. "Let's figure out a way for you to attend unit activities and still wash your hands."

"Let's figure out a way for you to attend unit activities and still wash your hands." Rationale: The most appropriate statement by the nurse is, "Let's figure out a way for you to attend unit activities and still wash your hands." This statement reflects the therapeutic communication technique of formulating a plan of action. The nurse attempts to work with the client to develop a plan without damaging the therapeutic relationship.

A client expresses fear and anxiety over the life changes associated with diabetes, stating, "I am scared that I can't do it all and will get so sick that I will be a burden on my family." What is the nurse's best response? "Let's tackle it piece by piece. What is most scary to you?" "It is overwhelming, isn't it?" "Let's see how much you can learn today, so you are less nervous." "Many people live with diabetes and do it just fine."

"Let's tackle it piece by piece. What is most scary to you?" The nurse's best response is to suggest that the client tackle it piece by piece and ask what is most scary to him or her. This is the best client-centered response, and acknowledges the client's concern, letting the client master survival skills first.Referring to the illness as overwhelming may reflect the client's feelings, but is a closed-ended question and does not encourage the client to express his feelings about the underlying fear. Trying to see how much the client can learn in 1 day may add to his anxiety by overwhelming him with information and the need to "do it all" in 1 day. Suggesting that other people handle the illness just fine criticizes the client and does not recognize his concerns.

What action will the nurse advise to prevent harm for a client with diabetes who has a 3-cm callus on the ball of the right foot? "Make an appointment with your podiatrist as soon as possible." "Make an appointment with a pedicurist and have them cut or file off the callus." "Soak your feet nightly in warm water and peel of a little of the callus every day." "Apply an over-the-counter callus-dissolving pad and follow the package directions."

"Make an appointment with your podiatrist as soon as possible." The client with diabetes is taught to see his or her diabetes health care provider or a podiatrist for calluses, corns, or any other foot lesion and never to self-treat such problems. The risk for development of an ongoing injury with chronic infection is very high could lead to eventual amputation.

Which specific action is a priority for the nurse to teach a client with diabetes who has peripheral neuropathy to prevent harm? "Wear a medical alert bracelet." "Never go barefoot." "Never reuse insulin syringes." "Drink at least 3 L of fluids daily."

"Never go barefoot." All the actions are important for the client with diabetes to perform for safety and to prevent a variety of complications. However, the most important action to prevent harm from peripheral neuropathy is to never go barefoot and wear shoes and slippers with firm soles.

A client with malabsorption syndrome asks the nurse, "What did I do to cause this disorder to develop?" How does the nurse respond? A. "An excessive intake of alcohol is associated with it, so your substance abuse could have contributed to its development." B. "It is inherited, so it could run in your family." C. "It might be caused by a virus, so you could have gotten it almost anywhere." D. "Nothing you did could have caused it; it is the result of flattening of the mucosa of your large intestine."

"Nothing you did could have caused it; it is the result of flattening of the mucosa of your large intestine." Stating that the disorder is the result of flattening of the mucosa of the large intestine is the only statement that is physiologically accurate. Malabsorption syndrome is not associated with an excessive intake of alcohol. It is not inherited, although a genetic immune defect is present in the related disease, celiac sprue. It is not caused by a virus.

A client has developed acute pancreatitis after also developing gallstones. Which is the highest priority instruction for this client to avoid further attacks of pancreatitis? A. "You may need a surgical consult for removal of your gallbladder." B. "See your health care provider immediately when experiencing symptoms of a gallbladder attack." C. "If you have a gallbladder attack and pain does not resolve within a few days, call your health care provider." D. "You'll need to drastically modify your alcohol intake."

"See your health care provider immediately when experiencing symptoms of a gallbladder attack." In this case, the client's pancreatitis was likely triggered by the development of gallstones. A diagnostic statement must come from the provider. Also, the client may not require removal of the gallbladder. The client must see the provider promptly when experiencing gallbladder disease and should not wait. Because this client's acute pancreatitis is likely related to gallstones, alcohol consumption need not be restricted.

15. An instructor is correcting a nursing student's clinical worksheet. Which instructor statement is the best example of effective feedback? 1. "Why did you use the client's name on your clinical worksheet?" 2. "You were very careless to refer to your client by name on your clinical worksheet." 3. "Surely you didn't do this deliberately, but you breeched confidentiality by using names." 4. "It is disappointing that after being told you're still using client names on your worksheet."

"Surely you didn't do this deliberately, but you breeched confidentiality by using names." Rationale: The instructor's statement, "Surely you didn't do this deliberately, but you breeched confidentiality by using names," is an example of effective feedback. Feedback is method of communication for helping others consider a modification of behavior. Feedback should be descriptive, specific, and directed toward a behavior that the person has the capacity to modify and should impart information rather than offer advice.

The nurse is assessing a client's alcohol intake to determine whether it is the underlying cause of the client's attacks of pancreatitis. Which question does the nurse ask to elicit this information? A. "Do you usually binge drink?" B. "Do you tend to drink more on holidays or weekends?" C. "Tell me more about your alcohol intake." D. "Estimate how many episodes of binge drinking you do in a week."

"Tell me more about your alcohol intake." Asking the client about his or her alcohol intake is the only way that will allow the client to provide information in the client's own words and to the extent that the client wishes to provide it. Asking the client if he or she binge drinks or tends to drink more on holidays or weekends may put the client on the defensive rather than provide the desired information. It has not yet been determined whether the client engages in binge drinking.

A client with colorectal cancer is scheduled for colostomy surgery. Which comment from the nurse is most therapeutic for this client? A. "Are you afraid of what your spouse will think of the colostomy?" B. "Don't worry. You will get used to the colostomy eventually." C. "Tell me what worries you the most about this procedure." D. "Why are you so afraid of having this procedure done?"

"Tell me what worries you the most about this procedure." Asking the client about what worries him or her is the only question that allows the client to express fears and anxieties about the diagnosis and treatment. Asking the client if he or she is afraid is a closed question (i.e., it requires only a "yes" or "no" response); it closes the dialogue and is not therapeutic. Telling the client not to worry offers reassurance and is a "pat" statement, making it nontherapeutic. "Why" questions place clients on the defense and are not therapeutic because they close the conversation.

Which statement made by a client who is learning about self-injection of insulin indicates to the nurse that clarification is needed about injection site selection and rotation? "The abdominal site is best because it is closest to the pancreas." "I can reach my thigh best, so I will use different areas of the same thigh." "If I change my injection site from the thigh to an arm, the inulin absorption may be different." "By rotating sites within one area, my chance of having skin changes is less."

"The abdominal site is best because it is closest to the pancreas." The abdominal site has the fastest and most consistent rate of absorption because of the blood vessels in the area and not because of its proximity to the pancreas. The other statements demonstrate correct understanding about injection site selection and rotation.

What is the nurse's best response to a client newly diagnosed with diabetes who asks why he is always so thirsty? "Without insulin, glucose is excreted rather than used in the cells. The loss of glucose directly triggers thirst, especially for sugared drinks." "The extra glucose in the blood increases the blood sodium level, which increases your sense of thirst." "Without insulin, glucose combines with blood cholesterol, which damages the kidneys, making you feel thirsty even when no water has been lost." "The extra glucose in the blood makes the blood thicker, which then triggers thirst so that the water you drink will dilute the blood glucose level."

"The extra glucose in the blood makes the blood thicker, which then triggers thirst so that the water you drink will dilute the blood glucose level." The high blood glucose levels that are present, because movement of glucose into cells is impaired, increase the osmolarity of the blood. The increased osmolarity stimulates the osmoreceptors in the hypothalamus, which triggers the thirst reflex. In response, the person drinks more water (not sugary fluids or hyperosmotic fluids), which helps dilute blood glucose levels and reduces blood osmolarity.

A male client's sister was recently diagnosed with colorectal cancer (CRC), and his brother died of CRC 5 years ago. He asks the nurse whether he will inherit the disease too. How does the nurse respond? A. "Have you asked your health care provider what he or she thinks your chances are?" B. "It is hard to know what can predispose a person to develop a certain disease." C. "No. Just because they both had CRC doesn't mean that you will have it, too." D. "The only way to know whether you are predisposed to CRC is by genetic testing."

"The only way to know whether you are predisposed to CRC is by genetic testing." Genetic testing is the only definitive way to determine whether the client has a predisposition to develop CRC. A higher incidence of the disease has been noted in families who have a history; however, it is not the responsibility of the nurse to engage in genetic counseling, and this client might not be predisposed to developing CRC. Asking the client what the health care provider thinks is an evasive response by the nurse and does not address the client's concerns.

A client diagnosed with acalculous cholecystitis asks the nurse how the gallbladder inflammation developed when there is no history of gallstones. What is the nurse's best response? A. "This may be an indication that you are developing sepsis." B. "The gallstones are present, but have become fibrotic and contracted." C. "This type of gallbladder inflammation is associated with hypovolemia." D. "This may be an indication of pancreatic disease."

"This type of gallbladder inflammation is associated with hypovolemia." This type of gallbladder inflammation is associated with hypovolemia. Although this type of gallbladder inflammation is associated with sepsis, it is not an indicator that sepsis is developing. Fibrotic and contracted gallstones are associated with chronic cholecystitis. The presence of acalculous cholecystitis is not an indicator that pancreatic disease has developed.

The nurse is caring for a client recently diagnosed with type 1 diabetes mellitus who has had an episode of acute pancreatitis. The client asks the nurse how he developed diabetes when the disease does not run in the family. What is the nurse's best response? A. "The diabetes could be related to your obesity." B. "What has your doctor told you about your disease?" C. "Do you consume alcohol on a frequent basis?" D. "Type 1 diabetes can occur when the pancreas is destroyed by disease."

"Type 1 diabetes can occur when the pancreas is destroyed by disease." Telling the client that type 1 diabetes can occur when the pancreas is destroyed by disease is the only response that accurately describes the relationship of the client's diabetes to pancreatic destruction. Type 2, not type 1, diabetes is usually related to obesity. Asking the client what the provider has said is an evasive response by the nurse and does not address the client's question. Many factors could produce acute pancreatitis other than alcohol consumption.

13. A client diagnosed with post-traumatic stress disorder related to a rape is admitted to an inpatient psychiatric unit for evaluation and medication stabilization. Which therapeutic communication technique might a nurse use that is an example of "broad openings"? 1. "What occurred prior to the rape, and when did you go to the emergency department?" 2. "What would you like to talk about?" 3. "I notice you seem uncomfortable discussing this." 4. "How can we help you feel safe during your stay here?"

"What would you like to talk about?" Rationale: The nurse's statement, "What would you like to talk about?" is an example of the therapeutic communication technique of a broad opening. Using broad openings allows the client to take the initiative in introducing the topic and emphasizes the importance of the client's role in the interaction.

A client has just been diagnosed with pancreatic cancer. The client's upset spouse tells the nurse that they have recently moved to the area, have no close relatives, and are not yet affiliated with a church. What is the nurse's best response? A. "Maybe you should find a support group to join." B. "Would you like me to contact the hospital chaplain for you?" C. "Do you want me to try to find a therapist for you?" D. "Do you have any friends whom you want me to call?"

"Would you like me to contact the hospital chaplain for you?" It is appropriate for the nurse to suggest contacting the hospital chaplain as a counseling option for the client and family. Suggesting that the client find a support group does not assist the client and the family with the problem. It is inappropriate for the nurse to suggest that the client and the family need a therapist. The spouse has already told the nurse that they have recently moved to the area, so it is unlikely that they have already made close friends.

11. The nurse is interviewing a newly admitted psychiatric client. Which of the following nursing statements is an example of offering a "general lead"? 1. "Do you know why you are here?" 2. "Are you feeling depressed or anxious?" 3. "Yes, I see. Go on." 4. "Can you order the specific events that led to your admission?"

"Yes, I see. Go on." Rationale: The nurse's statement, "Yes, I see. Go on," is an example of a general lead. Offering general leads encourages the client to continue sharing information.

How will the nurse reply when a client with type 2 diabetes tells the nurse that he would like to have a 12-ounce glass of beer with supper but believes that is now impossible? "You can have a beer with a meal if you test yourself for hypoglycemia an hour later." "You can have a beer with a meal if you test yourself for hyperglycemia an hour later." "There are nonalcoholic beers available that you can substitute for a regular beer." "If you gave up dessert, you can still have one beer."

"You can have a beer with a meal if you test yourself for hypoglycemia an hour later." Alcohol consumption contributes to hypoglycemia. This risk is reduced if the alcohol is consumed with or shortly after a meal. The client is instructed to check blood glucose levels about an hour after alcohol is consumed to determine if either more food is needed or if insulin dosage needs to be adjusted.

What is the nurse's best response to a client with type 2 diabetes controlled with metformin who asks why now that he is recovering from surgery, is he prescribed to receive insulin therapy for a few days? "Your insurance doesn't permit metformin to be used during hospitalization." "Your presurgical testing indicates that you now have type 1 diabetes and require daily insulin." "You just need insulin temporarily because the stress of surgery causes increased blood glucose levels for a day or two." "You must take insulin from now on because the surgery has aggravated the intensity of your diabetes."

"You just need insulin temporarily because the stress of surgery causes increased blood glucose levels for a day or two." The nurse's best response is that due to the stress of surgery and NPO status, short-term insulin therapy may be needed perioperatively for clients with diabetes who use oral antidiabetic agents. For those receiving insulin, dosage adjustments may be required until the stress of surgery subsides.No evidence suggests that the client's diabetes has worsened. However, surgery is stressful and may increase insulin requirements. Metformin may be taken in the hospital, but not on days when the client is NPO for surgery. When the client returns to his or her previous health state, oral agents will be resumed.

The client says, "I hate this stupid COPD." What is the best response by the nurse? "Stopping smoking will help your lungs heal." "You sound fed up with managing your illness." "Does anyone in your family have COPD?" "Most clients get used to it after a few months."

"You sound fed up with managing your illness." The best response by the nurse is "You sound fed up with managing your illness." This response encourages the client to express his or her feelings about the disease and its challenges.Lecturing the client regarding his smoking habits disregards the client's need for support. "Why" questions can seem accusatory and may make a client less likely to talk about what he or she is feeling. Asking the client if anyone in the family has COPD is a "yes" or "no" question and does not encourage the client to talk about his or her feelings. The client's feelings should never be minimized.

The nurse is providing preoperative teaching for the client with lung cancer for whom a lobectomy is planned. Which of these does the nurse include in the preoperative education session? Select all that apply. "You will wake up with a drain in your chest which removes blood and allows the remaining lung to expand." "You will be able to get out of bed after the chest tube is removed." "Plan to request pain medication before your pain becomes severe." "You may have a tube in your throat connected to a mechanical ventilator to assist you with breathing." "You will need to lie on the operative side until the area of tissue removal heals."

"You will wake up with a drain in your chest which removes blood and allows the remaining lung to expand." "Plan to request pain medication before your pain becomes severe." "You may have a tube in your throat connected to a mechanical ventilator to assist you with breathing." Preoperative teaching for a client scheduled to have a lobectomy for cancer includes telling the client that a chest drain will be in place, to request pain medication before the pain gets severe, and the possibility of having an endotracheal tube in the throat to assist with breathing.The nurse providing preoperative teaching for the lobectomy client would not tell the client that he or she will be able to get out of bed after the chest tube is removed. Bed rest may be necessary beyond the time the chest tube is removed in order to allow for proper healing; conversely the presence of the tube is not a contraindication for sitting in a chair. The nurse would not tell the client to lie on the operative side; this is typical after a pneumonectomy. Lying on either the operative or nonoperative side is a decision made by the surgeon.

18. A mother rescues two of her four children from a house fire. In an emergency department, she cries, "I should have gone back in to get them. I should have died, not them." What is the nurse's best response? 1. "The smoke was too thick. You couldn't have gone back in." 2. "You're experiencing feelings of guilt, because you weren't able to save your children." 3. "Focus on the fact that you could have lost all four of your children." 4. "It's best if you try not to think about what happened. Try to move on."

"You're experiencing feelings of guilt, because you weren't able to save your children." Rationale: The best response by the nurse is, "You're experiencing feelings of guilt, because you weren't able to save your children." This response uses the therapeutic communication technique of restating what the client has said. This lets the client know whether an expressed statement has been understood or if clarification is necessary.

The nurse remains respectful of a client who is engaging in behaviors opposed to the nurse's religious beliefs. Which describes the nurse's response? 1. Confidentiality 2. Unconditional positive regard 3. Genuineness 4. Concrete thinking

2 To show respect is to believe in the dignity and worth of an individual regardless of his or her unacceptable behavior.

What is the normal lab value for Serum Creatinine?

0.6-1.2 mg/dL

Which therapeutic communication technique would the nurse use for a client who is moving rapidly from one thought to another? 1. Focusing 2. Restating 3. Exploring 4. Reflecting

1 Focusing is the therapeutic communication technique that should be used in a client who is rapidly moving from one thought to another. It helps keep the client stable in one thought.

Which precaution should the nurse follow while caring for an Asian American client? 1. Limit touch with this client. 2. Refrain from taking financial gifts from the client. 3. Refrain from collecting background data of the client. 4. Refrain from giving excuses about the client's inappropriate behavior to others.

1 In Asian American culture, touch is not considered acceptable. The nurse should particularly be careful while taking care of an Asian American client.

A client with depression says, "I feel like I am alone out in the ocean." What would the nurse say in response to this statement using a therapeutic communication technique? 1. "You must be feeling very lonely." 2. "Are you feeling that no one understands you?" 3. "You are feeling like nobody cares about you." 4. "Please explain the situation more clearly."

1 In response to the client's statement, the nurse should try to desymbolize the statement and find an underlying clue in it. This statement is attempting to do so.

The nurse tells an angry client, "I see you have been walking back and forth frequently." Which therapeutic communication technique should the nurse follow while communicating with the client? 1. Making observations 2. Giving broad openings 3. Offering general leads 4. Placing the event in a timeline

1 Making observations or verbalizing the perceived behavior encourages the client to recognize specific behavior and evaluate the observation with the nurse. This gives the client an opening to discuss issues.

While caring for a client with deficit knowledge, the nurse says, "I will clarify all your questions about the therapy." What is the rationale behind this statement by the nurse? 1. To protect himself or herself from any verbal attack of the client 2. To present reality to the client 3. To prevent unmanageable anxiety of the client 4. To facilitate understanding between the client and the nurse

1 The statement by the nurse indicates defending. When the nurse speaks in a defending way, it protects from verbal attack.

Which category of touch does the nurse exhibit while greeting a client with a handshake? 1. Social-polite 2. Love-intimacy 3. Friendship-warmth 4. Functional-professional

1 Touch is a powerful communication tool that elicits negative and positive reactions. Social-polite touch is an impersonal touch that conveys an acceptance toward another person. The nurse accepting the client and giving a handshake is an example of social-polite touch.

The client becomes angered when a nurse comes in wearing a hat that reminds the client of the his abusive mother used to wear. Which term describes this emotion? 1. Transference 2. Countertransference 3. Overcoming resistance 4. Reality perception

1 Transference occurs when the client unconsciously displaces (or "transfers") feelings formed toward a person from his or her past onto another.

Which therapeutic communication technique is often used with clients experiencing delusional thinking? 1. Voicing doubt 2. Exploring 3. Verbalizing the implied 4. Belittling feelings expressed

1 Voicing doubt allows the client to express uncertainty of the reality about the client's perception. This technique is often used with clients experiencing delusion thinking.

Which actions of the nurse convey an attitude of respect towards the client? Select all that apply. 1. Being honest while interacting with the client 2. Calling the client by name 3. Spending time with the client 4. Understanding the situation from the client's point of view 5. Promoting an atmosphere of privacy during therapeutic interaction

1, 2, 3, 5 Option 1: Being honest while interacting with the client even when the truth may be difficult to discuss conveys respect. Option 2: Calling the client by name conveys respect towards the client. Option 3: Spending time with the client conveys respect towards the client. Option 4: Understanding the situation from the client's point of view conveys empathy. Option 5: Promoting an atmosphere of privacy during therapeutic interactions with the client conveys respect towards the client.

While obtaining data from a client, the nurse observes that the client's eyebrows are in the frowning position. Which type of associated feelings could the nurse interpret in the client? Select all that apply. 1. Anger 2. Surprise 3. Enthusiasm 4. Unhappiness 5. Concentration

1, 4, 5 When a client shows a facial expression with frowning eyebrows, it indicates that the client is angry. Frowning eyebrows indicate that the client is unhappy. When a client is concentrating on something, then an expression of frowning eyebrows is maintained.

Which gesture exemplifies the "O" in the acronym SOLER for nonverbal behaviors involved in active listening? 1. Sitting in front of the client 2. Lean in toward the client 3. Establish eye contact 4. Client sitting in a chair with arms and legs uncrossed

4 Posture is considered "open" when arms and legs remain uncrossed; observing this is the "O" in SOLER.

21. Which of the following individuals are communicating a message? (Select all that apply.) 1. A mother spanking her son for playing with matches 2. A teenage boy isolating himself and playing loud music 3. A biker sporting an eagle tattoo on his biceps 4. A teenage girl writing, "No one understands me" 5. A father checking for new e-mail on a regular basis

1. A mother spanking her son for playing with matches 2. A teenage boy isolating himself and playing loud music 3. A biker sporting an eagle tattoo on his biceps 4. A teenage girl writing, "No one understands me" Rationale: The nurse should determine that spanking, isolating, getting tattoos, and writing are all ways in which people communicate messages to others. It is estimated that about 70% to80% of communication is nonverbal.

The nurse is standing in front of a room of clients. Which term describes the distance between the nurse and the clients? 1. Intimate 2. Social 3. Personal 4. Public

4 Public distance is one that exceeds 12 feet. Examples include speaking in public or yelling to someone from a distance.

Which precaution is a priority for the nurse to teach a client prescribed semaglutide to prevent harm? A. Only take this drug once weekly B. Report any vision changes immediately C. Do not mix in the same syringe with insulin D. This drug can only be given by a health care professional

A. Only take this drug once weekly

A nurse assesses a client who is experiencing diabetic ketoacidosis (DKA). For which manifestations should the nurse monitor the client? (Select all that apply.) a.Deep and fast respirations b.Decreased urine output c.Tachycardia d.Dependent pulmonary crackles e.Orthostatic hypotension

Answer: A, C, E

What method of emptying the bladder is helpful for a client with a large cystocele? 1. Splinting 2. Credé method 3. Double-voiding 4. Valsalva maneuver

1. Splinting The client with a large cystocele or prolapse of the bladder into the vagina may use splinting to reduce renal urinary incontinence. This is achieved by inserting fingers into the vagina and lifting the cystocele to urinate. The client using the Credé method presses over the bladder area to increase pressure. The client may also trigger nerve stimulation by tugging at the pubic hair or massaging the genital area. In the double-voiding technique, the client empties the bladder once and then attempts a second voiding within a few minutes. The Valsalva maneuver is a breathing technique that increases chest and abdominal pressure. Increased pressure is directed toward the bladder during exhalation.

A client is prescribed trimethoprim/sulfamethoxazole (Septra) for urinary tract infection (UTI). What does the nurse instruct the client about this therapy? Select all that apply. 1. Disclose any allergies to sulfa drugs before beginning therapy. 2. Wear sunscreen and protective clothing when out in the sun. 3. Monitor the pulse twice daily while taking this drug. 4. Drink a full glass of water with each dose of the drug. 5. Avoid taking the drug within 2 hours of taking an antacid.

1. Disclose any allergies to sulfa drugs before beginning therapy. 2. Wear sunscreen and protective clothing when out in the sun. 4. Drink a full glass of water with each dose of the drug. The nurse should ensure that the client does not have any allergies to sulfa drugs before beginning therapy, since allergies to sulfa drugs are common and may require changing the drug therapy. The client should wear sunscreen and protective clothing when out in the sun because sulfamethoxazole increases sensitivity to the sun and can lead to severe sunburns. The client must consume a full glass of water with each dose because the drug can form crystals that precipitate in the kidney tubules; drinking at least 3 L of fluids daily prevents this complication. The client taking fluoroquinolone is asked to monitor the pulse twice daily as this class of drugs induces serious cardiac dysrhythmias. Fluoroquinolone must not be taken within 2 hours of taking an antacid; antacids containing magnesium or aluminum interfere with drug absorption.

The nurse is teaching a group of older adult women about the signs and symptoms of urinary tract infection (UTI). Which concepts does the nurse explain in the presentation? Select all that apply. 1. Dysuria 2. Enuresis 3. Frequency 4. Nocturia 5. Urgency 6. Polyuria

1. Dysuria 3. Frequency 4. Nocturia 5. Urgency Dysuria (painful urination), nocturia (frequent urinating at night), urgency (having the urge to urinate quickly), and frequency are symptoms of UTI. Enuresis (bed-wetting) and polyuria (increased amounts of urine production) are not signs of a UTI.

What nonsurgical methods does the nurse teach the client to manage stress incontinence? Select all that apply. 1. Reduce excess body weight. 2. Walk to strengthen pelvic muscles. 3. Perform Kegel exercises. 4. Use artificial sweeteners instead of sugar. 5. Practice vaginal cone therapy.

1. Reduce excess body weight. 3. Perform Kegel exercises. 5. Practice vaginal cone therapy. The client with stress incontinence should reduce excess weight because increased abdominal pressure aggravates stress incontinence. Kegel exercise therapy strengthens the muscles of the pelvic floor, and weighted vaginal cones are used to strengthen pelvic muscles and decrease stress incontinence. Walking is a good exercise; however, it does not help to increase pelvic strength. The client must avoid foods that irritate the bladder such as artificial sweeteners, alcohol, nicotine, citrus, and caffeine.

What clinical findings does the nurse attribute to the presence of kidney stones? Select all that apply. 1. Smoky urine 2. Odorless urine 3. Increased serum calcium 4. Increased serum phosphate 5. Urine pH of 6

1. Smoky urine 3. Increased serum calcium 4. Increased serum phosphate Smoky or rusty urine is common in clients with kidney stones, indicating hematuria. Increases in serum calcium and phosphate levels indicate that excess minerals are present and may contribute to stone formation. The urine generally has an odor, indicating infection. Normal urine is alkaline, with a pH between 5 and 6. If urine contains uric acid or cystine stones, it is highly acidic. If urine contains calcium phosphate and struvite stones, it is alkaline.

While collecting the history of a client with anger, the nurse observes that the client is blinking his or her eyes continuously. Which associated feeling does the nurse expect from the client's facial expression? 1. Surprise 2. Dislike 3. Low self-esteem 4. Frustration

2 A client who dislikes something often blinks his or her eyes or looks away from the nurse.

Which action of the nurse indicates the working phase of relationship development with a client? 1. Formulating nursing diagnoses 2. Examining the client's feelings, fears, and anxieties 3. Continuously evaluating the client's progress towards goal attainment 4. Developing a plan of action that is realistic for meeting the goals

3 In the working phase, the nurse continuously evaluates the progress of the client towards goal attainment.

Which is a task completed in the preinteraction phase of the client relationship? Gathering assessment information to build a strong client database 2. Promoting the client's insight and perception of reality 3. Using the problem-solving model 4. Examining one's own feelings, fears, and anxieties about working with a particular client

4 Examining one's own feelings, fears, and anxieties about working with a particular client happens in the preinteraction phase.

When (at which time) will the nurse plan to monitor for hypoglycemia in a client with type 1 diabetes received regular insulin at 7:00 a.m.? 7:30 a.m. 7:30 p.m. 11:00 a.m. 2:00 p.m.

11:00 a.m. Regular insulin is a short-acting type of insulin. Onset of action to regular insulin is ½ to 1 hour. The peak effect time is when hypoglycemia may start to occur. Peak time for regular insulin is 2 to 4 hours. Therefore, 11:00 a.m. is the anticipated peak time for regular insulin received at 7:00 a.m. The other options for peak times for regular insulin are incorrect.

While assessing a client who has facial trauma, the nurse auscultates stridor. The client is anxious and restless. Which action should the nurse take first? a. Contact the provider and prepare for intubation. b. Administer prescribed albuterol nebulizer therapy. c. Place the client in high-Fowlers position. d. Ask the client to perform deep-breathing exercises.

A

What is the normal lab value for urine Specific Gravity?

1.01-1.03

The nurse is caring for a psychiatric client who is experiencing concrete thinking. Which nursing intervention is most essential to develop a therapeutic relationship with the client? 1. Establishing an acquaintance with the client 2. Keeping promises made to the client 3. Considering the client's ideas when planning care 4. Being open and real while interacting with the client

2 A client who is experiencing concrete thinking focuses on specifics rather than generalities. Therefore, keeping promises made to the client helps the nurse fulfill the specifics and also develop trust.

The nurse knows touch is a powerful method of communication. Which type of physical contact is considered functional-professional? 1. A handshake 2. Nurse examining a client 3. Engaging in a strong embrace 4. Touching in the genital region

2 A nurse examining a client is functional-professional; it is impersonal and businesslike. It is used to accomplish a task.

A client's spouse presents a bouquet of flowers and chocolates to the nurse for taking care of the client. What is the most appropriate response of the nurse? 1. "I do not accept gifts." 2. "Thank you. I will share your gift with my colleagues." 3. "It is against the rules of the hospital to accept gifts." 4. "The flowers and chocolates are wonderful. Thank you for them."

2 Accepting small gifts as a token of appreciation from the client may be considered appropriate. If the nurse responds, "Thank you. I will share it with my colleagues," it would be more appropriate because the nurse is acknowledging the effort of other staff members involved in caring for the client.

What is an example of a negative attitude? 1. Having an idea that alcoholism is a disease 2. The negative stigma associated with mental illness 3. Conveying the truth to all psychotic clients about their medical illness 4. Conjecturing that all people with mental illness are dangerous

2 An attitude is a way of thinking, which can be selective and biased. The negative stigma associated with mental illness is an example of a negative attitude.

The nurse is caring for a client who is depressed due to the death of his or her partner. At dinnertime the nurse asks the client, "Can I sit and eat with you?" What is the rationale behind this nursing intervention? 1. To emphasize the client's importance 2. To increase the feeling of self-worth 3. To encourage a like response from the client 4. To increase the understanding between the nurse and the client

2 Asking the client to join for dinner indicates that the nurse is making him or herself available to the client and, therefore, increases the feeling of self-worth in the client.

The nurse begins a session with the client by saying, "What would you like to discuss today?" This is an example of which communication technique? 1. Offering general leads 2. Giving broad openings 3. Encouraging the description of perceptions 4. Offering self

2 Broad openings allow the client to take the initiative in introducing the topic.

A client with low self-esteem tells the nurse, "I am of no value to anybody." Which statement by the nurse in response to the client indicates a better example of therapeutic communication? 1. "Of course you are something. Everyone is something." 2. "You are feeling like nobody cares about you right now." 3. "What makes you say this?" 4. "You must be feeling very lonely right now."

2 Denial of the problem blocks discussion with the client and avoids helping the client identify the areas of difficulty. This statement made by the nurse is a better way of responding to the client.

In a therapeutic relationship empathy is extremely important. Which does the nurse know describes empathy? 1. Sharing the feelings of another person 2. Accurately perceiving and understanding another person's feelings 3. Identifying with what another is feeling 4. Experiencing a need to alleviate another's distress.

2 Empathy involves understanding what another person is feeling while staying emotionally separate.

The nurse is developing a therapeutic relationship with the client. Which statement made by the nurse indicates an empathetic response? 1. "I understand your feelings because I have gone through the same thing." 2. "If you felt bad about those harsh comments, it is ok to be sad and cry." 3. "I will surely incorporate your ideas and preferences when planning your care." 4. "Be assured that whatever we discuss will not leave the boundaries of our health-care team."

2 In an empathetic response, the nurse understands and perceives the feelings of the client accurately. The nurse encourages the client to explore his or her feelings. Therefore, this statement indicates an empathetic response.

Which therapeutic technique should the nurse use to encourage a client to recognize specific behaviors and compare perceptions with the nurse? 1. Giving recognition 2. Making observations 3. Giving broad openings 4. Encouraging comparison

2 Making observations is a therapeutic technique that involves what is observed or perceived. This will encourage the client to recognize specific behaviors and compare those perceptions with the nurse.

The nurse is disciplined for having dinner after hours with a client. Which type of boundary was breached? 1. Material boundaries 2. Professional boundaries 3. Personal boundaries 4. Social boundaries

2 Professional boundaries limit and outline expectations for appropriate professional relationships with clients.

When the client begins speaking in a session, the nurse says, "Let's not discuss that today." This is an example of which type of nontherapeutic communication technique? 1. Disapproving 2. Rejecting 3. Defending 4. Probing

2 Refusing to consider or showing contempt for a client's ideas or behavior may cause the client to discontinue interaction with the nurse for fear of further rejection.

The nurse accepts a client unconditionally and regards him or her as a worthy person. Which characteristic is exhibited by the nurse? 1. Trust 2. Respect 3. Empathy 4. Genuineness

2 Respect is to acknowledge the client's dignity, value, and worth. Therefore, if the nurse accepts the client unconditionally and regards him or her as a worthy person, it is a characteristic of respect exhibited by the nurse.

The client says, "All men are messy and untidy." What type of belief does the client have? 1. Faith 2. Stereotype 3. Rational belief 4. Irrational belief

2 Stereotypes are socially shared beliefs, which categorize all individuals with a common factor like age, sex, race, into one group. Therefore, the client's statement represents a stereotype.

The registered nurse is teaching a student nurse about skills for active listening. Which statement made by the student nurse needs correction? 1. "Always maintain an open posture while listening to someone." 2. "Maintain constant eye contact while listening to others." 3. "Always sit squarely facing the client while listening to him or her." 4. "Lean forward toward the client while listening to him or her."

2 The eye contact of the nurse should be intermittently directed, which shows the willingness of the nurse to listen. Thus, this statement needs correction.

The nurse is caring for a client with anxiety. The client asks the nurse, "Will I be normal?" Which nurse's statement reassures the client? 1. "Let's not discuss your thoughts." 2. "Don't worry about it. We can work on that as a team." 3. "Let's talk about how your behavior caused anxiety." 4. "What do you think you should do to become normal?"

2 The nurse reassures the client that there is no need to worry, thereby reassuring the client's feelings and discouraging further anxiety.

Which phase of the nurse-client relationship development deals with creating an environment for the establishment of trust and rapport with the client? 1. The working phase 2. The orientation phase 3. The termination phase 4. The preinteraction phase

2 The orientation phase is the introductory phase where the nurse and client get acquainted. Therefore, if the nurse is creating an environment to establish trust and rapport with the client, then it is the orientation phase.

The nurse is caring for a client who is in the isolation room. Which statement made by the nurse indicates that the nurse is trying to increase the client's feeling of self-worth? 1. "I see you put away your clothes." 2. "I'll sit in here with you for a while." 3. "I notice you are pacing a lot." 4. "Yes, I understand what you said."

2 This statement helps increase the client's feeling of self-worth.

A client who separated from his or her spouse tells the nurse, "I don't know why I am living. There is nothing to live for." Which statement made by the nurse serves as the best nontherapeutic communication technique? 1. "Tell me why you separated." 2. "You appear to be upset. Can you tell me how you're feeling?" 3. "Everybody gets down sometimes. I also feel this sometimes." 4. "It must have been very difficult for you when you separated from your spouse."

2 This statement made by the nurse is the best nontherapeutic communication technique because it conveys understanding and empathy to the client.

While collecting data from the client, the nurse observes that the client is communicating superficially. Which therapeutic communication technique should the nurse apply to respond to this client? 1. "Let's look at this a little more closely." 2. "Please explain the situation more clearly." 3. "Tell me if my understanding agrees with yours." 4. "I am not sure that I understood what you just told me. Would you please explain it again?"

2 When the client is communicating superficially, the nurse should respond by asking the client to explain the situation more clearly. This will make the client improve his or her communication.

When should an individual maintain a distance of 4 to 12 feet while communicating with others? Select all that apply. 1. While communicating with friends 2. While communicating with strangers 3. While communicating with colleagues 4. While communicating at a cocktail party 5. While communicating with others in a public building

2, 4 While interacting with strangers, an individual should maintain a social distance of 4 to 12 feet. While communicating with others at a cocktail party, an individual should maintain a social distance of 4 to 12 feet. (5)?(p.118)An individual while communicating with others in a public building should maintain a pubic distance of more than 12 feet.

The nurse is teaching a client with a neurogenic bladder to use intermittent self-catheterization for bladder emptying. Which client statement indicates a need for further clarification? 1. "A small-lumen catheter will help prevent injury to my urethra." 2. "I will use a new, sterile catheter each time I do the procedure." 3. "My family members can be taught to help me if I need it." 4. "Proper handwashing before I start the procedure is very important."

2. "I will use a new, sterile catheter each time I do the procedure." Catheters are cleaned and reused. With proper handwashing and cleaning of the catheter, no increase in bacterial complications has been shown. Catheters are replaced when they show signs of deteriorating. The smallest lumen possible and the use of a lubricant help reduce urethral trauma to this sensitive mucous tissue. Research shows that family members in the home can be taught to perform straight catheterizations using a clean (rather than a sterile) catheter with good outcomes. Proper handwashing is extremely important in reducing the risk for infection in clients who use intermittent self-catheterization and is a principle that should be stressed.

The nurse receives the change-of-shift report on four clients. Which client does the nurse decide to assess first? 1. 26-year-old admitted 2 days ago with urosepsis with an oral temperature of 99.4° F (37.4° C) 2. 28-year-old with urolithiasis who has been receiving morphine sulfate and has not voided for 8 hours 3. 32-year-old admitted with hematuria and possible bladder cancer who is scheduled for cystoscopy 4. 40-year-old with noninfectious urethritis who is reporting "burning" and has estrogen cream prescribed

2. 28-year-old with urolithiasis who has been receiving morphine sulfate and has not voided for 8 hours Anuria may indicate urinary obstruction at the bladder neck or urethra and is an emergency because obstruction can cause acute kidney failure. The client who has been receiving morphine sulfate may be oversedated and may not be aware of any discomfort caused by bladder distention. The 26-year-old admitted with urosepsis and slight fever, the 32-year-old scheduled for cystoscopy, and the 40-year-old with noninfectious urethritis are not at immediate risk for complications or deterioration.

A client is crying to the nurse because someone made a joke about her being overweight. Which of the nurse's responses is an example of empathy? 1. "I can identify with what you are feeling. I am overweight too." 2. "I get so angry when people are insensitive like that." 3. "You feel angry and embarrassed. It is alright to cry." 4. "It's typical of skinny people to be so rude."

3 Because the nurse identifies the client's feelings and makes it safe for the client to express those feelings, this is an example of empathy.

A client in the emergency department has several broken ribs. What care measure will best promote comfort? a. Allowing the client to choose the position in bed b. Humidifying the supplemental oxygen c. Offering frequent, small drinks of water d. Providing warmed blankets

A Allow the client with respiratory problems to assume a position of comfort if it does not interfere with care. Often the client will choose a more upright position, which also improves oxygenation. The other options are less effective comfort measures.

While escorting a client to the examination room, the nurse observes a smile on the client's face when the primary health-care provider approaches. What emotion is the client expressing? 1. Frustration 2. Disagreement 3. Happiness 4. Low self-esteem

3 Happiness is satisfaction or contentment. A smile on the client's face indicates that he or she is happy when looking at the primary health-care provider, whom he or she has met before.

The nurse is caring for a client with situational low self-esteem. Which statement made by the nurse helps the client recognize his or her life experiences? 1. "Explain what happened when you felt that way." 2. "I understand that you are telling me this happened." 3. "How did you respond when this happened in the past?" 4. "What might you do to handle this more appropriately?"

3 The nurse encourages the client to compare the similarities and differences of experiences and ideas, which helps the client recognize that life experiences may recur.

What is the minimum distance that the nurse should maintain while interacting with a client? 1. 18 inches 2. 30 inches 3. 60 inches 4. 156 inches

3 The nurse should communicate with a stranger within social distance by maintaining a distance of 4 to 12 feet.

During assessment of a client, what finding does the nurse associate with the presence of kidney stones? 1. Oliguria 2. Flank pain 3. Dysuria 4. Flank pain extending to the scrotum

2. Flank pain The major manifestation of kidney stones is renal colic, which begins suddenly and is described as "unbearable." Flank pain indicates that the stone is in the kidney or the upper ureter. Oliguria, or scant urine output, indicates an obstruction at the bladder neck or urethra. Dysuria and frequency occur when the stone reaches the bladder and causes irritation. Flank pain extending to the scrotum suggests that the stones are in the ureters or bladder. The pain is most intense when the stone is moving or when the ureter is obstructed.

What complication may the nurse expect in an older client with an indwelling catheter for urinary continence? 1. Skin breakdown 2. Risk for infection 3. Damage to tissues 4. Urine retention

2. Risk for infection An indwelling catheter increases the risk for infection in a client. An indwelling catheter can help protect the skin and reduce the risk of skin breakdown caused by urinary incontinence. Applied devices, such as intravaginal pessaries for women and penile clamps for men, can cause tissue damage. An indwelling catheter completely drains the bladder, so the client does not have urinary retention.

A client is prescribed prophylactic immunotherapy with intravesical instillation of bacille Calmette-Guérin (BCG) to prevent recurrence of bladder tumor. What does the nurse inform the client about this therapy? 1. The procedure is done in an inpatient setting. 2. The BCG virus compound is allowed to dwell in the bladder for 2 hours. 3. Flush the toilet three times after use. 4. Dispose of clothing that comes in contact with urine in 24 hours.

2. The BCG virus compound is allowed to dwell in the bladder for 2 hours. The BCG virus compound is allowed to dwell in the bladder for 2 hours. Live virus will be excreted when the client urinates. The instillation procedure is done in an outpatient setting. The client must not share the toilet with other household members for 24 hours after instillation; the toilet must be flushed and cleaned with a solution of 10% liquid bleach. The client must wash all clothing that has come into contact with urine during the 24 hours after instillation separately with 10% liquid bleach.

The nurse in the urology clinic is providing teaching for a female client with cystitis. Which instructions does the nurse include in the teaching plan? Select all that apply. 1. Cleanse the perineum from back to front after using the bathroom. 2. Try to take in 64 ounces of fluid each day. 3. Be sure to complete the full course of antibiotics. 4. If urine remains cloudy, call the clinic. 5. Expect some flank discomfort until the antibiotic has worked.

2. Try to take in 64 ounces of fluid each day. 3. Be sure to complete the full course of antibiotics. 4. If urine remains cloudy, call the clinic. Between 64 and 100 ounces (2-3 liters) of fluid should be taken daily to dilute bacteria and prevent infection. Not completing the course of antibiotics could suppress the bacteria, but would not destroy all bacteria, causing the infection to resurface. For persistent symptoms of infection, the client should contact the provider. The perineal area should be cleansed from front to back or "clean to dirty" to prevent infection. Cystitis produces suprapubic symptoms; flank pain occurs with infection or inflammation of the kidney.

The nurse is caring for a client who has pyuria. What does the urinalysis reveal in this client? 1. Kidney stones. 2. White blood cells in the urine. 3. Red blood cells in the urine. 4. Heavy bacteria in the urine.

2. White blood cells in the urine. Urinalysis showing pyuria means that the client has white blood cells in the urine without a large number of bacteria.

Which phase of development of the therapeutic relationship involves the evaluation of goals attained by the client after the therapy? 1. The working phase 2. The orientation phase 3. The termination phase 4. The preinteraction phase

3 In the termination phase, a conclusion is brought to the relationship. Therefore, if the nurse is evaluating the goals attained by the client after the therapy, it is the termination phase.

The nurse is caring for a psychiatric client who has been rejected by his partner. Which nursing intervention will increase feelings of self-worth in the client? 1. Expressing empathy towards the client 2. Getting acquainted with the client 3. Recognizing and respecting the client 4. Providing a safe environment to the client

3 A psychiatric client will have low self-esteem after being rejected by others. Recognizing and respecting the client will increase feelings of self-worth.

What is the normal lab value for serum PHOSPHORUS?

2.4-4.1 mg/dL

The nurse is caring for a client who has had a lobectomy and placement of a chest tube 8 hours ago. When performing an initial assessment, which of these requires immediate follow up? 200 mL red drainage from chest tube over 2 hours Client sleepy but able to be aroused 3 cm area of red drainage on the incisional dressing Report of pain at the chest tube insertion site

200 mL red drainage from chest tube over 2 hours The nurse must immediately report 200 mL of red drainage over a 2 hour span of time. Chest drainage should slow down after surgery. More than 70 mL of drainage/hour must be reported to the surgeon.A client who had a surgical procedure, anesthesia, and analgesia may spend most of the day sleeping, but should be able to be aroused. A small amount of drainage after surgery is expected, such as a 3 cm area. The nurse should circle the area and report increasing amounts to the surgeon. Pain at the surgical and chest tube insertion site is expected and will be managed by the nurse in collaboration with the provider after airway, breathing, and circulation are ensured.

What is the normal lab value for Urine Potassium?

25-100 mEq/L/day

Which client's facial expression is associated with a feeling of low self-esteem? 1. Frowning eyebrows 2. Sneered lips 3. Downcast eyes 4. Wrinkling nose

3 A client with low self-esteem will have downcast eyes or will lack eye contact while communicating with another person.

The nurse is caring for a client during an anger-management program. Which action represents the working phase of therapeutic relationship management? 1. The nurse preparing a plan for continuing care. 2. The nurse assessing the client's previous medical records. 3. The nurse helping the client practice various adaptive procedures to control anger. 4. The client and nurse setting goals to develop some adaptive ways to handle anger.

3 In the working phase, the therapeutic work of the relationship between the nurse and client is carried out. Therefore, the nurse helping the client practice various adaptive procedures to control anger represents the working phase.

A client in group therapy is uncomfortable speaking in front of other members but communicates openly in a one-to-one session with the nurse. Which element is contributing to the client's anxiety? 1. Religion 2. Values 3. Environment 4. Culture

3 Some individuals who feel uncomfortable and refuse to speak during a group therapy session may be open and willing to discuss problems privately on a one-to-one basis with the nurse because the environment feels safer.

While caring for a client with persistent depressive disorder, the nurse says, "I can understand what you are feeling now. I too have been in this same situation before." Which feeling of the nurse is indicated by these statements? 1. Rapport 2. Empathy 3. Sympathy 4. Genuineness

3 Sympathy is the ability to share the feelings of the client. In this case, the nurse is trying to share his or her feelings with the client about a similar situation that was experienced earlier.

The nurse is in the first phase of relationship development with a client who is an alcoholic. What should be the goal of the nurse during this phase? 1. Establishing trust 2. Promoting client change 3. Exploring self-perceptions 4. Ensuring therapeutic closure

3 The first phase is the preinteraction phase, where the nurse prepares for the first encounter with the client. Everyone brings attitudes and feelings from their own experiences to the clinical setting. Therefore, it is necessary to be aware of self-perceptions so as to not let them affect providing care to clients.

The nurse is counseling a group of clients on a one-to-one basis to obtain information regarding their current health situation. Which type of distance should the nurse maintain while communicating with the clients? 1. Public 2. Social 3. Personal 4. Intimate

3 The nurse can have a close conversation with the client in a personal distance while maintaining a distance of 18 to 40 inches.

The nurse is caring for a client who unconsciously transfers his or her feelings for a person in the client's past towards the nurse because the nurse's appearance reminds the client of that person. Which outcome in the client would indicate the effectiveness of the nursing care? 1. The client will formulate a plan with the nurse. 2. The client will develop problem-solving skills. 3. The client will assume responsibility for his or her own behavior. 4. The client will discuss and compare the exhibited behaviors with the nurse.

3 When the client assumes responsibility for his or her own behavior, it indicates that the client no longer shows the transference behavior and that the nursing care is effective.

Which facial expressions of a client may indicate disgust? Select all that apply. 1. Raised brows 2. Sticking out the tongue 3. Wrinkling up the nose 4. Eyelids squeezed shut 5. Canine-type snarl

3, 5 The facial expression of wrinkling up the nose indicates disgust. Canine-type snarl indicates disgust.

The nurse is instructing an older adult female client about interventions to decrease the risk for cystitis. Which client comment indicates that the teaching was effective? 1. "I must avoid drinking carbonated beverages." 2. "I need to douche vaginally once a week." 3. "I should drink 2½ liters of fluid every day." 4. "I will not drink fluids after 8 pm each evening."

3. "I should drink 2½ liters of fluid every day." Drinking 2½ liters of fluid a day flushes out the urinary system and helps reduce the risk for cystitis. Avoiding carbonated beverages is not necessary to reduce the risk for cystitis. Douching is not a healthy behavior because it removes beneficial organisms as well as the harmful ones. Avoiding fluids after 8 pm would help prevent nocturia but not cystitis. It is recommended that clients with incontinence problems limit their late-night fluid intake to 120 mL.

A 53-year-old postmenopausal woman reports "leaking urine" when she laughs, and is diagnosed with stress incontinence. What does the nurse tell the client about how certain drugs may be able to help with her stress incontinence? 1. "They can relieve your anxiety associated with incontinence." 2. "They help your bladder to empty." 3. "They may be used to improve urethral resistance." 4. "They decrease your bladder's tone."

3. "They may be used to improve urethral resistance." Bladder pressure is greater than urethral resistance; drugs may be used to improve urethral resistance. Relieving anxiety has not been shown to improve stress incontinence. No drugs have been shown to promote bladder emptying, and this is not usually the problem with stress incontinence. Emptying the bladder is accomplished by the individual, or if that is not possible, by using a catheter. Decreasing bladder tone would not be a desired outcome for a woman with incontinence.

A client is prescribed estrogen therapy for urinary incontinence. What does the nurse teach the client about this therapy? 1. Change positions slowly, especially in the mornings. 2. Report urine output that is significantly lower than fluid intake. 3. A thin application of cream is adequate. 4. Use hard candy to moisten the mouth.

3. A thin application of cream is adequate. Teach the client that a thin application of estrogen cream is all that is needed. The client taking tricyclic antidepressants must change positions slowly, especially in the mornings, because these drugs cause dizziness, orthostatic hypotension, and increase the risk for falls. The client taking antispasmodics or anticholinergics should report urine output that is significantly lower than fluid intake as these drugs cause urine retention. Dry mouth is another side effect of antispasmodics and anticholinergics; the client can use hard candy to moisten the mouth.

While caring for a client with anger, the nurse formulates a plan of action with the client. What would be the rationale behind this nursing intervention? 1. To nurture the client in the dependent role 2. To find clues to the underlying true feelings of the client 3. To encourage a like response from the client 4. To prevent anxiety or anger from escalating to an unmanageable level

4 Formulating a plan of action with the client helps prevent anxiety or anger from escalating to an unmanageable level.

The nurse tells the client the truth. What is this characteristic of the nurse called? 1. Respect 2. Empathy 3. Sympathy 4. Genuineness

4 Genuineness is the ability of the nurse to interact with the client by being honest and real. Therefore, if the nurse is telling the truth to the client, it represents genuineness.

The nurse knows which is an important characteristic of the therapeutic relationships? 1. Self-directed 2. Goal-oriented 3. One-sided 4. Collaborative

4 Ideally, the nurse and client 'decide together' what the goal of the therapeutic relationship will be.

A client with depression says, "Everyone has deserted me. I feel that my life has no value." Which nontherapeutic technique would the nurse use to take over this discussion? 1. Interpreting the situation 2. Using denial technique 3. Expressing belittling feelings 4. Introducing an unrelated topic

4 Introducing an unrelated topic allows the nurse to take over the discussion.

Which nonverbal behavior of the nurse indicates an open posture for attentive listening? 1. Establishing eye contact 2. Sitting squarely facing the client 3. Leaning forward toward the client 4. Keeping the arms and legs uncrossed

4 Keeping the arms and legs uncrossed indicates an open posture for attentive listening.

What is the normal Osmolality of Urine?

300-900 mOsm/kg

A cognitively impaired client has urge incontinence. Which method for achieving continence does the nurse include in the client's care plan? 1. Bladder training 2. Credé method 3. Habit training 4. Kegel exercises

3. Habit training Habit training (scheduled toileting) will be most effective in reducing incontinence for a cognitively impaired client because the caregiver is responsible for helping the client to a toilet on a scheduled basis. Bladder training, the Credé method, and learning Kegel exercises require that the client be alert, cooperative, and able to assist with his or her own training.

Which represents love-intimacy in an individual? 1. Accepting the hand of a person during a handshake 2. Expressing physical attraction towards others 3. Having a strong desire towards a person 4. Being emotionally attached to a person

4 Emotional attachment or attraction to a person represents love-intimacy in an individual.

The certified Wound, Ostomy, and Continence Nurse (CWOCN) or enterostomal therapist (ET) teaches a client who has had a cystectomy about which care principles for the client's postdischarge activities? 1. Nutritional and dietary care 2. Respiratory care 3. Stoma and pouch care 4. Wiping from front to back (asepsis)

3. Stoma and pouch care The enterostomal therapist demonstrates external pouch application, local skin care, pouch care, methods of adhesion, and drainage mechanisms. The registered dietitian (RD) teaches the cystectomy client about nutritional care. The respiratory therapist teaches the cystectomy client about respiratory care. The client with a cystectomy does not require instruction about front-to-back wiping.

A client is ordered phenazopyridine (Prodium) to reduce bladder pain and burning on urination. What does the nurse teach the client about this drug regimen? 1. Report if the urine turns red. 2. Report blurred vision. 3. Take the drug with a meal. 4. Wear dark glasses in sunlight.

3. Take the drug with a meal. The client should take the drug with a meal to prevent gastrointestinal disturbances. The client need not report if the urine turns red or orange because this is an expected response to the drug. The client taking antispasmodics for relieving bladder spasms is asked to report blurred vision,which is a manifestation of toxicity. The client taking antispasmodics, not analgesics, is asked to wear dark glasses in sunlight as the drug dilates the pupil and increases eye sensitivity to light.

While caring for an angry client, the nurse makes empty conversation. What could be the rationale behind this nursing intervention? 1. To explore the client's areas of difficulty 2. To tell the client about the meaning of the experience 3. To take over the direction of the discussion from the client 4. To encourage a like response from the client

4 Empty conversation from the nurse encourages a like response from the client. It is practiced as a nontherapeutic communication technique in which stereotyped comments are made to encourage a like response from the client.

What is the normal lab value for serum ALBUMIN?

3.5-5.5 g/dL

The nurse tells a client with a history of violence, "Let's discuss your behavior and see if we can clarify the situation." What could be the rationale behind this statement? 1. To help recognize life experiences that tend to recur 2. To define the perception of the situation for the client 3. To delve further into the subject with the client, beyond a superficial level of communication 4. To facilitate and increase the rapport for both client and nurse

4 Seeking clarification and validation facilitates and increases the mutual understanding between the nurse and the client. Therefore, this statement facilitates and increases the rapport for both client and nurse.

The nurse is conducting a process recording of a client with a history of anger. The nurse asks the client, "What were your feelings before taking all those pills the other night?" The client replies, "I was just so angry thinking that my husband wants a divorce." What may likely be the nurse's thoughts after the client's reply? 1. Feeling sorry 2. Feeling uncomfortable 3. Trying hard to remain objective 4. Feeling more comfortable

4 The nurse likely begins to feel comfortable after the client's reply because the client is willing to talk to the nurse and trusts him or her. This change in behavior helps the nurse to explore and understand the client.

The nurse is conducting a process recording of a client with anger who has attempted suicide. Which statement or question made by the nurse may make the nurse uncomfortable? 1. "How are you feeling about the situation now?" 2. "Seems like a pretty drastic way to make your point." 3. "You wanted to hurt yourself because you were cheated." 4. "Do you still have thoughts of harming yourself?"

4 The nurse may feel uncomfortable asking the client about thoughts of harming himself or herself. This question is asked to know whether the client has suicidal tendencies.

A client who lost his or her spouse in an accident tells the nurse, "Leave me alone. I can't talk to you." How should the nurse respond to this client using therapeutic communication techniques? 1. "Everything will be fine." 2. "Tell me what you are thinking?" 3. "I think you should come in here and discuss your feelings." 4. "Are you feeling that no one understands your feelings?"

4 The nurse should put into words what the client has implied or said indirectly. This statement made by the nurse is the best response to the client.

A client tells the nurse, "I can't concentrate on anything. My mind keeps wandering." How should the nurse respond to convey to the client that he or she has understood the client's statement? 1. Referring questions back to the client 2. Taking notice of a single idea of the client 3. Researching further into the client's feelings 4. Repeating the main idea of what the client has said

4 The nurse should repeat the main idea of what the client has said to make the client know that the statement is understood. Repeating the main idea will help both the client and the nurse to validate the statement.

The nurse is caring for a client with chronic low self-esteem. Which statement made by the nurse indicates an accepting attitude? 1. "I want to listen to what you have to say." 2. "Explain to me what you are feeling now." 3. "We can sit in the dayroom and eat dinner together." 4. "I understand what you're saying."

4 This statement indicates an accepting attitude of reception to and regard for the client.

The health care provider requests phenazopyridine (Pyridium) for a client with cystitis. What does the nurse tell the client about the drug? 1. "It will act as an antibacterial drug." 2. "This drug will treat your infection, not the symptoms of it." 3. "You need to take the drug on an empty stomach." 4. "Your urine will turn red or orange while on the drug."

4. "Your urine will turn red or orange while on the drug." Phenazopyridine will turn the client's urine red or orange. Clients should be warned about this effect of the drug because it will be alarming to them if they are not informed, and care should be taken because it will stain undergarments. Phenazopyridine reduces bladder pain and burning by exerting a local analgesic/anesthetic effect on the mucosa of the urinary tract. It does treat the symptoms of bladder infection; it has no antibacterial action. Phenazopyridine should be taken with a meal or immediately after eating.

The nurse is teaching a client who is scheduled for a neobladder and a Kock's pouch. Which client statement indicates a correct understanding of these procedures? 1 . "If I restrict my oral intake of fluids, the adjustment will be easier." 2. "I must go to the restroom more often because my urine will be excreted through my anus." 3. "I need to wear loose-fitting pants so the urine can flow into my ostomy bag." 4. "I will have to drain my pouch with a catheter."

4. "I will have to drain my pouch with a catheter." For the client with a neobladder and a Kock's pouch, urine is collected in a pouch and is drained with the use of a catheter. Fluids should not be restricted. A neobladder does not require the use of an ostomy bag.

The nurse is teaching a client about pelvic muscle exercises. What information does the nurse include? 1. "For the best effect, perform all of your exercises while you are seated on the toilet." 2. "Limit your exercises to 5 minutes twice a day, or you will injure yourself." 3. "Results should be visible to you within 72 hours." 4. "You know that you are exercising correct muscles if you can stop urine flow in midstream."

4. "You know that you are exercising correct muscles if you can stop urine flow in midstream." When the client can start and stop the urine stream, the pelvic muscles are being used. Pelvic muscle exercises can be performed anywhere and should be performed more often than 5 minutes twice daily. Noticeable results take several weeks.

Which client does the nurse manager on the medical unit assign to an experienced LPN/LVN? 1. 42-year-old with painless hematuria who needs an admission assessment 2. 46-year-old scheduled for cystectomy who needs help in selecting a stoma site 3. 48-year-old receiving intravesical chemotherapy for bladder cancer 4. 55-year-old with incontinence who has intermittent catheterization prescribed

4. 55-year-old with incontinence who has intermittent catheterization prescribed Insertion of catheters is within the education and legal scope of practice for LPN/LVNs. Admission assessments and intravesical chemotherapy should be done by an RN. Preoperative preparation for cystectomy and stoma site selection should be done by an RN and either a Certified Wound, Ostomy, and Continence Nurse (CWOCN) or an enterostomal therapy (ET) nurse.

What clinical finding in a postmenopausal client with urethritis does the nurse attribute to low estrogen levels? 1. The urinalysis indicates pyuria. 2. The urethral culture is positive for bacteria. 3. The urinalysis indicates presence of bacteria. 4. A pelvic examination shows tissue changes.

4. A pelvic examination shows tissue changes. A pelvic examination of a postmenopausal client shows tissue changes due to low estrogen. The client with urethritis does not have pyuria or white blood cells (WBCs) in the urine. The urethral culture and urinalysis is usually negative for bacteria. These clients may have improvement in their urethral symptoms with the use of estrogen vaginal cream.

What is the best way for the nurse to communicate with a client who is intubated and is receiving mechanical ventilation? a.Ask the client to point to words on a board. b.Ask the client to blink for "yes" and "no." c.Have the client mouth words slowly. d.Teach the client some simple sign language. The nurse should have the client point to words on a board to communicate needs. The endotracheal tube is positioned and placement is maintained with tape or some other type of appliance. Asking the client to move his or her mouth and lips could result in possible extubation. Communication is limited and could be misunderstood with blinking. Teaching the client sign language, even simple, would be an involved and unrealistic goal.

A

What is the normal lab value for Urine Protein?

<100 mg/dL

What is the normal lab value for Urine RBCs?

<4 RBC/HPF

The nurse is caring for a client with cystitis. What does the nurse ask the client to include in the diet as part of nutritional therapy? 1. Carbonated beverages 2. Tomato products 3. Caffeine 4. Cranberry juice

4. Cranberry juice The client with cystitis should consume 50 mL of concentrated cranberry juice daily because it is known to decrease the ability of bacteria to adhere to the epithelial cells lining the urinary tract, decreasing the incidence of symptomatic urinary tract infections in some clients. Cranberry juice must be consumed for 3 to 4 weeks to be effective. Caffeine, carbonated beverages, and tomato products must be avoided to decrease bladder irritation during cystitis.

What does the nurse teach a client to do to decrease the risk for urinary tract infection (UTI)? 1. Limit fluid intake. 2. Increase caffeine consumption. 3. Limit sugar intake. 4. Drink about 3 liters of fluid daily.

4. Drink about 3 liters of fluid daily. Drinking about 3 liters of fluid daily, if another medical problem does not require fluid restriction, helps prevent dehydration and UTIs. Fluids flush the system and should not be limited. Increased caffeine intake and limiting sugar intake will not prevent UTIs.

In what location would the nurse expect to find infection in a client with acute pyelonephritis? 1. Urethra 2. Urinary bladder 3. Prostate gland 4. Kidneys

4. Kidneys Acute pyelonephritis is a kidney infection. Urinary tract infections are described by their location in the tract. Urethritis is an acute infection in the urethra, cystitis in the bladder, and prostatitis in the prostate gland.

What procedure does the nurse expect the health care provider to prescribe for the removal of a large, impacted stone in a client's kidney? 1. Lithotripsy 2. Stenting 3. Pyelolithotomy 4. Nephrolithotomy

4. Nephrolithotomy Nephrolithotomy, an open surgical procedure, is often prescribed to remove a large, impacted stone in the kidney. This method is performed if all other procedures fail and there is a possible risk for a lasting injury to the ureter or kidney. Lithotripsy is the use of sound, laser, or dry shock waves to break stones into small fragments. Stenting is a minimally invasive procedure performed by placing a stent in the ureter by ureteroscopy. The stent dilates the ureter, enlarging the passageway for the stone or stone fragments. Pyelolithotomy is an open surgical procedure to remove stones in the kidney pelvis.

A client reports experiencing involuntary loss and constant dribbling of urine due to an enlarged prostate. How does the nurse document this incontinence? 1. Stress incontinence 2. Urge incontinence 3. Reflex incontinence 4. Overflow incontinence

4. Overflow incontinence This client's condition is known as overflow incontinence. The urethra in the client is obstructed due to the enlarged prostate; the urethra fails to relax sufficiently to allow urine to flow, resulting in incomplete bladder emptying or complete urinary retention. Loss of urine following physical exertion, cough, or sneeze is documented as stress incontinence. Stress incontinence occurs due to intrinsic sphincter deficiency or acquired anatomic damage to the urethral sphincter. The client with urge incontinence experiences an involuntary loss of urine with a strong desire to urinate. The client with reflex incontinence has a post-void residual less than 50 mL.

What is the normal lab value for Urine pH?

4.8-7.5

What is the normal lab value for Urine Sodium?

40-220 mEq/day

A client in the intensive care unit with acute kidney injury (AKI) must maintain a mean arterial pressure (MAP) of 65 mm Hg to promote kidney perfusion. What is the client's MAP if the blood pressure is 98/50 mm Hg? (Record your answer using a whole number.) _____ mm Hg

66 mmHg (sytolic + diastolic + diastolic)/3= MAP

A client admitted with respiratory difficulty and decreased oxygen saturation keeps pulling off the oxygen mask. What action does the nurse take? a.Stays with the client and replaces the oxygen mask b.Asks the client's spouse to hold the oxygen mask in place c.Restrains the client per facility policy d.Contacts the health care provider and requests sedation Restlessness and confusion are clinical manifestations of hypoxemia. It is important that the nurse stay with the client, ensure that the oxygen is maintained, and attempt to calm the client. Because of the client's restlessness, the nurse cannot delegate care to the spouse. Requesting a sedative might adversely affect the client's respiratory status further. Restraining the client could increase restlessness and increase oxygen demand.

A

A client with diabetes has all of the following changes after a percutaneous nephrolithotomy procedure. Which change is most important for the nurse need to immediately report to the health care provider? A. Difficulty breathing and an oxygen saturation of 88% on 2 L of oxygen by nasal cannula B. A point-of-care blood glucose of 150 mg/dL and client report of thirst C. A decreased hematocrit by 1% (compared with preoperative values and hematuria D. An oral temperature of 38° C (101° F) and cloudiness of urine draining from the nephrostomy tube right after IV administration of a broad-spectrum antibiotic

A

A nurse assesses a client who has a nasal fracture. The client reports constant nasal drainage, a headache, and difficulty with vision. Which action should the nurse take next? a. Collect the nasal drainage on a piece of filter paper. b. Encourage the client to blow his or her nose. c. Perform a test focused on a neurologic examination. d. Palpate the nose, face, and neck.

A

A nurse cares for a client after radiation therapy for lung cancer. The client reports a sore throat. Which action should the nurse take first? a. Ask the client to gargle with mouthwash containing lidocaine. b. Administer prescribed intravenous pain medications. c. Explain that soreness is normal and will improve in a couple days. d. Assess the clients neck for redness and swelling.

A

A nurse cares for a client who is experiencing epistaxis. Which action should the nurse take first? a. Initiate Standard Precautions. b. Apply direct pressure. c. Sit the client upright. d. Loosely pack the nares with gauze.

A

A nurse is caring for a client on mechanical ventilation and finds the client agitated and thrashing about. What action by the nurse is most appropriate? a. Assess the cause of the agitation. b. Reassure the client that he or she is safe. c. Restrain the client's hands. d. Sedate the client immediately. The nurse needs to determine the cause of the agitation. The inability to communicate often makes clients anxious, even to the point of panic. Pain and confusion can also cause agitation. Once the nurse determines the cause of the agitation, he or she can implement measures to relieve the underlying cause. Reassurance is also important but may not address the etiology of the agitation. Restraints and more sedation may be necessary, but not as a first step

A

The advanced-practice nurse is performing a digital rectal examination (DRE) and notes that the rectal sphincter contracts on digital insertion. How does the nurse interpret this finding? a. Nerve supply to the bladder is most likely intact. b. There is adequate strength in the pelvic floor. c. A rectocele is placing pressure on the bladder. d. Abnormal function for the bladder is unlikely.

A

The nurse is assessing arterial blood gases (ABGs). The client with which ABG reading requires the nurse's immediate attention? a.pH, 7.32; PaCO2, 55 mm Hg; PaO2, 70 mm Hg b.pH, 7.45; PaCO2, 42 mm Hg; PaO2, 70 mm Hg c.pH, 7.48; PaCO2, 38 mm Hg; PaO2, 60 mm Hg d.pH, 7.55; PaCO2, 32 mm Hg; PaO2, 50 mm Hg This client has the most severe hypoxia and respiratory alkalosis, indicated by low partial pressure of arterial carbon dioxide (PaCO2) values on ABG analysis.

A

The nurse is caring for a client who is intubated with an endotracheal tube and on a mechanical ventilator. The client is able to make sounds. What is the nurse's first action? a.Check cuff inflation on the endotracheal tube. b.Listen carefully to the client. c.Call the health care provider. d.Auscultate the lungs. If the client has the cuff on the endotracheal tube inflated, the cuff should prevent air from going around the cuff and through the vocal cords. If the client can talk with the cuff inflated, the cuff probably has a leak, causing it to become deflated and allowing air to pass through. The risk is that the client will not receive the prescribed tidal volume.

A

The nurse is caring for a client who is receiving mechanical ventilation accompanied by positive end-expiratory pressure (PEEP). What assessment findings require immediate intervention? a.Blood pressure drop from 110/90 mm Hg to 80/50 mm/Hg b.Pulse oximetry value of 96% c.Arterial blood gas (ABG): pH, 7.40; PaO2, 80 mm Hg; PaCO2, 45 mm Hg; HCO3-, 26 mEq/L d.Urinary output of 30 mL/hr Increased intrathoracic pressure can inhibit blood return to the heart and cause decreased cardiac output. This manifests with a drop in blood pressure. The pulse oximetry reading, ABGs, and urinary output are all normal.

A

An older adult woman who reports a change in bladder function says, "I feel like a child who sometimes pees her pants." What is the nurse's best response? "Have you tried using the toilet at least every couple of hours?" "How does that make you feel?" "We can fix that." "That happens when we get older."

A By emptying the bladder on a regular basis, urinary incontinence from overflow may be avoided, which may give the client some sense of control. The client has already stated how she feels; asking her again how she feels does not address her concern nor does it allow for nursing education. The nurse cannot assert that the problem can be fixed because this may be untrue. Suggesting that the problem occurs as we get older does not address the client's concern and does not provide for nursing education.

A client has returned from a captopril renal scan. Which teaching does the nurse provide when the client returns? "Arise slowly and call for assistance when ambulating." "I must measure your intake and output." "We must save your urine because it is radioactive." "I must attach you to this cardiac monitor."

A Captopril can cause severe hypotension during and after the procedure, so the client should be warned to avoid rapid position changes and about the risk for falling as a result of orthostatic (positional) hypotension. Intake and output measurement is not necessary after this procedure, unless it had been requested previously. A small amount of radionuclide is used in a renal scan; the urine is not radioactive, although the nurse should practice Standard Precautions, as always, and wear gloves. Cardiac monitoring is not needed, although the nurse should monitor for hypotension secondary to captopril.

Which urinary assessment information for a client indicates the potential need for increased fluids? Increased blood urea nitrogen Increased creatinine Pale-colored urine Decreased sodium

A Increased blood urea nitrogen can indicate dehydration. Increased creatinine indicates kidney impairment. Pale-colored urine signifies diluted urine, which indicates adequate fluid intake. Increased, not decreased, sodium indicates dehydration.

Which age-related change can cause nocturia? Decreased ability to concentrate urine Decreased production of antidiuretic hormone Increased production of erythropoietin Increased secretion of aldosterone

A Nocturia may result from decreased kidney-concentrating ability associated with aging. Increased production of antidiuretic hormone, decreased production of erythropoietin, and decreased secretion of aldosterone are age-related changes.

A client with these assessment data is preparing to undergo a computed tomography scan with contrast: Physical Assessment Diagnostic Findings Medications Flank pain BUN 54 mg/dL Captopril Dysuria Creatinine 2.4 mg/dL Metformin Bilateral knee pain Calcium 8.5 mg/dL Acetylcysteine Which medication does the nurse plan to administer before the procedure?] Acetylcysteine (Mucosil) Metformin (Glucophage) Captopril (Capoten) Acetaminophen (Tylenol)

A This client has kidney impairment demonstrated by increased creatinine. Acetylcysteine (an antioxidant) may be used to prevent contrast-induced nephrotoxic effects. Metformin is held at least 24 hours before procedures using contrast. Although captopril and acetaminophen may be administered with a sip of water with permission of the provider, this is not essential before the procedure.

A client has been diagnosed with a very large pulmonary embolism (PE) and has a dropping blood pressure. What medication should the nurse anticipate the client will need as the priority? a. Alteplase (Activase) b. Enoxaparin (Lovenox) c. Unfractionated heparin d. Warfarin sodium (Coumadin)

A Activase is a "clot-busting" agent indicated in large PEs in the setting of hemodynamic instability. The nurse knows this drug is the priority, although heparin may be started initially. Enoxaparin and warfarin are not indicated in this setting.

13. A nurse cares for a client admitted from a nursing home after several recent falls. What prescription should the nurse complete first? a. Obtain urine sample for culture and sensitivity. b. Administer intravenous antibiotics. c. Encourage protein intake and additional fluids. d. Consult physical therapy for gait training.

A Although all interventions are or might be important, obtaining a urine sample for urinalysis takes priority. Often urinary tract infection (UTI) symptoms in older adults are atypical, and a UTI may present with new onset of confusion or falling. The urine sample should be obtained before starting antibiotics. Dietary requirements and gait training should be implemented after obtaining the urine sample.

10. After teaching a client with a history of renal calculi, the nurse assesses the clients understanding. Which statement made by the client indicates a correct understanding of the teaching? a. I should drink at least 3 liters of fluid every day. b. I will eliminate all dairy or sources of calcium from my diet. c. Aspirin and aspirin-containing products can lead to stones. d. The doctor can give me antibiotics at the first sign of a stone.

A Dehydration contributes to the precipitation of minerals to form a stone. Although increased intake of calcium causes hypercalcemia and leads to excessive calcium filtered into the urine, if the client is well hydrated the calcium will be excreted without issues. Dehydration increases the risk for supersaturation of calcium in the urine, which contributes to stone formation. The nurse should encourage the client to drink more fluids, not decrease calcium intake. Ingestion of aspirin or aspirin-containing products does not cause a stone. Antibiotics neither prevent nor treat a stone.

6. A nurse cares for adult clients who experience urge incontinence. For which client should the nurse plan a habit training program? a. A 78-year-old female who is confused b. A 65-year-old male with diabetes mellitus c. A 52-year-old female with kidney failure d. A 47-year-old male with arthritis

A For a bladder training program to succeed in a client with urge incontinence, the client must be alert, aware, and able to resist the urge to urinate. Habit training will work best for a confused client. This includes going to the bathroom (or being assisted to the bathroom) at set times. The other clients may benefit from another type of bladder training.

5. After teaching a client who has stress incontinence, the nurse assesses the clients understanding. Which statement made by the client indicates a need for additional teaching? a. I will limit my total intake of fluids. b. I must avoid drinking alcoholic beverages. c. I must avoid drinking caffeinated beverages. d. I shall try to lose about 10% of my body weight.

A Limiting fluids concentrates urine and can irritate tissues, leading to increased incontinence. Many people try to manage incontinence by limiting fluids. Alcoholic and caffeinated beverages are bladder stimulants. Obesity increases intra-abdominal pressure, causing incontinence.

The client with sleep apnea asks her nurse how her new prescription for Xyrem (sodium oxybate) can help this problem. What is the nurse's best response? A. "The drug depresses your nervous system and allows you to have a deeper night's sleep." B. "Xyrem stimulates daytime wakefulness so that you are less likely to have narco-lepsy with your sleep apnea." C. "The drug reduces the water content of your oral mucous membranes, increasing the diameter of your throat so that your tongue does not obstruct your airway." D. "Xyrem constantly elevates the carbon dioxide content of your blood, which then triggers the respiratory centers of the brain so that you breathe more rapidly and more deeply."

A One problem with sleep apnea is that clients get so little restful sleep at night that they develop excessive daytime sleepiness (narcolepsy), often falling asleep while at work, driving, or per-forming other tasks. Xyrem does not help the apnea but does promote a deeper sleep, so that the client is less likely to fall asleep at inappropriate places or times during the day.

A client appears dyspneic, but the oxygen saturation is 97%. What action by the nurse is best? a. Assess for other manifestations of hypoxia. b. Change the sensor on the pulse oximeter. c. Obtain a new oximeter from central supply. d. Tell the client to take slow, deep breaths.

A Pulse oximetry is not always the most accurate assessment tool for hypoxia as many factors can interfere, producing normal or near-normal readings in the setting of hypoxia. The nurse should conduct a more thorough assessment. The other actions are not appropriate for a hypoxic client.

16. A nurse obtains the health history of a client with a suspected diagnosis of bladder cancer. Which question should the nurse ask when determining this clients risk factors? a. Do you smoke cigarettes? b. Do you use any alcohol? c. Do you use recreational drugs? d. Do you take any prescription drugs?

A Smoking is known to be a factor that greatly increases the risk of bladder cancer. Alcohol use, recreational drug use, and prescription drug use (except medications that contain phenacetin) are not known to increase the risk of developing bladder cancer.

The urine output of a patient with a kidney stone has decreased from 40 mL/hr to 5 mL/hr. What is the nurse's priority action? a. Ensure IV access and notify the health care provider. b. Perform the Credé maneuver on the patient's bladder. c. Test the urine for ketone bodies. d. Document the finding and continue monitoring.

A - Ensure IV access and notify the health care provider.

18. A nurse teaches a young female client who is prescribed amoxicillin (Amoxil) for a urinary tract infection. Which statement should the nurse include in this clients teaching? a. Use a second form of birth control while on this medication. b. You will experience increased menstrual bleeding while on this drug. c. You may experience an irregular heartbeat while on this drug. d. Watch for blood in your urine while taking this medication.

A The client should use a second form of birth control because penicillin seems to reduce the effectiveness of estrogen-containing contraceptives. She should not experience increased menstrual bleeding, an irregular heartbeat, or blood in her urine while taking the medication.

A nurse is caring for a client on mechanical ventilation and finds the client agitated and thrashing about. What action by the nurse is most appropriate? a. Assess the cause of the agitation. b. Reassure the client that he or she is safe. c. Restrain the client's hands. d. Sedate the client immediately.

A The nurse needs to determine the cause of the agitation. The inability to communicate often makes clients anxious, even to the point of panic. Pain and confusion can also cause agitation. Once the nurse determines the cause of the agitation, he or she can implement measures to relieve the underlying cause. Reassurance is also important but may not address the etiology of the agitation. Restraints and more sedation may be necessary, but not as a first step.

23. A nurse assesses a client who presents with renal calculi. Which question should the nurse ask? a. Do any of your family members have this problem? b. Do you drink any cranberry juice? c. Do you urinate after sexual intercourse? d. Do you experience burning with urination?

A There is a strong association between family history and stone formation and recurrence. Nephrolithiasis is associated with many genetic variations; therefore, the nurse should ask whether other family members have also had renal stones. The other questions do not refer to renal calculi but instead are questions that should be asked of a client with a urinary tract infection.

A young woman tells the nurse that she gets frequent UTIs that seem to follow sexual intercourse. Which questions would the nurse ask? (Select all that apply.) a. "Do you use a diaphragm or spermicides for contraception?" b. "Do you feel guilty or embarrassed about your sexual activities?" c. "Have you considered abstaining from intercourse?" d. "Do you and your partner(s) wash the perineal area before intercourse?" e. "Some positions cause more irritation during sex. Have you noticed this?"

A - "Do you use a diaphragm or spermicides for contraception?" D - "Do you and your partner(s) wash the perineal area before intercourse?" E - "Some positions cause more irritation during sex. Have you noticed this?"

The nurse is teaching a patient about self-care measures to prevent UTIs. Which daily fluid intake does the nurse recommend to the patient to prevent a bladder infection? a. 2 to 3 L of water b. 3 to 6 glasses of iced tea c. 4 to 6 cups of electrolyte fluid d. 3 to 4 glasses of juice

A - 2 to 3 L of water

Teaching intermittent self-catheterization for incontinence is appropriate for which patient? a. 25-year-old male patient with paraplegia b. 35-year-old female patient with stress incontinence c. 70-year-old patient who wears absorbent briefs d. 18-year-old patient with a severe head injury

A - 25-year-old male patient with paraplegia

Which patient is mostly likely to have mixed incontinence? a. 54-year-old woman who had four full-term pregnancies b. 52-year-old man who had a stroke with neurologic deficits c. 76-year-old man with benign prostatic hyperplasia d. 25-year-old woman who has a pelvic fracture

A - 54-year-old woman who had four full-term pregnancies

The nurse is caring for a patient with an indwelling catheter. What intervention does the nurse use to minimize catheter-related infections? a. Assess the patient daily to determine need for catheter. b. Irrigate the catheter daily with sterile solution to remove debris. c. Use sterile technique when opening system to obtain urine samples. d. Apply antiseptic solutions or antibiotic ointments to the perineal area.

A - Assess the patient daily to determine need for catheter.

The nurse is teaching a woman how to prevent UTIs. What information does the nurse include? a. Clean the perineal area from front to back. b. Always use a condom if spermicides are used for contraception. c. Obtain prescription for oral estrogen for vaginal dryness. d. Avoid urinary stasis by urinating every 6 to 8 hours.

A - Clean the perineal area from front to back.

Patients who have central nervous system lesions from stroke, multiple sclerosis, or parasacral spinal cord lesions may have which type of urinary incontinence? a. Detrusor hyperreflexia b. Mixed c. Stress d. Functional

A - Detrusor hyperreflexia

A patient is diagnosed with a urethral stricture. The nurse prepares the patient for which temporary treatment? a. Dilation of the urethra b. Antibiotic therapy c. Fluid restriction d. Urinary diversion

A - Dilation of the urethra

The nurse is counseling a patient with recurrent symptomatic UTIs about dietary therapy. What information does the nurse give to the patient? a. Drink 50 mL of concentrated cranberry juice every day. b. Low consumption of protein may prevent recurrent UTIs. c. Caffeine, carbonated beverages, and tomato products cause UTI. d. Cranberry tablets are more effective than juice or fluids.

A - Drink 50 mL of concentrated cranberry juice every day.

The nurse is teaching self-care measures to a patient who had lithotripsy for kidney stones. What information does the nurse include? (Select all that apply.) a. Finish the entire prescription of antibiotics to prevent UTIs. b. Balance regular exercise with sleep and rest. c. Drink at least 3 L of fluid a day. d. Watch for and immediately report bruising after lithotripsy. e. Urine may be bloody for several days. f. Pain in the region of the kidneys or bladder is expected.

A - Finish the entire prescription of antibiotics to prevent UTIs. B - Balance regular exercise with sleep and rest. C - Drink at least 3 L of fluid a day. E - Urine may be bloody for several days.

A patient's recurrent cystitis appears to be related to sexual intercourse. The patient seems uncomfortable talking about the situation. What communication technique does the nurse use to assist the patient? a. Have a frank and sensitive discussion with the patient. b. Give the patient reading material with instructions to call with any questions. c. Call the patient's partner and invite the partner to discuss the problem. d. Talk about other topics until the patient feels more comfortable disclosing.

A - Have a frank and sensitive discussion with the patient.

A patient reports intense urgency, frequency, and bladder pain. Urinalysis results show white blood cells (WBCs) and red blood cells (RBCs) and urine culture results are negative for infection. How does the nurse interpret these findings? a. Interstitial cystitis b. Urethritis c. Bacteriuria d. Infectious cystitis

A - Interstitial cystitis

What does the nurse include in the care plan for a patient who had pyelolithotomy? (Select all that apply.) a. Monitor the amount of bleeding from incisions. b. Restrict fluids to prevent edema and fluid overload. c. Strain the urine to monitor the passage of stone fragments. d. Encourage fluids to avoid dehydration and supersaturation. e. Monitor changes in urine output. f. Administer antibiotics to eliminate or prevent infections.

A - Monitor the amount of bleeding from incisions. C - Strain the urine to monitor the passage of stone fragments. D - Encourage fluids to avoid dehydration and supersaturation. E - Monitor changes in urine output. F - Administer antibiotics to eliminate or prevent infections.

The home health nurse is assessing an older adult patient who refuses to leave the house to see friends or participate in usual activities. She reports taking a bath several times a day and becomes very upset when she has an incontinent episode. What is the priority problem for this patient? a. Negative self-image b. Stress urinary incontinence c. Social isolation d. Potential for skin breakdown

A - Negative self-image

Which task related to care of patients who have indwelling catheters can be delegated to unlicensed assistive personnel (UAP)? a. Perform daily catheter care by washing the perineum and proximal portion of the catheter with soap and water. b. Use sterile technique when inserting the urinary catheter or when opening the system to obtain urine samples. c. Determine whether use of condom catheters is appropriate for male patients and apply the devices accordingly. d. Keep urine collection bag in a place that is readily visible to the patient, so that the patient is reassured of kidney function.

A - Perform daily catheter care by washing the perineum and proximal portion of the catheter with soap and water.

The health care provider has recommended intermittent self-catheterization for a patient with long-term problems of incomplete bladder emptying. Which information does the nurse give the patient about the procedure? a. Perform proper handwashing and cleaning of the catheter to reduce the risk for infection. b. Use a large-lumen catheter and good lubrication for rapid emptying of the bladder. c. Catheterize yourself whenever the bladder gets distended. d. Use sterile technique, especially if catheterization is done by a family member.

A - Perform proper handwashing and cleaning of the catheter to reduce the risk for infection.

The nurse is reviewing a care plan for a patient who has functional incontinence. There is a note that containment is recommended, especially at night. What is the major concern with this approach? a. Skin integrity b. Cost of care and materials c. Self-esteem of the patient d. Fall risk

A - Skin integrity

A patient returns to the medical-surgical unit after having shock wave lithotripsy (SWL). What is an appropriate nursing intervention for the postprocedural care of this patient? a. Strain the urine to monitor the passage of stone fragments. b. Report bruising that occurs on the flank of the affected side. c. Continuously monitor electrocardiogram (ECG) for dysrhythmias. d. Apply a local anesthetic cream to the skin of the affected side.

A - Strain the urine to monitor the passage of stone fragments.

A middle-aged woman has urinary stress incontinence related to weak pelvic muscles. The patient is highly motivated to participate in self-care. Which interventions does the nurse include in the treatment plan? (Select all that apply.) a. Suggest keeping a detailed diary of urine leakage, activities, and foods eaten. b. Suggest wearing absorbent undergarments during the assessment process. c. Teach pelvic floor (Kegel) exercise therapy. d. Teach about vaginal cone therapy. e. Encourage drinking orange juice every day for 4 to 6 weeks. f. Refer to a nutritionist for diet therapy for weight reduction.

A - Suggest keeping a detailed diary of urine leakage, activities, and foods eaten. B - Suggest wearing absorbent undergarments during the assessment process. C - Teach pelvic floor (Kegel) exercise therapy. D - Teach about vaginal cone therapy. F - Refer to a nutritionist for diet therapy for weight reduction.

The nurse is designing a habit training bladder program for an older adult patient who is alert but mildly confused. What task associated with the training program is delegated to the UAP? a. Tell the patient it is time to go to the toilet and assist him to go on a regular schedule. b. Help the patient record the incidents of incontinence in a bladder diary. c. Change the patient's incontinence pants (or pad) every 4 hours. d. Gradually encourage independence and increase the intervals between voidings.

A - Tell the patient it is time to go to the toilet and assist him to go on a regular schedule.

A patient reports symptoms indicating a UTI. Results from which diagnostic test will verify a UTI? a. Urinalysis to test for leukocyte esterase and nitrate b. Urinalysis for glucose and red blood cells c. Urinalysis to test for ketones and protein d. Urinalysis for pH and specific gravity

A - Urinalysis to test for leukocyte esterase and nitrate

The nurse has just received report on a group of clients. Which client is the nurse's first priority? A 26 year old with type 1 diabetes whose insulin pump is beeping "occlusion." A 30 year old with type 1 diabetes who is reporting thirst. A 40 year old with type 2 diabetes who has a blood glucose of 150 mg/dL (8.3 mmol/L). A 50 year old with type 2 diabetes with a blood pressure of 150/90 mm Hg.

A 26 year old with type 1 diabetes whose insulin pump is beeping "occlusion." The client the nurse sees first is the client with type 1 diabetes whose insulin pump is beeping "occlusion." Because glucose levels will increase quickly in clients whose continuous insulin pumps malfunction, the nurse must assess this client and the insulin pump first to avoid hyperglycemia or diabetic ketoacidosis.Thirst is an expected symptom of hyperglycemia and, although important, is not a priority. The nurse could delegate fingerstick blood glucose to unlicensed assistive personnel while assessing the client whose insulin pump is beeping. Although a blood glucose reading of 150 mg/dL (8.3 mmol/L) is mildly elevated, this does not require immediate action. Mild hypertension does not require immediate action. The nurse can later assess if this is within the client's usual range or represents a change before taking action.

The RN on the medical-surgical unit receives a shift report about four clients. Which client does the nurse assess first? A. A 34-year-old who has returned to the unit after a colon resection with a new colostomy stoma, which is dark pink B. A 36-year-old admitted after a motor vehicle crash with areas of ecchymoses on the abdomen in a "lap-belt" pattern C. A 40-year-old with pneumonia who has abdominal distention and decreased bowel sounds in all quadrants D. A 51-year-old with familial adenomatous polyposis (FAP) who is scheduled for a colonoscopy

A 36-year-old admitted after a motor vehicle crash with areas of ecchymoses on the abdomen in a "lap-belt" pattern Ecchymoses in the abdominal area may indicate intraperitoneal or intra-abdominal bleeding; this client requires rapid assessment and interventions. The client who is post colon resection, the client with pneumonia, and the client with FAP do not have an urgent need for further assessment or intervention.

Which client does the medical-surgical unit charge nurse assign to an LPN/LVN? A. A 41-year-old who needs assistance with choosing a site for a colostomy stoma B. A 47-year-old who needs to receive "whole gut" lavage before a colon resection C. A 51-year-old who has recently arrived on the unit after having an open herniorrhaphy D. A 56-year-old who has obstipation and a recent emesis of foul-smelling liquid

A 47-year-old who needs to receive "whole gut" lavage before a colon resection Because administration of medications is within the LPN/LVN scope of practice, this preoperative client can be assigned to the LPN/LVN. Assistance with choosing a site for a colostomy stoma is an intervention that should be provided by an RN. The recent postoperative client and the critically ill client will need assessments and interventions that can only be done by an RN.

The nurse has just received report on a group of clients. Which client is the nurse's first priority? A 50 year old taking repaglinide who has nausea and back pain. A 55 year old taking pioglitazone who has bilateral ankle swelling. A 45 year old taking metformin who has abdominal cramps. A 40 year old taking glyburide who is dizzy and sweaty.

A 45 year old taking metformin who has abdominal cramps. The nurse needs to first assess the client taking glyburide who is dizzy and sweaty and has symptoms consistent with hypoglycemia. Because hypoglycemia is the most serious adverse effect of antidiabetic medications, this client must be assessed as soon as possible.Nausea is a documented side effect of repaglinide. Checking the client's back pain requires assessment, which can be performed after the nurse assesses the client displaying signs and symptoms of hypoglycemia. Metformin may cause abdominal cramping and diarrhea, but the client taking it does not require immediate assessment. Ankle swelling is an expected side effect of pioglitazone.

Which client does the nurse caution to avoid self-monitoring of blood glucose (SMBG) at alternate sites? A 55-year-old client who has hypoglycemic unawareness An 80-year-old client with type 2 diabetes mellitus A 45-year-old client with type 1 diabetes mellitus A 75-year-old client whose blood glucose levels show little variation

A 75-year-old client whose blood glucose levels show little variation Comparison studies have shown wide variation between fingertip and alternate sites, and variation is most evident during times when blood glucose levels are rapidly changing. Clients are taught that there is a lag time for blood glucose levels between the fingertip and other sites when blood glucose levels are changing rapidly and that the fingertip reading is the only safe choice at those times. Because of this lag time, clients who have hypoglycemic unawareness are warned to not ever use alternate sites for SMBG.

For which clients scheduled for a computed tomography (CT) scan with contrast does the nurse communicate safety concerns to the health care provider? (Select all that apply.) Client with an allergy to shrimp Client with a history of asthma Client who requests morphine sulfate every 3 hours Client with a blood urea nitrogen of 62 mg/dL and a creatinine of 2.0 mg/dL Client who took metformin (Glucophage) 4 hours ago

A B D E The client who will be undergoing a CT scan with contrast should be asked about known hay fever or food or drug allergies, especially to seafood, eggs, milk, or chocolate. Contrast reactions have been reported to be as high as 15% in these clients. Clients with asthma have been shown to be at greater risk for contrast reactions than the general public. When reactions do occur, they are more likely to be severe. The risk for contrast-induced nephropathy is increased in clients who have pre-existing renal insufficiency (e.g., serum creatinine levels greater than 1.5 mg/dL or estimated glomerular filtration rate less than 45 mL/min). Metformin must be discontinued at least 24 hours before and for at least 48 hours after any study using contrast media because the life-threatening complication of lactic acidosis, although rare, could occur. There are no contraindications to undergo CT with contrast while taking morphine sulfate. CT with contrast may help to identify the underlying cause of pain.

A nurse is assessing clients on a rehabilitation unit. Which clients are at greatest risk for asphyxiation related to inspissated oral and nasopharyngeal secretions? (Select all that apply.) a. A 24-year-old with a traumatic brain injury b. A 36-year-old who fractured his left femur c. A 58-year-old at risk for aspiration following radiation therapy d. A 66-year-old who is a quadriplegic and has a sacral ulcer e. An 80-year-old who is aphasic after a cerebral vascular accident

A, C, D, E

The nurse on a medical surgical unit is planning bed assignments for a new admission who has cystic fibrosis (CF) and is infected with Burkholderia cepacia. Which of these room assignments is most appropriate for this client? A room with laminar air flow A room with a client who has Down syndrome and pneumonia A room with another client who has cystic fibrosis A private room with a bathroom

A private room with a bathroom The most appropriate room for this client is a private room and separate bathroom. This provides maximum protection from organisms which can easily cause infection in the client with CF. A serious bacterial infection for clients with CF is Burkholderia cepacia, which is spread by casual contact from one CF client to another. To reduce spread of infection, measures include separating infected CF clients from noninfected CF clients on hospital units and seeing them in the clinic on different days.Laminar air flow is used in operating rooms and other areas where removing circulating air will provide for infection prevention. This is not required for those with CF. A client with Down syndrome may be unable to be careful with covering the mouth when coughing, using tissues, and handwashing, and would not be cohorted with a client who has high risk for infection.

The nurse is teaching a client with chronic kidney disease (CKD) about the sodium restriction needed in the diet to prevent edema and hypertension. Which statement by the client indicates more teaching is needed? a. I am thrilled that I can continue to eat fast food. b. I will cut out bacon with my eggs every morning. c. My cooking style will change by not adding salt. d. I will probably lose weight by cutting out potato chips.

A ~ Fast food restaurants usually serve food that is high in sodium. This statement indicates that more teaching needs to occur. The other statements show a correct understanding of the teaching.

A client is taking furosemide (Lasix) 40 mg/day for management of chronic kidney disease (CKD). To detect the positive effect of the medication, what action of the nurse is best? a. Obtain daily weights of the client. b. Auscultate heart and breath sounds. c. Palpate the client's abdomen. d. Assess the client's diet history.

A ~ Furosemide (Lasix) is a loop diuretic that helps reduce fluid overload and hypertension in clients with early stages of CKD. One kilogram of weight equals about 1 liter of fluid retained in the client, so daily weights are necessary to monitor the response of the client to the medication. Heart and breath sounds should be assessed if there is fluid retention, as in heart failure. Palpation of the client's abdomen is not necessary, but the nurse should check for edema. The diet history of the client would be helpful to assess electrolyte replacement since potassium is lost with this diuretic, but this does not assess the effect of the medication.

A client in the intensive care unit is started on continuous venovenous hemofiltration (CVVH). Which finding is the cause of immediate action by the nurse? a. Blood pressure of 76/58 mm Hg b. Sodium level of 138 mEq/L c. Potassium level of 5.5 mEq/L d. Pulse rate of 90 beats/min

A ~ Hypotension can be a problem with CVVH if replacement fluid does not provide enough volume to maintain blood pressure. The specially trained nurse needs to monitor for ongoing fluid and electrolyte replacement. The sodium level is normal and the potassium level is slightly elevated, which could be normal findings for someone with acute kidney injury. A pulse rate of 90 beats/min is normal.

A 70-kg adult with chronic renal failure is on a 40-g protein diet. The client has a reduced glomerular filtration rate and is not undergoing dialysis. Which result would give the nurse the most concern? a. Albumin level of 2.5 g/dL b. Phosphorus level of 5 mg/dL c. Sodium level of 135 mmol/L d. Potassium level of 5.5 mmol/L

A ~ Protein restriction is necessary with chronic renal failure due to the buildup of waste products from protein breakdown. The nurse would be concerned with the low albumin level (normal = 3.5-5.5) since this indicates that the protein in the diet is not enough for the client's metabolic needs. The electrolyte values are not related to the protein-restricted diet.

A male client with chronic kidney disease (CKD) is refusing to take his medication and has missed two hemodialysis appointments. What is the best initial action for the nurse? a. Discuss what the treatment regimen means to him. b. Refer the client to a mental health nurse practitioner. c. Reschedule the appointments to another date and time. d. Discuss the option of peritoneal dialysis.

A ~ The initial action for the nurse is to assess anxiety, coping styles, and the client's acceptance of the required treatment for CKD. The client may be in denial of the diagnosis. While rescheduling hemodialysis appointments may help, and referral to a mental health practitioner and the possibility of peritoneal dialysis are all viable options, assessment of the client's acceptance of the treatment should come first.

A client has a serum potassium level of 6.5 mmol/L, a serum creatinine level of 2 mg/dL, and a urine output of 350 mL/day. What is the best action by the nurse? a. Place the client on a cardiac monitor immediately. b. Teach the client to limit high-potassium foods. c. Continue to monitor the client's intake and output. d. Ask to have the laboratory redraw the blood specimen.

A ~ The priority action by the nurse should be to check the cardiac status with a monitor. High potassium levels can lead to dysrhythmias. The other choices are logical nursing interventions for acute kidney injury but not the best immediate action.

A male client comes into the emergency department with a serum creatinine of 2.2 mg/dL and a blood urea nitrogen (BUN) of 24 mL/dL. What question should the nurse ask first when taking this client's history? a. Have you been taking any aspirin, ibuprofen, or naproxen recently? b. Do you have anyone in your family with renal failure? c. Have you had a diet that is low in protein recently? d. Has a relative had a kidney transplant lately?

A ~ There are some medications that are nephrotoxic, such as the nonsteroidal anti-inflammatory drugs ibuprofen, aspirin, and naproxen. This would be a good question to initially ask the client since both the serum creatinine and BUN are elevated, indicating some renal problems. A family history of renal failure and kidney transplantation would not be part of the questioning and could cause anxiety in the client. A diet high in protein could be a factor in an increased BUN.

A client with chronic kidney disease (CKD) is experiencing nausea, vomiting, visual changes, and anorexia. Which action by the nurse is best? a. Check the client's digoxin (Lanoxin) level. b. Administer an anti-nausea medication. c. Ask if the client is able to eat crackers. d. Get a referral to a gastrointestinal provider.

A ~ These signs and symptoms are indications of digoxin (Lanoxin) toxicity. The nurse should check the level of this medication. Administering antiemetics, asking if the client can eat, and obtaining a referral to a specialist all address the client's symptoms but do not lead to the cause of the symptoms.

A marathon runner comes into the clinic and states "I have not urinated very much in the last few days." The nurse notes a heart rate of 110 beats/min and a blood pressure of 86/58 mm Hg. Which action by the nurse is the priority? a. Give the client a bottle of water immediately. b. Start an intravenous line for fluids. c. Teach the client to drink 2 to 3 liters of water daily. d. Perform an electrocardiogram.

A ~ This athlete is mildly dehydrated as evidenced by the higher heart rate and lower blood pressure. The nurse can start hydrating the client with a bottle of water first, followed by teaching the client to drink 2 to 3 liters of water each day. An intravenous line may be ordered later, after the client's degree of dehydration is assessed. An electrocardiogram is not necessary at this time.

What is the rationale for using CPAP to treat sleep apnea? A) positive air pressure holds the airway open B) negative air pressure holds the airway closed C) delivery of oxygen facilitates respiratory effort D) alternating waves of air stimulate breathing

A) positive air pressure holds the airway open

A client admitted for difficulty breathing becomes worse. Which assessment findings indicate that the client has developed acute respiratory distress syndrome (ARDS)? (Select all that apply.) a.Oxygen administered at 100%, PaO2 60 b.Increased dyspnea c.Anxiety d.Chest pain e.Pitting pedal edema f.Clubbing of fingertips A client who is developing ARDS presents with a decrease in oxygen despite an increase in the fraction of inspired oxygen. Increased dyspnea goes along with the increased hypoxemia, as does anxiety. Chest pain is not specific to ARDS; although chest pain can occur with ARDS, it occurs with many other conditions as well. Pitting edema would not be an assessment factor that confirms ARDS. Clubbing occurs in chronic, not acute, respiratory conditions.

A, B, C

The nurse caring for mechanically ventilated clients uses best practices to prevent ventilator-associated pneumonia. What actions are included in this practice? (Select all that apply.) a. Adherence to proper hand hygiene b. Administering anti-ulcer medication c. Elevating the head of the bed d. Providing oral care per protocol e. Suctioning the client on a regular schedule

A, B, C, D The "ventilator bundle" is a group of care measures to prevent ventilator-associated pneumonia. Actions in the bundle include using proper hand hygiene, giving anti-ulcer medications, elevating the head of the bed, providing frequent oral care per policy, preventing aspiration, and providing pulmonary hygiene measures. Suctioning is done as needed.

7. A nurse teaches a client about self-care after experiencing a urinary calculus treated by lithotripsy. Which statements should the nurse include in this clients discharge teaching? (Select all that apply.) a. Finish the prescribed antibiotic even if you are feeling better. b. Drink at least 3 liters of fluid each day. c. The bruising on your back may take several weeks to resolve. d. Report any blood present in your urine. e. It is normal to experience pain and difficulty urinating.

A, B, C The client should be taught to finish the prescribed antibiotic to ensure that he or she does not get a urinary tract infection. The client should drink at least 3 liters of fluid daily to dilute potential stone-forming crystals, prevent dehydration, and promote urine flow. After lithotripsy, the client should expect bruising that may take several weeks to resolve. The client should also experience blood in the urine for several days. The client should report any pain, fever, chills, or difficulty with urination to the provider as these may signal the beginning of an infection or the formation of another stone.

The nurse is caring for five clients on the medical-surgical unit. Which clients would the nurse consider to be at risk for post-renal acute kidney injury (AKI)? (SATA) a. Man with prostate cancer b. Woman with blood clots in the urinary tract c. Client with ureterolithiasis d. Firefighter with severe burns e. Young woman with lupus

A, B, C ~ Urine flow obstruction, such as prostate cancer, blood clots in the urinary tract, and kidney stones (ureterolithiasis), causes post-renal AKI. Severe burns would be a pre-renal cause. Lupus would be an intrarenal cause for AKI.

A nurse assesses a client who is 6 hours post-surgery for a nasal fracture and has nasal packing in place. Which actions should the nurse take? (Select all that apply.) a. Observe for clear drainage. b. Assess for signs of bleeding. c. Watch the client for frequent swallowing. d. Ask the client to open his or her mouth. e. Administer a nasal steroid to decrease edema. f. Change the nasal packing

A, B, C, D

A nurse teaches a client who is being discharged after a fixed centric occlusion for a mandibular fracture. Which statements should the nurse include in this clients teaching? (Select all that apply.) a. You will need to cut the wires if you start vomiting. b. Eat six soft or liquid meals each day while recovering. c. Irrigate your mouth every 2 hours to prevent infection. d. Sleep in a semi-Fowlers position after the surgery. e. Gargle with mouthwash that contains Benadryl once a day.

A, B, C, D

The nurse caring for mechanically ventilated clients uses best practices to prevent ventilator-associated pneumonia. What actions are included in this practice? (Select all that apply.) a. Adherence to proper hand hygiene b. Administering anti-ulcer medication c. Elevating the head of the bed d. Providing oral care per protocol e. Suctioning the client on a regular schedule The "ventilator bundle" is a group of care measures to prevent ventilator-associated pneumonia. Actions in the bundle include using proper hand hygiene, giving anti-ulcer medications, elevating the head of the bed, providing frequent oral care per policy, preventing aspiration, and providing pulmonary hygiene measures. Suctioning is done as needed.

A, B, C, D

A client with a new pulmonary embolism (PE) is anxious. What nursing actions are most appropriate? (Select all that apply.) a. Acknowledge the frightening nature of the illness. b. Delegate a back rub to the unlicensed assistive personnel (UAP). c. Give simple explanations of what is happening. d. Request a prescription for antianxiety medication. e. Stay with the client and speak in a quiet, calm voice.

A, B, C, E Clients with PEs are often anxious. The nurse can acknowledge the client's fears, delegate comfort measures, give simple explanations the client will understand, and stay with the client. Using a calm, quiet voice is also reassuring. Sedatives and antianxiety medications are not used routinely because they can contribute to hypoxia. If the client's anxiety is interfering with diagnostic testing or treatment, they can be used, but there is no evidence that this is the case.

The nurse caring for mechanically ventilated clients knows that older adults are at higher risk for weaning failure. What age-related changes contribute to this? (Select all that apply.) a. Chest wall stiffness b. Decreased muscle strength c. Inability to cooperate d. Less lung elasticity e. Poor vision and hearing Age-related changes that increase the difficulty of weaning older adults from mechanical ventilation include increased stiffness of the chest wall, decreased muscle strength, and less elasticity of lung tissue. Not all older adults have an inability to cooperate or poor sensory acuity.

A, B, D

The nurse caring for mechanically ventilated clients knows that older adults are at higher risk for weaning failure. What age-related changes contribute to this? (Select all that apply.) a. Chest wall stiffness b. Decreased muscle strength c. Inability to cooperate d. Less lung elasticity e. Poor vision and hearing

A, B, D Age-related changes that increase the difficulty of weaning older adults from mechanical ventilation include increased stiffness of the chest wall, decreased muscle strength, and less elasticity of lung tissue. Not all older adults have an inability to cooperate or poor sensory acuity.

1. A nurse assesses a client who has had two episodes of bacterial cystitis in the last 6 months. Which questions should the nurse ask? (Select all that apply.) a. How much water do you drink every day? b. Do you take estrogen replacement therapy? c. Does anyone in your family have a history of cystitis? d. Are you on steroids or other immune-suppressing drugs? e. Do you drink grapefruit juice or orange juice daily?

A, B, D Fluid intake, estrogen levels, and immune suppression all can increase the chance of recurrent cystitis. Family history is usually insignificant, and cranberry juice, not grapefruit or orange juice, has been found to increase the acidic pH and reduce the risk for bacterial cystitis.

5. A nurse cares for clients with urinary incontinence. Which types of incontinence are correctly paired with their clinical manifestation? (Select all that apply.) a. Stress incontinence Urine loss with physical exertion b. Urge incontinence Large amount of urine with each occurrence c. Functional incontinence Urine loss results from abnormal detrusor contractions d. Overflow incontinence Constant dribbling of urine e. Reflex incontinence Leakage of urine without lower urinary tract disorder

A, B, D Stress incontinence is a loss of urine with physical exertion, coughing, sneezing, or exercising. Urge incontinence presents with an abrupt and strong urge to void and usually has a large amount of urine released with each occurrence. Overflow incontinence occurs with bladder distention and results in a constant dribbling of urine. Functional incontinence is the leakage of urine caused by factors other than a disorder of the lower urinary tract. Reflex incontinence results from abnormal detrusor contractions from a neurologic abnormality.

A client is undergoing hemodialysis. The client's blood pressure at the beginning of the procedure was 136/88 mm Hg, and now it is 110/54 mm Hg. What actions should the nurse perform to maintain blood pressure? (SATA) a. Adjust the rate of extracorporeal blood flow. b. Place the client in the Trendelenburg position. c. Stop the hemodialysis treatment. d. Administer a 250-mL bolus of normal saline. e. Contact the health care provider for orders.

A, B, D ~ Hypotension occurs often during hemodialysis treatments as a result of vasodilation from the warmed dialysate. Modest decreases in blood pressure, as is the case with this client, can be maintained with rate adjustment, Trendelenburg positioning, and a fluid bolus. If the blood pressure drops considerably after two boluses and cooling dialysate, the hemodialysis can be stopped and the health care provider contacted.

A client is unsure of the decision to undergo peritoneal dialysis (PD) and wishes to discuss the advantages of this treatment with the nurse. Which statements by the nurse are accurate regarding PD? (SATA) a. You will not need vascular access to perform PD. b. There is less restriction of protein and fluids. c. You will have no risk for infection with PD. d. You have flexible scheduling for the exchanges. e. It takes less time than hemodialysis treatments.

A, B, D ~ PD is based on exchanges of waste, fluid, and electrolytes in the peritoneal cavity. There is no need for vascular access. Protein is lost in the exchange, which allows for more protein and fluid in the diet. There is flexibility in the time for exchanges, but the treatment takes a longer period of time compared to hemodialysis. There still is risk for infection with PD, especially peritonitis.

A nurse is caring for a client who is on mechanical ventilation. What actions will promote comfort in this client? (Select all that apply.) a. Allow visitors at the client's bedside. b. Ensure the client can communicate if awake. c. Keep the television tuned to a favorite channel. d. Provide back and hand massages when turning. e. Turn the client every 2 hours or more. There are many basic care measures that can be employed for the client who is on a ventilator. Allowing visitation, providing a means of communication, massaging the client's skin, and routinely turning and repositioning the client are some of them. Keeping the TV on will interfere with sleep and rest.

A, B, D, E

A nurse is caring for a client who is on mechanical ventilation. What actions will promote comfort in this client? (Select all that apply.) a. Allow visitors at the client's bedside. b. Ensure the client can communicate if awake. c. Keep the television tuned to a favorite channel. d. Provide back and hand massages when turning. e. Turn the client every 2 hours or more.

A, B, D, E There are many basic care measures that can be employed for the client who is on a ventilator. Allowing visitation, providing a means of communication, massaging the client's skin, and routinely turning and repositioning the client are some of them. Keeping the TV on will interfere with sleep and rest.

Select the results (in italics) that are normal in a urinalysis. A. pH 6 B. Specific gravity 1.015 C. Protein small D. Sugar negative E. Nitrate small F. Leukocyte esterase positive G. Bilirubin negative

A, B, D, G Rationale: The abnormal values are indicative of a urinary tract infection. As a result of protein, nitrates, and leukoesterase in the urine, the nurse can expect the laboratory to analyze microscopic sediment including evaluating the sample for the presence of crystals, casts, WBCs, and RBCs.

6. A nurse teaches a client with a history of calcium phosphate urinary stones. Which statements should the nurse include in this clients dietary teaching? (Select all that apply.) a. Limit your intake of food high in animal protein. b. Read food labels to help minimize your sodium intake. c. Avoid spinach, black tea, and rhubarb. d. Drink white wine or beer instead of red wine. e. Reduce your intake of milk and other dairy products.

A, B, E Clients with calcium phosphate urinary stones should be taught to limit the intake of foods high in animal protein, sodium, and calcium. Clients with calcium oxalate stones should avoid spinach, black tea, and rhubarb. Clients with uric acid stones should avoid red wine.

8. A nurse teaches a female client who has stress incontinence. Which statements should the nurse include about pelvic muscle exercises? (Select all that apply.) a. When you start and stop your urine stream, you are using your pelvic muscles. b. Tighten your pelvic muscles for a slow count of 10 and then relax for a slow count of 10. c. Pelvic muscle exercises should only be performed sitting upright with your feet on the floor. d. After you have been doing these exercises for a couple days, your control of urine will improve. e. Like any other muscle in your body, you can make your pelvic muscles stronger by contracting them.

A, B, E The client should be taught that the muscles used to start and stop urination are pelvic muscles, and that pelvic muscles can be strengthened by contracting and relaxing them. The client should tighten pelvic muscles for a slow count of 10 and then relax the muscles for a slow count of 10, and perform this exercise 15 times while in lying-down, sitting-up, and standing positions. The client should begin to notice improvement in control of urine after several weeks of exercising the pelvic muscles.

A client is hospitalized in the oliguric phase of acute kidney injury (AKI) and is receiving tube feedings. The nurse is teaching the client's spouse about the kidney-specific formulation for the enteral solution compared to standard formulas. What components should be discussed in the teaching plan? (SATA) a. Lower sodium b. Higher calcium c. Lower potassium d. Higher phosphorus e. Higher calories

A, C, E ~ Many clients with AKI are too ill to meet caloric goals and require tube feedings with kidney-specific formulas that are lower in sodium, potassium, and phosphorus, and higher in calories than are standard formulas.

A nurse is caring for a postoperative 70-kg client who had major blood loss during surgery. Which findings by the nurse should prompt immediate action to prevent acute kidney injury? (SATA) a. Urine output of 100 mL in 4 hours b. Urine output of 500 mL in 12 hours c. Large amount of sediment in the urine d. Amber, odorless urine e. Blood pressure of 90/60 mm Hg

A, C, E ~ The low urine output, sediment, and blood pressure should be reported to the provider. Postoperatively, the nurse should measure intake and output, check the characteristics of the urine, and report sediment, hematuria, and urine output of less than 0.5 mL/kg/hour for 3 to 4 hours. A urine output of 100 mL is low, but a urine output of 500 mL in 12 hours should be within normal limits. Perfusion to the kidneys is compromised with low blood pressure. The amber odorless urine is normal.

The nurse is caring for a client on a ventilator when the high-pressure alarm sounds. What actions are most appropriate? (Select all that apply.) a.Assess the tubing for kinks. b.Assess whether the tubing has become disconnected. c.Determine the need for suctioning. d.Call the health care provider. e.Call the Rapid Response Team. f.Auscultate the client's lungs. Reasons for a high-pressure alarm include water or a kink impeding airflow or mucus in the airway. The nurse first should assess the client and determine whether he or she needs to be suctioned; then the nurse should auscultate the lungs. The nurse also should assess the tubing for kinks. The high-pressure alarm sounding would not be a reason to call the health care provider or the Rapid Response Team. If the tubing became disconnected, the low-pressure alarm would sound.

A, C, F

2. A nurse teaches a client about self-catheterization in the home setting. Which statements should the nurse include in this clients teaching? (Select all that apply.) a. Wash your hands before and after self-catheterization. b. Use a large-lumen catheter for each catheterization. c. Use lubricant on the tip of the catheter before insertion. d. Self-catheterize at least twice a day or every 12 hours. e. Use sterile gloves and sterile technique for the procedure. f. Maintain a specific schedule for catheterization.

A, C, F The key points in self-catheterization include washing hands, using lubricants, and maintaining a regular schedule to avoid distention and retention of urine that leads to bacterial growth. A smaller rather than a larger lumen catheter is preferred. The client needs to catheterize more often than every 12 hours. Self-catheterization in the home is a clean procedure.

A nurse assesses a client who has facial trauma. Which assessment findings require immediate intervention? (Select all that apply.) a. Stridor b. Nasal stuffiness c. Edema of the cheek d. Ecchymosis behind the ear e. Eye pain f. Swollen chin

A, D

A patient with sleep apnea is given a noninvasive method of monitoring rest and activity cycles. The patient is required to wear a small watch device on the wrist. What is the name of this method? A Actigraphy IncorrectB Electromyogram C Polysomnography D Electrooculogram

A. Actigraphy is a noninvasive method of monitoring rest and activity cycles. In this method, a small actigraph watch is worn on the wrist to measure gross motor activity. An electromyogram records muscle tone. Polysomnography is also used to measure sleep apnea but through electrodes that record the main stages of sleep and wakefulness. Eye movements are recorded with an electrooculogram.

The spouse of a patient tells the nurse that, during sleep, the patient's respiration ceases for 10 seconds. This happens repeatedly during the night. As a result, the patient feels sleepy throughout the day. What is this condition known as? A Apnea B Insomnia C Hypopnea D Hypercapnia

A. Apnea is a medical condition in which there is a cessation of spontaneous respirations for 10 seconds or more. It can prevent the patient from sleeping soundly and cause daytime sleepiness. Insomnia is a lack of sleep. Hypopnea is a condition characterized by shallow respirations. Hypercapnia is a condition characterized by an increased concentration of carbon dioxide in the body.

Which assessment question is most appropriate when the nurse is assessing a patient who is receiving care for suspected obstructive sleep apnea (OSA)? A. "Do you smoke?" B "Do you tend to awaken early in the morning?" C "Are you under a lot of stress at work or at home right now?" D "Do you have a history of chronic obstructive pulmonary disease (COPD)?"

A. Smoking is a major etiologic factor in OSA. Early wakening and stress are associated with insomnia, not OSA in particular. COPD exacerbates the hypoxemia associated with OSA, but does not precipitate the onset of OSA itself.

Which statement by a client with chronic obstructive pulmonary disease (COPD) and a 10 pound (4.5 kg) weight loss indicates the need for additional follow-up instruction? A. "I should consume plenty of fluids with my meal." B. "I will try eating smaller more frequent meals." C. "I will try to eat more protein." D. "I will perform mouth care prior to eating."

A. "I should consume plenty of fluids with my meal."

The nurse is providing preoperative teaching for the client with lung cancer for whom a lobectomy is planned. Which of these does the nurse include in the preoperative education session? (Select all that apply) A. "You will wake up with a drain in your chest which removes blood and allows the remaining lung to expand." B. "You will be able to get out of bed after the chest tube is removed." C. "Plan to request pain medication before your pain becomes severe." D. "You may have a tube in your throat connected to a mechanical ventilator to assist you with breathing." E. "You will need to lie on the operative side until the area of tissue removal heals."

A. "You will wake up with a drain in your chest which removes blood and allows the remaining lung to expand" C. "Plan to request pain medication before your pain becomes severe D. "You may have a tube in your throat connected to a mechanical ventilator to assist you with breathing

What information will the nurse provide to a client who is scheduled for extracorporeal shock wave lithotripsy? Select all that apply. A. "Your urine will be strained after the procedure." B. "Be sure to finish all of your antibiotics." C. "Immediately call the health care provider if you notice bruising." D. "Remember to drink at least 3 liters of fluid a day to promote urine flow." E. "You will need to change the incisional dressing once a day."

A. "Your urine will be strained after the procedure." B. "Be sure to finish all of your antibiotics." D. "Remember to drink at least 3 liters of fluid a day to promote urine flow." After lithotripsy, urine is strained to monitor the passage of stone fragments. Clients must finish the entire antibiotic prescription to decrease the risk of developing a urinary tract infection. Drinking at least 3 L of fluid a day dilutes potential stone-forming crystals, prevents dehydration, and promotes urine flow. Bruising on the flank of the affected side is expected after lithotripsy as a result of the shock waves that break the stone into small fragments. The client must notify the health care provider if he or she develops pain, fever, chills, or difficulty with urination because these signs and symptoms may signal the beginning of an infection or the formation of another stone. There is no incision with extracorporeal shock wave lithotripsy. There may be a small incision when intracorporeal lithotripsy is performed.

The nurse is caring for a client who has had a lobectomy and placement of a chest tube 8 hours ago. When performing an initial assessment, which of these requires immediate follow up? A. 200 mL of red drainage from chest tube over 2 hours B. Client sleepy but able to be aroused C. 3 cm area of red drainage on the incisional dressing D. Report of pain at the chest tube insertion site

A. 200 mL of red drainage from chest tube over 2 hours

The nurse is teaching a group of clients with irritable bowel syndrome (IBS) about complementary and alternative therapies. What does the nurse suggest as possible treatment modalities? (Select all that apply.) A. Acupuncture B. Decreasing physical activities C. Herbs (moxibustion) D. Meditation E. Peppermint oil capsules F. Yoga

A. Acupuncture C. Herbs (moxibustion) D. Meditation E. Peppermint oil capsules F. Yoga Acupuncture is recommended as a complementary therapy for IBS. Moxibustion is helpful for some clients with IBS. Meditation, yoga, and other relaxation techniques help many clients manage stress and their IBS symptoms. Research has shown that peppermint oil capsules may be effective in reducing symptoms of IBS. Regular exercise is important for managing stress and promoting bowel elimination.

A client with acute exacerbation of asthma has been admitted to the medical surgical unit for treatment. The client is reporting increased shortness of breath with inspiratory and expiratory wheezes. When planning care for this client, which medication will the nurse administer first? A. Albuterol-2 inhalations B. Fluticasone-2 inhalations C. Ipratropium-2 inhalations D. Salmeterol-2 inhalations

A. Albuterol-2 inhalations

When caring for the client returning from thoracotomy and placement of a chest tube, the client reports severe pain. What does the nurse do first? A. Assess location and quality of pain. B. Call for the Rapid Response Team (RRT). C. Check the patency of the chest tubes. D. Call the health care provider.

A. Assess location and quality of pain

Which interventions are important for the nurse to teach a client with severe chronic obstructive pulmonary disease (COPD) to help ensure adequate nutrition? (Select all that apply) a. Avoid eating gas-producing foods b. Cough to clear mucus right before eating c. Drink plenty of fluid with every meal d. Eat smaller meals more frequently e. Rest immediately following a meal f. Eat more raw fruits and vegetables g. Use your bronchodilator about 30 minutes before each meal

A. Avoid eating gas-producing foods B. Cough to clear mucus right before eating D. Eat smaller meals more frequently G. Use your bronchodilator about 30 minutes before each meal

The nurse is teaching a client with a newly created colostomy about foods to limit or avoid because of flatulence or odors. Which foods are included? (Select all that apply.) A. Broccoli B. Buttermilk C. Mushrooms D. Onions E. Peas F. Yogurt

A. Broccoli C. Mushrooms D. Onions E. Peas Broccoli, mushrooms, onions, and peas often cause flatus. Buttermilk will help prevent odors. Yogurt can help prevent flatus.

A client has undergone the Whipple procedure (radical pancreaticoduodenectomy) for pancreatic cancer. Which precautionary measures does the nurse implement to prevent potential complications? (Select all that apply.) A. Check blood glucose often. B. Check bowel sounds and stools. C. Ensure that drainage color is clear. D. Monitor mental status. E. Place the client in the supine position.

A. Check blood glucose often. B. Check bowel sounds and stools. D. Monitor mental status. Glucose should be checked often to monitor for diabetes mellitus. Bowels sounds and stools should be checked to monitor for bowel obstruction. A change in mental status or level of consciousness could be indicative of hemorrhage. Clear, colorless, bile-tinged drainage or frank blood with increased output may indicate disruption or leakage of a site of anastomosis. The client should be placed in semi-Fowler's position to reduce tension on the suture line and the anastomosis site and to optimize lung expansion.

The charge nurse is making assignments for clients cared for on the intensive care stepdown unit. Which client will the charge nurse assign to the RN who has floated from the pediatric unit? A. Client with acute asthma episode who is receiving oxygen at FiO2 of 60% by non-rebreather mask B. Client with chronic pleural effusions who is scheduled for a paracentesis in the next hour C. Client with emphysema who requires instruction about correct use of oxygen at home D. Client with lung cancer who has just been transferred from the intensive care unit after a left lower lobectomy yesterday

A. Client with acute asthma episode who is receiving oxygen at FiO2 of 60% by non-rebreather mask

Which action best exemplifies the expected outcome of appropriate negative feedback control over endocrine gland hormone secretion? A. Decreased secretion of glucagon when blood glucose approaches normal levels B. Increased secretion of parathyroid hormone in response to a calcium-containing intravenous infusion C. Increased secretion of thyroid-stimulating hormone in response to long-term exogenous thyroid hormone replacement therapy D. Decreased secretion of cortisol in response to a pituitary tumor stimulating the increased secretion of adrenocorticotropic hormone

A. Decreased secretion of glucagon when blood glucose approaches normal levels

A college student who attempted suicide by overdose is hospitalized. When the parents are contacted, they respond: "there must be a mistake. This could not have happened. We have given our child everything." The parents reaction reflects: A. Denial B. Anger C. Anxiety D. Rescue feelings

A. Denial: The parents statements indicate denial. Denial or minimization of suicidal ideation or attempts is a defense against uncomfortable feelings. Family members are often unable to acknowledge suicidal ideation in someone close to them. The feelings suggested in the distractors are not clearly described in the scenario.

When caring for a client who had a lobectomy the nurse notes small bubbles in the water seal chamber of the disposable chest drainage device during coughing. Which of these reflects the appropriate action by the nurse? A. Document the finding in the medical record. B. Check the tube for blood clots. C. Briefly increase the amount of suction. D. Add additional sterile water to the water seal chamber.

A. Document the finding in the medical record

A client with CF who is 2 months post-operative from a bilateral lung transplant wants to begin riding his bicycle again, as his pulmonary specialist has said he can do, but his wife is concerned that this will "wear out" his new lungs faster. How will the nurse advise this couple? a. Remind the wife that activity does not damage or "wear out" the lungs and that exercise will reduce the risk for other health complications. b. Tell the wife that because the client has a reduced life expectancy, she should allow him to do whatever he wants. c. Remind the client that this is the "honeymoon phase" of recovery and that he will not feel well for very long. d. Advise the client to protect his lungs at all cost.

A. Remind the wife that activity does not damage or "wear out" the lungs and that exercise will reduce the risk for other health complications

A client has just been admitted to the intensive care unit after having a left lower lobectomy via video-assisted thoracoscopic surgery. Which of these prescriptions will the nurse implement first? A. Titrate oxygen flow rate to keep O2 saturation at or greater than 93%. B. Administer 2 g of cephazolin IV now. C. Give morphine sulfate 4 to 6 mg IV for pain. D. Transfuse 1 unit of packed red blood cells (PRBCs) over 2 hours.

A. Titrate oxygen flow rate to keep O2 saturation at or greater than 93%

A client with COPD calls the pulmonary clinic reporting the last 24 hours the peak flow meter readings have been in the yellow range. Which of these interventions by the nurse is appropriate at this time? A. Use your prescription for rescue medication and retest yourself. B. This is a satisfactory reading, continue your present regimen. C. Go to the nearest emergency department. D. Increase your controller medication dose.

A. Use your prescription for rescue medication and retest yourself

An emergency department nurse assesses a client with ketoacidosis. Which clinical manifestation should the nurse correlate with this condition? A. Increased rate and depth of respiration B. Extremity tremors followed by seizure activity C. Oral temperature of 102F (38.9C) D. Severe orthostatic hypotension

Answer: A

A patient has obstructive sleep apnea (OSA). What should the nurse include in the teaching related to ways the patient can minimize the negative effects of OSA? Select all that apply. A Raise the head end of the bed slightly. B Use an oral appliance that prevents the airway from collapsing. C Sleep in a side-lying position. D Avoid consuming alcoholic beverages several hours before going to sleep. E Take sleeping pills before going to bed.

A.B.C.D Raising the head end of the bed keeps the airway open and prevents blockage to the airway. An oral appliance serves as special mouth guard to prevent the airway from collapsing. Sleeping on the side may help the patient to breathe easily. Alcoholic beverages may worsen OSA by decreasing the muscle tone and causing airway collapse. Use of sedatives will exacerbate OSA due to respiratory depression.

A patient has undergone an uvulopalatopharyngoplasty. Which instructions should the nurse give this patient at the time of discharge? Select all that apply. A "The throat may feel sore. It is normal." B "Snoring may persist until inflammation subsides." C "If you feel soreness in the throat, it is a sign of complication." D "If snoring doesn't stop immediately, you may need another surgery." E "There may be foul breath. You can reduce it by using a diluted mouthwash."

A.B.E The nurse has to inform the patient that a sore throat after surgery is normal. Snoring may also be present until the inflammation after surgery has subsided. The patient may have a foul breath odor that may be reduced by rinsing with diluted mouthwash and then salt water. Snoring and sore throat are commonly seen after surgery.

A patient's partner informs the nurse that the patient wakes up with a startle and gasps for breath several times at night. The nurse understands the patient is experiencing sleep apnea. What are the common risk factors in this patient for sleep apnea? Select all that apply. A Body mass index (BMI) 30 kg/m2 B Age 44 years C Habit of smoking D.Neck circumference 18 inches E .Occasional consumption of alcohol

A.C.D. The common risk factors for sleep apnea include BMI greater than 28 kg/m2, smoking habit, and neck circumference greater than 17 inches. Sleep apnea is often observed in patients older than 65 years. Occasional consumption of alcohol is not a risk factor by itself.

Which problem is associated with obesity, heavy snoring, and shallow breathing? A) Sleep apnea B) Narcolepsy C) Hypersomnia D) Hyperpnea

A.Sleep apnea refers to recurrent periods of absence of breathing for 10 seconds or longer, occurring at least 5 times per hour.

1. An emergency department nurse cares for a client who is severely dehydrated and is prescribed 3 L of intravenous fluid over 6 hours. At what rate (mL/hr) should the nurse set the intravenous pump to infuse the fluids? (Record your answer using a whole number.) ____ mL/hr

ANS: 500 mL/hr Because IV pumps deliver in units of milliliters per hour, the pump would have to be set at 500 mL/hr to deliver 3 L (3000 mL) over 6 hours. 6x = 3000 x = 500

1. A client in the intensive care unit with acute kidney injury (AKI) must maintain a mean arterial pressure (MAP) of 65 mm Hg to promote kidney perfusion. What is the client's MAP if the blood pressure is 98/50 mm Hg? (Record your answer using a whole number.) _____ mm Hg

ANS: 66 mm Hg 98+(2*50) / 3 = MAP

An older adult is admitted to the emergency department with respiratory symptoms. Which client symptom requires the nurse to intervene immediately? a. Confusion b. Scattered wheezing c. Crackles d. Flushed cheeks

ANS: A Confusion in an older adult can signify hypoxia. If the nurse waited to intervene until the older adult showed more traditional symptoms of pneumonia, the client may become critically ill. The other manifestations also require intervention but not as the priority.

5. Which symptoms should a nurse recognize that differentiate a client diagnosed with obsessive-compulsive disorder (OCD) from a client diagnosed with obsessive-compulsive personality disorder? 1. Clients diagnosed with OCD experience both obsessions and compulsions, and clients diagnosed with obsessive-compulsive personality disorder do not. 2. Clients diagnosed with obsessive-compulsive personality disorder experience both obsessions and compulsions, and clients diagnosed with OCD do not. 3. Clients diagnosed with obsessive-compulsive personality disorder experience only obsessions, and clients diagnosed with OCD experience only compulsions. 4. Clients diagnosed with OCD experience only obsessions, and clients diagnosed with obsessive-compulsive personality disorder experience only compulsions.

ANS: 1 Rationale: A client diagnosed with OCD experiences both obsessions and compulsions. Clients with obsessive-compulsive personality disorder exhibit a pervasive pattern of preoccupation with orderliness, perfectionism, mental and interpersonal control, but do not experience obsessions and compulsions. Cognitive Level: Analysis Integrated Process: Evaluation

14. A nurse is providing discharge teaching to a client taking a benzodiazepine. Which client statement would indicate a need for further follow-up instructions? 1. "I will need scheduled blood work in order to monitor for toxic levels of this drug." 2. "I won't stop taking this medication abruptly because there could be serious complications." 3. "I will not drink alcohol while taking this medication." 4. "I won't take extra doses of this drug because I can become addicted."

ANS: 1 Rationale: The client indicates a need for additional information about taking benzodiazepines when stating the need for blood work to monitor for toxic levels. This intervention is used when taking lithium (Eskalith) for the treatment of bipolar disorder. The client should understand that taking extra doses of a benzodiazepine may result in addiction and that the drug should not be taken in conjunction with alcohol. Cognitive Level: Application Integrated Process: Evaluation

. A client diagnosed with panic disorder states, "When an attack happens, I feel like I am going to die." Which is the most appropriate nursing response? 1. "I know it's frightening, but try to remind yourself that this will only last a short time." 2. "Death from a panic attack happens so infrequently that there is no need to worry." 3. "Most people who experience panic attacks have feelings of impending doom." 4. "Tell me why you think you are going to die every time you have a panic attack."

ANS: 1 Rationale: The most appropriate nursing response to the client's concerns is to empathize with the client and provide encouragement that panic attacks only last a short period. Panic attacks usually last minutes but can, rarely, last hours. When the nurse states that "Most people who experience panic attacks..." the nurse depersonalizes and belittles the client's feeling. Cognitive Level: Application Integrated Process: Implementation

What is the most essential task for a nurse to accomplish prior to forming a therapeutic relationship with a client? 1. Clarify personal attitudes, values, and beliefs. 2. Obtain thorough assessment data. 3. Determine the clients length of stay. 4. Establish personal goals for the interaction.

ANS: 1 Rationale: The most essential task for a nurse to accomplish prior to forming a therapeutic relationship with a client is to clarify personal attitudes, values, and beliefs. Understanding one's own attitudes, values, and beliefs is called self-awareness.

Which therapeutic communication technique is being used in the following nurse-client interaction? Client: My father spanked me often. Nurse: Your father was a harsh disciplinarian. 1. Restatement 2. Offering general leads 3. Focusing 4. Accepting

ANS: 1 Rationale: The nurse is using the therapeutic communication technique of restatement. Restatement involves repeating the main idea of what the client has said. It allows the client to know whether the statement has been understood and provides an opportunity to continue.

Which client action should a nurse expect during the working phase of the nurse-client relationship? 1. The client gains insight and incorporates alternative behaviors. 2. The client establishes rapport with the nurse and mutually develops treatment goals. 3. The client explores feelings related to reentering the community. 4. The client explores personal strengths and weaknesses that impact behavioral choices.

ANS: 1 Rationale: The nurse should expect that that the client will gain insight and incorporate alternative behaviors during the working phase of the nurse-client relationship. The client may also overcome resistance, problem-solve, and continually evaluate progress toward goals.

8. A nursing instructor is teaching about the medications used to treat panic disorder. Which student statement indicates that learning has occurred? 1. "Clonazepam (Klonopin) is particularly effective in the treatment of panic disorder." 2. "Clozapine (Clozaril) is used off-label in long-term treatment of panic disorder." 3. "Doxepin (Sinequan) can be used in low doses to relieve symptoms of panic attacks." 4. "Buspirone (BuSpar) is used for its immediate effect to lower anxiety during panic attacks."

ANS: 1 Rationale: The student indicates learning has occurred when he or she states that clonazepam is a particularly effective treatment for panic disorder. Clonazepam is a type of benzodiazepine in which the major risk is physical dependence and tolerance, which may encourage abuse. It can be used on an as-needed basis to reduce anxiety and the related symptoms. Cognitive Level: Application Integrated Process: Evaluation

24. An attractive female client presents with high anxiety levels because of her belief that her facial features are large and grotesque. Body dysmorphic disorder (BDD) is suspected. Which of the following additional symptoms would support this diagnosis? (Select all that apply.) 1. Mirror checking 2. Excessive grooming 3. History of an eating disorder 4. History of delusional thinking 5. Skin picking Multiple Response

ANS: 1, 2, 5 Rationale: The DSM-5 lists preoccupation with one or more perceived defects or flaws in physical appearance that are not observable or appear slight to others as a diagnostic criteria for the diagnosis of BDD. Also listed is that at some point during the course of the disorder, the person has performed repetitive behaviors, such as mirror checking, excessive grooming, skin picking, or reassurance seeking.

23. A nurse has been caring for a client diagnosed with generalized anxiety disorder (GAD). Which of the following nursing interventions would address this client's symptoms? (Select all that apply.) 1. Encourage the client to recognize the signs of escalating anxiety. 2. Encourage the client to avoid any situation that causes stress. 3. Encourage the client to employ newly learned relaxation techniques. 4. Encourage the client to cognitively reframe thoughts about situations that generate anxiety. 5. Encourage the client to avoid caffeinated products. Multiple Response

ANS: 1, 3, 4, 5 Rationale: Nursing interventions that address GAD symptoms should include encouraging the client to recognize signs of escalating anxiety, to employ relaxation techniques, to cognitively reframe thoughts about anxiety-provoking situations, and to avoid caffeinated products. Avoiding situations that cause stress is not an appropriate intervention, because avoidance does not help the client overcome anxiety and because not all situations are easily avoidable. Cognitive Level: Application Integrated Process: Implementation

21. A college student has been diagnosed with generalized anxiety disorder (GAD). Which of the following symptoms should a campus nurse expect this client to exhibit? (Select all that apply.) 1. Fatigue 2. Anorexia 3. Hyperventilation 4. Insomnia 5. Irritability Multiple Response

ANS: 1, 4, 5 Rationale: The nurse should expect that a client diagnosed with GAD would experience fatigue, insomnia, and irritability. GAD is characterized by chronic, unrealistic, and excessive anxiety and worry. Cognitive Level: Application Integrated Process: Assessment

13. A client diagnosed with obsessive-compulsive disorder is admitted to a psychiatric unit. The client has an elaborate routine for toileting activities. Which would be an appropriate initial client outcome during the first week of hospitalization? 1. The client will refrain from ritualistic behaviors during daylight hours. 2. The client will wake early enough to complete rituals prior to breakfast. 3. The client will participate in three unit activities by day three. 4. The client will substitute a productive activity for rituals by day one.

ANS: 2 Rationale: An appropriate initial client outcome is for the client to wake early enough to complete rituals prior to breakfast. The nurse should also provide a structured schedule of activities and begin to gradually limit the time allowed for rituals. Cognitive Level: Application Integrated Process: Planning

A mother rescues two of her four children from a house fire. In an emergency department, she cries, I should have gone back in to get them. I should have died, not them. What is the nurse's best response? 1. The smoke was too thick. You couldn't have gone back in. 2. You're experiencing feelings of guilt, because you weren't able to save your children. 3. Focus on the fact that you could have lost all four of your children. 4. It's best if you try not to think about what happened. Try to move on.

ANS: 2 Rationale: The best response by the nurse is, You're experiencing feelings of guilt, because you weren't able to save your children. This response uses the therapeutic communication technique of restating what the client has said. This lets the client know whether an expressed statement has been understood or if clarification is necessary.

17. A nursing student questions an instructor regarding the order for fluvoxamine (Luvox) 300 mg daily for a client diagnosed with obsessive-compulsive disorder (OCD). Which instructor response is most accurate? 1. High doses of tricyclic medications will be required for effective treatment of OCD. 2. Selective serotonin reuptake inhibitor (SSRI) doses, in excess of what is effective for treating depression, may be required for OCD. 3. The dose of Luvox is low because of the side effect of daytime drowsiness. 4. The dose of this selective serotonin reuptake inhibitor (SSRI) is outside the therapeutic range and needs to be questioned.

ANS: 2 Rationale: The most accurate instructor response is that SSRI doses in excess of what is effective for treating depression may be required in the treatment of OCD. SSRIs have been approved by the Food and Drug Administration for the treatment of OCD. Common side effects include headache, sleep disturbances, and restlessness. Cognitive Level: Application Integrated Process: Implementation

9. A family member is seeking advice about an older parent who seems to worry unnecessarily about everything. The family member states, "Should I seek psychiatric help for my mother?" Which is an appropriate nursing response? 1. "My mother also worries unnecessarily. I think it is part of the aging process." 2. "Anxiety is considered abnormal when it is out of proportion to the stimulus causing it and when it impairs functioning." 3. "From what you have told me, you should get her to a psychiatrist as soon as possible." 4. "Anxiety is a complex phenomenon and is effectively treated only with psychotropic medications."

ANS: 2 Rationale: The most appropriate response by the nurse is to explain to the family member that anxiety is considered abnormal when it is out of proportion and impairs functioning. Anxiety is a normal reaction to a realistic danger or threat to biological integrity or self-concept. Cognitive Level: Application Integrated Process: Implementation

A nurse maintains an uncrossed arm and leg posture when communicating with a client. This nonverbal behavior is reflective of which letter of the SOLER acronym for active listening? 1. S 2. O 3. L 4. E 5. R

ANS: 2 Rationale: The nurse should identify that maintaining an uncrossed arm and leg posture is nonverbal behavior that reflects the O in the active-listening acronym SOLER. The acronym SOLER includes sitting squarely facing the client (S), observing and open posture (O), leaning forward toward the client (L), establishing eye contact (E), and relaxing (R).

A client diagnosed with post-traumatic stress disorder related to a rape is admitted to an inpatient psychiatric unit for evaluation and medication stabilization. Which therapeutic communication technique might a nurse use that is an example of broad openings? 1. What occurred prior to the rape, and when did you go to the emergency department? 2. What would you like to talk about? 3. I notice you seem uncomfortable discussing this. 4. How can we help you feel safe during your stay here?

ANS: 2 Rationale: The nurses statement, What would you like to talk about? is an example of the therapeutic communication technique of a broad opening. Using broad openings allows the client to take the initiative in introducing the topic and emphasizes the importance of the client's role in the interaction.

1. A nursing instructor is teaching about specific phobias. Which student statement indicates to the instructor that learning has occurred? a. "These clients recognize their fear as excessive and frequently seek treatment." b."These clients have a panic level of fear that is overwhelming and unreasonable." c. "These clients experience symptoms that mirror a cerebrovascular accident (CVA)." d. "These clients experience the symptoms of tachycardia, dysphagia, and diaphoresis."

ANS: 2 Rationale: The nursing instructor should evaluate that learning has occurred when the student knows that clients with phobias have a panic level of fear that is overwhelming and unreasonable. Phobia is fear cued by a specific object or situation in which exposure to the stimuli produces an immediate anxiety response. Even though the disorder is relatively common among the general population, people seldom seek treatment unless the phobia interferes with ability to function. Cognitive Level: Application Integrated Process: Evaluation

20. A nursing instructor is teaching about the symptoms of agoraphobia. Which student statement indicates that learning has occurred? 1. Onset of symptoms most commonly occurs in early adolescence and persists until midlife. 2. Onset of symptoms most commonly occurs in the 20s and 30s and persists for many years. 3. Onset of symptoms most commonly occurs in the 40s and 50s and persists until death. 4. Onset of symptoms most commonly occurs after the age of 60 and persists for at least 6 years.

ANS: 2 Rationale: The onset of the symptoms of agoraphobia most commonly occurs in the 20s and 30s and persists for many years. Cognitive Level: Application Integrated Process: Assessment

What should be the priority nursing action during the orientation (introductory) phase of the nurse-client relationship? 1. Acknowledge the clients actions and generate alternative behaviors. 2. Establish rapport and develop treatment goals. 3. Attempt to find alternative placement. 4. Explore how thoughts and feelings about this client may adversely impact nursing care.

ANS: 2 Rationale: The priority nursing action during the orientation phase of the nurse-client relationship should be to establish rapport and develop treatment goals. Rapport implies feelings on the part of both the nurse and the client, based on respect, acceptance, a sense of trust, and a nonjudgmental attitude. It is the essential foundation of the nurse-client relationship.

When an individual is two-faced, which characteristic essential to the development of a therapeutic relationship should a nurse identify as missing? 1. Respect 2. Genuineness 3. Sympathy 4. Rapport

ANS: 2 Rationale: When an individual is two-faced, which means double-dealing or deceitful, the nurse should identify that genuineness is missing in the relationship. Genuineness refers to the nurses ability to be open and honest and maintain congruence between what is felt and what is communicated. When a nurse fails to bring genuineness to the relationship, trust cannot be established.

22. A nurse is discussing treatment options with a client whose life has been negatively impacted by claustrophobia. Which of the following commonly used behavioral therapies for phobias should the nurse explain to the client? (Select all that apply.) 1. Benzodiazepine therapy 2. Systematic desensitization 3. Imploding (flooding) 4. Assertiveness training 5. Aversion therapy Multiple Response

ANS: 2, 3 Rationale: The nurse should explain to the client that systematic desensitization and imploding are the most common behavioral therapies used for treating phobias. Systematic desensitization involves the gradual exposure of the client to anxiety-provoking stimuli. Imploding is the intervention used in which the client is exposed to extremely frightening stimuli for prolonged periods of time. Cognitive Level: Application Integrated Process: Implementation

2. Which nursing statement to a client about social anxiety disorder versus schizoid personality disorder (SPD) is most accurate? a. "Clients diagnosed with social anxiety disorder can manage anxiety without medications, whereas clients diagnosed with SPD can only manage anxiety with medications." b. "Clients diagnosed with SPD are distressed by the symptoms experienced in social settings, whereas clients diagnosed with social anxiety disorder are not." c. "Clients diagnosed with social anxiety disorder avoid interactions only in social settings, whereas clients diagnosed with SPD avoid interactions in all areas of life." d. "Clients diagnosed with SPD avoid interactions only in social settings, whereas clients diagnosed with social anxiety disorder tend to avoid interactions in all areas of life."

ANS: 3 Rationale: Clients diagnosed with social anxiety disorder avoid interactions only in social settings, whereas clients diagnosed with SPD avoid interactions in all areas of life. Social anxiety disorder is an excessive fear of situations in which a person might do something embarrassing or be evaluated negatively by others. Cognitive Level: Analysis Integrated Process: Implementation

An instructor is correcting a nursing students clinical worksheet. Which instructor statement is the best example of effective feedback? 1. Why did you use the clients name on your clinical worksheet? 2. You were very careless to refer to your client by name on your clinical worksheet. 3. Surely you didn't do this deliberately, but you breached confidentiality by using names. 4. It is disappointing that after being told you're still using client names on your worksheet.

ANS: 3 Rationale: The instructors statement, Surely you didn't do this deliberately, but you breached confidentiality by using names, is an example of effective feedback. Feedback is method of communication for helping others consider a modification of behavior. Feedback should be descriptive, specific, and directed toward a behavior that the person has the capacity to modify and should impart information rather than offer advice.

A client exhibiting dependent behaviors says, Do you think I should move from my parents house and get a job? Which nursing response is most appropriate? 1. It would be best to do that in order to increase independence. 2. Why would you want to leave a secure home? 3. Let's discuss and explore all of your options. 4. I'm afraid you would feel very guilty leaving your parents.

ANS: 3 Rationale: The most appropriate response by the nurse is, Let's discuss and explore all of your options. In this example, the nurse is encouraging the client to formulate ideas and decide independently the appropriate course of action.

10. A client is experiencing a severe panic attack. Which nursing intervention would meet this client's physiological need? 1. Teach deep breathing relaxation exercises. 2. Place the client in a Trendelenburg position. 3. Have the client breathe into a paper bag. 4. Administer the ordered prn buspirone (BuSpar).

ANS: 3 Rationale: The nurse can meet this client's physiological need by having the client breathe into a paper bag. Hyperventilation may occur during periods of extreme anxiety. Hyperventilation causes the amount of carbon dioxide (CO2) in the blood to decrease, possibly resulting in lightheadedness, rapid heart rate, shortness of breath, numbness or tingling in the hands or feet, and syncope. If hyperventilation occurs, assist the client to breathe into a small paper bag held over the mouth and nose. Six to twelve natural breaths should be taken, alternating with short periods of diaphragmatic breathing. Cognitive Level: Application Integrated Process: Implementation

Which therapeutic communication technique is being used in the following nurse-client interaction? Client: When I am anxious, the only thing that calms me down is alcohol. Nurse: Other than drinking, what alternatives have you explored to decrease anxiety? 1. Reflecting 2. Making observations 3. Formulating a plan of action 4. Giving recognition

ANS: 3 Rationale: The nurse is using the therapeutic communication technique of formulating a plan of action to help the client explore alternatives to drinking. The use of this technique may serve to prevent anger or anxiety from escalating.

12. A client living in a beachfront community is seeking help with an extreme fear of bridges, which is interfering with daily functioning. A psychiatric nurse practitioner decides to try systematic desensitization. Which explanation of this treatment should the nurse provide? 1. "Using your imagination, we will attempt to achieve a state of relaxation." 2. "Because anxiety and relaxation are mutually exclusive states, we can attempt to substitute a relaxation response for the anxiety response." 3. "Through a series of increasingly anxiety-provoking steps, we will gradually increase your tolerance to anxiety." 4. "In one intense session, you will be exposed to a maximum level of anxiety that you will learn to tolerate."

ANS: 3 Rationale: The nurse should explain to the client that when participating in systematic desensitization he or she will go through a series of increasingly anxiety-provoking steps that will gradually increase tolerance. Systematic desensitization was introduced by Joseph Wolpe in 1958 and is based on behavioral conditioning principles. Cognitive Level: Application Integrated Process: Implementation

4. Which treatment should a nurse identify as most appropriate for clients diagnosed with generalized anxiety disorder (GAD)? 1. Long-term treatment with diazepam (Valium) 2. Acute symptom control with citalopram (Celexa) 3. Long-term treatment with buspirone (BuSpar) 4. Acute symptom control with ziprasidone (Geodon)

ANS: 3 Rationale: The nurse should identify that an appropriate treatment for clients diagnosed with GAD is long-term treatment with buspirone. Buspirone is an anxiolytic medication that is effective in 60% to 80% of clients diagnosed with GAD. Buspirone takes 10 to 14 days for alleviation of symptoms but does not have the dependency concerns of other anxiolytics. Cognitive Level: Application Integrated Process: Evaluation

Which client statement should a nurse identify as a typical response to stress most often experienced in the working phase of the nurse-client relationship? 1. I can't bear the thought of leaving here and failing. 2. I might have a hard time working with you, because you remind me of my mother. 3. I really don't want to talk any more about my childhood abuse. 4. I'm not sure that I can count on you to protect my confidentiality.

ANS: 3 Rationale: The nurse should identify that the client statement, I really don't want to talk any more about my childhood abuse, reflects that the client is in the working phase of the nurse-client relationship. The working phase includes overcoming resistance behaviors on the part of the client as the level of anxiety rises in response to discussion of painful issues.

The nurse is interviewing a newly admitted psychiatric client. Which of the following nursing statements is an example of offering a general lead? 1. Do you know why you are here? 2. Are you feeling depressed or anxious? 3. Yes, I see. Go on. 4. Can you order the specific events that led to your admission?

ANS: 3 Rationale: The nurses statement, Yes, I see. Go on, is an example of a general lead. Offering general leads encourages the client to continue sharing information.

A newly admitted client, diagnosed with obsessive-compulsive disorder (OCD), washes his hands continually. This behavior prevents unit activity attendance. Which nursing statement best addresses this situation? 1. Everyone diagnosed with OCD needs to control their ritualistic behaviors. 2. It is important for you to discontinue these ritualistic behaviors. 3. Why are you asking for help, if you won't participate in unit therapy? 4. Let's figure out a way for you to attend unit activities and still wash your hands.

ANS: 4 Rationale: The most appropriate statement by the nurse is, Let's figure out a way for you to attend unit activities and still wash your hands. This statement reflects the therapeutic communication technique of formulating a plan of action. The nurse attempts to work with the client to develop a plan without damaging the therapeutic relationship.

16. A client is newly diagnosed with obsessive-compulsive disorder and spends 45 minutes folding clothes and rearranging them in drawers. Which nursing intervention would best address this client's problem? 1. Distract the client with other activities whenever ritual behaviors begin. 2. Report the behavior to the psychiatrist to obtain an order for medication dosage increase. 3. Lock the room to discourage ritualistic behavior. 4. Discuss the anxiety-provoking triggers that precipitate the ritualistic behaviors.

ANS: 4 Rationale: The nurse should discuss with the client the anxiety-provoking triggers that precipitate the ritualistic behavior. If the client is going to be able to control interrupting anxiety, he or she must first learn to recognize precipitating factors. Attempting to distract the client, seeking medication increase, and locking the client's room are not appropriate interventions, because they do not help the client gain insight. Cognitive Level: Application Integrated Process: Implementation

15. A client diagnosed with an obsessive-compulsive disorder spends hours bathing and grooming. During a one-on-one interaction, the client discusses the rituals in detail but avoids any feelings that the rituals generate. Which defense mechanism should the nurse identify? 1. Sublimation 2. Dissociation 3. Rationalization 4. Intellectualization

ANS: 4 Rationale: The nurse should identify that the client is using the defense mechanism of intellectualization when discussing the rituals of obsessive-compulsive disorder in detail while avoiding discussion of feelings. Intellectualization is an attempt to avoid expressing emotions associated with a stressful situation by using the intellectual process of logic, reasoning, and analysis. Cognitive Level: Application Integrated Process: Assessment

On which task should a nurse place priority during the working phase of relationship development? 1. Establishing a contract for intervention 2. Examining feelings about working with a particular client 3. Establishing a plan for continuing aftercare 4. Promoting the clients insight and perception of reality

ANS: 4 Rationale: The nurse should place priority on promoting the clients insight and perception of reality during the working phase of relationship development. Establishing a contract for intervention would occur in the orientation phase. Examining feelings about working with a client should occur in the pre-interaction phase. Establishing a plan for aftercare would occur in the termination phase.

3. What symptoms should a nurse recognize that differentiate a client diagnosed with panic disorder from a client diagnosed with generalized anxiety disorder (GAD)? 1. GAD is acute in nature, and panic disorder is chronic. 2. Chest pain is a common GAD symptom, whereas this symptom is absent in panic disorders. 3. Hyperventilation is a common symptom in GAD and rare in panic disorder. 4. Depersonalization is commonly seen in panic disorder and absent in GAD.

ANS: 4 Rationale: The nurse should recognize that a client diagnosed with panic disorder experiences depersonalization, whereas a client diagnosed with GAD would not. Depersonalization refers to being detached from oneself when experiencing extreme anxiety. Cognitive Level: Analysis Integrated Process: Assessment

If a client demonstrates transference toward a nurse, how should the nurse respond? 1. Promote safety and immediately terminate the relationship with the client. 2. Encourage the client to ignore these thoughts and feelings. 3. Immediately reassign the client to another staff member. 4. Help the client to clarify the meaning of the relationship, based on the present situation.

ANS: 4 Rationale: The nurse should respond to a client's transference by clarifying the meaning of the nurse-client relationship based on the present situation. Transference occurs when the client unconsciously displaces feelings about a person from the past toward the nurse. The nurse should assist the client in separating the past from the present.

6. A cab driver, stuck in traffic, becomes lightheaded, tremulous, diaphoretic, tachycardic and dyspneic. A workup in an emergency department reveals no pathology. Which medical diagnosis should a nurse suspect, and what nursing diagnosis should be the nurse's first priority? 1. Generalized anxiety disorder and a nursing diagnosis of fear 2. Altered sensory perception and a nursing diagnosis of panic disorder 3. Pain disorder and a nursing diagnosis of altered role performance 4. Panic disorder and a nursing diagnosis of anxiety

ANS: 4 Rationale: The nurse should suspect that the client has exhibited signs and symptoms of a panic disorder. The priority nursing diagnosis should be anxiety. Panic disorder is characterized by recurrent, sudden-onset panic attacks in which the person feels intense fear, apprehension, or terror. Cognitive Level: Analysis Integrated Process: Diagnosis

A mother who is notified that her child was killed in a tragic car accident states, I can't bear to go on with my life. Which nursing statement conveys empathy? 1. This situation is very sad, but time is a great healer. 2. You are sad, but you must be strong for your other children. 3. Once you cry it all out, things will seem so much better. 4. It must be horrible to lose a child, and I'll stay with you until your husband arrives.

ANS: 4 Rationale: The nurses response, It must be horrible to lose a child, and I'll stay with you until your husband arrives, conveys empathy to the client. Empathy is the ability to see the situation from the client's point of view. Empathy is considered to be one of the most important characteristics of the therapeutic relationship.

A nurse says to a client, Things will look better tomorrow after a good night's sleep. This is an example of which communication technique? 1. The therapeutic technique of giving advice 2. The therapeutic technique of defending 3. The non therapeutic technique of presenting reality 4. The non therapeutic technique of giving reassurance

ANS: 4 Rationale: The nurses statement, Things will look better tomorrow after a good night's sleep, is an example of the nontherapeutic communication technique of giving reassurance. Giving reassurance indicates to the client that there is no cause for anxiety, thereby devaluing the clients feelings.

What is a nurses purpose for providing appropriate feedback? 1. To give the client good advice 2. To advise the client on appropriate behaviors 3. To evaluate the client's behavior 4. To give the client critical information

ANS: 4 Rationale: The purpose of providing appropriate feedback is to give the client critical information. Feedback should not be used to give advice or evaluate behaviors.

The nurse has determined that a client has an acute sore throat. What is the nurse's best action? a. Assess whether the client can speak. b. Call an ear-nose-throat specialist. c. Administer an antibiotic. d. Give the client ice chips.

ANS: A A dry cough and difficulty swallowing may indicate that the client is developing laryngitis. The nurse should assess whether the client can speak or shows any changes in his or her voice. The other interventions are not appropriate.

Which assessment is most important for the nurse to perform for the client admitted to the postanesthesia care unit (PACU) after surgery under general anesthesia? a. Determining the client's level of consciousness b. Checking for pain on dorsi and plantar flexion of the foot c. Assessing the response to pin prick stimulation from feet to mid chest level d. Comparing blood pressure taken in the right arm to blood pressure taken in the left arm

ANS: A After general anesthesia, which affects the entire body, the priority assessment is to determine that the client's level of consciousness has returned. All other assessment can be performed subsequently.

A client has acute rhinitis. What is the most important intervention for the nurse to perform? a. Assess for symptoms of infection. b. Ascertain whether the client has allergies. c. Question the client on the use of nasal sprays. d. Do blood and urine screenings for drug use

ANS: A Bacterial infection often occurs with acute rhinitis. The nurse should assess for symptoms because treatment may be warranted. It is not essential to assess for allergies or the use of nasal spray, or to determine whether drug use is occurring. All of these interventions are focused on determining a cause for repeated acute rhinitis and are primarily the responsibility of the health care provider. The nurse should focus on client assessment and should determine whether a secondary infection is present.

A nurse assesses a client who has a 15-year history of diabetes and notes decreased tactile sensation in both feet. Which action should the nurse take first? A. Document the finding in the client's chart B. Assess tactile sensation in the client's hands C. Examine the client's feet for signs of injury D. Notify the health care provider

Answer: C

The nurse is teaching a client about nutrition and diverticulosis. Which food will the nurse teach the client to avoid? a. Popcorn b. Oatmeal c. Bran d. Lettuce

ANS: A Foods such as nuts, corn, popcorn, cucumbers, tomatoes, figs, and strawberries—all of which contain seeds or indigestible material—may block a diverticulum and should be avoided. Oatmeal, bran, and lettuce are acceptable for patients with diverticulosis to eat.

The nurse is worried that a client who is not entirely reliable is being discharged home on therapy for multidrug-resistant tuberculosis. What strategy is the best to use for this client? a. Directly observed therapy b. IV drug administration c. Remaining in the hospital d. Isolation

ANS: A If a client is "not reliable," the risk is that the client will not take medications as required, causing spread of an organism that may become more drug resistant. The other answers are not correct.

An older adult client with heart failure asks if she should get a flu shot. Which is the nurse's best response? a. "Yes, because of your heart failure you are at greater risk for complications." b. "Yes, if it has been longer than 5 years since your last flu vaccination." c. "No, your heart failure makes you too weak to get the live virus vaccine." d. "No, the vaccine will interact with your heart medications."

ANS: A People older than 50 years and those with chronic disease should be vaccinated against the flu each year early in the fall because they are at higher risk of developing complications if they do get ill. Flu shots appear to be effective for only one flu season, so the client should get one annually. The live vaccine is recommended only for healthy people up to age 49. This vaccination should not have interactions with heart medications.

Which finding best indicates that the goal Demonstrate mentally healthy behavior was achieved? A patient: a. sees self as capable of achieving ideals and meeting demands. b. behaves without considering the consequences of personal actions. c. aggressively meets own needs without considering the rights of others. d. seeks help from others when assuming responsibility for major areas of own life.

ANS: A The correct response describes an adaptive, healthy behavior. The distracters describe maladaptive behaviors.

A client who has had acute tonsillitis develops drooling and reports severe throat pain. What is the nurse's priority intervention? a. Assess the throat for deviation of the uvula. b. Prepare the client for surgery. c. Teach the client about antibiotic therapy. d. Prepare the client for percutaneous needle aspiration.

ANS: A The nurse should first assess the throat for signs of peritonsillar abscess. If present, the nurse should call the health care provider immediately because aspiration of the abscess may be needed to maintain the airway.

In the majority culture of the United States, which individual has the greatest risk to be labeled mentally ill? One who: a. describes hearing Gods voice speaking. b. is usually pessimistic but strives to meet personal goals. c. is wealthy and gives away $20 bills to needy individuals. d. always has an optimistic viewpoint about life and having own needs met.

ANS: A The question asks about risk. Hearing voices is generally associated with mental illness, but in charismatic religious groups, hearing the voice of God or a prophet is a desirable event. Cultural norms vary, which makes it more difficult to make an accurate diagnosis. The individuals described in the other options are less likely to be labeled mentally ill.

Which statement provides the best rationale for why a nurse should closely monitor a severely depressed patient during antidepressant medication therapy? A. As depression lifts, physical energy becomes available to carry out suicide. B. Suicide may be precipitated by a variety of internal and external events C. Suicidal patients have difficulty using social supports D. Suicide is an impulsive act.

ANS: A Antidepressant medication has the objective of relieving depression. The risk for suicide is greater as the depression lifts, primarily because the patient has more physical energy at a time when he or she may still have suicidal ideation. The other options have little to do with nursing interventions relating to antidepressant medication therapy.

11. A nurse plans care for a client who is experiencing dyspnea and must stop multiple times when climbing a flight of stairs. Which intervention should the nurse include in this clients plan of care? a. Assistance with activities of daily living b. Physical therapy activities every day c. Oxygen therapy at 2 liters per nasal cannula d. Complete bedrest with frequent repositioning

ANS: A A client with dyspnea and difficulty completing activities such as climbing a flight of stairs has class III dyspnea. The nurse should provide assistance with activities of daily living. These clients should be encouraged to participate in activities as tolerated. They should not be on complete bedrest, may not be able to tolerate daily physical therapy, and only need oxygen if hypoxia is present.

24. A nurse auscultates a clients lung fields. Which pathophysiologic process should the nurse associate with this breath sound? (Click the media button to hear the audio clip.) a. Inflammation of the pleura b. Constriction of the bronchioles c. Upper airway obstruction d. Pulmonary vascular edema

ANS: A A pleural friction rub can be heard when the pleura is inflamed and rubbing against the lung wall. The other pathophysiologic processes would not cause a pleural friction rub. Constriction of the bronchioles may be heard as a wheeze, upper airway obstruction may be heard as stridor, and pulmonary vascular edema may be heard as crackles.

16. According to Peplau, which nursing intervention is most appropriate when the nurse is functioning in the role of a surrogate? A. The nurse functions as a nurturing parent in order to build a trusting relationship. B. The nurse plays cards with a small group of clients. C. The nurse discusses childhood events that may affect personality development. D. The nurse provides a safe social environment.

ANS: A According to Peplau, when a client is acutely ill, he or she may incur the role of infant or child, while the nurse is perceived as the mother surrogate. KEY: Cognitive Level: Application | Integrated Processes: Nursing Process: Implementation | Client Need: Psychosocial Integrity

According to Peplau, which nursing intervention is most appropriate when the nurse is functioning in the role of a surrogate? A. The nurse functions as a nurturing parent in order to build a trusting relationship. B. The nurse plays cards with a small group of clients. C. The nurse discusses childhood events that may affect personality development. D. The nurse provides a safe social environment.

ANS: A According to Peplau, when a client is acutely ill, he or she may incur the role of infant or child, while the nurse is perceived as the mother surrogate.

21.A nurse admits a client from the emergency department. Client data are listed below: History 70 years of age History of diabetes On insulin twice a day Reports new-onset dyspnea and productive cough Physical Assessment Crackles and rhonchi heard throughout the lungs Dullness to percussion LLL Afebrile Oriented to person only Laboratory Values WBC: 5,200/mm3 PaO2 on room air 65mm Hg What action by the nurse is the priority? a. Administer oxygen at 4 liters per nasal cannula. b. Begin broad-spectrum antibiotics. c. Collect a sputum sample for culture. d. Start an IV of normal saline at 50 mL/hr.

ANS: A All actions are appropriate for this client who has manifestations of pneumonia. However, airway and breathing come first, so begin oxygen administration and titrate it to maintain saturations greater than 95%. Start the IV and collect a sputum culture, and then begin antibiotics.

A nurse admits a client from the emergency department. Client data are listed below: 70 years of age History of diabetes On insulin twice a day Reports new-onset dyspnea and productive cough Crackles and rhonchi heard throughout the lungs Dullness to percussion LLL Afebrile Oriented to person only WBC: 5,200/mm3 PaO2 on room air 65 mm Hg What action by the nurse is the priority? a. Administer oxygen at 4 liters per nasal cannula. b. Begin broad-spectrum antibiotics. c. Collect a sputum sample for culture. d. Start an IV of normal saline at 50 mL/hr.

ANS: A All actions are appropriate for this client who has manifestations of pneumonia. However, airway and breathing come first, so begin oxygen administration and titrate it to maintain saturations greater than 95%. Start the IV and collect a sputum culture, and then begin antibiotics.

A nurse assesses a client who is being treated for hyperglycemic - hyperosmolar state. Which clinical manifestation indicates to the nurse that the therapy needs to be adjusted? A. Serum potassium level has increased B. Blood osmolarity has decreased C. Glasgow Coma Scale score is unchanged D. Urine remains negative for ketone bodies

Answer: C

14. A nurse auscultates a harsh hollow sound over a clients trachea and larynx. Which action should the nurse take first? a. Document the findings. b. Administer oxygen therapy. c. Position the client in high-Fowlers position. d. Administer prescribed albuterol.

ANS: A Bronchial breath sounds, including harsh, hollow, tubular, and blowing sounds, are a normal finding over the trachea and larynx. The nurse should document this finding. There is no need to implement oxygen therapy, administer albuterol, or change the clients position because the finding is normal.

12. A nurse is caring for a client who has threatened to commit suicide by hanging. The client states, Im going to use a knotted shower curtain when no one is around. Which information would determine the nurses plan of care for this client? A. The more specific the plan is, the more likely the client will attempt suicide. B. Clients who talk about suicide never actually commit it. C. Clients who threaten suicide should be observed every 15 minutes. D. After a brief assessment, the nurse should avoid the topic of suicide.

ANS: A Clients who have specific plans are at greater risk for suicide.

5. A nurse evaluates a client with acute glomerulonephritis (GN). Which manifestation should the nurse recognize as a positive response to the prescribed treatment? a. The client has lost 11 pounds in the past 10 days. b. The client's urine specific gravity is 1.048. c. No blood is observed in the client's urine. d. The client's blood pressure is 152/88 mm Hg.

ANS: A Fluid retention is a major feature of acute GN. This weight loss represents fluid loss, indicating that the glomeruli are performing the function of filtration. A urine specific gravity of 1.048 is high. Blood is not usually seen in GN, so this finding would be expected. A blood pressure of 152/88 mm Hg is too high; this may indicate kidney damage or fluid overload.

11.The emergency department (ED) manager is reviewing client charts to determine how well the staff performs when treating clients with community-acquired pneumonia. What outcome demonstrates that goals for this client type have been met? a. Antibiotics started before admission b. Blood cultures obtained within 20 minutes c. Chest x-ray obtained within 30 minutes d. Pulse oximetry obtained on all clients

ANS: A Goals for treatment of community-acquired pneumonia include initiating antibiotics prior to inpatient admission or within 6 hours of presentation to the ED. Timely collection of blood cultures, chest x-ray, and pulse oximetry are important as well but do not coincide with established goals.

The emergency department (ED) manager is reviewing client charts to determine how well the staff performs when treating clients with community-acquired pneumonia. What outcome demonstrates that goals for this client type have been met? a. Antibiotics started before admission b. Blood cultures obtained within 20 minutes c. Chest x-ray obtained within 30 minutes d. Pulse oximetry obtained on all clients

ANS: A Goals for treatment of community-acquired pneumonia include initiating antibiotics prior to inpatient admission or within 6 hours of presentation to the ED. Timely collection of blood cultures, chest x-ray, and pulse oximetry are important as well but do not coincide with established goals.

8. A client in the intensive care unit is started on continuous venovenous hemofiltration (CVVH). Which finding is the cause of immediate action by the nurse? a. Blood pressure of 76/58 mm Hg b. Sodium level of 138 mEq/L c. Potassium level of 5.5 mEq/L d. Pulse rate of 90 beats/min

ANS: A Hypotension can be a problem with CVVH if replacement fluid does not provide enough volume to maintain blood pressure. The specially trained nurse needs to monitor for ongoing fluid and electrolyte replacement. The sodium level is normal and the potassium level is slightly elevated, which could be normal findings for someone with acute kidney injury. A pulse rate of 90 beats/min is normal.

A nurse assesses a client who is prescribed alosetron (Lotronex). Which assessment question should the nurse ask this client? a. "Have you been experiencing any constipation?" b. "Are you eating a diet high in fiber and fluids?" c. "Do you have a history of high blood pressure?" d. "What vitamins and supplements are you taking?"

ANS: A Ischemic colitis is a life-threatening complication of alosetron. The nurse should assess the client for constipation. The other questions do not identify complications related to alosetron.

24. A client has been brought to the emergency department for signs and symptoms of Chronic Obstructive Pulmonary Disease (COPD). The client has a history of a suicide attempt 1 year ago. Which nursing intervention would take priority in this situation? A. Assessing the clients pulse oximetry and vital signs B. Developing a plan for safety for the client C. Assessing the client for suicidal ideations D. Establishing a trusting nurse client relationship

ANS: A It is important to prioritize client interventions that assess the symptoms of COPD prior to any other nursing intervention. Physical needs must be prioritized according to Maslow's hierarchy of needs. This clients problems with oxygenation will take priority over assessing for current suicidal ideations.

11. A client is receiving oxygen at 4 liters per nasal cannula. What comfort measure may the nurse delegate to unlicensed assistive personnel (UAP)? a. Apply water-soluble ointment to nares and lips. b. Periodically turn the oxygen down or off. c. Remove the tubing from the clients nose. d. Turn the client every 2 hours or as needed.

ANS: A Oxygen can be drying, so the UAP can apply water-soluble lubricant to the clients lips and nares. The UAP should not adjust the oxygen flow rate or remove the tubing. Turning the client is not related to comfort measures for oxygen.

18. A client is prescribed alprazolam (Xanax) for acute anxiety. What client history should cause a nurse to question this order? A. History of alcohol use disorder B. History of personality disorder C. History of schizophrenia D. History of hypertension

ANS: A Rationale: The nurse should question a prescription of alprazolam for acute anxiety if the client has a history of alcohol use disorder. Alprazolam is a benzodiazepine used in the treatment of anxiety and has an increased risk for physiological dependence and tolerance. A client with a history of substance use disorder may be more likely to abuse other addictive substances. Cognitive Level: Application Integrated Process: Evaluation

15. A nurse cares for a client with a 40-year smoking history who is experiencing distended neck veins and dependent edema. Which physiologic process should the nurse correlate with this clients history and clinical manifestations? a. Increased pulmonary pressure creating a higher workload on the right side of the heart b. Exposure to irritants resulting in increased inflammation of the bronchi and bronchioles c. Increased number and size of mucus glands producing large amounts of thick mucus d. Left ventricular hypertrophy creating a decrease in cardiac output

ANS: A Smoking increases pulmonary hypertension, resulting in cor pulmonale, or right-sided heart failure. Increased pressures in the lungs make it more difficult for blood to flow through the lungs. Blood backs up into the right side of the heart and then into the peripheral venous system, creating distended neck veins and dependent edema. Inflammation in bronchi and bronchioles creates an airway obstruction which manifests as wheezes. Thick mucus in the lungs has no impact on distended neck veins and edema. Left ventricular hypertrophy is associated with left heart failure and is not caused by a 40-year smoking history.

A nurse cares for a client with a 40-year smoking history who is experiencing distended neck veins and dependent edema. Which physiologic process should the nurse correlate with this client's history and clinical manifestations? a. Increased pulmonary pressure creating a higher workload on the right side of the heart b. Exposure to irritants resulting in increased inflammation of the bronchi and bronchioles c. Increased number and size of mucus glands producing large amounts of thick mucus d. Left ventricular hypertrophy creating a decrease in cardiac output

ANS: A Smoking increases pulmonary hypertension, resulting in cor pulmonale, or right-sided heart failure. Increased pressures in the lungs make it more difficult for blood to flow through the lungs. Blood backs up into the right side of the heart and then into the peripheral venous system, creating distended neck veins and dependent edema. Inflammation in bronchi and bronchioles creates an airway obstruction which manifests as wheezes. Thick mucus in the lungs has no impact on distended neck veins and edema. Left ventricular hypertrophy is associated with left heart failure and is not caused by a 40-year smoking history.

25. A nurse auscultates a clients lung fields. Which action should the nurse take based on the lung sounds? (Click the media button to hear the audio clip.) a. Assess for airway obstruction. b. Initiate oxygen therapy. c. Assess vital signs. d. Elevate the clients head.

ANS: A Stridor is the sound heard, and it indicates severe airway constriction. The nurse must administer a bronchodilator to get air into the lungs. Administering oxygen, assessing vital signs, and elevating the clients head will not help until the clients airways are open.

7. A student is practicing suctioning a tracheostomy in the skills laboratory. What action by the student demonstrates that more teaching is needed? a. Applying suction while inserting the catheter b. Preoxygenating the client prior to suctioning c. Suctioning for a total of three times if needed d. Suctioning for only 10 to 15 seconds each time

ANS: A Suction should only be applied while withdrawing the catheter. The other actions are appropriate.

7. A nurse is assessing a client who is recovering from a lung biopsy. Which assessment finding requires immediate action? a. Increased temperature b. Absent breath sounds c. Productive cough d. Incisional discomfort

ANS: B Absent breath sounds may indicate that the client has a pneumothorax, a serious complication after a needle biopsy or open lung biopsy. The other manifestations are not life threatening.

6.A client admitted for pneumonia has been tachypneic for several days. When the nurse starts an IV to give fluids, the client questions this action, saying I have been drinking tons of water. How am I dehydrated? What response by the nurse is best? a. Breathing so quickly can be dehydrating. b. Everyone with pneumonia is dehydrated. c. This is really just to administer your antibiotics. d. Why do you think you are so dehydrated?

ANS: A Tachypnea and mouth breathing, both seen in pneumonia, increase insensible water loss and can lead to a degree of dehydration. The other options do not give the client useful information.

A client admitted for pneumonia has been tachypneic for several days. When the nurse starts an IV to give fluids, the client questions this action, saying "I have been drinking tons of water. How am I dehydrated?" What response by the nurse is best? a. "Breathing so quickly can be dehydrating." b. "Everyone with pneumonia is dehydrated." c. "This is really just to administer your antibiotics." d. "Why do you think you are so dehydrated?"

ANS: A Tachypnea and mouth breathing, both seen in pneumonia, increase insensible water loss and can lead to a degree of dehydration. The other options do not give the client useful information.

11. A nurse assesses a client who is prescribed fluticasone (Flovent) and notes oral lesions. Which action should the nurse take? a. Encourage oral rinsing after fluticasone administration. b. Obtain an oral specimen for culture and sensitivity. c. Start the client on a broad-spectrum antibiotic. d. Document the finding as a known side effect.

ANS: A The drug reduces local immunity and increases the risk for local infection, especially Candida albicans. Rinsing the mouth after using the inhaler will decrease the risk for developing this infection. Use of mouthwash and broad-spectrum antibiotics is not warranted in this situation. The nurse should document the finding, but the best action to take is to have the client start rinsing his or her mouth after using fluticasone. An oral specimen for culture and sensitivity will not provide information necessary to care for this client.

A nurse assesses a client who is prescribed fluticasone (Flovent) and notes oral lesions. Which action should the nurse take? a. Encourage oral rinsing after fluticasone administration. b. Obtain an oral specimen for culture and sensitivity. c. Start the client on a broad-spectrum antibiotic. d. Document the finding as a known side effect.

ANS: A The drug reduces local immunity and increases the risk for local infection, especially Candida albicans. Rinsing the mouth after using the inhaler will decrease the risk for developing this infection. Use of mouthwash and broad-spectrum antibiotics is not warranted in this situation. The nurse should document the finding, but the best action to take is to have the client start rinsing his or her mouth after using fluticasone. An oral specimen for culture and sensitivity will not provide information necessary to care for this client.

10. After assertiveness training, a formerly passive client appropriately confronts a peer in group therapy. The group leader states, Im so proud of you for being assertive. You are so good! Which communication technique has the leader employed? A. The nontherapeutic technique of giving approval B. The nontherapeutic technique of interpreting C. The therapeutic technique of presenting reality D. The therapeutic technique of making observations

ANS: A The group leader has employed the nontherapeutic technique of giving approval. Giving approval implies that the nurse has the right to pass judgment on whether the clients ideas or behaviors are good or bad. This creates a conditional acceptance of the client. KEY: Cognitive Level: Application | Integrated Processes: Nursing Process: Implementation | Client Need: Psychosocial Integrity

After assertiveness training, a formerly passive client appropriately confronts a peer in group therapy. The group leader states, Im so proud of you for being assertive. You are so good! Which communication technique has the leader employed? A. The nontherapeutic technique of giving approval B. The nontherapeutic technique of interpreting C. The therapeutic technique of presenting reality D. The therapeutic technique of making observations

ANS: A The group leader has employed the nontherapeutic technique of giving approval. Giving approval implies that the nurse has the right to pass judgment on whether the clients ideas or behaviors are good or bad. This creates a conditional acceptance of the client.

11. A male client with chronic kidney disease (CKD) is refusing to take his medication and has missed two hemodialysis appointments. What is the best initial action for the nurse? a. Discuss what the treatment regimen means to him. b. Refer the client to a mental health nurse practitioner. c. Reschedule the appointments to another date and time. d. Discuss the option of peritoneal dialysis.

ANS: A The initial action for the nurse is to assess anxiety, coping styles, and the client's acceptance of the required treatment for CKD. The client may be in denial of the diagnosis. While rescheduling hemodialysis appointments may help, and referral to a mental health practitioner and the possibility of peritoneal dialysis are all viable options, assessment of the client's acceptance of the treatment should come first.

Chapter 07. Relationship Development Multiple Choice 1. What is the most essential task for a nurse to accomplish prior to forming a therapeutic relationship with a client? A. To clarify personal attitudes, values, and beliefs B. To obtain thorough assessment data C. To determine the clients length of stay D. To establish personal goals for the interaction

ANS: A The most essential task for a nurse to accomplish prior to forming a therapeutic relationship with a client is to clarify personal attitudes, values, and beliefs. Understanding ones own attitudes, values, and beliefs is called self-awareness. KEY: Cognitive Level: Comprehension | Integrated Processes: Nursing Process: Implementation | Client Need: Psychosocial Integrity

7. A nurse is assessing a client diagnosed with schizophrenia for the presence of hallucinations. Which therapeutic communication technique used by the nurse is an example of making observations? A. You appear to be talking to someone I do not see. B. Please describe what you are seeing. C. Why do you continually look in the corner of this room? D. If you hum a tune, the voices may not be so distracting.

ANS: A The nurse is making an observation when stating, You appear to be talking to someone I do not see. Making observations involves verbalizing what is observed or perceived. This encourages the client to recognize specific behaviors and make comparisons with the nurses perceptions. KEY: Cognitive Level: Application | Integrated Processes: Nursing Process: Implementation | Client Need: Psychosocial Integrity

A nurse is assessing a client diagnosed with schizophrenia for the presence of hallucinations. Which therapeutic communication technique used by the nurse is an example of making observations? A. You appear to be talking to someone I do not see. B. Please describe what you are seeing. C. Why do you continually look in the corner of this room? D. If you hum a tune, the voices may not be so distracting.

ANS: A The nurse is making an observation when stating, You appear to be talking to someone I do not see. Making observations involves verbalizing what is observed or perceived. This encourages the client to recognize specific behaviors and make comparisons with the nurses perceptions.

2. Which therapeutic communication technique is being used in this nurseclient interaction? Client: My father spanked me often. Nurse: Your father was a harsh disciplinarian. A. Restatement B. Offering general leads C. Focusing D. Accepting

ANS: A The nurse is using the therapeutic communication technique of restatement. Restatement involves repeating the main idea of what the client has said. The nurse uses this technique to communicate that the clients statement has been heard and understood. KEY: Cognitive Level: Application | Integrated Processes: Nursing Process: Evaluation | Client Need: Psychosocial Integrity

15.A client has been hospitalized with tuberculosis (TB). The clients spouse is fearful of entering the room where the client is in isolation and refuses to visit. What action by the nurse is best? a. Ask the spouse to explain the fear of visiting in further detail. b. Inform the spouse the precautions are meant to keep other clients safe. c. Show the spouse how to follow the isolation precautions to avoid illness. d. Tell the spouse that he or she has already been exposed, so its safe to visit.

ANS: A The nurse needs to obtain further information about the spouses specific fears so they can be addressed. This will decrease stress and permit visitation, which will be beneficial for both client and spouse. Precautions for TB prevent transmission to all who come into contact with the client. Explaining isolation precautions and what to do when entering the room will be helpful, but this is too narrow in scope to be the best answer. Telling the spouse its safe to visit is demeaning of the spouses feelings.

A nurse cares for a client who has a new colostomy. Which action should the nurse take? a. Empty the pouch frequently to remove excess gas collection. b. Change the ostomy pouch and wafer every morning. c. Allow the pouch to completely fill with stool prior to emptying it. d. Use surgical tape to secure the pouch and prevent leakage.

ANS: A The nurse should empty the new ostomy pouch frequently because of excess gas collection, and empty the pouch when it is one-third to one-half full of stool. The ostomy pouch does not need to be changed every morning. Ostomy wafers with paste should be used to secure and seal the ostomy appliance; surgical tape should not be used.

5. Which client response should a nurse expect during the working phase of the nurseclient relationship? A. The client gains insight and incorporates alternative behaviors. B. The client and nurse establish rapport and mutually develop treatment goals. C. The client explores feelings related to reentering the community. D. The client explores personal strengths and weaknesses that impact behaviors.

ANS: A The nurse should expect that the client would gain insight and incorporate alternative behaviors during the working phase of the nurseclient relationship. The client may also overcome resistance, problem-solve, and continually evaluate progress toward goals. KEY: Cognitive Level: Application | Integrated Processes: Nursing Process: Implementation | Client Need: Psychosocial Integrity

9. A client has made the decision to leave her alcoholic husband. She is feeling very depressed. Which nontherapeutic statement by the nurse conveys sympathy? A. You are feeling very depressed. I felt the same way when I decided to leave my husband. B. I can understand you are feeling depressed. It was a difficult decision. Ill sit with you. C. You seem depressed. It was a difficult decision to make. Would you like to talk about it? D. I know this is a difficult time for you. Would you like a prn medication for anxiety?

ANS: A The nurses statement, You are feeling very depressed. I felt the same when I decided to leave my husband, is a nontherapeutic statement that conveys sympathy. Sympathy implies that the nurse shares what the client is feeling and by this personal expression alleviates the clients distress. KEY: Cognitive Level: Application | Integrated Processes: Nursing Process: Implementation | Client Need: Psychosocial Integrity

A client has made the decision to leave her alcoholic husband. She is feeling very depressed. Which nontherapeutic statement by the nurse conveys sympathy? A. You are feeling very depressed. I felt the same way when I decided to leave my husband. B. I can understand you are feeling depressed. It was a difficult decision. Ill sit with you. C. You seem depressed. It was a difficult decision to make. Would you like to talk about it? D. I know this is a difficult time for you. Would you like a prn medication for anxiety?

ANS: A The nurses statement, You are feeling very depressed. I felt the same when I decided to leave my husband, is a nontherapeutic statement that conveys sympathy. Sympathy implies that the nurse shares what the client is feeling and by this personal expression alleviates the clients distress.

An adult attempts suicide after declaring bankruptcy. The patient is hospitalized and takes an antidepressant medication for five days. The patient is now more talkative and shows increased energy. Select the highest priority nursing intervention. a. Supervise the patient 24 hours a day. b. Begin discharge planning for the patient. c. Refer the patient to art and music therapists. d. Consider the discontinuation of suicide precautions.

ANS: A The patient now has more energy and may have decided on suicide, especially considering the history of the prior suicide attempt. The patient is still a suicide risk; therefore, continuous supervision is indicated.

6. A client has a serum potassium level of 6.5 mmol/L, a serum creatinine level of 2 mg/dL, and a urine output of 350 mL/day. What is the best action by the nurse? a. Place the client on a cardiac monitor immediately. b. Teach the client to limit high-potassium foods. c. Continue to monitor the client's intake and output. d. Ask to have the laboratory redraw the blood specimen.

ANS: A The priority action by the nurse should be to check the cardiac status with a monitor. High potassium levels can lead to dysrhythmias. The other choices are logical nursing interventions for acute kidney injury but not the best immediate action.

The cystoscopy results for a patient include a small-capacity bladder, the presence of Hunner's ulcers, and small hemorrhages after bladder distention. How does the nurse interpret this report? a. Urosepsis b. Complicated cystitis c. Interstitial cystitis d. Urethritis

C - Interstitial cystitis

5. An unlicensed assistive personnel (UAP) was feeding a client with a tracheostomy. Later that evening, the UAP reports that the client had a coughing spell during the meal. What action by the nurse takes priority? a. Assess the clients lung sounds. b. Assign a different UAP to the client. c. Report the UAP to the manager. d. Request thicker liquids for meals.

ANS: A The priority is to check the clients oxygenation because he or she may have aspirated. Once the client has been assessed, the nurse can consult with the registered dietitian about appropriately thickened liquids. The UAP should have reported the incident immediately, but addressing that issue is not the immediate priority.

26. The nurse instructs a client on the steps needed to obtain a peak expiratory flow rate. In which order should these steps occur? 1. Take as deep a breath as possible. 2. Stand up (unless you have a physical disability). 3. Place the meter in your mouth, and close your lips around the mouthpiece. 4. Make sure the device reads zero or is at base level. 5. Blow out as hard and as fast as possible for 1 to 2 seconds. 6. Write down the value obtained. 7. Repeat the process two additional times, and record the highest number in your chart. a. 4, 2, 1, 3, 5, 6, 7 b. 3, 4, 1, 2, 5, 7, 6 c. 2, 1, 3, 4, 5, 6, 7 d. 1, 3, 2, 5, 6, 7, 4

ANS: A The proper order for obtaining a peak expiratory flow rate is as follows. Make sure the device reads zero or is at base level. The client should stand up (unless he or she has a physical disability). The client should take as deep a breath as possible, place the meter in the mouth, and close the lips around the mouthpiece. The client should blow out as hard and as fast as possible for 1 to 2 seconds. The value obtained should be written down. The process should be repeated two more times, and the highest of the three numbers should be recorded in the clients chart.

The nurse instructs a client on the steps needed to obtain a peak expiratory flow rate. In which order should these steps occur? 1. "Take as deep a breath as possible." 2. "Stand up (unless you have a physical disability)." 3. "Place the meter in your mouth, and close your lips around the mouthpiece." 4. "Make sure the device reads zero or is at base level." 5. "Blow out as hard and as fast as possible for 1 to 2 seconds." 6. "Write down the value obtained." 7. "Repeat the process two additional times, and record the highest number in your chart." a. 4, 2, 1, 3, 5, 6, 7 b. 3, 4, 1, 2, 5, 7, 6 c. 2, 1, 3, 4, 5, 6, 7 d. 1, 3, 2, 5, 6, 7, 4

ANS: A The proper order for obtaining a peak expiratory flow rate is as follows. Make sure the device reads zero or is at base level. The client should stand up (unless he or she has a physical disability). The client should take as deep a breath as possible, place the meter in the mouth, and close the lips around the mouthpiece. The client should blow out as hard and as fast as possible for 1 to 2 seconds. The value obtained should be written down. The process should be repeated two more times, and the highest of the three numbers should be recorded in the client's chart.

19. Which nursing intervention strategy is most appropriate to implement initially with a suicidal client? A. Ask a direct question such as, Do you ever think about killing yourself? B. Ask client, Please rate your mood on a scale from 1 to 10. C. Establish a trusting nurse client relationship. D. Apply the nursing process to the planning of client care.

ANS: A The risk of suicide is greatly increased if the client has suicidal ideations, if the client has developed a plan, and particularly if the means exist for the client to execute the plan.

A nurse cares for a client who had a colostomy placed in the ascending colon 2 weeks ago. The client states, "The stool in my pouch is still liquid." How should the nurse respond? a. "The stool will always be liquid with this type of colostomy." b. "Eating additional fiber will bulk up your stool and decrease diarrhea." c. "Your stool will become firmer over the next couple of weeks." d. "This is abnormal. I will contact your health care provider."

ANS: A The stool from an ascending colostomy can be expected to remain liquid because little large bowel is available to reabsorb the liquid from the stool. This finding is not abnormal. Liquid stool from an ascending colostomy will not become firmer with the addition of fiber to the client's diet or with the passage of time.

8.A client has been diagnosed with tuberculosis (TB). What action by the nurse takes highest priority? a. Educating the client on adherence to the treatment regimen b. Encouraging the client to eat a well-balanced diet c. Informing the client about follow-up sputum cultures d. Teaching the client ways to balance rest with activity

ANS: A The treatment regimen for TB ranges from 6 to 12 months, making adherence problematic for many people. The nurse should stress the absolute importance of following the treatment plan for the entire duration of prescribed therapy. The other options are appropriate topics to educate this client on but do not take priority.

3. A male client comes into the emergency department with a serum creatinine of 2.2 mg/dL and a blood urea nitrogen (BUN) of 24 mL/dL. What question should the nurse ask first when taking this client's history? a. "Have you been taking any aspirin, ibuprofen, or naproxen recently?" b. "Do you have anyone in your family with renal failure?" c. "Have you had a diet that is low in protein recently?" d. "Has a relative had a kidney transplant lately?"

ANS: A There are some medications that are nephrotoxic, such as the nonsteroidal anti-inflammatory drugs ibuprofen, aspirin, and naproxen. This would be a good question to initially ask the client since both the serum creatinine and BUN are elevated, indicating some renal problems. A family history of renal failure and kidney transplantation would not be part of the questioning and could cause anxiety in the client. A diet high in protein could be a factor in an increased BUN.

2. A marathon runner comes into the clinic and states "I have not urinated very much in the last few days." The nurse notes a heart rate of 110 beats/min and a blood pressure of 86/58 mm Hg. Which action by the nurse is the priority? a. Give the client a bottle of water immediately. b. Start an intravenous line for fluids. c. Teach the client to drink 2 to 3 liters of water daily. d. Perform an electrocardiogram.

ANS: A This athlete is mildly dehydrated as evidenced by the higher heart rate and lower blood pressure. The nurse can start hydrating the client with a bottle of water first, followed by teaching the client to drink 2 to 3 liters of water each day. An intravenous line may be ordered later, after the client's degree of dehydration is assessed. An electrocardiogram is not necessary at this time.

3. A client has a tracheostomy that is 3 days old. Upon assessment, the nurse notes the clients face is puffy and the eyelids are swollen. What action by the nurse takes priority? a. Assess the clients oxygen saturation. b. Notify the Rapid Response Team. c. Oxygenate the client with a bag-valve-mask. d. Palpate the skin of the upper chest.

ANS: A This client may have subcutaneous emphysema, which is air that leaks into the tissues surrounding the tracheostomy. The nurse should first assess the clients oxygen saturation and other indicators of oxygenation. If the client is stable, the nurse can palpate the skin of the upper chest to feel for the air. If the client is unstable, the nurse calls the Rapid Response Team. Using a bag-valve-mask device may or may not be appropriate for the unstable client.

17. Which example of a therapeutic communication technique would be effective in the planning phase of the nursing process? A. Weve discussed past coping skills. Lets see if these coping skills can be effective now. B. Please tell me in your own words what brought you to the hospital. C. This new approach worked for you. Keep it up. D. I notice that you seem to be responding to voices that I do not hear.

ANS: A This is an example of the therapeutic communication technique of formulating a plan of action. By the use of this technique the nurse can help the client plan in advance to deal with a stressful situation, which may prevent anger and/or anxiety from escalating to an unmanageable level. KEY: Cognitive Level: Application | Integrated Process: Nursing Process: Implementation | Client Need: Psychosocial Integrity

Which example of a therapeutic communication technique would be effective in the planning phase of the nursing process? A. We've discussed past coping skills. Lets see if these coping skills can be effective now. B. Please tell me in your own words what brought you to the hospital. C. This new approach worked for you. Keep it up. D. I notice that you seem to be responding to voices that I do not hear.

ANS: A This is an example of the therapeutic communication technique of formulating a plan of action. By the use of this technique the nurse can help the client plan in advance to deal with a stressful situation, which may prevent anger and/or anxiety from escalating to an unmanageable level.

21. A client is trying to explore and solve a problem. Which nursing statement would be an example of verbalizing the implied? A. You seem to be motivated to change your behavior. B. How will these changes affect your family relationships? C. Why dont you make a list of the behaviors you need to change. D. The team recommends that you make only one behavioral change at a time.

ANS: A This is an example of the therapeutic communication technique of verbalizing the implied. Verbalizing the implied puts into words what the client has only implied or said indirectly. KEY: Cognitive Level: Application | Integrated Processes: Nursing Process: Implementation | Client Need: Psychosocial Integrity

A client is trying to explore and solve a problem. Which nursing statement would be an example of verbalizing the implied? A. You seem to be motivated to change your behavior. B. How will these changes affect your family relationships? C. Why dont you make a list of the behaviors you need to change. D. The team recommends that you make only one behavioral change at a time.

ANS: A This is an example of the therapeutic communication technique of verbalizing the implied. Verbalizing the implied puts into words what the client has only implied or said indirectly.

27. During a nurseclient interaction, which nursing statement may belittle the clients feelings and concerns? A. Dont worry. Everything will be alright. B. You appear uptight. C. I notice you have bitten your nails to the quick. D. You are jumping to conclusions.

ANS: A This nursing statement is an example of the nontherapeutic communication block of belittling feelings. Belittling feelings occurs when the nurse misjudges the degree of the clients discomfort, suggesting a lack of empathy and understanding. KEY: Cognitive Level: Application | Integrated Processes: Nursing Process: Implementation | Client Need: Psychosocial Integrity

During a nurse client interaction, which nursing statement may belittle the client's feelings and concerns? A. Don't worry. Everything will be alright. B. You appear uptight. C. I notice you have bitten your nails to the quick. D. You are jumping to conclusions.

ANS: A This nursing statement is an example of the nontherapeutic communication block of belittling feelings. Belittling feelings occurs when the nurse misjudges the degree of the clients discomfort, suggesting a lack of empathy and understanding.

29. Which nursing response is an example of the nontherapeutic communication block of requesting an explanation? A. Can you tell me why you said that? B. Keep your chin up. Ill explain the procedure to you. C. There is always an explanation for both good and bad behaviors. D. Are you not understanding the explanation I provided?

ANS: A This nursing statement is an example of the nontherapeutic communication block of requesting an explanation. Requesting an explanation is when the client is asked to provide the reason for thoughts, feelings, behaviors, and events. Asking why a client did something or feels a certain way can be very intimidating and implies that the client must defend his or her behavior or feelings. KEY: Cognitive Level: Application | Integrated Processes: Nursing Process: Implementation | Client Need: Psychosocial Integrity

The nurse is caring for an older adult client who experiences an exacerbation of ulcerative colitis with severe diarrhea that have lasted a week. For which complications will the nurse assess? (Select all that apply.) a. Dehydration b. Hypokalemia c. Skin breakdown d. Deep vein thrombus e. Hyperkalemia

ANS: A, B, C The nurse will assess for complications such as dehydration, hypokalemia, and skin breakdown, all which can occur when diarrhea is profuse and fluid has been lost. Deep vein thrombosis and hyperkalemia are not complications that are associated with ulcerative colitis with severe diarrhea.

Which nursing response is an example of the nontherapeutic communication block of requesting an explanation? A. Can you tell me why you said that? B. Keep your chin up. I'll explain the procedure to you. C. There is always an explanation for both good and bad behaviors. D. Are you not understanding the explanation I provided?

ANS: A This nursing statement is an example of the nontherapeutic communication block of requesting an explanation. Requesting an explanation is when the client is asked to provide the reason for thoughts, feelings, behaviors, and events. Asking why a client did something or feels a certain way can be very intimidating and implies that the client must defend his or her behavior or feelings.

24. A student nurse is learning about the appropriate use of touch when communicating with clients diagnosed with psychiatric disorders. Which statement by the instructor best provides information about this aspect of therapeutic communication? A. Touch carries a different meaning for different individuals. B. Touch is often used when deescalating volatile client situations. C. Touch is used to convey interest and warmth. D. Touch is best combined with empathy when dealing with anxious clients.

ANS: A Touch can elicit both negative and positive reactions, depending on the people involved and the circumstances of the interaction. KEY: Cognitive Level: Application | Integrated Processes: Nursing Process: Implementation | Client Need: Psychosocial Integrity

A student nurse is learning about the appropriate use of touch when communicating with clients diagnosed with psychiatric disorders. Which statement by the instructor best provides information about this aspect of therapeutic communication? A. Touch carries a different meaning for different individuals. B. Touch is often used when deescalating volatile client situations. C. Touch is used to convey interest and warmth. D. Touch is best combined with empathy when dealing with anxious clients.

ANS: A Touch can elicit both negative and positive reactions, depending on the people involved and the circumstances of the interaction.

19. Which client statement may indicate a transference reaction? A. I need a real nurse. You are young enough to be my daughter and I dont want to tell you about my personal life. B. I deserve more than I am getting here. Do you know who I am and what I do? Let me talk to your supervisor. C. I dont seem to be able to relate to people. I would rather stay in my room and be by myself. D. My mother is the source of my problems. She has always told me what to do and what to say.

ANS: A Transference occurs when a client unconsciously displaces or transfers to the nurse feelings formed toward a person from the past. KEY: Cognitive Level: Application | Integrated Processes: Nursing Process: Assessment | Client Need: Psychosocial Integrity

Which client statement may indicate a transference reaction? A. I need a real nurse. You are young enough to be my daughter and I don't want to tell you about my personal life. B. I deserve more than I am getting here. Do you know who I am and what I do? Let me talk to your supervisor. C. I don't seem to be able to relate to people. I would rather stay in my room and be by myself. D. My mother is the source of my problems. She has always told me what to do and what to say.

ANS: A Transference occurs when a client unconsciously displaces or transfers to the nurse feelings formed toward a person from the past.

A 32-year-old morbidly obese male complains of excessive fatigue, snoring, and awakening in the middle of the night, which prevents restorative sleep. He is sluggish during the day due to the lack of sleep and feels like he is going "fall asleep at the wheel" when driving to work. Occupation: dishwasher. Medical history includes hypertension and type 2 diabetes. Current medications include ACE inhibitor and metformin. Denies use of alcohol, tobacco, or drugs. On physical examination, the patient is afebrile, pulse 88, resps 20/min, BP 178/95. BMI is 45. These are signs and symptoms of: A. Obstructive sleep apnea B. Primary insomnia C. Heart failure D. All of the above

ANS: A With the obesity epidemic, the incidence of obstructive sleep apnea (OSA) is also increasing. Besides obesity, other predisposing and risk factors include narrowed upper airways, macroglossia, tonsillar hypertrophy, sleep medicines, alcohol, smoking, nasal obstruction, and hypothyroidism. It occurs more in middle-aged men. A thorough medication history, respiratory history, neurologic history, and mental health assessment should be performed. One of the more common symptoms of OSA is snoring, usually reported by the spouse. The patient may complain of frequent nighttime awakening, morning drowsiness, headache (caused by carbon dioxide buildup in the brain), cognitive impairment, as well as impotence and weight gain, which can be both a cause and an effect. Systemic hypertension is a complication of OSA but often resolves when the cause of the apnea is corrected.

The client scheduled for intravenous urography informs the nurse of the following allergies. Which one should the nurse report to the physician immediately? A. Seafood B. Penicillin C. Bee stings D. Red food dye

ANS: A Clients with seafood allergies often have severe allergic reactions to the standard dyes used during intravenous urography.

The client scheduled to have an intravenous urogram is a diabetic and taking the antidiabetic agent metformin. What should the nurse tell this client? A. "Call your diabetes doctor and tell him or her that you are having an intravenous urogram performed using dye." B. "Do not take your metformin the morning of the test because you are not going to be eating anything and could become hypoglycemic." C. "You must start on an antibiotic before this test because your risk of infection is greater as a result of your diabetes." D. "You must take your metformin immediately before the test is performed because the IV fluid and the dye contain a significant amount of sugar."

ANS: A Metformin can cause a lactic acidosis and renal impairment as an interaction with the dye. This drug must be discontinued for 48 hours before the procedure and not started again after the procedure until urine output is well established.

What would be the response if a person's nephrons were not able to filter normally due to scarring of the proximal convoluted tubule leading to inhibition of reabsorption? A. Increased urine output, fluid volume deficit B. Decreased urine output, fluid volume deficit C. Increased urine output, fluid volume overload D. Decreased urine output, fluid volume overload

ANS: A The nephrons filter about 120 mL/min. Most of this filtrate is reabsorbed in the proximal convoluted tubule. If the tubule were not able to reabsorb the fluid that has been filtered, urine output would greatly increase, leading to rapid and severe dehydration.

What are the possible implications of the patient's consumption of soda prior to surgery?

ANS: A patient's stomach needs to be empty prior to receiving anesthesia and undergoing surgery to decrease the chance of aspiration.

An experienced nurse says to a new graduate, When youve practiced as long as I have, you instantly know how to take care of psychotic patients. What information should the new graduate consider when analyzing this comment? Select all that apply. a. The experienced nurse may have lost sight of patients individuality, which may compromise the integrity of practice. b. New research findings should be integrated continuously into a nurses practice to provide the most effective care. c. Experience provides mental health nurses with the essential tools and skills needed for effective professional practice. d. Experienced psychiatric nurses have learned the best ways to care for mentally ill patients through trial and error. e. An intuitive sense of patients needs guides effective psychiatric nurses.

ANS: A, B Evidence-based practice involves using research findings and standards of care to provide the most effective nursing care. Evidence is continuously emerging, so nurses cannot rely solely on experience. The effective nurse also maintains respect for each patient as an individual. Overgeneralization compromises that perspective. Intuition and trial and error are unsystematic approaches to care.

A client enters the clinic with an acute sore throat and a temperature of 101.5° F (38.5° C). What diagnostic testing does the nurse educate the client about? (Select all that apply.) a. Complete blood count (CBC) b. Throat culture c. Monospot test d. Arterial blood gas e. Biopsy f. HIV testing

ANS: A, B, C CBC, throat culture, and monospot testing can help to determine the causes of sore throat and fever. A biopsy is not needed. Human immune deficiency virus (HIV) testing would not be indicated unless the symptoms were a recurrent problem. Arterial blood gases would not be performed unless the client had dyspnea and a low oxygen saturation reading.

2.A hospital nurse is participating in a drill during which many clients with inhalation anthrax are being admitted. What drugs should the nurse anticipate administering? (Select all that apply.) a. Amoxicillin (Amoxil) b. Ciprofloxacin (Cipro) c. Doxycycline (Vibramycin) d. Ethambutol (Myambutol) e. Sulfamethoxazole-trimethoprim (SMX-TMP) (Septra)

ANS: A, B, C Amoxicillin, ciprofloxacin, and doxycycline are all possible treatments for inhalation anthrax. Ethambutol is used for tuberculosis. SMX-TMP is commonly used for urinary tract infections and other common infections.

3. A nurse teaches a client who has chronic obstructive pulmonary disease. Which statements related to nutrition should the nurse include in this clients teaching? (Select all that apply.) a. Avoid drinking fluids just before and during meals. b. Rest before meals if you have dyspnea. c. Have about six small meals a day. d. Eat high-fiber foods to promote gastric emptying. e. Increase carbohydrate intake for energy.

ANS: A, B, C Fluids can make a client feel bloated and should be avoided with meals. Resting before the meal will help a client with dyspnea. Six small meals a day also will help to decrease bloating. Fibrous foods can produce gas, which can cause abdominal bloating and can increase shortness of breath. The client should increase calorie and protein intake to prevent malnourishment. The client should not increase carbohydrate intake as this will increase carbon dioxide production and increase the clients risk of for acidosis.

A nurse teaches a client who has chronic obstructive pulmonary disease. Which statements related to nutrition should the nurse include in this client's teaching? (Select all that apply.) a. "Avoid drinking fluids just before and during meals." b. "Rest before meals if you have dyspnea." c. "Have about six small meals a day." d. "Eat high-fiber foods to promote gastric emptying." e. "Increase carbohydrate intake for energy."

ANS: A, B, C Fluids can make a client feel bloated and should be avoided with meals. Resting before the meal will help a client with dyspnea. Six small meals a day also will help to decrease bloating. Fibrous foods can produce gas, which can cause abdominal bloating and can increase shortness of breath. The client should increase calorie and protein intake to prevent malnourishment. The client should not increase carbohydrate intake as this will increase carbon dioxide production and increase the client's risk of for acidosis.

1. A home health nurse is visiting a new client who uses oxygen in the home. For which factors does the nurse assess when determining if the client is using the oxygen safely? (Select all that apply.) a. The client does not allow smoking in the house. b. Electrical cords are in good working order. c. Flammable liquids are stored in the garage. d. Household light bulbs are the fluorescent type. e. The client does not have pets inside the home.

ANS: A, B, C Oxygen is an accelerant, which means it enhances combustion, so precautions are needed whenever using it. The nurse should assess if the client allows smoking near the oxygen, whether electrical cords are in good shape or are frayed, and if flammable liquids are stored (and used) in the garage away from the oxygen. Light bulbs and pets are not related to oxygen safety.

25. After a teenager reveals that he is gay, the father responds by beating him. The next morning, the teenager is found hanging in his closet. Which paternal grief responses should a nurse anticipate? Select all that apply. A. I cant believe this is happening. B. If only I had been more understanding. C. How dare he do this to me! D. I'm just going to have to accept that he was gay. E. Well, that was a selfish thing to do.

ANS: A, B, C Suicide of a family member can induce a whole gamut of feelings in the survivors. Shock, disbelief, guilt, remorse, anger, and resentment are all feelings that may be experienced by this father. The last two possible responses suggest acceptance and understanding. It is far more common for survivors of suicide to have a sense of feeling wounded and as if they will never get over it.

1. The nurse is caring for five clients on the medical-surgical unit. Which clients would the nurse consider to be at risk for post-renal acute kidney injury (AKI)? (Select all that apply.) a. Man with prostate cancer b. Woman with blood clots in the urinary tract c. Client with ureterolithiasis d. Firefighter with severe burns e. Young woman with lupus

ANS: A, B, C Urine flow obstruction, such as prostate cancer, blood clots in the urinary tract, and kidney stones (ureterolithiasis), causes post-renal AKI. Severe burns would be a pre-renal cause. Lupus would be an intrarenal cause for AKI.

4.A client has been diagnosed with an empyema. What interventions should the nurse anticipate providing to this client? (Select all that apply.) a. Assisting with chest tube insertion b. Facilitating pleural fluid sampling c. Performing frequent respiratory assessment d. Providing antipyretics as needed e. Suctioning deeply every 4 hours

ANS: A, B, C, D The client with an empyema is often treated with chest tube insertion, which facilitates obtaining samples of the pleural fluid for analysis and re-expands the lungs. The nurse should perform frequent respiratory system assessments. Antipyretic medications are also used. Suction is only used when needed and is not done deeply to prevent tissue injury.

A client has been diagnosed with an empyema. What interventions should the nurse anticipate providing to this client? (Select all that apply.) a. Assisting with chest tube insertion b. Facilitating pleural fluid sampling c. Performing frequent respiratory assessment d. Providing antipyretics as needed e. Suctioning deeply every 4 hours

ANS: A, B, C, D The client with an empyema is often treated with chest tube insertion, which facilitates obtaining samples of the pleural fluid for analysis and re-expands the lungs. The nurse should perform frequent respiratory system assessments. Antipyretic medications are also used. Suction is only used when needed and is not done deeply to prevent tissue injury.

34. Which of the following individuals are communicating a message? Select all that apply. A. A mother spanking her son for playing with matches B. A teenage boy isolating himself and playing loud music C. A biker sporting an eagle tattoo on his biceps D. A teenage girl writing, No one understands me E. A father checking for new e-mail on a regular basis

ANS: A, B, C, D The nurse should determine that spanking, isolating, getting tattoos, and writing are all ways in which people communicate messages to others. It is estimated that about 70% to 90% of communication is nonverbal. KEY: Cognitive Level: Application | Integrated Processes: Nursing Process: Assessment | Client Need: Psychosocial Integrity

5. A nurse is teaching a client about possible complications and hazards of home oxygen therapy. About which complications does the nurse plan to teach the client? (Select all that apply.) a. Absorptive atelectasis b. Combustion c. Dried mucous membranes d. Oxygen-induced hyperventilation e. Toxicity

ANS: A, B, C, E Complications of oxygen therapy include absorptive atelectasis, combustion, dried mucous membranes, and oxygen toxicity. Oxygen-induced hypoventilation is also a complication.

5. A nurse teaches a client with polycystic kidney disease (PKD). Which statements should the nurse include in this client's discharge teaching? (Select all that apply.) a. "Take your blood pressure every morning." b. "Weigh yourself at the same time each day." c. "Adjust your diet to prevent diarrhea." d. "Contact your provider if you have visual disturbances." e. "Assess your urine for renal stones."

ANS: A, B, D A client who has PKD should measure and record his or her blood pressure and weight daily, limit salt intake, and adjust dietary selections to prevent constipation. The client should notify the provider if urine smells foul or has blood in it, as these are signs of a urinary tract infection or glomerular injury. The client should also notify the provider if visual disturbances are experienced, as this is a sign of a possible berry aneurysm, which is a complication of PKD. Diarrhea and renal stones are not manifestations or complications of PKD; therefore, teaching related to these concepts would be inappropriate.

6. A client is undergoing hemodialysis. The client's blood pressure at the beginning of the procedure was 136/88 mm Hg, and now it is 110/54 mm Hg. What actions should the nurse perform to maintain blood pressure? (Select all that apply.) a. Adjust the rate of extracorporeal blood flow. b. Place the client in the Trendelenburg position. c. Stop the hemodialysis treatment. d. Administer a 250-mL bolus of normal saline. e. Contact the health care provider for orders.

ANS: A, B, D Hypotension occurs often during hemodialysis treatments as a result of vasodilation from the warmed dialysate. Modest decreases in blood pressure, as is the case with this client, can be maintained with rate adjustment, Trendelenburg positioning, and a fluid bolus. If the blood pressure drops considerably after two boluses and cooling dialysate, the hemodialysis can be stopped and the health care provider contacted.

6. A nurse plans care for a client who has chronic obstructive pulmonary disease and thick, tenacious secretions. Which interventions should the nurse include in this clients plan of care? (Select all that apply.) a. Ask the client to drink 2 liters of fluids daily. b. Add humidity to the prescribed oxygen. c. Suction the client every 2 to 3 hours. d. Use a vibrating positive expiratory pressure device. e. Encourage diaphragmatic breathing.

ANS: A, B, D Interventions to decrease thick tenacious secretions include maintaining adequate hydration and providing humidified oxygen. These actions will help to thin secretions, making them easier to remove by coughing. The use of a vibrating positive expiratory pressure device can also help clients remove thick secretions. Although suctioning may assist with the removal of secretions, frequent suctioning can cause airway trauma and does not support the clients ability to successfully remove secretions through normal coughing. Diaphragmatic breathing is not used to improve the removal of thick secretions.

A nurse plans care for a client who has chronic obstructive pulmonary disease and thick, tenacious secretions. Which interventions should the nurse include in this client's plan of care? (Select all that apply.) a. Ask the client to drink 2 liters of fluids daily. b. Add humidity to the prescribed oxygen. c. Suction the client every 2 to 3 hours. d. Use a vibrating positive expiratory pressure device. e. Encourage diaphragmatic breathing.

ANS: A, B, D Interventions to decrease thick tenacious secretions include maintaining adequate hydration and providing humidified oxygen. These actions will help to thin secretions, making them easier to remove by coughing. The use of a vibrating positive expiratory pressure device can also help clients remove thick secretions. Although suctioning may assist with the removal of secretions, frequent suctioning can cause airway trauma and does not support the client's ability to successfully remove secretions through normal coughing. Diaphragmatic breathing is not used to improve the removal of thick secretions.

2. A nurse assesses a client with nephrotic syndrome. For which clinical manifestations should the nurse assess? (Select all that apply.) a. Proteinuria b. Hypoalbuminemia c. Dehydration d. Lipiduria e. Dysuria f. Costovertebral angle (CVA) tenderness

ANS: A, B, D Nephrotic syndrome is caused by glomerular damage and is characterized by proteinuria (protein level higher than 3.5 g/24 hr), hypoalbuminemia, edema, and lipiduria. Fluid overload leading to edema and hypertension is common with nephrotic syndrome; dehydration does not occur. Dysuria is present with cystitis. CVA tenderness is present with inflammatory changes in the kidney.

7. A client is unsure of the decision to undergo peritoneal dialysis (PD) and wishes to discuss the advantages of this treatment with the nurse. Which statements by the nurse are accurate regarding PD? (Select all that apply.) a. "You will not need vascular access to perform PD." b. "There is less restriction of protein and fluids." c. "You will have no risk for infection with PD." d. "You have flexible scheduling for the exchanges." e. "It takes less time than hemodialysis treatments."

ANS: A, B, D PD is based on exchanges of waste, fluid, and electrolytes in the peritoneal cavity. There is no need for vascular access. Protein is lost in the exchange, which allows for more protein and fluid in the diet. There is flexibility in the time for exchanges, but the treatment takes a longer period of time compared to hemodialysis. There still is risk for infection with PD, especially peritonitis.

4. A nurse is planning discharge teaching on tracheostomy care for an older client. What factors does the nurse need to assess before teaching this particular client? (Select all that apply.) a. Cognition b. Dexterity c. Hydration d. Range of motion e. Vision

ANS: A, B, D, E The older adult is at risk for having impairments in cognition, dexterity, range of motion, and vision that could limit the ability to perform tracheostomy care and should be assessed. Hydration is not directly related to the ability to perform self-care.

A client who previously had a bacillus Calmette-Guérin (BCG) vaccine has a positive tuberculosis (TB) test. What symptoms assist in determining that the client has active disease? (Select all that apply.) a. Nausea b. Weight loss c. Insomnia d. Ankle edema e. Night sweats f. Increased urination

ANS: A, B, E TB symptoms include nausea and weight loss, as well as night sweats. Inability to sleep and ankle edema are not typical symptoms. Increased urination also is not a typical symptom.

A nurse assesses a client with irritable bowel syndrome (IBS). Which questions should the nurse include in this client's assessment? (Select all that apply.) a. "Which food types cause an exacerbation of symptoms?" b. "Where is your pain and what does it feel like?" c. "Have you lost a significant amount of weight lately?" d. "Are your stools soft, watery, and black in color?" e. "Do you experience nausea associated with defecation?"

ANS: A, B, E The nurse should ask the client about factors that may cause exacerbations of symptoms, including food, stress, and anxiety. The nurse should also assess the location, intensity, and quality of the client's pain, and nausea associated with defecation or meals. Clients who have IBS do not usually lose weight and stools are not black in color.

Which assessment maneuvers should the nurse perform first when assessing the renal system at the same time as the abdomen? A. Abdominal percussion B. Abdominal auscultation C. Abdominal palpation D. Renal palpation

ANS: B Auscultation precedes percussion and palpation because the nurse needs to auscultate for abdominal bruits before palpation or percussion of the abdominal and renal components of a physical assessment.

2. A nurse plans care for a client who is at high risk for a pulmonary infection. Which interventions should the nurse include in this clients plan of care? (Select all that apply.) a. Encourage deep breathing and coughing. b. Implement an air mattress overlay. c. Ambulate the client three times each day. d. Provide a diet high in protein and vitamins. e. Administer acetaminophen (Tylenol) twice daily.

ANS: A, C, D Regular pulmonary hygiene and activities to maintain health and fitness help to maximize functioning of the respiratory system and prevent infection. A client at high risk for a pulmonary infection may need a specialty bed to help with postural drainage or percussion; this would not include an air mattress overlay, which is used to prevent pressure ulcers. Tylenol would not decrease the risk of a pulmonary infection.

26. A nursing student is developing a study guide related to historical facts about suicide. Which of the following facts should the student include? Select all that apply. A. In the Middle Ages, suicide was viewed as a selfish and criminal act. B. During the Roman Empire, suicide was followed by incineration of the body. C. Suicide was an offense in ancient Greece, and a common-site burial was denied. D. During the Renaissance, suicide was discussed and viewed more philosophically. E. Old Norse traditionally set a person who committed suicide adrift in the North Sea.

ANS: A, C, D These are true historical facts about suicide and should be included in the students study guide.

1.A nurse is providing pneumonia vaccinations in a community setting. Due to limited finances, the event organizers must limit giving the vaccination to priority groups. What clients would be considered a priority Test Bank - Medical-Surgical Nursing: Concepts for Interprofessional Collaborative Care 9e 253 when administering the pneumonia vaccination? (Select all that apply.) a. 22-year-old client with asthma b. Client who had a cholecystectomy last year c. Client with well-controlled diabetes d. Healthy 72-year-old client e. Client who is taking medication for hypertension

ANS: A, C, D, E Clients over 65 years of age and any client (no matter what age) with a chronic health condition would be considered a priority for a pneumonia vaccination. Having a cholecystectomy a year ago does not qualify as a chronic health condition.

The nurse assesses a client with pneumonia and notes decreased lung sounds on the left side and decreased lung expansion. What is the nurse's best action? a. Have the client cough and deep breathe. b. Check oxygen saturation and notify the health care provider. c. Perform an arterial blood gas analysis. d. Increase oxygen flow to 10 L/min.

ANS: B Decreased lung sounds and decreased lung expansion could indicate the development of a complication such as empyema or pus in the pleural space. The nurse should check the client's oxygen saturation and notify the provider. Infection can also move into the bloodstream and result in sepsis, so quick treatment is needed.

A nurse is providing pneumonia vaccinations in a community setting. Due to limited finances, the event organizers must limit giving the vaccination to priority groups. What clients would be considered a priority when administering the pneumonia vaccination? (Select all that apply.) a. 22-year-old client with asthma b. Client who had a cholecystectomy last year c. Client with well-controlled diabetes d. Healthy 72-year-old client e. Client who is taking medication for hypertension

ANS: A, C, D, E Clients over 65 years of age and any client (no matter what age) with a chronic health condition would be considered a priority for a pneumonia vaccination. Having a cholecystectomy a year ago does not qualify as a chronic health condition.

Which assessment findings are most important for the nurse to determine when assessing a client with dyspnea? (Select all that apply.) a. Onset of or when the client first noticed dyspnea b. Results of most recent pulmonary function test c. Conditions that relieve the dyspnea sensation d. Whether or not dyspnea interferes with ADLs e. Inspection of the external nose and its symmetry f. Whether stridor is present with dyspnea

ANS: A, C, D, F Rationale: Dyspnea, especially if it is new onset, is a sensitive indicator of the possible presence of life-threatening respiratory problems. Dyspnea is subjective and determining onset, relieving factors, interference with ADLs, and presence of stridor should be elicited from the client to help assess severity and determine the level of intervention needed. Pulmonary functioning and inspection of the external nose are objective data.

What teaching is appropriate for a client with acute rhinitis and sinusitis? (Select all that apply.) a. Using hot packs over the sinuses b. Fluid restriction c. Saline irrigations d. Staying in a dry climate e. Taking echinacea f. Antifungal medications

ANS: A, C, E Treatment of sinusitis includes the use of broad-spectrum antibiotics (e.g., amoxicillin), analgesics for pain and fever, decongestants, steam humidification, hot and wet packs over the sinus area, and nasal saline irrigations. As complementary therapy, echinacea is recommended for the symptom of rhinitis. Antifungal medications, fluid restrictions, and staying in a dry climate are not recommended. DIF: Cognitive Level: Application

7. A nurse cares for a client who is prescribed an intravenous prostacyclin agent. Which actions should the nurse take to ensure the clients safety while on this medication? (Select all that apply.) a. Keep an intravenous line dedicated strictly to the infusion. b. Teach the client that this medication increases pulmonary pressures. c. Ensure that there is always a backup drug cassette available. d. Start a large-bore peripheral intravenous line. e. Use strict aseptic technique when using the drug delivery system.

ANS: A, C, E Intravenous prostacyclin agents should be administered in a central venous catheter with a dedicated intravenous line for this medication. Death has been reported when the drug delivery system is interrupted; therefore, a backup drug cassette should also be available. The nurse should use strict aseptic technique when using the drug delivery system. The nurse should teach the client that this medication decreases pulmonary pressures and increases lung blood flow.

3. A client is hospitalized in the oliguric phase of acute kidney injury (AKI) and is receiving tube feedings. The nurse is teaching the client's spouse about the kidney-specific formulation for the enteral solution compared to standard formulas. What components should be discussed in the teaching plan? (Select all that apply.) a. Lower sodium b. Higher calcium c. Lower potassium d. Higher phosphorus e. Higher calories

ANS: A, C, E Many clients with AKI are too ill to meet caloric goals and require tube feedings with kidney-specific formulas that are lower in sodium, potassium, and phosphorus, and higher in calories than are standard formulas.

A nurse inserts a nasogastric (NG) tube for an adult client who has a bowel obstruction. Which actions does the nurse perform correctly? (Select all that apply.) a. Performs hand hygiene and positions the client in high-Fowler's position, with pillows behind the head and shoulders b. Instructs the client to extend the neck against the pillow once the NG tube has reached the oropharynx c. Checks for correct placement by checking the pH of the fluid aspirated from the tube d. Secures the NG tube by taping it to the client's nose and pinning the end to the pillowcase e. Connects the NG tube to intermittent medium suction with an anti-reflux valve on the air vent

ANS: A, C, E The client's head should be flexed forward once the NG tube has reached the oropharynx. The NG tube should be secured to the client's gown, not to the pillowcase, because it could become dislodged easily. All the other actions are appropriate.

2. A nurse is caring for a postoperative 70-kg client who had major blood loss during surgery. Which findings by the nurse should prompt immediate action to prevent acute kidney injury? (Select all that apply.) a. Urine output of 100 mL in 4 hours b. Urine output of 500 mL in 12 hours c. Large amount of sediment in the urine d. Amber, odorless urine e. Blood pressure of 90/60 mm Hg

ANS: A, C, E The low urine output, sediment, and blood pressure should be reported to the provider. Postoperatively, the nurse should measure intake and output, check the characteristics of the urine, and report sediment, hematuria, and urine output of less than 0.5 mL/kg/hour for 3 to 4 hours. A urine output of 100 mL is low, but a urine output of 500 mL in 12 hours should be within normal limits. Perfusion to the kidneys is compromised with low blood pressure. The amber odorless urine is normal.

A client started on therapy for tuberculosis infection is reporting nausea. What does the nurse teach this client? (Select all that apply.) a. Eat a diet rich in protein, iron, and vitamins. b. Do not drink fluids with medications. c. Take medications at bedtime. d. Space medications 12 hours apart. e. Take medications with milk. f. Take an antiemetic daily.

ANS: A, C, F Taking the daily dose of medications at bedtime may help to decrease nausea. A well-balanced diet with foods rich in iron, protein, and vitamins C and B also helps to decrease nausea. Antiemetics are often prescribed. Drinking fluids with medications should not influence the nausea; neither should taking medications with milk. Spacing medications 12 hours apart is not recommended therapy.

Which conditions are most likely to cause a "left shift" of the oxyhemoglobin dissociation curve? (Select all that apply.) a. Reduced blood and tissue levels of diphosphoglycerate (DPG) b. Reduced blood and tissue pH c. Increased metabolic demands d. Alkalosis e. Increased body temperature f. Reduced blood and tissue levels of oxygen

ANS: A, D Rationale: The oxyhemoglobin dissociation curve is shifted to the left when conditions are present that reduce overall oxygen needs. This left shift makes it harder for oxygen to dissociate from the hemoglobin molecule. Such conditions are those associated with slower or lower metabolism and oxygen need. These include less DPG, and alkalosis (fewer hydrogen ions). Reduced pH, increased metabolic demand, increased body temperature, and hypoxia are all associated with increased oxygen need and a right shift in the oxyhemoglobin dissociation curve.

2. A nurse is caring for a client who has a tracheostomy tube. What actions may the nurse delegate to unlicensed assistive personnel (UAP)? (Select all that apply.) a. Applying water-soluble lip balm to the clients lips b. Ensuring the humidification provided is adequate c. Performing oral care with alcohol-based mouthwash d. Reminding the client to cough and deep breathe often e. Suctioning excess secretions through the tracheostomy

ANS: A, D The UAP can perform hygiene measures such as applying lip balm and reinforce teaching such as reminding the client to perform coughing and deep-breathing exercises. Oral care can be accomplished with normal saline, not products that dry the mouth. Ensuring the humidity is adequate and suctioning through the tracheostomy are nursing functions.

1. A nurse assesses a client who is prescribed varenicline (Chantix) for smoking cessation. Which manifestations should the nurse identify as adverse effects of this medication? (Select all that apply.) a. Visual hallucinations b. Tachycardia c. Decreased cravings d. Impaired judgment e. Increased thirst

ANS: A, D Varenicline (Chantix) has a black box warning stating that the drug can cause manic behavior and hallucinations. The nurse should assess for changes in behavior and thought processes, including impaired judgment and visual hallucinations. Tachycardia and increased thirst are not adverse effects of this medication. Decreased cravings is a therapeutic response to this medication.

Which findings are signs of a person who is mentally healthy? Select all that apply. a. Says, I have some weaknesses, but I feel Im important to my family and friends. b. Adheres strictly to religious beliefs of parents and family of origin. c. Spends all holidays alone watching old movies on television. d. Considers past experiences when deciding about the future. e. Experiences feelings of conflict related to changing jobs.

ANS: A, D, E Mental health is a state of well-being in which each individual is able to realize his or her own potential, cope with the normal stresses of life, work productively, and make a contribution to the community. Mental health provides people with the capacity for rational thinking, communication skills, learning, emotional growth, resilience, and self-esteem.

3. While obtaining a clients health history, the client states, I am allergic to avocados. Which responses by the nurse are best? (Select all that apply.) a. What response do you have when you eat avocados? b. I will remove any avocados that are on your lunch tray. c. When was the last time you ate foods containing avocados? d. I will document this in your record so all of your providers will know. e. Have you ever been treated for this allergic reaction?

ANS: A, D, E Nurses should assess clients who have allergies for the specific cause, treatment, and response to treatment. The nurse should also document the allergies in a prominent place in the clients medical record. The nurse should collaborate with food services to ensure no avocados are placed on the clients meal trays. Asking about the last time the client ate avocados does not provide any pertinent information for the clients plan of care.

3. A client is being discharged home after having a tracheostomy placed. What suggestions does the nurse offer to help the client maintain self-esteem? (Select all that apply.) a. Create a communication system. b. Dont go out in public alone. c. Find hobbies to enjoy at home. d. Try loose-fitting shirts with collars. e. Wear fashionable scarves.

ANS: A, D, E The client with a tracheostomy may be shy and hesitant to go out in public. The client should have a sound communication method to ease frustration. The nurse can also suggest ways of enhancing appearance so the client is willing to leave the house. These can include wearing scarves and loose-fitting shirts to hide the stoma. Keeping the client homebound is not good advice.

5. A nurse teaches a client who is interested in smoking cessation. Which statements should the nurse include in this clients teaching? (Select all that apply.) a. Find an activity that you enjoy and will keep your hands busy. b. Keep snacks like potato chips on hand to nibble on. c. Identify a punishment for yourself in case you backslide. d. Drink at least eight glasses of water each day. e. Make a list of reasons you want to stop smoking.

ANS: A, D, E The nurse should teach a client who is interested in smoking cessation to find an activity that keeps the hands busy, to keep healthy snacks on hand to nibble on, to drink at least 8 glasses of water each day, and to make a list of reasons for quitting smoking. The nurse should also encourage the client not to be upset if he or she backslides and has a cigarette.

4. A nurse collaborates with a respiratory therapist to complete pulmonary function tests (PFTs) for a client. Which statements should the nurse include in communications with the respiratory therapist prior to the tests? (Select all that apply.) a. I held the clients morning bronchodilator medication. b. The client is ready to go down to radiology for this examination. c. Physical therapy states the client can run on a treadmill. d. I advised the client not to smoke for 6 hours prior to the test. e. The client is alert and can follow your commands.

ANS: A, D, E To ensure the PFTs are accurate, the therapist needs to know that no bronchodilators have been administered in the past 4 to 6 hours, the client did not smoke within 6 to 8 hours prior to the test, and the client can follow basic commands, including different breathing maneuvers. The respiratory therapist can perform PFTs at the bedside. A treadmill is not used for this test.

Which person is at greatest risk for developing a community-acquired pneumonia? a. Middle-aged teacher who typically eats a diet of Asian foods b. Older adult who smokes and has a substance abuse problem c. Older adult with exercise-induced wheezing d. Young adult aerobics instructor who is a vegetarian

ANS: B Although age is a factor in the development of community-acquired pneumonia, other lifestyle and exposure factors increase the risk to a greater extent than age. Two conditions that heavily predispose to the development of pneumonia are cigarette smoking and alcoholism. Dietary choices typically do not predispose to the development of pneumonia. Cigarette smoking interferes with the ciliary function of removal of invasive materials. Alcoholism usually results in unbalanced nutrition, as well as decreased immune function. A middle-aged adult, an older adult with wheezing induced by exercise, and a young adult vegetarian would not be at risk for community-acquired pneumonia because they have no predisposing conditions.

Complete this analogy. NANDA: clinical judgment: NIC: _________________ a. patient outcomes c. diagnosis b. nursing actions d. symptoms

ANS: B Analogies show parallel relationships. NANDA, the North American Nursing Diagnosis Association, identifies diagnostic statements regarding human responses to actual or potential health problems. These statements represent clinical judgments. NIC (Nursing Interventions Classification) identifies actions provided by nurses that enhance patient outcomes. Nursing care activities may be direct or indirect.

4. A nurse is caring for an older adult client who has a pulmonary infection. Which action should the nurse take first? a. Encourage the client to increase fluid intake. b. Assess the clients level of consciousness. c. Raise the head of the bed to at least 45 degrees. d. Provide the client with humidified oxygen.

ANS: B Assessing the clients level of consciousness will be most important because it will show how the client is responding to the presence of the infection. Although it will be important for the nurse to encourage the client to turn, cough, and frequently breathe deeply; raise the head of the bed; increase oral fluid intake; and humidify the oxygen administered, none of these actions will be as important as assessing the level of consciousness. Also, the client who has a pulmonary infection may not be able to cough effectively if an area of abscess is present.

The nurse is caring for a client who has just been prescribed a glucocorticoid to treat an exacerbation of ulcerative colitis. What teaching will the nurse provide? a. Decrease the drug dose during the next exacerbation. b. Report fever to healthcare provider immediately. c. Determine if the client's insurance covers payment for this medication. d. This drug will act as an antidiarrheal.

ANS: B Glucocorticoid medication, a form of steroid therapy, can cause immunosuppression (not act as an antidiarrheal); therefore, any sign of infection should be immediately reported to the healthcare provider. The drug dose may be increased by the health care provider during the next exacerbation. The client will determine with the insurance provider (and possibly a social worker) whether the drug is covered for payment.

A staff nurse completes orientation to a psychiatric unit. This nurse may expect an advanced practice nurse to perform which additional intervention? a. Conduct mental health assessments. b. Prescribe psychotropic medication. c. Establish therapeutic relationships. d. Individualize nursing care plans.

ANS: B In most states, prescriptive privileges are granted to masters-prepared nurse practitioners who have taken special courses on prescribing medication. The nurse prepared at the basic level is permitted to perform mental health assessments, establish relationships, and provide individualized care planning

Which is the highest priority goal to set for a client with pneumonia? a. Absence of cyanosis b. Maintenance of SaO2 of 95% c. Walking 20 feet three times daily d. Absence of confusion

ANS: B Maintenance of an SaO2 of at least 95% is a clear goal that indicates that the client has adequate oxygenation. Absence of cyanosis and the presence of confusion are assessment factors that contribute to evaluation of oxygen; however, they are not absolute measures. Likewise, walking three times a day does not directly address oxygenation.

Which client does the nurse caution to avoid taking over-the-counter decongestants for manifestations of a cold or flu? a. Young man with a latex allergy b. Middle-aged woman with hypertension c. Teenage woman who is taking oral contraceptives d. Older man who has had type 1 diabetes mellitus for 20 years

ANS: B Most decongestants work by increasing blood vessel constriction. This action increases peripheral vascular resistance and blood pressure. The client who already has hypertension may develop dangerously high blood pressure when taking a decongestant. The client who has a latex allergy, is taking oral contraceptives, or has type 1 diabetes would not be likely to be affected by the decongestant in such a life-threatening manner as the client who is hypertensive.

The nurse assessing the respiratory status of a client discovers that tactile fremitus has increased from the assessment performed yesterday. For which possible respiratory problem should the nurse assess further? a. Pneumothorax b. Pneumonia c. Pleural effusion d. Emphysema

ANS: B Tactile (vocal) fremitus is a vibration of the chest wall produced when the patient speaks. This vibration can be felt on the chest wall. Fremitus is decreased if the transmission of sound waves from the larynx to the chest wall is slowed, such as when the pleural space is filled with air (pneumothorax) or fluid (such as with a pleural effusion) or when the bronchus is obstructed. Fremitus is increased with pneumonia and lung abscesses because the increased density of the chest enhances transmission of the vibrations.

A nurse wants to find a description of diagnostic criteria for anxiety disorders. Which resource would have the most complete information? a. Nursing Outcomes Classification (NOC) b. Diagnostic and Statistical Manual of Mental Disorders (DSM-5) c. The ANAs Psychiatric-Mental Health Nursing Scope and Standards of Practice d. International Statistical Classification of Diseases and Related Health Problems (ICD-10)

ANS: B The DSM-5 details the diagnostic criteria for psychiatric clinical conditions. The other references are good resources but do not define the diagnostic criteria.

A nurse encounters an unfamiliar psychiatric disorder on a new patients admission form. Which resource should the nurse consult to determine criteria used to establish this diagnosis? a. International Statistical Classification of Diseases and Related Health Problems (ICD-10) b. Diagnostic and Statistical Manual of Mental Disorders (DSM-5) c. A behavioral health reference manual d. Wikipedia

ANS: B The DSM-5 gives the criteria used to diagnose each mental disorder. The distracters may not contain diagnostic criteria for a psychiatric illness.

A client is admitted with suspected avian influenza. The family asks the nurse what kind of care the client will get. Which statement by the nurse is correct? a. "He will be given standard antibiotic agents and will be placed in contact isolation." b. "He will be placed on airborne and contact isolation." c. "Oseltamivir (Tamiflu) will reduce complications of this infection." d. "All family members should be tested for evidence of the same disease."

ANS: B The client who is experiencing avian influenza should be on both airborne and contact isolation. Standard antibiotic agents would be ineffective with this disease process, as would most of the standard antiviral medications commonly used for influenza. Human-to-human contact through family members is likely only in very close living arrangements, so only specific members of the client's family should consider diagnostic testing.

. A client is admitted with left lower lung pneumonia. Which assessment finding does the nurse correlate with this condition? a. Expiratory wheeze on the right side b. Dullness to percussion on the lower left side c. Crepitus of the skin around the left lung d. Crackles heard on expiration bilaterally

ANS: B The client with pneumonia may have dullness to percussion on the affected side. The other options are all inconsistent with pneumonia.

Which assessment finding most clearly indicates that a patient may be experiencing a mental illness? The patient: a. reports occasional sleeplessness and anxiety. b. reports a consistently sad, discouraged, and hopeless mood. c. is able to describe the difference between as if and for real. d. perceives difficulty making a decision about whether to change jobs.

ANS: B The correct response describes a mood alteration, which reflects mental illness. The distracters describe behaviors that are mentally healthy or within the usual scope of human experience.

Which component of treatment of mental illness is specifically recognized by Quality and Safety Education for Nurses (QSEN)? a. All genomes are unique. b. Care is centered on the patient. c. Healthy development is vital to mental health. d. Recovery occurs on a continuum from illness to health.

ANS: B The key areas of care promoted by QSEN are patient-centered care, teamwork and collaboration, evidence-based practice, quality improvement, safety, and informatics.

When a nursing student expresses concerns about how mental health nurses lose all their nursing skills, the best response by the mental health nurse is: a. Psychiatric nurses practice in safer environments than other specialties. Nurse-to-patient ratios must be better because of the nature of the patients problems. b. Psychiatric nurses use complex communication skills as well as critical thinking to solve multidimensional problems. I am challenged by those situations. c. Thats a misconception. Psychiatric nurses frequently use high technology monitoring equipment and manage complex intravenous therapies. d. Psychiatric nurses do not have to deal with as much pain and suffering as medical-surgical nurses do. That appeals to me.

ANS: B The practice of psychiatric nursing requires a different set of skills than medical-surgical nursing, though there is substantial overlap. Psychiatric nurses must be able to help patients with medical as well as mental health problems, reflecting the holistic perspective these nurses must have. Nurse-patient ratios and workloads in psychiatric settings have increased, just like other specialties. Psychiatric nursing involves clinical practice, not just documentation. Psychosocial pain and suffering are as real as physical pain and suffering.

A client is worried about contracting influenza. What is the nurse's best response to the client? a. "Flu is no longer a prevalent problem." b. "Did you receive a flu vaccine this year?" c. "Current flu strains are generally mild." d. "If you develop symptoms, antibiotics will cure you."

ANS: B Vaccines for influenza are widely available and are recommended to prevent flu. Flu continues to be a major problem, affecting up to 20% of the U.S. population and causing 36,000 deaths annually.

A staff nurse tells another nurse, I evaluated a new patient using the SAD PERSONS scale and got a score of 10. Im wondering if I should send the patient home. Select the best reply by the second nurse. a. That action would seem appropriate. b. A score over 8 requires immediate hospitalization. c. I think you should strongly consider hospitalization for this patient. d. Give the patient a follow-up appointment. Hospitalization may be needed soon.

ANS: B A SAD PERSONS scale score of 0 to 5 suggests home care with follow-up. A score of 6 to 8 requires psychiatric consultation. A score over 8 calls for hospitalization.

12. A nurse provides health screening for a community health center with a large population of African-American clients. Which priority assessment should the nurse include when working with this population? a. Measure height and weight. b. Assess blood pressure. c. Observe for any signs of abuse. d. Ask about medications.

ANS: B All interventions are important for the visiting nurse to accomplish. However, African Americans have a high rate of hypertension leading to end-stage renal disease. Each encounter that the nurse has with an African-American client provides a chance to detect hypertension and treat it. If the client is already on antihypertensive medication, assessing blood pressure monitors therapy.

16. Which is a correctly written, appropriate outcome for a client with a history of suicide attempts who is currently exhibiting symptoms of low self-esteem by isolating self? A. The client will not physically harm self. B. The client will express three positive self-attributes by day 4. C. The client will reveal a suicide plan. D. The client will establish a trusting relationship.

ANS: B Although the client has a history of suicide attempts, the current problem is isolative behaviors based on low self-esteem. Outcomes should be client centered, specific, realistic, and measureable and contain a time frame.

24. A client is having a peritoneal dialysis treatment. The nurse notes an opaque color to the effluent. What is the priority action by the nurse? a. Warm the dialysate solution in a microwave before instillation. b. Take a sample of the effluent and send to the laboratory. c. Flush the tubing with normal saline to maintain patency of the catheter. d. Check the peritoneal catheter for kinking and curling.

ANS: B An opaque or cloudy effluent is the first sign of peritonitis. A sample of the effluent would need to be sent to the laboratory for culture and sensitivity in order to administer the correct antibiotic. Warming the dialysate in a microwave and flushing the tubing are not safe actions by the nurse. Checking the catheter for obstruction is a viable option but will not treat the peritonitis.

3.Which teaching point is most important for the client with bacterial pharyngitis? a. Gargle with warm salt water. b. Take all antibiotics as directed. c. Use a humidifier in the bedroom. d. Wash hands frequently.

ANS: B Any client on antibiotics must be instructed to complete the entire course of antibiotics. Not completing them can lead to complications or drug-resistant strains of bacteria. The other instructions are appropriate, just not the most important.

Which teaching point is most important for the client with bacterial pharyngitis? a. Gargle with warm salt water. b. Take all antibiotics as directed. c. Use a humidifier in the bedroom. d. Wash hands frequently.

ANS: B Any client on antibiotics must be instructed to complete the entire course of antibiotics. Not completing them can lead to complications or drug-resistant strains of bacteria. The other instructions are appropriate, just not the most important

2. A nurse cares for a client with arthritis who reports frequent asthma attacks. Which action should the nurse take first? a. Review the clients pulmonary function test results. b. Ask about medications the client is currently taking. c. Assess how frequently the client uses a bronchodilator. d. Consult the provider and request arterial blood gases.

ANS: B Aspirin and other nonsteroidal anti-inflammatory drugs (NSAIDs) can trigger asthma in some people. This results from increased production of leukotriene when aspirin or NSAIDs suppress other inflammatory pathways and is a high priority given the clients history. Reviewing pulmonary function test results will not address the immediate problem of frequent asthma attacks. This is a good intervention for reviewing response to bronchodilators. Questioning the client about the use of bronchodilators will address interventions for the attacks but not their cause. Reviewing arterial blood gas results would not be of use in a client between attacks because many clients are asymptomatic when not having attacks.

27. A client is recovering from a kidney transplant. The client's urine output was 1500 mL over the last 12-hour period since transplantation. What is the priority assessment by the nurse? a. Checking skin turgor b. Taking blood pressure c. Assessing lung sounds d. Weighing the client

ANS: B By taking blood pressure, the nurse is assessing for hypotension that could compromise perfusion to the new kidney. The nurse then should notify the provider immediately. Skin turgor, lung sounds, and weight could give information about the fluid status of the client, but they are not the priority assessment.

A nurse cares for a client with arthritis who reports frequent asthma attacks. Which action should the nurse take first? a. Review the client's pulmonary function test results. b. Ask about medications the client is currently taking. c. Assess how frequently the client uses a bronchodilator. d. Consult the provider and request arterial blood gases.

ANS: B Aspirin and other nonsteroidal anti-inflammatory drugs (NSAIDs) can trigger asthma in some people. This results from increased production of leukotriene when aspirin or NSAIDs suppress other inflammatory pathways and is a high priority given the client's history. Reviewing pulmonary function test results will not address the immediate problem of frequent asthma attacks. This is a good intervention for reviewing response to bronchodilators. Questioning the client about the use of bronchodilators will address interventions for the attacks but not their cause. Reviewing arterial blood gas results would not be of use in a client between attacks because many clients are asymptomatic when not having attacks.

A nurse assesses a client who is recovering from a hemorrhoidectomy that was done the day before. The nurse notes that the client has lower abdominal distention accompanied by dullness to percussion over the distended area. Which action should the nurse take? a. Assess the client's heart rate and blood pressure. b. Determine when the client last voided. c. Ask if the client is experiencing flatus. d. Auscultate all quadrants of the client's abdomen

ANS: B Assessment findings indicate that the client may have an over-full bladder. In the immediate postoperative period, the client may experience difficulty voiding due to urinary retention. The nurse should assess when the client last voided. The client's vital signs may be checked after the nurse determines the client's last void. Asking about flatus and auscultating bowel sounds are not related to a hemorrhoidectomy.

9. A stockbroker commits suicide after being convicted of insider trading. In speaking with the family, which statement by the nurse demonstrates accurate and appropriate sharing of information? A. Your grieving will subside within 1 year; until then I recommend antidepressants. B. Support groups are available specifically for survivors of suicide, and I would be glad to help you locate one in this area. C. The only way to deal effectively with this kind of grief is to write a letter to the brokerage firm to express your anger with them. D. Since stigmatization often occurs in these situations, it would be best if you avoid discussing the suicide with anyone.

ANS: B Bereavement following suicide is complicated by the complex psychological impact of the act on those close to the victim. Support groups for survivors can provide a meaningful resource for grief work.

23. A nurse administers medications to a client who has asthma. Which medication classification is paired correctly with its physiologic response to the medication? a. Bronchodilator Stabilizes the membranes of mast cells and prevents the release of inflammatory mediators b. Cholinergic antagonist Causes bronchodilation by inhibiting the parasympathetic nervous system c. Corticosteroid Relaxes bronchiolar smooth muscles by binding to and activating pulmonary beta2 receptors d. Cromone Disrupts the production of pathways of inflammatory mediators

ANS: B Cholinergic antagonist drugs cause bronchodilation by inhibiting the parasympathetic nervous system. This allows the sympathetic nervous system to dominate and release norepinephrine that actives beta2 receptors. Bronchodilators relax bronchiolar smooth muscles by binding to and activating pulmonary beta2 receptors. Corticosteroids disrupt the production of pathways of inflammatory mediators. Cromones stabilize the membranes of mast cells and prevent the release of inflammatory mediators.

A nurse administers medications to a client who has asthma. Which medication classification is paired correctly with its physiologic response to the medication? a. Bronchodilator - Stabilizes the membranes of mast cells and prevents the release of inflammatory mediators b. Cholinergic antagonist - Causes bronchodilation by inhibiting the parasympathetic nervous system c. Corticosteroid - Relaxes bronchiolar smooth muscles by binding to and activating pulmonary beta2 receptors d. Cromone - Disrupts the production of pathways of inflammatory mediators

ANS: B Cholinergic antagonist drugs cause bronchodilation by inhibiting the parasympathetic nervous system. This allows the sympathetic nervous system to dominate and release norepinephrine that actives beta2 receptors. Bronchodilators relax bronchiolar smooth muscles by binding to and activating pulmonary beta2 receptors. Corticosteroids disrupt the production of pathways of inflammatory mediators. Cromones stabilize the membranes of mast cells and prevent the release of inflammatory mediators.

The female client's urinalysis shows all the following characteristics. Which should the nurse document as abnormal? A. pH 5.6 B. Ketone bodies present C. Specific gravity is 1.030 D. Two white blood cells per high-power field

ANS: B Ketone bodies are byproducts of incomplete metabolism of fatty acids. Normally, there are no ketones in urine. Ketone bodies are produced when fat sources are used instead of glucose to provide cellular energy.

After teaching a client with irritable bowel syndrome (IBS), a nurse assesses the client's understanding. Which menu selection indicates that the client correctly understands the dietary teaching? a. Ham sandwich on white bread, cup of applesauce, glass of diet cola b. Broiled chicken with brown rice, steamed broccoli, glass of apple juice c. Grilled cheese sandwich, small banana, cup of hot tea with lemon d. Baked tilapia, fresh green beans, cup of coffee with low-fat milk

ANS: B Clients with IBS are advised to eat a high-fiber diet (30 to 40 g/day), with 8 to 10 cups of liquid daily. Chicken with brown rice, broccoli, and apple juice has the highest fiber content. They should avoid alcohol, caffeine, and other gastric irritants.

3. After teaching a client with early polycystic kidney disease (PKD) about nutritional therapy, the nurse assesses the client's understanding. Which statement made by the client indicates a correct understanding of the teaching? a. "I will take a laxative every night before going to bed." b. "I must increase my intake of dietary fiber and fluids." c. "I shall only use salt when I am cooking my own food." d. "I'll eat white bread to minimize gastrointestinal gas."

ANS: B Clients with PKD often have constipation, which can be managed with increased fiber, exercise, and drinking plenty of water. Laxatives should be used cautiously. Clients with PKD should be on a restricted salt diet, which includes not cooking with salt. White bread has a low fiber count and would not be included in a high-fiber diet.

8. An emergency department nurse assesses a client with kidney trauma and notes that the client's abdomen is tender and distended and blood is visible at the urinary meatus. Which prescription should the nurse consult the provider about before implementation? a. Assessing vital signs every 15 minutes b. Inserting an indwelling urinary catheter c. Administering intravenous fluids at 125 mL/hr d. Typing and crossmatching for blood products

ANS: B Clients with blood at the urinary meatus should not have a urinary catheter inserted via the urethra before additional diagnostic studies are done. The urethra could be torn. The nurse should question the provider about the need for a catheter; if one is needed, the provider can insert a suprapubic catheter. The nurse should monitor the client's vital signs closely, send blood for type and crossmatch in case the client needs blood products, and administer intravenous fluids.

4. A client has a tracheostomy tube in place. When the nurse suctions the client, food particles are noted. What action by the nurse is best? a. Elevate the head of the clients bed. b. Measure and compare cuff pressures. c. Place the client on NPO status. d. Request that the client have a swallow study.

ANS: B Constant pressure from the tracheostomy tube cuff can cause tracheomalacia, leading to dilation of the tracheal passage. This can be manifested by food particles seen in secretions or by noting that larger and larger amounts of pressure are needed to keep the tracheostomy cuff inflated. The nurse should measure the pressures and compare them to previous ones to detect a trend. Elevating the head of the bed, placing the client on NPO status, and requesting a swallow study will not correct this situation.

13. A nurse is caring for a client who received benzocaine spray prior to a recent bronchoscopy. The client presents with continuous cyanosis even with oxygen therapy. Which action should the nurse take next? a. Administer an albuterol treatment. b. Notify the Rapid Response Team. c. Assess the clients peripheral pulses. d. Obtain blood and sputum cultures.

ANS: B Cyanosis unresponsive to oxygen therapy is a manifestation of methemoglobinemia, which is an adverse effect of benzocaine spray. Death can occur if the level of methemoglobin rises and cyanosis occurs. The nurse should notify the Rapid Response Team to provide advanced nursing care. An albuterol treatment would not address the clients oxygenation problem. Assessment of pulses and cultures will not provide data necessary to treat this client.

23. A student nurse tells the instructor, Im concerned that when a client asks me for advice I wont have a good solution. Which should be the nursing instructors best response? A. Its scary to feel put on the spot by a client. Nurses dont always have the answer. B. Remember, clients, not nurses, are responsible for their own choices and decisions. C. Just keep the clients best interests in mind and do the best that you can. D. Set a goal to continue to work on this aspect of your practice.

ANS: B Giving advice tells the client what to do or how to behave. It implies that the nurse knows what is best and that the client is incapable of any self-direction. It discourages independent thinking. KEY: Cognitive Level: Application | Integrated Processes: Nursing Process: Implementation | Client Need: Psychosocial Integrity

A student nurse tells the instructor, Im concerned that when a client asks me for advice I wont have a good solution. Which should be the nursing instructors best response? A. Its scary to feel put on the spot by a client. Nurses dont always have the answer. B. Remember, clients, not nurses, are responsible for their own choices and decisions. C. Just keep the clients best interests in mind and do the best that you can. D. Set a goal to continue to work on this aspect of your practice.

ANS: B Giving advice tells the client what to do or how to behave. It implies that the nurse knows what is best and that the client is incapable of any self-direction. It discourages independent thinking.

13.A client has been taking isoniazid (INH) for tuberculosis for 3 weeks. What laboratory results need to be reported to the health care provider immediately? a. Albumin: 5.1 g/dL b. Alanine aminotransferase (ALT): 180 U/L c. Red blood cell (RBC) count: 5.2/mm3 d. White blood cell (WBC) count: 12,500/mm3

ANS: B INH can cause liver damage, especially if the client drinks alcohol. The ALT (one of the liver enzymes) is extremely high and needs to be reported immediately. The albumin and RBCs are normal. The WBCs are slightly high, but that would be an expected finding in a client with an infection.

10. While assessing a client who is 12 hours postoperative after a thoracotomy for lung cancer, a nurse notices that the lower chest tube is dislodged. Which action should the nurse take first? a. Assess for drainage from the site. b. Cover the insertion site with sterile gauze. c. Contact the provider and obtain a suture kit. d. Reinsert the tube using sterile technique.

ANS: B Immediately covering the insertion site helps prevent air from entering the pleural space and causing a pneumothorax. The area will not reseal quickly enough to prevent air from entering the chest. The nurse should not leave the client to obtain a suture kit. An occlusive dressing may cause a tension pneumothorax. The site should only be assessed after the insertion site is covered. The provider should be called to reinsert the chest tube or prescribe other treatment options.

6. After teaching a client with nephrotic syndrome and a normal glomerular filtration, the nurse assesses the client's understanding. Which statement made by the client indicates a correct understanding of the nutritional therapy for this condition? a. "I must decrease my intake of fat." b. "I will increase my intake of protein." c. "A decreased intake of carbohydrates will be required." d. "An increased intake of vitamin C is necessary."

ANS: B In nephrotic syndrome, the renal loss of protein is significant, leading to hypoalbuminemia and edema formation. If glomerular filtration is normal or near normal, increased protein loss should be matched by increased intake of protein. The client would not need to adjust fat, carbohydrates, or vitamins based on this disorder.

7.An older adult is brought to the emergency department by a family member, who reports a moderate change in mental status and mild cough. The client is afebrile. The health care provider orders a chest x-ray. The family member questions why this is needed since the manifestations seem so vague. What response by the nurse is best? a. Chest x-rays are always ordered when we suspect pneumonia. b. Older people often have vague symptoms, so an x-ray is essential. c. The x-ray can be done and read before laboratory work is reported. d. We are testing for any possible source of infection in the client.

ANS: B It is essential to obtain an early chest x-ray in older adults suspected of having pneumonia because symptoms are often vague. Waiting until definitive manifestations are present to obtain the x-ray leads to a costly delay in treatment. Stating that chest x-rays are always ordered does not give the family definitive information. The xray can be done while laboratory values are still pending, but this also does not provide specific information about the importance of a chest x-ray in this client. The client has manifestations of pneumonia, so the staff is not testing for any possible source of infection but rather is testing for a suspected disorder.

An older adult is brought to the emergency department by a family member, who reports a moderate change in mental status and mild cough. The client is afebrile. The health care provider orders a chest x-ray. The family member questions why this is needed since the manifestations seem so vague. What response by the nurse is best? a. "Chest x-rays are always ordered when we suspect pneumonia." b. "Older people often have vague symptoms, so an x-ray is essential." c. "The x-ray can be done and read before laboratory work is reported." d. "We are testing for any possible source of infection in the client."

ANS: B It is essential to obtain an early chest x-ray in older adults suspected of having pneumonia because symptoms are often vague. Waiting until definitive manifestations are present to obtain the x-ray leads to a costly delay in treatment. Stating that chest x-rays are always ordered does not give the family definitive information. The x-ray can be done while laboratory values are still pending, but this also does not provide specific information about the importance of a chest x-ray in this client. The client has manifestations of pneumonia, so the staff is not testing for any possible source of infection but rather is testing for a suspected disorder.

9. The nurse is caring for four clients with chronic kidney disease. Which client should the nurse assess first upon initial rounding? a. Woman with a blood pressure of 158/90 mm Hg b. Client with Kussmaul respirations c. Man with skin itching from head to toe d. Client with halitosis and stomatitis

ANS: B Kussmaul respirations indicate a worsening of chronic kidney disease (CKD). The client is increasing the rate and depth of breathing to excrete carbon dioxide through the lungs. Hypertension is common in most clients with CKD, and skin itching increases with calcium-phosphate imbalances, another common finding in CKD. Uremia from CKD causes ammonia to be formed, resulting in the common findings of halitosis and stomatitis.

Which statement by a patient during an assessment interview should alert the nurse to the patients need for immediate, active intervention? A. I am mixed up but I know I need help B. I have no one for help or support C. It is worse when you are a person of color D. I tried to get attention before i shot myself.

ANS: B Lack of social support and social isolation increase the suicide risk. The willingness to seek help lowers the risk. Being a person of color does not suggest a higher risk; more whites commit suicide than do individuals of other racial groups. Attention seeking is not correlated with a higher risk of suicide.

19.A client is in the family medicine clinic reporting a dry, sore throat. The provider asks the nurse to assess for odynophagia. What assessment technique is most appropriate? a. Ask the client what foods cause trouble swallowing. b. Assess the client for pain when swallowing. c. Determine if the client can swallow saliva. d. Palpate the clients jaw while swallowing.

ANS: B Odynophagia is painful swallowing. The nurse should assess the client for this either by asking or by having the client attempt to drink water. It is not related to specific foods and is not assessed by palpating the jaw. Being unable to swallow saliva is not odynophagia, but it would be a serious situation.

A client is in the family medicine clinic reporting a dry, sore throat. The provider asks the nurse to assess for odynophagia. What assessment technique is most appropriate? a. Ask the client what foods cause trouble swallowing. b. Assess the client for pain when swallowing. c. Determine if the client can swallow saliva. d. Palpate the client's jaw while swallowing.

ANS: B Odynophagia is painful swallowing. The nurse should assess the client for this either by asking or by having the client attempt to drink water. It is not related to specific foods and is not assessed by palpating the jaw. Being unable to swallow saliva is not odynophagia, but it would be a serious situation.

An emergency room nurse assesses a client after a motor vehicle crash and notes ecchymotic areas across the client's lower abdomen. Which action should the nurse take first? a. Measure the client's abdominal girth. b. Assess for abdominal guarding or rigidity. c. Check the client's hemoglobin and hematocrit. d. Obtain the client's complete health history.

ANS: B On noticing the ecchymotic areas, the nurse should check to see if abdominal guarding or rigidity is present, because this could indicate major organ injury. The nurse should then notify the provider. Measuring abdominal girth or obtaining a complete health history is not appropriate at this time. Laboratory test results can be checked after assessment for abdominal guarding or rigidity.

12. A client is wearing a Venturi mask to deliver oxygen and the dinner tray has arrived. What action by the nurse is best? a. Assess the clients oxygen saturation and, if normal, turn off the oxygen. b. Determine if the client can switch to a nasal cannula during the meal. c. Have the client lift the mask off the face when taking bites of food. d. Turn the oxygen off while the client eats the meal and then restart it.

ANS: B Oxygen is a drug that needs to be delivered constantly. The nurse should determine if the provider has approved switching to a nasal cannula during meals. If not, the nurse should consult with the provider about this issue. The oxygen should not be turned off. Lifting the mask to eat will alter the FiO2 delivered.

8. A nurse is caring for a client using oxygen while in the hospital. What assessment finding indicates that goals for a priority diagnosis are being met? a. 100% of meals being eaten by the client b. Intact skin behind the ears c. The client understanding the need for oxygen d. Unchanged weight for the past 3 days

ANS: B Oxygen tubing can cause pressure ulcers, so clients using oxygen have the nursing diagnosis of Risk for Impaired Skin Integrity. Intact skin behind the ears indicates that goals for this diagnosis are being met. Nutrition and weight are not related to using oxygen. Understanding the need for oxygen is important but would not take priority over a physical problem.

1. A nurse assesses a client with polycystic kidney disease (PKD). Which assessment finding should alert the nurse to immediately contact the health care provider? a. Flank pain b. Periorbital edema c. Bloody and cloudy urine d. Enlarged abdomen

ANS: B Periorbital edema would not be a finding related to PKD and should be investigated further. Flank pain and a distended or enlarged abdomen occur in PKD because the kidneys enlarge and displace other organs. Urine can be bloody or cloudy as a result of cyst rupture or infection.

17.A client is in the family practice clinic reporting a severe cough that has lasted for 5 weeks. The client is so exhausted after coughing that work has become impossible. What action by the nurse is most appropriate? a. Arrange for immediate hospitalization. b. Facilitate polymerase chain reaction testing. c. Have the client produce a sputum sample. d. Obtain two sets of blood cultures.

ANS: B Polymerase chain reaction testing is used to diagnose pertussis, which this client is showing manifestations of. Hospitalization may or may not be needed but is not the most important action. The client may or may not be able to produce sputum, but sputum cultures for this disease must be obtained via deep suctioning. Blood cultures will be negative.

1. The nurse is assessing a client with a diagnosis of pre-renal acute kidney injury (AKI). Which condition would the nurse expect to find in the client's recent history? a. Pyelonephritis b. Myocardial infarction c. Bladder cancer d. Kidney stones

ANS: B Pre-renal causes of AKI are related to a decrease in perfusion, such as with a myocardial infarction. Pyelonephritis is an intrinsic or intrarenal cause of AKI related to kidney damage. Bladder cancer and kidney stones are post-renal causes of AKI related to urine flow obstruction.

5.The charge nurse on a medical unit is preparing to admit several clients who have possible pandemic flu during a preparedness drill. What action by the nurse is best? a. Admit the clients on Contact Precautions. b. Cohort the clients in the same area of the unit. c. Do not allow pregnant caregivers to care for these clients. d. Place the clients on enhanced Droplet Precautions.

ANS: B Preventing the spread of pandemic flu is equally important as caring for the clients who have it. Clients can be cohorted together in the same set of rooms on one part of the unit to use distancing to help prevent the spread of the disease. The other actions are not appropriate.

The charge nurse on a medical unit is preparing to admit several "clients" who have possible pandemic flu during a preparedness drill. What action by the nurse is best? a. Admit the "clients" on Contact Precautions. b. Cohort the "clients" in the same area of the unit. c. Do not allow pregnant caregivers to care for these "clients." d. Place the "clients" on enhanced Droplet Precautions.

ANS: B Preventing the spread of pandemic flu is equally important as caring for the clients who have it. Clients can be cohorted together in the same set of rooms on one part of the unit to use distancing to help prevent the spread of the disease. The other actions are not appropriate.

1. A nursing student caring for a client removes the clients oxygen as prescribed. The client is now breathing what percentage of oxygen in the room air? a. 14% b. 21% c. 28% d. 31%

ANS: B Room air is 21% oxygen.

2. A client is scheduled to have a tracheostomy placed in an hour. What action by the nurse is the priority? a. Administer prescribed anxiolytic medication. b. Ensure informed consent is on the chart. c. Reinforce any teaching done previously. d. Start the preoperative antibiotic infusion.

ANS: B Since this is an operative procedure, the client must sign an informed consent, which must be on the chart. Giving anxiolytics and antibiotics and reinforcing teaching may also be required but do not take priority.

Which intervention should the nurse recommend for the distressed family and friends of someone who has committed suicide? A. Participating in reminiscence therapy B. Attending a self-help group for survivors C. Contracting for two sessions of group therapy D. Completing a psychological postmortem assessment

ANS: B Survivors need outlets for their feelings about the loss and the deceased person. Self-help groups provide peer support while survivors work through feelings of loss, anger, and guilt. Psychological postmortem assessment would not provide the support necessary to work through feelings of loss associated with the suicide of a family member. Reminiscence therapy is not geared to loss resolution. Contracting for two sessions of group therapy would probably not provide sufficient time to work through the issues associated with a death by suicide.

13. After teaching a client with renal cancer who is prescribed temsirolimus (Torisel), the nurse assesses the client's understanding. Which statement made by the client indicates a correct understanding of the teaching? a. "I will take this medication with food and plenty of water." b. "I shall keep my appointment at the infusion center each week." c. "I'll limit my intake of green leafy vegetables while on this medication." d. "I must not take this medication if I have an infection or am feeling ill."

ANS: B Temsirolimus is administered as a weekly intravenous infusion. This medication blocks protein that is needed for cell division and therefore inhibits cell cycle progression. This medication is not taken orally, and clients do not need to follow a specific diet.

15. A mother rescues two of her four children from a house fire. In an emergency department, she cries, I should have gone back in to get them. I should have died, not them. Which of the following responses by the nurse is an example of reflection? A. The smoke was too thick. You couldnt have gone back in. B. Youre feeling guilty because you werent able to save your children. C. Focus on the fact that you could have lost all four of your children. D. Its best if you try not to think about what happened. Try to move on.

ANS: B The best response by the nurse is, Youre experiencing feelings of guilt because you werent able to save your children. This response utilizes the therapeutic communication technique of reflection, which identifies a clients emotional response and reflects these feelings back to the client so that they may be recognized and accepted. KEY: Cognitive Level: Application | Integrated Processes: Nursing Process: Implementation | Client Need: Psychosocial Integrity

10. A client with a new tracheostomy is being seen in the oncology clinic. What finding by the nurse best indicates that goals for the nursing diagnosis Impaired Self-Esteem are being met? a. The client demonstrates good understanding of stoma care. b. The client has joined a book club that meets at the library. c. Family members take turns assisting with stoma care. d. Skin around the stoma is intact without signs of infection.

ANS: B The client joining a book club that meets outside the home and requires him or her to go out in public is the best sign that goals for Impaired Self-Esteem are being met. The other findings are all positive signs but do not relate to this nursing diagnosis.

9. After teaching a client with hypertension secondary to renal disease, the nurse assesses the client's understanding. Which statement made by the client indicates a need for additional teaching? a. "I can prevent more damage to my kidneys by managing my blood pressure." b. "If I have increased urination at night, I need to drink less fluid during the day." c. "I need to see the registered dietitian to discuss limiting my protein intake." d. "It is important that I take my antihypertensive medications as directed."

ANS: B The client should not restrict fluids during the day due to increased urination at night. Clients with renal disease may be prescribed fluid restrictions. These clients should be assessed thoroughly for potential dehydration. Increased nocturnal voiding can be decreased by consuming fluids earlier in the day. Blood pressure control is needed to slow the progression of renal dysfunction. When dietary protein is restricted, refer the client to the registered dietitian as needed.

18. A pulmonary nurse cares for clients who have chronic obstructive pulmonary disease (COPD). Which client should the nurse assess first? a. A 46-year-old with a 30pack-year history of smoking b. A 52-year-old in a tripod position using accessory muscles to breathe c. A 68-year-old who has dependent edema and clubbed fingers d. A 74-year-old with a chronic cough and thick, tenacious secretions

ANS: B The client who is in a tripod position and using accessory muscles is working to breathe. This client must be assessed first to establish how well the client is breathing and provide interventions to minimize respiratory failure. The other clients are not in acute distress.

A pulmonary nurse cares for clients who have chronic obstructive pulmonary disease (COPD). Which client should the nurse assess first? a. A 46-year-old with a 30-pack-year history of smoking b. A 52-year-old in a tripod position using accessory muscles to breathe c. A 68-year-old who has dependent edema and clubbed fingers d. A 74-year-old with a chronic cough and thick, tenacious secretions

ANS: B The client who is in a tripod position and using accessory muscles is working to breathe. This client must be assessed first to establish how well the client is breathing and provide interventions to minimize respiratory failure. The other clients are not in acute distress.

A nurse cares for a client who is recovering from a hemorrhoidectomy. The client states, "I need to have a bowel movement." Which action should the nurse take? a. Obtain a bedside commode for the client to use. b. Stay with the client while providing privacy. c. Make sure the call light is in reach to signal completion. d. Gather supplies to collect a stool sample for the laboratory.

ANS: B The first bowel movement after hemorrhoidectomy can be painful enough to induce syncope. The nurse should stay with the client. The nurse should instruct clients who are discharged the same day to have someone nearby when they have their first postoperative bowel movement. Making sure the call light is within reach is an important nursing action too, but it does not take priority over client safety. Obtaining a bedside commode and taking a stool sample are not needed in this situation.

18. What is the main goal of the working phase of the nurseclient therapeutic relationship? A. Role modeling to improve interaction with others B. Resolution of the clients problems C. Using therapeutic communication to clarify perceptions D. Helping the client access outpatient treatment

ANS: B The goal of the working phase of the nurseclient therapeutic relationship is to resolve client problems by promoting behavioral change. KEY: Cognitive Level: Comprehension | Integrated Processes: Nursing Process: Implementation | Client Need: Psychosocial Integrity

What is the main goal of the working phase of the nurse client therapeutic relationship? A. Role modeling to improve interaction with others B. Resolution of the clients problems C. Using therapeutic communication to clarify perceptions D. Helping the client access outpatient treatment

ANS: B The goal of the working phase of the nurseclient therapeutic relationship is to resolve client problems by promoting behavioral change.

5. A nurse recently admitted a client to an inpatient unit after a suicide attempt. A health-care provider orders amitriptyline (Elavil) for the client. Which intervention related to this medication should be initiated to maintain this clients safety upon discharge? A. Provide a 6-month supply of Elavil to ensure long-term compliance. B. Provide a 1-week supply of Elavil with refills contingent on follow-up appointments. C. Provide a pill dispenser as a memory aid. D. Provide education regarding the avoidance of foods containing tyramine.

ANS: B The health-care provider should provide a 1-week supply of Elavil with refills contingent on follow-up appointments as an appropriate intervention to maintain the clients safety. Tricyclic antidepressants have a narrow therapeutic range and can be used in overdose to commit suicide. Distributing limited amounts of the medication decreases this potential.

22. Which statement indicates that the nurse is acting as an advocate for a client who was hospitalized after a suicide attempt and is now nearing discharge? A. I must observe you continually for 1 hour in order to keep you safe. B. Lets confer with the treatment team about the resources that you may need after discharge. C. You must have been very upset to do what you did today. D. Are you currently thinking about harming yourself?

ANS: B The nurse is functioning in an advocacy role when collaborating with the client and treatment team to discuss client problems and needs.

6. A nurse observes that a clients anteroposterior (AP) chest diameter is the same as the lateral chest diameter. Which question should the nurse ask the client in response to this finding? a. Are you taking any medications or herbal supplements? b. Do you have any chronic breathing problems? c. How often do you perform aerobic exercise? d. What is your occupation and what are your hobbies?

ANS: B The normal chest has a lateral diameter that is twice as large as the AP diameter. When the AP diameter approaches or exceeds the lateral diameter, the client is said to have a barrel chest. Most commonly, barrel chest occurs as a result of a long-term chronic airflow limitation problem, such as chronic obstructive pulmonary disease or severe chronic asthma. It can also be seen in people who have lived at a high altitude for many years. Therefore, an AP chest diameter that is the same as the lateral chest diameter should be rechecked but is not as indicative of underlying disease processes as an AP diameter that exceeds the lateral diameter. Medications, herbal supplements, and aerobic exercise are not associated with a barrel chest. Although occupation and hobbies may expose a client to irritants that can cause chronic lung disorders and barrel chest, asking about chronic breathing problems is more direct and should be asked first.

2. A hungry, homeless client, diagnosed with schizophrenia, refuses to participate in an admission interview. When the nurse postpones the admission interview, verbally assures safety, and provides a warm meal, he or she is promoting which of the following? A. Sympathy B. Trust C. Veracity D. Manipulation

ANS: B The nurse is promoting trust by postponing the admission interview, assuring safety, and providing a warm meal. Trust implies a feeling of confidence that a person is reliable and sincere and has integrity and veracity. Trustworthiness is demonstrated through nursing interventions that convey a sense of warmth and caring to the client. KEY: Cognitive Level: Application | Integrated Processes: Nursing Process: Implementation | Client Need: Psychosocial Integrity

A hungry, homeless client, diagnosed with schizophrenia, refuses to participate in an admission interview. When the nurse postpones the admission interview, verbally assures safety, and provides a warm meal, he or she is promoting which of the following? A. Sympathy B. Trust C. Veracity D. Manipulation

ANS: B The nurse is promoting trust by postponing the admission interview, assuring safety, and providing a warm meal. Trust implies a feeling of confidence that a person is reliable and sincere and has integrity and veracity. Trustworthiness is demonstrated through nursing interventions that convey a sense of warmth and caring to the client.

11. If an individual is two-faced, which characteristic essential to the development of a therapeutic relationship should a nurse identify as missing? A. Respect B. Genuineness C. Sympathy D. Rapport

ANS: B The nurse should identify that genuineness is missing in the relationship. Genuineness refers to an individuals ability to be open and honest and maintain congruence between what is felt and what is communicated. Genuineness is essential to establishing trust in a relationship. KEY: Cognitive Level: Comprehension | Integrated Processes: Nursing Process: Evaluation | Client Need: Psychosocial Integrity

8. A nurse maintains an uncrossed arm and leg posture. This nonverbal behavior is reflective of which letter of the SOLER acronym for active listening? A. S B. O C. L D. E E. R

ANS: B The nurse should identify that maintaining an uncrossed arm and leg posture is nonverbal behavior that reflects the O in the active-listening acronym SOLER. The acronym SOLER includes sitting squarely facing the client (S), open posture when interacting with the client (O), leaning forward toward the client (L), establishing eye contact (E), and relaxing (R). KEY: Cognitive Level: Application | Integrated Processes: Communication and Documentation | Client Need: Psychosocial Integrity

13. A client diagnosed with paranoid schizophrenia becomes agitated when asked to play a game. The client responds, Do you want to be my girlfriend? Which nursing response is most appropriate? A. You are upset now. It would be best if you go to your room until you feel better. B. Remember, we have a professional relationship. Are you feeling uncomfortable? C. We have discussed this before. I am not allowed to date clients. D. I think you should discuss your fantasies with your therapist.

ANS: B The nurse should promote the clients insight and perception of reality by confirming appropriate roles in the nurseclient relationship and identifying what is troubling the client in this situation. KEY: Cognitive Level: Analysis | Integrated Processes: Nursing Process: Implementation | Client Need: Psychosocial Integrity

A client diagnosed with paranoid schizophrenia becomes agitated when asked to play a game. The client responds, Do you want to be my girlfriend? Which nursing response is most appropriate? A. You are upset now. It would be best if you go to your room until you feel better. B. Remember, we have a professional relationship. Are you feeling uncomfortable? C. We have discussed this before. I am not allowed to date clients. D. I think you should discuss your fantasies with your therapist.

ANS: B The nurse should promote the clients insight and perception of reality by confirming appropriate roles in the nurseclient relationship and identifying what is troubling the client in this situation.

20. Which nursing action demonstrates the role of the teacher in a therapeutic milieu? A. The nurse implements a self-affirmation exercise during a one-to-one client interaction. B. The nurse holds a group meeting to present common side effects of psychiatric medications. C. The nurse introduces the concept of fair play while playing cards with a group of clients. D. The nurse models adaptive and effective coping mechanisms with clients on the psychiatric unit.

ANS: B The nurse, in the role of teacher, identifies learning needs and provides information required by the client or family to improve the clients health. KEY: Cognitive Level: Application | Integrated Processes: Nursing Process: Implementation | Client Need: Psychosocial Integrity

Which nursing action demonstrates the role of the teacher in a therapeutic milieu? A. The nurse implements a self-affirmation exercise during a one-to-one client interaction. B. The nurse holds a group meeting to present common side effects of psychiatric medications. C. The nurse introduces the concept of fair play while playing cards with a group of clients. D. The nurse models adaptive and effective coping mechanisms with clients on the psychiatric unit.

ANS: B The nurse, in the role of teacher, identifies learning needs and provides information required by the client or family to improve the clients health.

6. A client diagnosed with post-traumatic stress disorder is admitted to an inpatient psychiatric unit for evaluation and medication stabilization. Which therapeutic communication technique used by the nurse is an example of a broad opening? A. What occurred prior to the rape, and when did you go to the emergency department? B. What would you like to talk about? C. I notice you seem uncomfortable discussing this. D. How can we help you feel safe during your stay here?

ANS: B The nurses statement, What would you like to talk about? is an example of the therapeutic communication technique of giving broad openings. Using a broad opening allows the client to take the initiative in introducing the topic and emphasizes the importance of the clients role in the interaction. KEY: Cognitive Level: Application | Integrated Processes: Nursing Process: Implementation | Client Need: Psychosocial Integrity

7. Which phase of the nurseclient relationship begins when the individuals first meet and is characterized by an agreement to continue to meet and work on setting client-centered goals? A. Preinteraction B. Orientation C. Working D. Termination

ANS: B The orientation phase is when the individuals first meet and is characterized by an agreement to continue to meet and work on setting client-centered goals. There are four phases of relationship development: preinteraction, orientation, working, and termination. KEY: Cognitive Level: Comprehension | Integrated Process: Nursing Process: Implementation | Client Need: Psychosocial Integrity

Which phase of the nurseclient relationship begins when the individuals first meet and is characterized by an agreement to continue to meet and work on setting client-centered goals? A. Preinteraction B. Orientation C. Working D. Termination

ANS: B The orientation phase is when the individuals first meet and is characterized by an agreement to continue to meet and work on setting client-centered goals. There are four phases of relationship development: pre interaction, orientation, working, and termination.

Which condition would trigger the release of antidiuretic hormone (ADH)? A. Plasma osmolarity decreased secondary to overhydration. B. Plasma osmolarity increased secondary to dehydration. C. Plasma volume decreased secondary to hemorrhage. D. Plasma volume increased with edema formation.

ANS: B Antidiuretic hormone is triggered by a rising ECF osmolarity, especially hypernatremia.

4. What is the priority nursing action during the orientation (introductory) phase of the nurseclient relationship? A. Acknowledge the clients actions and generate alternative behaviors. B. Establish rapport and develop treatment goals. C. Attempt to find alternative placement. D. Explore how thoughts and feelings about this client may adversely impact care.

ANS: B The priority nursing action during the orientation phase of the nurseclient relationship should be to establish rapport and develop treatment goals. Rapport implies feelings on the part of both the nurse and the client based on respect, acceptance, a sense of trust, and a nonjudgmental attitude. It is the essential foundation of the nurseclient relationship. KEY: Cognitive Level: Comprehension | Integrated Processes: Nursing Process: Implementation | Client Need: Psychosocial Integrity

14.A client seen in the emergency department reports fever, fatigue, and dry cough but no other upper respiratory symptoms. A chest x-ray reveals mediastinal widening. What action by the nurse is best? a. Collect a sputum sample for culture by deep suctioning. b. Inform the client that antibiotics will be needed for 60 days. c. Place the client on Airborne Precautions immediately. d. Tell the client that directly observed therapy is needed.

ANS: B This client has manifestations of early inhalation anthrax. For treatment, after IV antibiotics are finished, oral antibiotics are continued for at least 60 days. Sputum cultures are not needed. Anthrax is not transmissible from person to person, so Standard Precautions are adequate. Directly observed therapy is often used for tuberculosis.

10. After years of dialysis, an 84-year-old states, Im exhausted, depressed, and done with these attempts to keep me alive. Which question should the nurse ask the spouse when preparing a discharge plan of care? A. Have there been any changes in appetite or sleep? B. How often is your spouse left alone? C. Has your spouse been following a diet and exercise program consistently? D. How would you characterize your relationship with your spouse?

ANS: B This client has many risk factors for suicide. The client should have increased supervision to decrease likelihood of self-harm.

4. A client is admitted with acute kidney injury (AKI) and a urine output of 2000 mL/day. What is the major concern of the nurse regarding this client's care? a. Edema and pain b. Electrolyte and fluid imbalance c. Cardiac and respiratory status d. Mental health status

ANS: B This client may have an inflammatory cause of AKI with proteins entering the glomerulus and holding the fluid in the filtrate, causing polyuria. Electrolyte loss and fluid balance is essential. Edema and pain are not usually a problem with fluid loss. There could be changes in the client's cardiac, respiratory, and mental health status if the electrolyte imbalance is not treated.

11. A nursing instructor is teaching about suicide in the elderly population. Which information should the instructor include? A. Elderly people use less lethal means to commit suicide. B. Although the elderly make up less than 13% of the population, they account for 16% of all suicides. C. Suicide is the second leading cause of death among the elderly. D. It is normal for elderly individuals to express a desire to die, because they have come to terms with their mortality.

ANS: B This factual information should be included in the nursing instructors teaching plan. An expressed desire to die is not normal in any age group.

26. After fasting from 10 p.m. the previous evening, a client finds out that the blood test has been canceled. The client swears at the nurse and states, You are incompetent! Which is the nurses best response? A. Do you believe that I was the cause of your blood test being canceled? B. I see that you are upset, but I feel uncomfortable when you swear at me. C. Have you ever thought about ways to express anger appropriately? D. Ill give you some space. Let me know if you need anything.

ANS: B This is an example of the appropriate use of feedback. Feedback should be directed toward behavior that the client has the capacity to modify. KEY: Cognitive Level: Application | Integrated Processes: Nursing Process: Implementation | Client Need: Psychosocial Integrity

After fasting from 10 p.m. the previous evening, a client finds out that the blood test has been canceled. The client swears at the nurse and states, You are incompetent! Which is the nurses best response? A. Do you believe that I was the cause of your blood test being canceled? B. I see that you are upset, but I feel uncomfortable when you swear at me. C. Have you ever thought about ways to express anger appropriately? D. I'll give you some space. Let me know if you need anything.

ANS: B This is an example of the appropriate use of feedback. Feedback should be directed toward behavior that the client has the capacity to modify.

32. Which nursing statement is a good example of the therapeutic communication technique of offering self? A. I think it would be great if you talked about that problem during our next group session. B. Would you like me to accompany you to your electroconvulsive therapy treatment? C. I notice that you are offering help to other peers in the milieu. D. After discharge, would you like to meet me for lunch to review your outpatient progress?

ANS: B This is an example of the therapeutic communication technique of offering self. Offering self makes the nurse available on an unconditional basis, increasing clients feelings of self-worth. Professional boundaries must be maintained when using the technique of offering self. KEY: Cognitive Level: Application | Integrated Processes: Nursing Process: Implementation | Client Need: Psychosocial Integrity

Which nursing statement is a good example of the therapeutic communication technique of offering self? A. I think it would be great if you talked about that problem during our next group session. B. Would you like me to accompany you to your electroconvulsive therapy treatment? C. I notice that you are offering help to other peers in the milieu. D. After discharge, would you like to meet me for lunch to review your outpatient progress?

ANS: B This is an example of the therapeutic communication technique of offering self. Offering self makes the nurse available on an unconditional basis, increasing clients feelings of self-worth. Professional boundaries must be maintained when using the technique of offering self.

31. Which therapeutic communication technique should the nurse use when communicating with a client who is experiencing auditory hallucinations? A. My sister has the same diagnosis as you and she also hears voices. B. I understand that the voices seem real to you, but I do not hear any voices. C. Why not turn up the radio so that the voices are muted. D. I wouldnt worry about these voices. The medication will make them disappear.

ANS: B This is an example of the therapeutic communication technique of presenting reality. Presenting reality is when the client has a misperception of the environment. The nurse defines reality or indicates his or her perception of the situation for the client. KEY: Cognitive Level: Application | Integrated Processes: Nursing Process: Implementation | Client Need: Psychosocial Integrity

Which therapeutic communication technique should the nurse use when communicating with a client who is experiencing auditory hallucinations? A. My sister has the same diagnosis as you and she also hears voices. B. I understand that the voices seem real to you, but I do not hear any voices. C. Why not turn up the radio so that the voices are muted. D. I wouldn't worry about these voices. The medication will make them disappear.

ANS: B This is an example of the therapeutic communication technique of presenting reality. Presenting reality is when the client has a misperception of the environment. The nurse defines reality or indicates his or her perception of the situation for the client.

33. A client slammed a door on the unit several times. The nurse responds, You seem angry. The client states, Im not angry. What therapeutic communication technique has the nurse employed, and what defense mechanism is the client unconsciously demonstrating? A. Making observations and the defense mechanism of suppression B. Verbalizing the implied and the defense mechanism of denial C. Reflection and the defense mechanism of projection D. Encouraging descriptions of perceptions and the defense mechanism of displacement

ANS: B This is an example of the therapeutic communication technique of verbalizing the implied. The nurse is putting into words what the client has only implied by words or actions. Denial is the refusal of the client to acknowledge the existence of a real situation, the feelings associated with it, or both. KEY: Cognitive Level: Application | Integrated Processes: Nursing Process: Implementation | Client Need: Psychosocial Integrity Multiple Response

A client slammed a door on the unit several times. The nurse responds, You seem angry. The client states, Im not angry. What therapeutic communication technique has the nurse employed, and what defense mechanism is the client unconsciously demonstrating? A. Making observations and the defense mechanism of suppression B. Verbalizing the implied and the defense mechanism of denial C. Reflection and the defense mechanism of projection D. Encouraging descriptions of perceptions and the defense mechanism of displacement

ANS: B This is an example of the therapeutic communication technique of verbalizing the implied. The nurse is putting into words what the client has only implied by words or actions. Denial is the refusal of the client to acknowledge the existence of a real situation, the feelings associated with it, or both.

4. After teaching a client how to perform diaphragmatic breathing, the nurse assesses the clients understanding. Which action demonstrates that the client correctly understands the teaching? a. The client lays on his or her side with his or her knees bent. b. The client places his or her hands on his or her abdomen. c. The client lays in a prone position with his or her legs straight. d. The client places his or her hands above his or her head.

ANS: B To perform diaphragmatic breathing correctly, the client should place his or her hands on his or her abdomen to create resistance. This type of breathing cannot be performed effectively while lying on the side or with hands over the head. This type of breathing would not be as effective lying prone.

After teaching a client how to perform diaphragmatic breathing, the nurse assesses the client's understanding. Which action demonstrates that the client correctly understands the teaching? a. The client lays on his or her side with his or her knees bent. b. The client places his or her hands on his or her abdomen. c. The client lays in a prone position with his or her legs straight. d. The client places his or her hands above his or her head.

ANS: B To perform diaphragmatic breathing correctly, the client should place his or her hands on his or her abdomen to create resistance. This type of breathing cannot be performed effectively while lying on the side or with hands over the head. This type of breathing would not be as effective lying prone.

The client is taking a medication for an endocrine problem that inhibits aldosterone secretion and release. To what complications of this therapy should the nurse be alert? A. Dehydration, hypokalemia B. Dehydration, hyperkalemia C. Overhydration, hyponatremia D. Overhydration, hypernatremia

ANS: B Aldosterone is a mineralocorticoid that increases the reabsorption of water and sodium in the kidney at the same time that it promotes excretion of potassium. Any drug or condition that disrupts aldosterone secretion or release increases the client's risk for excessive water loss and potassium reabsorption.

Which of the following conditions are associated with oversecretion of rennin? A. Alzheimer's disease B. Hypertension C. Diabetes mellitus D. Diabetes insipidus

ANS: B Renin is secreted when special cells in the DCT, called the macula densa, sense changes in blood volume and pressure. When the macula densa cells sense that blood volume is low, blood pressure is low, or blood sodium levels are low, renin is secreted. Renin then converts angiotensinogen into angiotensin I. This leads to a series of reactions that cause the secretion of the hormone aldosterone. This hormone increases kidney reabsorption of sodium and water, increasing blood pressure, blood volume, and blood sodium levels. Inappropriate or excessive renin secretion is a major cause of persistent hypertension.

The client's urine specific gravity is 1.018. What is the nurse's best action? A. Ask the client for a 24-hour recall of liquid intake. B. Document the finding as the only action. C. Obtain a specimen for culture. D. Notify the physician.

ANS: B This specific gravity is within the normal range for urine.

Which of the following muscle actions results in voluntary urination? A. Detrusor contraction, external sphincter contraction B. Detrusor contraction, external sphincter relaxation C. Detrusor relaxation, external sphincter contraction D. Detrusor relaxation, external sphincter relaxation

ANS: B Voiding becomes a voluntary act as a result of learned responses controlled by the cerebral cortex that cause contraction of the bladder detrusor muscle and simultaneous relaxation of the external urethral sphincter muscle.

The nurse is caring for a client who has celiac disease. Which food will the nurse remove from the client's dietary tray? (Select all that apply.) a. Rice b. Graham crackers c. Croissant d. Fresh peaches e. Chicken breast

ANS: B, C Clients with celiac disease cannot eat gluten. Foods containing gluten, such as graham crackers and toast, must be removed from the client's tray. The other foods listed do not contain gluten.

3.A client in the emergency department is taking rifampin (Rifadin) for tuberculosis. The client reports yellowing of the sclera and skin and bleeding after minor trauma. What laboratory results correlate to this condition? (Select all that apply.) a. Blood urea nitrogen (BUN): 19 mg/dL b. International normalized ratio (INR): 6.3 c. Prothrombin time: 35 seconds d. Serum sodium: 130 mEq/L e. White blood cell (WBC) count: 72,000/mm3

ANS: B, C Rifampin can cause liver damage, evidenced by the clients high INR and prothrombin time. The BUN and WBC count are normal. The sodium level is low, but that is not related to this clients problem.

5. A nurse assesses a client who has a chest tube. For which manifestations should the nurse immediately intervene? (Select all that apply.) a. Production of pink sputum b. Tracheal deviation c. Sudden onset of shortness of breath d. Pain at insertion site e. Drainage of 75 mL/hr

ANS: B, C Tracheal deviation and sudden onset of shortness of breath are manifestations of a tension pneumothorax. The nurse must intervene immediately for this emergency situation. Pink sputum is associated with pulmonary edema and is not a complication of a chest tube. Pain at the insertion site and drainage of 75 mL/hr are normal findings with a chest tube.

A nurse assesses a client who has a chest tube. For which manifestations should the nurse immediately intervene? (Select all that apply.) a. Production of pink sputum b. Tracheal deviation c. Sudden onset of shortness of breath d. Pain at insertion site e. Drainage of 75 mL/hr

ANS: B, C Tracheal deviation and sudden onset of shortness of breath are manifestations of a tension pneumothorax. The nurse must intervene immediately for this emergency situation. Pink sputum is associated with pulmonary edema and is not a complication of a chest tube. Pain at the insertion site and drainage of 75 mL/hr are normal findings with a chest tube.

A client who recently had laparoscopic surgery to treat a ruptured appendix has developed subsequent peritonitis. The client currently has two Jackson-Pratt drains placed in the abdomen. Which finding(s) would the nurse report immediately to the surgeon? (Select all that apply.) a. Serosanguineous drainage b. Fever c. Cloudy drainage d. Painful abdominal distension e. Pain level "3" on a scale of 1 to 10

ANS: B, C, D Fever, cloudy drainage, and painful abdominal distention can be signs of infection that should be immediately reported to the surgeon. Serosanguineous drainage is expected at this time, as is a manageable pain level of "3" on a 1 to 10 scale.

What is the best way for the nurse to decrease the risk of ventilator-associated pneumonia (VAP) in a ventilator-dependent client? (Select all that apply.) a. Provide prophylactic antibiotics. b. Provide frequent oral care. c. Keep the head of the bed elevated. d. Maintain good hand hygiene. e. Perform chest percussion frequently.

ANS: B, C, D Providing frequent oral care, keeping the head of the bed elevated, and maintaining good hand hygiene are currently stated as the best ways to help prevent VAP. Prophylactic antibiotics are not recommended; neither is taking the client off the ventilator. Likewise, frequent chest percussion is not stated as an intervention to decrease VAP.

23. The nurseclient therapeutic relationship includes which of the following characteristics? Select all that apply. A. Meeting the psychological needs of the nurse and the client B. Ensuring therapeutic termination C. Promoting client insight into problematic behavior D. Collaborating to set appropriate goals E. Meeting both the physical and psychological needs of the client

ANS: B, C, D, E The nurseclient therapeutic relationship should include promoting client insight into problematic behavior, collaboration to set appropriate goals, meeting the physical and psychological needs of the client, and ensuring therapeutic termination. The nurses psychological needs should not be addressed within the nurseclient relationship. KEY: Cognitive Level: Comprehension | Integrated Processes: Nursing Process: Implementation | Client Need: Psychosocial Integrity

A patient in the emergency department says, Voices say someone is stalking me. They want to kill me because I developed a cure for cancer. I have a knife and will stab anyone who is a threat. Which aspects of the patients mental health have the greatest and most immediate concern to the nurse? Select all that apply. a. Happiness b. Appraisal of reality c. Control over behavior d. Effectiveness in work e. Healthy self-concept

ANS: B, C, E The aspects of mental health of greatest concern are the patients appraisal of and control over behavior. The appraisal of reality is inaccurate. There are auditory hallucinations, delusions of persecution, and delusions of grandeur. In addition, the patients control over behavior is tenuous, as evidenced by the plan to stab anyone who seems threatening. A healthy self-concept is lacking, as evidenced by the delusion of grandeur. Data are not present to suggest that the other aspects of mental health (happiness and effectiveness in work) are of immediate concern.

1. A nurse assesses a client who has a family history of polycystic kidney disease (PKD). For which clinical manifestations should the nurse assess? (Select all that apply.) a. Nocturia b. Flank pain c. Increased abdominal girth d. Dysuria e. Hematuria f. Diarrhea

ANS: B, C, E Clients with PKD experience abdominal distention that manifests as flank pain and increased abdominal girth. Bloody urine is also present with tissue damage secondary to PKD. Clients with PKD often experience constipation, but would not report nocturia or dysuria.

4. A nurse assesses a client with chronic obstructive pulmonary disease. Which questions should the nurse ask to determine the clients activity tolerance? (Select all that apply.) a. What color is your sputum? b. Do you have any difficulty sleeping? c. How long does it take to perform your morning routine? d. Do you walk upstairs every day? e. Have you lost any weight lately?

ANS: B, C, E Difficulty sleeping could indicate worsening breathlessness, as could taking longer to perform activities of daily living. Weight loss could mean increased dyspnea as the client becomes too fatigued to eat. The color of the clients sputum would not assist in determining activity tolerance. Asking whether the client walks upstairs every day is not as pertinent as determining if the client becomes short of breath on walking upstairs, or if the client goes upstairs less often than previously.

A nurse assesses a client with chronic obstructive pulmonary disease. Which questions should the nurse ask to determine the client's activity tolerance? (Select all that apply.) a. "What color is your sputum?" b. "Do you have any difficulty sleeping?" c. "How long does it take to perform your morning routine?" d. "Do you walk upstairs every day?" e. "Have you lost any weight lately?"

ANS: B, C, E Difficulty sleeping could indicate worsening breathlessness, as could taking longer to perform activities of daily living. Weight loss could mean increased dyspnea as the client becomes too fatigued to eat. The color of the client's sputum would not assist in determining activity tolerance. Asking whether the client walks upstairs every day is not as pertinent as determining if the client becomes short of breath on walking upstairs, or if the client goes upstairs less often than previously.

Which assessment data finding for a client scheduled for total knee replacement surgery is most important for the nurse to communicate to the surgeon and the anesthesia provider before the procedure? Select all that apply. a. The oxygen saturation is 97%. b. The serum potassium level is 3.0 mEq/L (3.0 mmol/L). c. The client took a total of 1300 mg of aspirin yesterday. d. The client requests to talk with a registered dietitian about weight loss. e. The client took a regularly scheduled antihypertensive drug with a sip of water 2 hours ago. f. After receiving the preoperative medications, the client tells the nurse that he lied on the assessment form and that he really is a current smoker.

ANS: B, C, F This is a low potassium value (3.0 mEq/L) which should be communicated to the surgeon and anesthesia provider prior to surgery. Taking aspirin prior to surgery can increase the risk for bleeding. This should be communicated to the surgeon and anesthesia provider prior to surgery. The client's smoking status can change important assessment information collected by the surgeon and anesthesia provider; therefore, this should be immediately communicated. The oxygen saturation level is normal, and it is acceptable that the regularly scheduled antihypertensive was taken with a sip of water 2 hours ago.

Which white blood cell types are involved in the development of antibody-mediated immunity? (Select all that apply.) a. Basophils b. B-lymphocytes c. Cytotoxic/cytolytic T-cells d. Helper/inducer T-cells e. Macrophages f. Natural killer cells g. Neutrophils

ANS: B, D, E Basophils, cytotoxic/cytolytic T-cells, natural killer cells, and neutrophils have no role in antibody production, which is the basis of antibody-mediated immunity. Antibody production requires the interaction of macrophages, helper/inducer T-cells, and B-lymphocytes. The macrophages initially recognize and process the antigen. The helper/inducer T-cell presents to and assists the unsensitized B-lymphocyte to recognize the antigen as an invader. The B-lymphocyte then becomes sensitized to the antigen and begins producing antibodies against it.

4. A nurse assesses a client who is recovering from a nephrostomy. Which assessment findings should alert the nurse to urgently contact the health care provider? (Select all that apply.) a. Clear drainage b. Bloody drainage at site c. Client reports headache d. Foul-smelling drainage e. Urine draining from site

ANS: B, D, E After a nephrostomy, the nurse should assess the client for complications and urgently notify the provider if drainage decreases or stops, drainage is cloudy or foul-smelling, the nephrostomy sites leaks blood or urine, or the client has back pain. Clear drainage is normal. A headache would be an unrelated finding.

A client has multidrug-resistant tuberculosis (TB). What is the most important fact for the nurse to teach the client? a. "You will need to take medications longer than clients with other strains." b. "You will need to remain in the hospital until cultures are negative." c. "You will need to wear a mask when you go out in public." d. "You will need to have drug cultures done weekly."

ANS: C The client should wear a mask when out of the home environment and in crowds to prevent spread of the infection. The other statements are not accurate.

2. A nurse assesses a client who has a mediastinal chest tube. Which symptoms require the nurses immediate intervention? (Select all that apply.) a. Production of pink sputum b. Tracheal deviation c. Pain at insertion site d. Sudden onset of shortness of breath e. Drainage greater than 70 mL/hr f. Disconnection at Y site

ANS: B, D, E, F Immediate intervention is warranted if the client has tracheal deviation because this could indicate a tension pneumothorax. Sudden shortness of breath could indicate dislodgment of the tube, occlusion of the tube, or pneumothorax. Drainage greater than 70 mL/hr could indicate hemorrhage. Disconnection at the Y site could result in air entering the tubing. Production of pink sputum, oxygen saturation less than 95%, and pain at the insertion site are not signs/symptoms that would require immediate intervention.

A nurse assesses a client who has a mediastinal chest tube. Which symptoms require the nurse's immediate intervention? (Select all that apply.) a. Production of pink sputum b. Tracheal deviation c. Pain at insertion site d. Sudden onset of shortness of breath e. Drainage greater than 70 mL/hr f. Disconnection at Y site

ANS: B, D, E, F Immediate intervention is warranted if the client has tracheal deviation because this could indicate a tension pneumothorax. Sudden shortness of breath could indicate dislodgment of the tube, occlusion of the tube, or pneumothorax. Drainage greater than 70 mL/hr could indicate hemorrhage. Disconnection at the Y site could result in air entering the tubing. Production of pink sputum, oxygen saturation less than 95%, and pain at the insertion site are not signs/symptoms that would require immediate intervention.

The nurse is caring for a client who is suspected of having severe acute respiratory syndrome (SARS). What actions by the nurse are most appropriate? (Select all that apply.) a. Wash hands when entering the client's room and use Standard Precautions. b. Wear a gown and goggles when entering the client's room. c. Teach the client to wear a mask at all times when someone is in the room. d. Use a disposable particulate mask respirator when the client is coughing. e. Keep the door to the client's room open to allow close monitoring. f. Place the client in a negative airflow room, if available in the facility.

ANS: B, D, F The nurse should follow Airborne Precautions when caring for clients suspected of SARS. Wear a gown and goggles when in the room and caring for the client. Use a disposable particulate mask respirator if the client is coughing, or if particles are being aerosolized. Handwashing and Standard Precautions are not enough. The client does not have to wear a mask while others are in the room because they should be protecting themselves by using Airborne Precautions.

Which client statement indicates that termination of the therapeutic nurse client relationship has been handled successfully? A. I know I can count on you for continued support. B. I am looking forward to discharge, but I am surprised that we will no longer work together. C. Reviewing the changes that have happened during our time together has helped me put things in perspective. D. I don't know how comfortable I will feel when talking to someone else.

ANS: C

2. A nurse assesses a client after an open lung biopsy. Which assessment finding is matched with the correct intervention? a. Client states he is dizzy. Nurse applies oxygen and pulse oximetry. b. Clients heart rate is 55 beats/min. Nurse withholds pain medication. c. Client has reduced breath sounds. Nurse calls physician immediately. d. Clients respiratory rate is 18 breaths/min. Nurse decreases oxygen flow rate.

ANS: C A potentially serious complication after biopsy is pneumothorax, which is indicated by decreased or absent breath sounds. The physician needs to be notified immediately. Dizziness after the procedure is not an expected finding. If the clients heart rate is 55 beats/min, no reason is known to withhold pain medication. A respiratory rate of 18 breaths/min is a normal finding and would not warrant changing the oxygen flow rate.

A client is scheduled for surgery at noon. The surgeon is delayed and the surgery is now scheduled for 3:00 PM. How will the nurse plan to administer the preoperative prophylactic antibiotic? a. Give at noon as originally prescribed. b. Cancel orders; preoperative prophylactic antibiotics are given optionally. c. Adjust the administration time to be given within one hour prior to surgery. d. Hold the preoperative antibiotic so it can be administered immediately following surgery.

ANS: C According to the Surgical Care Improvement Project (SCIP) guidelines, prophylactic antibiotics should be given within one hour before the surgical incision

Which is the nurse's best response to an older adult client who is hesitant to take the pneumococcal vaccination and influenza vaccine in the same year? a. "You need both injections. A risk factor for getting pneumonia is infection with influenza." b. "Take both injections. They will protect you against respiratory problems for this year." c. "The flu shot may protect you against influenza but not against bacteria that cause pneumonia." d. "You should get the pneumococcal vaccination so you won't infect other people."

ANS: C Although influenza can lead to pneumonia, and preventing influenza with a flu shot reduces the risk for a secondary pneumonia, bacterial pneumonia can be acquired without influenza as a precipitating event and can be life threatening. Getting both injections will not protect the client from respiratory problems, nor will it prevent the client from being infectious to other people.

When a new bill introduced in Congress reduces funding for care of persons with mental illness, a group of nurses writes letters to their elected representatives in opposition to the legislation. Which role have the nurses fulfilled? a. Recovery b. Attending c. Advocacy d. Evidence-based practice

ANS: C An advocate defends or asserts anothers cause, particularly when the other person lacks the ability to do that for self. Examples of individual advocacy include helping patients understand their rights or make decisions. On a community scale, advocacy includes political activity, public speaking, and publication in the interest of improving the human condition. Since funding is necessary to deliver quality programming for persons with mental illness, the letter- writing campaign advocates for that cause on behalf of patients who are unable to articulate their own needs

Which comment best indicates that a patient perceived the nurse was caring? My nurse: a. always asks me which type of juice I want to help me swallow my medication. b. explained my treatment plan to me and asked for my ideas about how to make it better. c. spends time listening to me talk about my problems. That helps me feel like I am not alone. d. told me that if I take all the medicines the doctor prescribes, then I will get discharged sooner.

ANS: C Caring evidences empathetic understanding as well as competency. It helps change pain and suffering into a shared experience, creating a human connection that alleviates feelings of isolation. The distracters give examples of statements that demonstrate advocacy or giving advice.

Which disorder is a culture-bound syndrome? a. Epilepsy c. Running amok b. Schizophrenia d. Major depression

ANS: C Culture-bound syndromes occur in specific sociocultural contexts and are easily recognized by people in those cultures. A syndrome recognized in parts of Southeast Asia is running amok, in which a person (usually a male) runs around engaging in furious, almost indiscriminate violent behavior.

What client teaching will the nurse provide regarding postoperative leg exercises, to minimize the risk for development of deep vein thrombosis after surgery? a. Only perform each exercise one time to prevent overuse. b. Begin exercises by sitting at a 90-degree angle on the side of the bed. c. Point toes of one foot toward bottom of bed, then point toes of same leg toward their face. Repeat several times, then switch legs. d. Bend knee, and push heel of foot into the bed until the calf and thigh muscles contract. Repeat several times, then switch legs.

ANS: C Exercises should be repeated several times for each leg. Clients should begin by lying in the bed in a 45-degree angle. Pointing toes, as described, promotes circulation. Clients should push the ball of the foot into the bed until the calf and thigh muscles contract.

A client who is immune compromised develops muscle aches and fever. The client is admitted to the hospital for several days and is diagnosed with influenza. At discharge, the client asks when he can go back to work. What is the nurse's best response? a. "You should be able to return to work in 5 days." b. "You can return to work as soon as you feel ready." c. "You cannot return to work for several weeks." d. "You will need to have cultures performed before returning to work."

ANS: C Immune compromised clients are contagious for several weeks. The client should remain at home until he is not contagious.

A category 5 tornado occurred in a community of 400 people resulting in destruction of many homes and businesses. In the 2 years after this disaster, 140 individuals were diagnosed with posttraumatic stress disorder (PTSD). Which term best applies to these newly diagnosed cases? a. Prevalence b. Co-morbidity c. Incidence d. Parity

ANS: C Incidence refers to the number of new cases of mental disorders in a healthy population within a given period of time. Prevalence describes the total number of cases, new and existing, in a given population during a specific period of time, regardless of when they became ill. Parity refers to equivalence, and legislation required insurers that provide mental health coverage to offer annual and lifetime benefits at the same level provided for medical/surgical coverage. Co-morbidity refers to having more than one mental disorder at a time.

The nurse is teaching a client with pneumonia ways to clear secretions. Which intervention is the most effective? a. Administering an antitussive medication b. Administering an antiemetic medication c. Increasing fluids to 2 L/day if tolerated d. Having the client cough and deep breathe hourly

ANS: C Increasing fluids has been proven to decrease the thickness of secretions, thus allowing them to be expectorated quickly. The other interventions would not be as effective.

The spouse of a patient diagnosed with schizophrenia says, I dont understand how events from childhood have anything to do with this disabling illness. Which response by the nurse will best help the spouse understand the cause of this disorder? a. Psychological stress is the basis of most mental disorders. b. This illness results from developmental factors rather than stress. c. Research shows that this condition more likely has a biological basis. d. It must be frustrating for you that your spouse is sick so much of the time.

ANS: C Many of the most prevalent and disabling mental disorders have strong biological influences. Genetics are only one part of biological factors. Empathy does not address increasing the spouses level of knowledge about the cause of the disorder. The other distracters are not established facts.

Which description of respiratory physiologic features is correct? a. The elastic tissues of the tracheobronchial tree are the major structures responsible for gas exchange. b. The epiglottis closes during speech to divert air movement into and through the vocal cords to produce sound. c. Any problem with the right lung interferes with gas exchange and perfusion to a greater degree than a problem in the left lung. d. The left lung is responsible for approximately 60% of gas exchange and the right lung is responsible for 60% of pulmonary perfusion.

ANS: C Rationale: The right lung is larger and has more diffusing surface and more blood vessels than does the left lung. All lung functions (gas exchange and perfusion) are greater in the right lung, which means that problems in the right lung more severely affect (reduce) gas exchange than do similar problems in the left lung. Surfactant reduces surface tension rather than increases it. Gas exchange does not occur within the tracheobronchial tree because the tissues are too thick for adequate diffusion of gas in either direction.

A client who works in a day care facility is admitted to the emergency department. The client is diagnosed with pneumonia, and a sputum culture is taken. Infection with Streptococcus pneumoniae is confirmed. What is the nurse's primary action? a. Have emergency intubation equipment nearby. b. Teach the client about the treatment. c. Isolate the client. d. Perform chest physiotherapy.

ANS: C The client who works in a day care facility and is infected with Streptococcus pneumoniae may have a drug-resistant pneumonia. It is extremely important that this organism does not spread to other clients; the client should be isolated.

1. A nurse obtains the health history of a client who is recently diagnosed with lung cancer and identifies that the client has a 60pack-year smoking history. Which action is most important for the nurse to take when interviewing this client? a. Tell the client that he needs to quit smoking to stop further cancer development. b. Encourage the client to be completely honest about both tobacco and marijuana use. c. Maintain a nonjudgmental attitude to avoid causing the client to feel guilty. d. Avoid giving the client false hope regarding cancer treatment and prognosis.

ANS: C Smoking history includes the use of cigarettes, cigars, pipe tobacco, marijuana, and other controlled substances. Because the client may have guilt or denial about this habit, assume a nonjudgmental attitude during the interview. This will encourage the client to be honest about the exposure. Ask the client whether any of these substances are used now or were used in the past. Assess whether the client has passive exposure to smoke in the home or workplace. If the client smokes, ask for how long, how many packs per day, and whether he or she has quit smoking (and how long ago). Document the smoking history in pack-years (number of packs smoked daily multiplied by the number of years the client has smoked). Quitting smoking may not stop further cancer development. This statement would be giving the client false hope, which should be avoided, but is not as important as maintaining a nonjudgmental attitude.

A client has a white blood cell change in which the number of suppressor T-cells is well below normal and asks the nurse what type of health problem(s) could be expected as a result of this deficiency. What is the nurse's best response? a. "You will need to receive booster vaccinations more often because your ability to make antibodies is reduced." b. "Try to avoid crowds and people who are ill because you are now more susceptible to bacterial and viral infections." c. "You will be more prone to allergic reactions when exposed to allergens or drugs." d. "Your risk for cancer development is increased."

ANS: C Suppressor T-cells have the opposite action of helper/inducer T-cells. For optimal CMI, then, a balance between helper/inducer T-cell activity and suppressor T-cell activity must be maintained. This balance occurs when the helper/inducer T-cells outnumber the suppressor T-cells by a ratio of 2:1. When this ratio increases, indicating that helper/inducer T-cells vastly outnumber the suppressor cells, in this case because of way too few suppressor T-cells, overreactions can occur. These include allergies to almost anything, including drugs. Some of these overreactions are tissue damaging and dangerous, as well as unpleasant.

The newly employed nurse received a bacillus Calmette-Guérin (BCG) vaccine before moving to the United States. The nurse needs to receive a tuberculin (TB) test as part of the pre-employment physical. What does the nurse do? a. The nurse should not receive the tuberculin test. b. The nurse will need a two-step TB test. c. The nurse will need a chest x-ray instead. d. A physician should examine the nurse before the TB test is given.

ANS: C The bacillus Calmette-Guérin (BCG) vaccine contains attenuated tubercle bacilli and is used in many countries to produce increased resistance to TB. The nurse will have a positive skin test. The client should be evaluated for TB with a chest x-ray. A physician examination is not necessary.

What is the priority nursing intervention when caring for a client with severe acute respiratory syndrome (SARS)? a. Maintaining Standard Precautions b. Administering antibiotics c. Assessing oxygenation d. Making sure the client stays hydrated

ANS: C The client with SARS can rapidly develop hypoxia. Assessing oxygenation is a priority because intubation and mechanical ventilation may be needed. Maintaining precautions is essential for preventing the spread of this illness, but oxygenation and client safety are the highest priorities. Antibiotics are administered if bacterial pneumonia occurs with this disease. Hydration is important to make sure secretions stay liquefied; this is also secondary to oxygenation.

Which client statement regarding appropriate pain control requires nursing intervention? a. "I will listen to music when I feel pain." b. "Before exercise, I will be sure I have taken my medication." c. "If the prescribed dose of medication doesn't help my pain, I'll take an extra dose." d. "I plan to keep a pain diary so that I can see trends about when my pain worsens."

ANS: C The nurse should intervene to discuss appropriate doses of medication, and risks associated with taking more medication than has been prescribed. The nurse should also educate the client to call the health care provider if pain is not being adequately managed with the prescribed pain reliever.

A nurse assesses a client with diabetes mellitus 3 hours after a surgical procedure and notes the client's breath has a "fruity" odor. Which action should the nurse take? A. Encourage the client to use an incentive spirometer B. Increase the client's intravenous fluid flow rate C. Consult the provider to test for ketoacidosis D. Perform meticulous pulmonary hygiene care

Answer: C

21. Which client data indicate that a suicidal client is participating in a plan for safety? A. Compliance with antidepressant therapy B. A mood rating of 9/10 C. Disclosing a plan for suicide to staff D. Expressing feelings of hopelessness to nurse

ANS: C A degree of the responsibility for the suicidal clients safety is given to the client. When a client shares with staff a plan for suicide, the client is participating in a plan for safety by communicating thoughts of self-harm that would initiate interventions to prevent suicide.

23. A client is newly admitted to an inpatient psychiatric unit. Which of the following is most critical to assess when determining risk for suicide? A. Family history of depression B. The clients orientation to reality C. The clients history of suicide attempts D. Family support systems

ANS: C A history of suicide attempts places a client at a higher risk for current suicide behaviors. Knowing this specific data will alert the nurse to the clients risk.

A nurse prepares a client for a colonoscopy scheduled for tomorrow. The client states, "My doctor told me that the fecal occult blood test was negative for colon cancer. I don't think I need the colonoscopy and would like to cancel it." How should the nurse respond? a. "Your doctor should not have given you that information prior to the colonoscopy." b. "The colonoscopy is required due to the high percentage of false negatives with the blood test." c. "A negative fecal occult blood test does not rule out the possibility of colon cancer." d. "I will contact your doctor so that you can discuss your concerns about the procedure."

ANS: C A negative result from a fecal occult blood test does not completely rule out the possibility of colon cancer. To determine whether the client has colon cancer, a colonoscopy should be performed so the entire colon can be visualized and a tissue sample taken for biopsy. The client may want to speak with the provider, but the nurse should address the client's concerns prior to contacting the provider.

12. A nurse cares for a client who is infected with Burkholderia cepacia. Which action should the nurse take first when admitting this client to a pulmonary care unit? a. Instruct the client to wash his or her hands after contact with other people. b. Implement Droplet Precautions and don a surgical mask. c. Keep the client isolated from other clients with cystic fibrosis. d. Obtain blood, sputum, and urine culture specimens.

ANS: C Burkholderia cepacia infection is spread through casual contact between cystic fibrosis clients, thus the need for these clients to be separated from one another. Strict isolation measures will not be necessary. Although the client should wash his or her hands frequently, the most important measure that can be implemented on the unit is isolation of the client from other clients with cystic fibrosis. There is no need to implement Droplet Precautions or don a surgical mask when caring for this client. Obtaining blood, sputum, and urine culture specimens will not provide information necessary to care for a client with Burkholderia cepacia infection.

14. A nurse cares for a client who has pyelonephritis. The client states, "I am embarrassed to talk about my symptoms." How should the nurse respond? a. "I am a professional. Your symptoms will be kept in confidence." b. "I understand. Elimination is a private topic and shouldn't be discussed." c. "Take your time. It is okay to use words that are familiar to you." d. "You seem anxious. Would you like a nurse of the same gender to care for you?"

ANS: C Clients may be uncomfortable discussing issues related to elimination and the genitourinary area. The nurse should encourage the client to use language that is familiar to the client. The nurse should not make promises that cannot be kept, like keeping the client's symptoms confidential. The nurse must assess the client and cannot take the time to stop the discussion or find another nurse to complete the assessment.

A nurse teaches a client who is recovering from a colon resection. Which statement should the nurse include in this client's plan of care? a. "You may experience nausea and vomiting for the first few weeks." b. "Carbonated beverages can help decrease acid reflux from anastomosis sites." c. "Take a stool softener to promote softer stools for ease of defecation." d. "You may return to your normal workout schedule, including weight lifting."

ANS: C Clients recovering from a colon resection should take a stool softener as prescribed to keep stools a soft consistency for ease of passage. Nausea and vomiting are symptoms of intestinal obstruction and perforation and should be reported to the provider immediately. The client should be advised to avoid gas-producing foods and carbonated beverages, and avoid lifting heavy objects or straining on defecation.

6. The nurse is caring for a client with lung cancer who states, I dont want any pain medication because I am afraid Ill become addicted. How should the nurse respond? a. I will ask the provider to change your medication to a drug that is less potent. b. Would you like me to use music therapy to distract you from your pain? c. It is unlikely you will become addicted when taking medicine for pain. d. Would you like me to give you acetaminophen (Tylenol) instead?

ANS: C Clients should be encouraged to take their pain medications; addiction usually is not an issue with a client in pain. The nurse would not request that the pain medication be changed unless it was not effective. Other methods to decrease pain can be used, in addition to pain medication.

12. A nurse teaches a client who is prescribed nicotine replacement therapy. Which statement should the nurse include in this clients teaching? a. Make a list of reasons why smoking is a bad habit. b. Rise slowly when getting out of bed in the morning. c. Smoking while taking this medication will increase your risk of a stroke. d. Stopping this medication suddenly increases your risk for a heart attack.

ANS: C Clients who smoke while using drugs for nicotine replacement therapy increase the risk of stroke and heart attack. Nurses should teach clients not to smoke while taking this drug. The other responses are inappropriate.

9. A nurse is teaching a client who has cystic fibrosis (CF). Which statement should the nurse include in this clients teaching? a. Take an antibiotic each day. b. Contact your provider to obtain genetic screening. c. Eat a well-balanced, nutritious diet. d. Plan to exercise for 30 minutes every day.

ANS: C Clients with CF often are malnourished due to vitamin deficiency and pancreatic malfunction. Maintaining nutrition is essential. Daily antibiotics and daily exercise are not essential actions. Genetic screening would not help the client manage CF better.

4. A nurse cares for a middle-aged female client with diabetes mellitus who is being treated for the third episode of acute pyelonephritis in the past year. The client asks, "What can I do to help prevent these infections?" How should the nurse respond? a. "Test your urine daily for the presence of ketone bodies and proteins." b. "Use tampons rather than sanitary napkins during your menstrual period." c. "Drink more water and empty your bladder more frequently during the day." d. "Keep your hemoglobin A1c under 9% by keeping your blood sugar controlled."

ANS: C Clients with long-standing diabetes mellitus are at risk for pyelonephritis for many reasons. Chronically elevated blood glucose levels spill glucose into the urine, changing the pH and providing a favorable climate for bacterial growth. The neuropathy associated with diabetes reduces bladder tone and reduces the client's sensation of bladder fullness. Thus, even with large amounts of urine, the client voids less frequently, allowing stasis and overgrowth of microorganisms. Increasing fluid intake (specifically water) and voiding frequently prevent stasis and bacterial overgrowth. Testing urine and using tampons will not help prevent pyelonephritis. A hemoglobin A1c of 9% is too high.

16. A nurse cares for a client with chronic obstructive pulmonary disease (COPD) who appears thin and disheveled. Which question should the nurse ask first? a. Do you have a strong support system? b. What do you understand about your disease? c. Do you experience shortness of breath with basic activities? d. What medications are you prescribed to take each day?

ANS: C Clients with severe COPD may not be able to perform daily activities, including bathing and eating, because of excessive shortness of breath. The nurse should ask the client if shortness of breath is interfering with basic activities. Although the nurse should know about the clients support systems, current knowledge, and medications, these questions do not address the clients appearance.

A nurse cares for a client with chronic obstructive pulmonary disease (COPD) who appears thin and disheveled. Which question should the nurse ask first? a. "Do you have a strong support system?" b. "What do you understand about your disease?" c. "Do you experience shortness of breath with basic activities?" d. "What medications are you prescribed to take each day?"

ANS: C Clients with severe COPD may not be able to perform daily activities, including bathing and eating, because of excessive shortness of breath. The nurse should ask the client if shortness of breath is interfering with basic activities. Although the nurse should know about the client's support systems, current knowledge, and medications, these questions do not address the client's appearance.

19. The nurse is teaching a client with chronic obstructive pulmonary disease who has been prescribed continuous oxygen therapy at home. Which statement indicates the client correctly understands the teaching? a. I plan to wear my oxygen when I exercise and feel short of breath. b. I will use my portable oxygen when grilling burgers in the backyard. c. I plan to use cotton balls to cushion the oxygen tubing on my ears. d. I will only smoke while I am wearing my oxygen via nasal cannula.

ANS: C Cotton balls can decrease pressure ulcers from the oxygen tubing. Continuous oxygen orders mean the client should wear the oxygen at all times. Oxygen fuels a fire. Wearing oxygen while grilling and smoking increases the risk for fire.

The nurse is teaching a client with chronic obstructive pulmonary disease who has been prescribed continuous oxygen therapy at home. Which statement indicates the client correctly understands the teaching? a. "I plan to wear my oxygen when I exercise and feel short of breath." b. "I will use my portable oxygen when grilling burgers in the backyard." c. "I plan to use cotton balls to cushion the oxygen tubing on my ears." d. "I will only smoke while I am wearing my oxygen via nasal cannula."

ANS: C Cotton balls can decrease pressure ulcers from the oxygen tubing. Continuous oxygen orders mean the client should wear the oxygen at all times. Oxygen fuels a fire. Wearing oxygen while grilling and smoking increases the risk for fire.

22. A nurse cares for a female client who has a family history of cystic fibrosis. The client asks, Will my children have cystic fibrosis? How should the nurse respond? a. Since many of your family members are carriers, your children will also be carriers of the gene. b. Cystic fibrosis is an autosomal recessive disorder. If you are a carrier, your children will have the disorder. c. Since you have a family history of cystic fibrosis, I would encourage you and your partner to be tested. d. Cystic fibrosis is caused by a protein that controls the movement of chloride. Adjusting your diet will decrease the spread of this disorder.

ANS: C Cystic fibrosis is an autosomal recessive disorder in which both gene alleles must be mutated for the disorder to be expressed. The nurse should encourage both the client and partner to be tested for the abnormal gene. The other statements are not true.

18. A nurse is caring for four clients diagnosed with major depressive disorder. When considering each clients belief system, the nurse should conclude which client would potentially be at highest risk for suicide? A. Roman Catholic B. Protestant C. Atheist D. Muslim

ANS: C Depressed men and women who consider themselves affiliated with a religion are less likely to attempt suicide than their nonreligious counterparts.

A tearful, anxious patient at the outpatient clinic reports, "I should be dead." The initial task of the nurse conducting the assessment interview is to: a. assess lethality of suicide plan. b. encourage expression of anger. c. establish rapport with the patient. d. determine risk factors for suicide.

ANS: C Establishing rapport will allow the nurse to obtain relevant assessment data such as the presence of a suicide plan, the lethality of a suicide plan, and the presence of risk factors for suicide.

1.A nurse working in a geriatric clinic sees clients with cold symptoms and rhinitis. Which drug would be appropriate to teach these clients to take for their symptoms? a. Chlorpheniramine (Chlor- Trimeton) b. Diphenhydramine (Benadryl) c. Fexofenadine (Allegra) d. Hydroxyzine (Vistaril)

ANS: C First-generation antihistamines are not appropriate for use in the older population. These drugs include chlorpheniramine, diphenhydramine, and hydroxyzine. Fexofenadine is a second-generation antihistamine.

A nurse working in a geriatric clinic sees clients with "cold" symptoms and rhinitis. Which drug would be appropriate to teach these clients to take for their symptoms? a. Chlorpheniramine (Chlor-Trimeton) b. Diphenhydramine (Benadryl) c. Fexofenadine (Allegra) d. Hydroxyzine (Vistaril)

ANS: C First-generation antihistamines are not appropriate for use in the older population. These drugs include chlorpheniramine, diphenhydramine, and hydroxyzine. Fexofenadine is a second-generation antihistamine.

13. The nurse assesses the client using the device pictured below to deliver 50% O2: The nurse finds the mask fits snugly, the skin under the mask and straps is intact, and the flow rate of the oxygen is 3 L/min. What action by the nurse is best? a. Assess the clients oxygen saturation. b. Document these findings in the chart. c. Immediately increase the flow rate. d. Turn the flow rate down to 2 L/min.

ANS: C For the Venturi mask to deliver high flow of oxygen, the flow rate must be set correctly, usually between 4 and 10 L/min. The clients flow rate is too low and the nurse should increase it. After increasing the flow rate, the nurse assesses the oxygen saturation and documents the findings.

12.A nurse has educated a client on isoniazid (INH). What statement by the client indicates teaching has been effective? a. I need to take extra vitamin C while on INH. b. I should take this medicine with milk or juice. c. I will take this medication on an empty stomach. d. My contact lenses will be permanently stained.

ANS: C INH needs to be taken on an empty stomach, either 1 hour before or 2 hours after meals. Extra vitamin B needs to be taken while on the drug. Staining of contact lenses commonly occurs while taking rifampin (Rifadin).

Which understanding about individuals who attempt suicide will help a nurse plan the care for a suicidal patient? Every suicidal person should be considered: a. mentally ill. b. intent on dying. c. cognitively impaired. d. experiencing hopelessness.

ANS: D Hopelessness is the characteristic common among people who attempt suicide. The incorrect options reflect myths about suicide. Not all who attempt suicide are intent on dying. Not all are mentally ill or cognitively impaired.

25. The nurse is teaching a client how to increase the flow of dialysate into the peritoneal cavity during dialysis. Which statement by the client demonstrates a correct understanding of the teaching? a. "I should leave the drainage bag above the level of my abdomen." b. "I could flush the tubing with normal saline if the flow stops." c. "I should take a stool softener every morning to avoid constipation." d. "My diet should have low fiber in it to prevent any irritation."

ANS: C Inflow and outflow problems of the dialysate are best controlled by preventing constipation. A daily stool softener is the best option for the client. The drainage bag should be below the level of the abdomen. Flushing the tubing will not help with the flow. A diet high in fiber will also help with a constipation problem.

3. After teaching a client who is prescribed a long-acting beta2 agonist medication, a nurse assesses the clients understanding. Which statement indicates the client comprehends the teaching? a. I will carry this medication with me at all times in case I need it. b. I will take this medication when I start to experience an asthma attack. c. I will take this medication every morning to help prevent an acute attack. d. I will be weaned off this medication when I no longer need it.

ANS: C Long-acting beta2 agonist medications will help prevent an acute asthma attack because they are long acting. The client will take this medication every day for best effect. The client does not have to always keep this medication with him or her because it is not used as a rescue medication. This is not the medication the client will use during an acute asthma attack because it does not have an immediate onset of action. The client will not be weaned off this medication because this is likely to be one of his or her daily medications.

After teaching a client who is prescribed a long-acting beta2 agonist medication, a nurse assesses the client's understanding. Which statement indicates the client comprehends the teaching? a. "I will carry this medication with me at all times in case I need it." b. "I will take this medication when I start to experience an asthma attack." c. "I will take this medication every morning to help prevent an acute attack." d. "I will be weaned off this medication when I no longer need it."

ANS: C Long-acting beta2 agonist medications will help prevent an acute asthma attack because they are long acting. The client will take this medication every day for best effect. The client does not have to always keep this medication with him or her because it is not used as a rescue medication. This is not the medication the client will use during an acute asthma attack because it does not have an immediate onset of action. The client will not be weaned off this medication because this is likely to be one of his or her daily medications.

8. A nurse cares for a client with chronic obstructive pulmonary disease (COPD). The client states that he no longer enjoys going out with his friends. How should the nurse respond? a. There are a variety of support groups for people who have COPD. b. I will ask your provider to prescribe you with an antianxiety agent. c. Share any thoughts and feelings that cause you to limit social activities. d. Friends can be a good support system for clients with chronic disorders.

ANS: C Many clients with moderate to severe COPD become socially isolated because they are embarrassed by frequent coughing and mucus production. They also can experience fatigue, which limits their activities. The nurse needs to encourage the client to verbalize thoughts and feelings so that appropriate interventions can be selected. Joining a support group would not decrease feelings of social isolation if the client does not verbalize feelings. Antianxiety agents will not help the client with social isolation. Encouraging a client to participate in activities without verbalizing concerns also would not be an effective strategy for decreasing social isolation.

11. A college student is unable to take a final exam owing to severe test anxiety. Instead of studying, the student relieves stress by attending a movie. Which priority nursing diagnosis should a campus nurse assign for this client? A. Non-adherence R/T test taking B. Ineffective role performance R/T helplessness C. Altered coping R/T anxiety D. Powerlessness R/T fear

ANS: C Rationale: The priority nursing diagnosis for this client is altered coping R/T anxiety. The nurse should assist in implementing interventions that will improve the client's healthy coping skills and reduce anxiety. Cognitive Level: Analysis Integrated Process: Diagnosis

A nurse cares for a client with chronic obstructive pulmonary disease (COPD). The client states that he no longer enjoys going out with his friends. How should the nurse respond? a. "There are a variety of support groups for people who have COPD." b. "I will ask your provider to prescribe you with an antianxiety agent." c. "Share any thoughts and feelings that cause you to limit social activities." d. "Friends can be a good support system for clients with chronic disorders."

ANS: C Many clients with moderate to severe COPD become socially isolated because they are embarrassed by frequent coughing and mucus production. They also can experience fatigue, which limits their activities. The nurse needs to encourage the client to verbalize thoughts and feelings so that appropriate interventions can be selected. Joining a support group would not decrease feelings of social isolation if the client does not verbalize feelings. Antianxiety agents will not help the client with social isolation. Encouraging a client to participate in activities without verbalizing concerns also would not be an effective strategy for decreasing social isolation.

A nurse is caring for a client who has sleep apnea and is prescribed modafinil (Provigil). The client asks, "How will this medication help me?" How should the nurse respond? a. "This medication will treat your sleep apnea." b. "This sedative will help you to sleep at night." c. "This medication will promote daytime wakefulness." d. "This analgesic will increase comfort while you sleep."

ANS: C Modafinil is helpful for clients who have narcolepsy (uncontrollable daytime sleep) related to sleep apnea. This medication promotes daytime wakefulness.

10.A nurse is caring for several older clients in the hospital that the nurse identifies as being at high risk for healthcare-associated pneumonia. To reduce this risk, what activity should the nurse delegate to the unlicensed assistive personnel (UAP)? a. Encourage between-meal snacks. b. Monitor temperature every 4 hours. c. Provide oral care every 4 hours. d. Report any new onset of cough.

ANS: C Oral colonization by gram-negative bacteria is a risk factor for healthcare-associated pneumonia. Good, frequent oral care can help prevent this from developing and is a task that can be delegated to the UAP. Encouraging good nutrition is important, but this will not prevent pneumonia. Monitoring temperature and reporting new cough in clients is important to detect the onset of possible pneumonia but do not prevent it.

A nurse is caring for several older clients in the hospital that the nurse identifies as being at high risk for healthcare-associated pneumonia. To reduce this risk, what activity should the nurse delegate to the unlicensed assistive personnel (UAP)? a. Encourage between-meal snacks. b. Monitor temperature every 4 hours. c. Provide oral care every 4 hours. d. Report any new onset of cough.

ANS: C Oral colonization by gram-negative bacteria is a risk factor for healthcare-associated pneumonia. Good, frequent oral care can help prevent this from developing and is a task that can be delegated to the UAP. Encouraging good nutrition is important, but this will not prevent pneumonia. Monitoring temperature and reporting new cough in clients is important to detect the onset of possible pneumonia but do not prevent it.

19. During her aunt's wake, a four-year-old child runs up to the casket before a mother can stop her. An appointment is made with a nurse practitioner when the child starts twisting and pulling out hair. Which nursing diagnosis should the nurse practitioner assign to this child? A. Complicated grieving B. Altered family processes C. Ineffective coping D. Body image disturbance

ANS: C Rationale: Ineffective coping is defined as an inability to form a valid appraisal of the stressors, inadequate choices of practiced responses, and/or inability to use available resources. This child is coping with the anxiety generated by viewing her deceased aunt by pulling out hair. If this behavior continues, a diagnosis of hair-pulling disorder, or trichotillomania, may be assigned. Cognitive Level: Analysis Integrated Process: Diagnosis

A nurse teaches a client who is prescribed an insulin pump. Which statement should the nurse include in this client's discharge education? A. "Test your urine daily for ketones." B. "Use only buffered insulin in your pump." C. "Store the insulin in the freezer until you need it." D. "Change the needle every 3 days."

Answer: D

7. After teaching a client who is prescribed salmeterol (Serevent), the nurse assesses the clients understanding. Which statement by the client indicates a need for additional teaching? a. I will be certain to shake the inhaler well before I use it. b. It may take a while before I notice a change in my asthma. c. I will use the drug when I have an asthma attack. d. I will be careful not to let the drug escape out of my nose and mouth.

ANS: C Salmeterol is designed to prevent an asthma attack; it does not relieve or reverse symptoms. Salmeterol has a slow onset of action; therefore, it should not be used as a rescue drug. The drug must be shaken well because it has a tendency to separate easily. Poor technique on the clients part allows the drug to escape through the nose and mouth.

After teaching a client who is prescribed salmeterol (Serevent), the nurse assesses the client's understanding. Which statement by the client indicates a need for additional teaching? a. "I will be certain to shake the inhaler well before I use it." b. "It may take a while before I notice a change in my asthma." c. "I will use the drug when I have an asthma attack." d. "I will be careful not to let the drug escape out of my nose and mouth."

ANS: C Salmeterol is designed to prevent an asthma attack; it does not relieve or reverse symptoms. Salmeterol has a slow onset of action; therefore, it should not be used as a rescue drug. The drug must be shaken well because it has a tendency to separate easily. Poor technique on the client's part allows the drug to escape through the nose and mouth.

5. A nurse cares for a client who has developed esophagitis after undergoing radiation therapy for lung cancer. Which diet selection should the nurse provide for this client? a. Spaghetti with meat sauce, ice cream b. Chicken soup, grilled cheese sandwich c. Omelet, soft whole wheat bread d. Pasta salad, custard, orange juice

ANS: C Side effects of radiation therapy may include inflammation of the esophagus. Clients should be taught that bland, soft, high-calorie foods are best, along with liquid nutritional supplements. Tomato sauce may prove too spicy for a client with esophagitis. A grilled cheese sandwich is too difficult to swallow with this condition, and orange juice and other foods with citric acid are too caustic.

14. A new nursing graduate asks the psychiatric nurse manager how to best classify suicide. Which is the nurse managers best reply? A. Suicide is a DSM-5 diagnosis. B. Suicide is a mental disorder. C. Suicide is a behavior. D. Suicide is an antisocial affliction.

ANS: C Suicide is not a diagnosis, disorder, or affliction. It is a behavior.

21. Which client statement indicates that termination of the therapeutic nurseclient relationship has been handled successfully? A. I know I can count on you for continued support. B. I am looking forward to discharge, but I am surprised that we will no longer work together. C. Reviewing the changes that have happened during our time together has helped me put things in perspective. D. I dont know how comfortable I will feel when talking to someone else.

ANS: C Termination should begin in the orientation phase to minimize feelings of loss when the nurseclient relationship ends. Bringing a therapeutic conclusion to the relationship occurs when progress has been made toward attainment of mutually set goals. KEY: Cognitive Level: Application | Integrated Processes: Nursing Process: Evaluation | Client Need: Psychosocial Integrity

14. A nurse moving out of state speaks to a client about the need to work with a new nurse. The client states, Im not well enough to switch to a different nurse. What does this client response indicate to the nurse? A. The client is using manipulation to receive secondary gain. B. The client is using the defense mechanism of denial. C. The client is having trouble terminating the relationship. D. The client is using splitting as a way to remain dependent on the nurse.

ANS: C Termination should begin in the orientation phase to minimize feelings of loss when the nurseclient relationship ends. When a client feels sadness and loss, behaviors to delay termination may become evident. KEY: Cognitive Level: Application | Integrated Processes: Nursing Process: Evaluation | Client Need: Psychosocial Integrity

A nurse moving out of state speaks to a client about the need to work with a new nurse. The client states, Im not well enough to switch to a different nurse. What does this client response indicate to the nurse? A. The client is using manipulation to receive secondary gain. B. The client is using the defense mechanism of denial. C. The client is having trouble terminating the relationship. D. The client is using splitting as a way to remain dependent on the nurse.

ANS: C Termination should begin in the orientation phase to minimize feelings of loss when the nurseclient relationship ends. When a client feels sadness and loss, behaviors to delay termination may become evident.

Which client statement should a nurse identify as a typical response to stress most often experienced in the working phase of the nurse client relationship? A. I can't bear the thought of leaving here and failing. B. I might have a hard time working with you. You remind me of my mother. C. I cant tell my husband how I feel; he wouldn't listen anyway. D. Im not sure that I can count on you to protect my confidentiality.

ANS: C The nurse should identify that the client statement I cant tell my husband how I feel; he wouldn't listen anyway reflects resistance to change, which is a common behavior in the working phase of the nurse client relationship. The working phase includes overcoming resistant behaviors on the part of the client as the level of anxiety rises in response to discussion of painful issues.

10. The charge nurse of the medical-surgical unit is making staff assignments. Which staff member should be assigned to a client with chronic kidney disease who is exhibiting a low-grade fever and a pericardial friction rub? a. Registered nurse who just floated from the surgical unit b. Registered nurse who just floated from the dialysis unit c. Registered nurse who was assigned the same client yesterday d. Licensed practical nurse with 5 years' experience on this floor

ANS: C The client is exhibiting symptoms of pericarditis, which can occur with chronic kidney disease. Continuity of care is important to assess subtle differences in clients. Therefore, the registered nurse (RN) who was assigned to this client previously should again give care to this client. The float nurses would not be as knowledgeable about the unit and its clients. The licensed practical nurse may not have the education level of the RN to assess for pericarditis.

28. A nurse reviews these laboratory values of a client who returned from kidney transplantation 12 hours ago: Sodium 136 mEq/L Potassium 5 mEq/L Blood urea nitrogen (BUN) 44 mg/dL Serum creatinine 2.5 mg/dL What initial intervention would the nurse anticipate? a. Start hemodialysis immediately. b. Discuss the need for peritoneal dialysis. c. Increase the dose of immunosuppression. d. Return the client to surgery for exploration.

ANS: C The client may need a higher dose of immunosuppressive medication as evidenced by the elevated BUN and serum creatinine levels. This increased dose may reverse the possible acute rejection of the transplanted kidney. The client does not need hemodialysis, peritoneal dialysis, or further surgery at this point.

A nurse teaches a client who is at risk for colon cancer. Which dietary recommendation should the nurse teach this client? a. "Eat low-fiber and low-residual foods." b. "White rice and bread are easier to digest." c. "Add vegetables such as broccoli and cauliflower to your new diet." d. "Foods high in animal fat help to protect the intestinal mucosa."

ANS: C The client should be taught to modify his or her diet to decrease animal fat and refined carbohydrates. The client should also increase high-fiber foods and Brassica vegetables, including broccoli and cauliflower, which help to protect the intestinal mucosa from colon cancer.

21. A nurse cares for a client who tests positive for alpha1-antitrypsin (AAT) deficiency. The client asks, What does this mean? How should the nurse respond? a. Your children will be at high risk for the development of chronic obstructive pulmonary disease. b. I will contact a genetic counselor to discuss your condition. c. Your risk for chronic obstructive pulmonary disease is higher, especially if you smoke. d. This is a recessive gene and should have no impact on your health.

ANS: C The gene for AAT is a recessive gene. Clients with only one allele produce enough AAT to prevent chronic obstructive pulmonary disease (COPD) unless the client smokes. A client with two alleles is at high risk for COPD even if not exposed to smoke or other irritants. The client is a carrier, and children may or may not be at high risk depending on the partners AAT levels. Contacting a genetic counselor may be helpful but does not address the clients current question.

A nurse cares for a client who tests positive for alpha1-antitrypsin (AAT) deficiency. The client asks, "What does this mean?" How should the nurse respond? a. "Your children will be at high risk for the development of chronic obstructive pulmonary disease." b. "I will contact a genetic counselor to discuss your condition." c. "Your risk for chronic obstructive pulmonary disease is higher, especially if you smoke." d. "This is a recessive gene and should have no impact on your health."

ANS: C The gene for AAT is a recessive gene. Clients with only one allele produce enough AAT to prevent chronic obstructive pulmonary disease (COPD) unless the client smokes. A client with two alleles is at high risk for COPD even if not exposed to smoke or other irritants. The client is a carrier, and children may or may not be at high risk depending on the partner's AAT levels. Contacting a genetic counselor may be helpful but does not address the client's current question.

3. A client diagnosed with major depressive disorder with psychotic features hears voices commanding self- harm. The client refuses to commit to developing a plan for safety. What should be the nurses priority intervention at this time? A. Obtaining an order for locked seclusion until client is no longer suicidal B. Conducting 15-minute checks to ensure safety C. Placing the client on one-to-one observation while monitoring suicidal ideations D. Encouraging client to express feelings related to suicide

ANS: C The nurses priority intervention when a client hears voices commanding self-harm is to place the client on one-to-one observation while continuing to monitor suicidal ideation.

9. An instructor is correcting a nursing students clinical worksheet. Which instructor statement is the best example of effective feedback? A. Why did you use the clients name on your clinical worksheet? B. You were very careless to refer to your client by name on your clinical worksheet. C. I noticed that you used the clients name in your written process recording. That is a breach of confidentiality. D. It is disappointing that after being told, youre still using client names on your worksheet.

ANS: C The instructors statement, I noticed that you used the clients name in your written process recording, is an example of effective feedback. Feedback is a method of communication to help others consider a modification of behavior. Feedback should be descriptive, specific, and directed toward a behavior that the person has the capacity to modify and should impart information rather than offer advice or criticism. KEY: Cognitive Level: Application | Integrated Processes: Nursing Process: Implementation | Client Need: Psychosocial Integrity

13. A client diagnosed with dependent personality disorder states, Do you think I should move from my parents house and get a job? Which nursing response is most appropriate? A. It would be best to do that in order to increase independence. B. Why would you want to leave a secure home? C. Lets discuss and explore all of your options. D. Im afraid you would feel very guilty leaving your parents.

ANS: C The most appropriate response by the nurse is, Lets discuss and explore all of your options. In this example, the nurse is encouraging the client to formulate ideas and decide independently the appropriate course of action. KEY: Cognitive Level: Application | Integrated Processes: Nursing Process: Implementation | Client Need: Psychosocial Integrity

3. Which therapeutic communication technique is being used in this nurseclient interaction? Client: When I am anxious, the only thing that calms me down is alcohol. Nurse: Other than drinking, what alternatives have you explored to decrease anxiety? A. Reflecting B. Making observations C. Formulating a plan of action D. Giving recognition

ANS: C The nurse is using the therapeutic communication technique of formulating a plan of action to help the client explore alternatives to drinking alcohol. The use of this technique, rather than direct confrontation regarding the clients poor coping choice, may serve to prevent anger or anxiety from escalating. KEY: Cognitive Level: Application | Integrated Processes: Nursing Process: Evaluation | Client Need: Psychosocial Integrity

4. A client with a history of three suicide attempts has been taking fluoxetine (Prozac) for 1 month. The client suddenly presents with a bright affect, rates mood at 9/10, and is much more communicative. Which action should be the nurses priority at this time? A. Give the client off-unit privileges as positive reinforcement. B. Encourage the client to share mood improvement in group. C. Increase frequency of client observation. D. Request that the psychiatrist reevaluate the current medication protocol.

ANS: C The nurse should be aware that a sudden increase in mood rating and change in affect could indicate that the client is at risk for suicide and client observation should be more frequent. Suicide risk may occur early during treatment with antidepressants. The return of energy may bring about an increased ability to act out self- destructive behaviors prior to attaining the full therapeutic effect of the antidepressant medication.

A clients younger daughter is ignoring curfew. The client states, Im afraid she will get pregnant. The nurse responds, Hang in there. Dont you think she has a lot to learn about life? This is an example of which communication block? A. Requesting an explanation B. Belittling the client C. Making stereotyped comments D. Probing

ANS: C This is an example of the nontherapeutic communication block of making stereotyped comments. Clichs and trite expressions are meaningless in a therapeutic nurseclient relationship.

8. Which client statement should a nurse identify as a typical response to stress most often experienced in the working phase of the nurseclient relationship? A. I cant bear the thought of leaving here and failing. B. I might have a hard time working with you. You remind me of my mother. C. I cant tell my husband how I feel; he wouldnt listen anyway. D. Im not sure that I can count on you to protect my confidentiality.

ANS: C The nurse should identify that the client statement I cant tell my husband how I feel; he wouldnt listen anyway reflects resistance to change, which is a common behavior in the working phase of the nurseclient relationship. The working phase includes overcoming resistant behaviors on the part of the client as the level of anxiety rises in response to discussion of painful issues. KEY: Cognitive Level: Analysis | Integrated Processes: Nursing Process: Evaluation | Client Need: Psychosocial Integrity

20. A client is newly committed to an inpatient psychiatric unit. Which nursing intervention best lowers this clients risk for suicide? A. Encouraging participation in the milieu to promote hope B. Developing a strong personal relationship with the client C. Observing the client at intervals determined by assessed data D. Encouraging and redirecting the client to concentrate on happier times

ANS: C The nurse should observe the actively suicidal client continuously for the first hour after admission. After a full assessment the treatment team will determine the observation status of the client. Observation of the client allows the nurse to interrupt any observed suicidal behaviors.

6. What should be the nurses primary goal during the preinteraction phase of the nurseclient relationship? A. To evaluate goal attainment and ensure therapeutic closure B. To establish trust and formulate a contract for intervention C. To explore self-perceptions D. To promote client change

ANS: C The nurses primary goal of the preinteraction phase should be to explore self-perceptions. The nurse should be aware of how any preconceptions may affect his or her ability to care for individual clients. Another goal of the preinteraction phase is to obtain available client information. KEY: Cognitive Level: Comprehension | Integrated Processes: Nursing Process: Planning | Client Need: Psychosocial Integrity

What should be the nurses primary goal during the pre interaction phase of the nurse client relationship? A. To evaluate goal attainment and ensure therapeutic closure B. To establish trust and formulate a contract for intervention C. To explore self-perceptions D. To promote client change

ANS: C The nurses primary goal of the preinteraction phase should be to explore self-perceptions. The nurse should be aware of how any preconceptions may affect his or her ability to care for individual clients. Another goal of the preinteraction phase is to obtain available client information.

4. The nurse is interviewing a newly admitted psychiatric client. Which nursing statement is an example of offering a general lead? A. Do you know why you are here? B. Are you feeling depressed or anxious? C. Yes, I see. Go on. D. Can you chronologically order the events that led to your admission?

ANS: C The nurses statement, Yes, I see. Go on, is an example of the therapeutic communication technique of a general lead. Offering a general lead encourages the client to continue sharing information. KEY: Cognitive Level: Application | Integrated Process: Nursing Process: Implementation | Client Need: Psychosocial Integrity

A nurse assessing a client with colorectal cancer auscultates high-pitched bowel sounds and notes the presence of visible peristaltic waves. Which action should the nurse take? a. Ask if the client is experiencing pain in the right shoulder. b. Perform a rectal examination and assess for polyps. c. Contact the provider and recommend computed tomography. d. Administer a laxative to increase bowel movement activity.

ANS: C The presence of visible peristaltic waves, accompanied by high-pitched or tingling bowel sounds, is indicative of partial obstruction caused by the tumor. The nurse should contact the provider with these results and recommend a computed tomography scan for further diagnostic testing. This assessment finding is not associated with right shoulder pain; peritonitis and cholecystitis are associated with referred pain to the right shoulder. The registered nurse is not qualified to complete a rectal examination for polyps, and laxatives would not help this client.

1. A nurse discovers a clients suicide note that details the time, place, and means to commit suicide. What should be the priority nursing intervention and the rationale for this action? A. Administering lorazepam (Ativan) prn, because the client is angry about the discovery of the note B. Establishing room restrictions, because the clients threat is an attempt to manipulate the staff C. Placing this client on one-to-one suicide precautions, because the more specific the plan, the more likely the client will attempt suicide D. Calling an emergency treatment team meeting, because the clients threat must be addressed

ANS: C The priority nursing action should be to place this client on one-to-one suicide precautions, because the more specific the plan, the more likely the client will attempt suicide. The appropriate nursing diagnosis for this client would be risk for suicide.

27. The nurse instructs a client on how to correctly use an inhaler with a spacer. In which order should these steps occur? 1. Press down firmly on the canister to release one dose of medication. 2. Breathe in slowly and deeply. 3. Shake the whole unit vigorously three or four times. 4. Insert the mouthpiece of the inhaler into the nonmouthpiece end of the spacer. 5. Place the mouthpiece into your mouth, over the tongue, and seal your lips tightly around the mouthpiece. 6. Remove the mouthpiece from your mouth, keep your lips closed, and hold your breath for at least 10 seconds. a. 2, 3, 4, 5, 6, 1 b. 3, 4, 5, 1, 6, 2 c. 4, 3, 5, 1, 2, 6 d. 5, 3, 6, 1, 2, 4

ANS: C The proper order for correctly using an inhaler with a spacer is as follows. Insert the mouthpiece of the inhaler into the nonmouthpiece end of the spacer. Shake the whole unit vigorously three or four times. Place the mouthpiece into the mouth, over the tongue, and seal the lips tightly around it. Press down firmly on the canister of the inhaler to release one dose of medication into the spacer. Breathe in slowly and deeply. Remove the mouthpiece from the mouth, and, keeping the lips closed, hold the breath for at least 10 seconds. Then breathe out slowly. Wait at least 1 minute between puffs.

The nurse instructs a client on how to correctly use an inhaler with a spacer. In which order should these steps occur? 1. "Press down firmly on the canister to release one dose of medication." 2. "Breathe in slowly and deeply." 3. "Shake the whole unit vigorously three or four times." 4. "Insert the mouthpiece of the inhaler into the nonmouthpiece end of the spacer." 5. "Place the mouthpiece into your mouth, over the tongue, and seal your lips tightly around the mouthpiece." 6. "Remove the mouthpiece from your mouth, keep your lips closed, and hold your breath for at least 10 seconds." a. 2, 3, 4, 5, 6, 1 b. 3, 4, 5, 1, 6, 2 c. 4, 3, 5, 1, 2, 6 d. 5, 3, 6, 1, 2, 4

ANS: C The proper order for correctly using an inhaler with a spacer is as follows. Insert the mouthpiece of the inhaler into the nonmouthpiece end of the spacer. Shake the whole unit vigorously three or four times. Place the mouthpiece into the mouth, over the tongue, and seal the lips tightly around it. Press down firmly on the canister of the inhaler to release one dose of medication into the spacer. Breathe in slowly and deeply. Remove the mouthpiece from the mouth, and, keeping the lips closed, hold the breath for at least 10 seconds. Then breathe out slowly. Wait at least 1 minute between puffs.

A client on an inpatient psychiatric unit tells the nurse, I should have died, because I am totally worthless. In order to encourage the client to continue talking about feelings, which should be the nursing initial response? A. How would your family feel if you died? B. You feel worthless now, but that can change with time. C. Youve been feeling sad and alone for some time now? D. It is great that you have come in for help.

ANS: C This nursing statement is an example of the therapeutic communication technique of reflection. When reflection is used, questions and feelings are referred back to the client so that they may be recognized and accepted.

10. A nurse cares for a client who had a bronchoscopy 2 hours ago. The client asks for a drink of water. Which action should the nurse take next? a. Call the physician and request a prescription for food and water. b. Provide the client with ice chips instead of a drink of water. c. Assess the clients gag reflex before giving any food or water. d. Let the client have a small sip to see whether he or she can swallow.

ANS: C The topical anesthetic used during the procedure will have affected the clients gag reflex. Before allowing the client anything to eat or drink, the nurse must check for the return of this reflex.

20.A client is admitted with suspected pneumonia from the emergency department. The client went to the primary care provider a few days ago and shows the nurse the results of what the client calls an allergy test, as shown below: What action by the nurse takes priority? a. Assess the client for possible items to which he or she is allergic. b. Call the primary care providers office to request records. c. Immediately place the client on Airborne Precautions. d. Prepare to begin administration of intravenous antibiotics.

ANS: C This allergy test is actually a positive tuberculosis test. The client should be placed on Airborne Precautions immediately. The other options do not take priority over preventing the spread of the disease.

2.A nurse in a family practice clinic is preparing discharge instructions for a client reporting facial pain that is worse when bending over, tenderness across the cheeks, and postnasal discharge. What instruction will be most helpful? a. Ice packs may help with the facial pain. b. Limit fluids to dry out your sinuses. c. Try warm, moist heat packs on your face. d. We will schedule you for a computed tomography scan this week.

ANS: C This client has rhinosinusitis. Comfort measures for this condition include breathing in warm steam, hot packs, nasal saline irrigations, sleeping with the head elevated, increased fluids, and avoiding cigarette smoke. The client does not need a CT scan.

A nurse in a family practice clinic is preparing discharge instructions for a client reporting facial pain that is worse when bending over, tenderness across the cheeks, and postnasal discharge. What instruction will be most helpful? a. "Ice packs may help with the facial pain." b. "Limit fluids to dry out your sinuses." c. "Try warm, moist heat packs on your face." d. "We will schedule you for a computed tomography scan this week."

ANS: C This client has rhinosinusitis. Comfort measures for this condition include breathing in warm steam, hot packs, nasal saline irrigations, sleeping with the head elevated, increased fluids, and avoiding cigarette smoke. The client does not need a CT scan.

19. A clients younger daughter is ignoring curfew. The client states, Im afraid she will get pregnant. The nurse responds, Hang in there. Dont you think she has a lot to learn about life? This is an example of which communication block? A. Requesting an explanation B. Belittling the client C. Making stereotyped comments D. Probing

ANS: C This is an example of the nontherapeutic communication block of making stereotyped comments. Clichs and trite expressions are meaningless in a therapeutic nurseclient relationship. KEY: Cognitive Level: Application | Integrated Processes: Nursing Process: Implementation | Client Need: Psychosocial Integrity

22. The nurse says to a newly admitted client, Tell me more about what led up to your hospitalization. What is the purpose of this therapeutic communication technique? A. To reframe the clients thoughts about mental health treatment B. To put the client at ease C. To explore a subject, idea, experience, or relationship D. To communicate that the nurse is listening to the conversation

ANS: C This is an example of the therapeutic communication technique of exploring. The purpose of exploring is to delve further into the subject, idea, experience, or relationship. This technique is especially helpful with clients who tend to remain on a superficial level of communication. KEY: Cognitive Level: Application | Integrated Processes: Nursing Process: Implementation | Client Need: Psychosocial Integrity

The nurse says to a newly admitted client, Tell me more about what led up to your hospitalization. What is the purpose of this therapeutic communication technique? A. To reframe the clients thoughts about mental health treatment B. To put the client at ease C. To explore a subject, idea, experience, or relationship D. To communicate that the nurse is listening to the conversation

ANS: C This is an example of the therapeutic communication technique of exploring. The purpose of exploring is to delve further into the subject, idea, experience, or relationship. This technique is especially helpful with clients who tend to remain on a superficial level of communication.

20. Which nursing statement is a good example of the therapeutic communication technique of giving recognition? A. You did not attend group today. Can we talk about that? B. Ill sit with you until it is time for your family session. C. I notice you are wearing a new dress and you have washed your hair. D. Im happy that you are now taking your medications. They will really help.

ANS: C This is an example of the therapeutic communication technique of giving recognition. Giving recognition acknowledges and indicates awareness. This technique is more appropriate than complimenting the client, which reflects the nurses judgment. KEY: Cognitive Level: Application | Integrated Processes: Nursing Process: Implementation | Client Need: Psychosocial Integrity

Which nursing statement is a good example of the therapeutic communication technique of giving recognition? A. You did not attend group today. Can we talk about that? B. Ill sit with you until it is time for your family session. C. I notice you are wearing a new dress and you have washed your hair. D. I'm happy that you are now taking your medications. They will really help.

ANS: C This is an example of the therapeutic communication technique of giving recognition. Giving recognition acknowledges and indicates awareness. This technique is more appropriate than complimenting the client, which reflects the nurses judgment.

28. A client on an inpatient psychiatric unit tells the nurse, I should have died, because I am totally worthless. In order to encourage the client to continue talking about feelings, which should be the nursing initial response? A. How would your family feel if you died? B. You feel worthless now, but that can change with time. C. Youve been feeling sad and alone for some time now? D. It is great that you have come in for help.

ANS: C This nursing statement is an example of the therapeutic communication technique of reflection. When reflection is used, questions and feelings are referred back to the client so that they may be recognized and accepted. KEY: Cognitive Level: Application | Integrated Processes: Nursing Process: Implementation | Client Need: Psychosocial Integrity

6. A student nurse is providing tracheostomy care. What action by the student requires intervention by the instructor? a. Holding the device securely when changing ties b. Suctioning the client first if secretions are present c. Tying a square knot at the back of the neck d. Using half-strength peroxide for cleansing

ANS: C To prevent pressure ulcers and for client safety, when ties are used that must be knotted, the knot should be placed at the side of the clients neck, not in back. The other actions are appropriate.

A nurse assesses a patient who reports a 3-week history of depression and crying spells. The patient says, my business is bankrupt, and I was served with divorce papers. Which subsequent statement by the patient alerts the nurse to a concealed suicidal message? A. I wish I were dead B. Life is not worth living C. I have a plan that will fix everything D. My family will be better off without me

ANS: C Verbal clues to suicide may be overt or covert. The incorrect options are overt references to suicide. The correct option is more veiled. It alludes to the patient's suicide as being a way to "fix everything" but does not say it outright.

5. A nurse is providing care after auscultating clients breath sounds. Which assessment finding is correctly matched to the nurses primary intervention? a. Hollow sounds are heard over the trachea. The nurse increases the oxygen flow rate. b. Crackles are heard in bases. The nurse encourages the client to cough forcefully. c. Wheezes are heard in central areas. The nurse administers an inhaled bronchodilator. d. Vesicular sounds are heard over the periphery. The nurse has the client breathe deeply.

ANS: C Wheezes are indicative of narrowed airways, and bronchodilators help to open the air passages. Hollow sounds are typically heard over the trachea, and no intervention is necessary. If crackles are heard, the client may need a diuretic. Crackles represent a deep interstitial process, and coughing forcefully will not help the client expectorate secretions. Vesicular sounds heard in the periphery are normal and require no intervention.

14. When interviewing a client, which nonverbal behavior should a nurse employ? A. Maintaining indirect eye contact with the client B. Providing space by leaning back away from the client C. Sitting squarely, facing the client D. Maintaining open posture with arms and legs crossed

ANS: C When interviewing a client, the nurse should employ the nonverbal behavior of sitting squarely, facing the client. Facilitative skills for active listening can be identified by the acronym SOLER. SOLER includes sitting squarely facing the client (S), open posture when interacting with a client (O), leaning forward toward the client (L), establishing eye contact (E), and relaxing (R). KEY: Cognitive Level: Application | Integrated Processes: Nursing Process: Implementation | Client Need: Psychosocial Integrity

When interviewing a client, which nonverbal behavior should a nurse employ? A. Maintaining indirect eye contact with the client B. Providing space by leaning back away from the client C. Sitting squarely, facing the client D. Maintaining open posture with arms and legs crossed

ANS: C When interviewing a client, the nurse should employ the nonverbal behavior of sitting squarely, facing the client. Facilitative skills for active listening can be identified by the acronym SOLER. SOLER includes sitting squarely facing the client (S), open posture when interacting with a client (O), leaning forward toward the client (L), establishing eye contact (E), and relaxing (R).

17. As the client and nurse move from the orientation stage to the working stage of the therapeutic relationship, which is the nurses most therapeutic statement? A. I want to assure you that I will maintain your confidentiality. B. A long-term goal for someone your age would be to develop better job skills. C. Which identified problems would you like for us to initially address? D. I think first we need to focus on your relationship issues.

ANS: C When moving on a continuum from the orientation to working phase of the nurseclient relationship, the clients identified goals are addressed through mutual therapeutic work to promote client behavioral change. KEY: Cognitive Level: Application | Integrated Processes: Nursing Process: Implementation | Client Need: Psychosocial Integrity

As the client and nurse move from the orientation stage to the working stage of the therapeutic relationship, which is the nurses most therapeutic statement? A. I want to assure you that I will maintain your confidentiality. B. A long-term goal for someone your age would be to develop better job skills. C. Which identified problems would you like for us to initially address? D. I think first we need to focus on your relationship issues.

ANS: C When moving on a continuum from the orientation to working phase of the nurseclient relationship, the clients identified goals are addressed through mutual therapeutic work to promote client behavioral change.

Which nursing intervention below is part of the scope of an advanced practice psychiatric/mental health nurse only? a. Coordination of care b. Health teaching c. Milieu therapy d. Psychotherapy

ANS: D Psychotherapy is part of the scope of practice of an advanced practice nurse. The distracters are within a staff nurses scope of practice.

Two hours after a closed percutaneous kidney biopsy, the client reports a dramatic increase in pain. What is the nurse's best first action? A. Reposition the client on the operative side. B. Administer prescribed opioid analgesic. C. Assess pulse rate and blood pressure. D. Check the Foley catheter for kinks.

ANS: C An increase in the intensity of pain after a percutaneous kidney biopsy is a symptom of internal hemorrhage.

With a renal threshold for glucose of 220 mg/dL, what is the expected response when a client has a blood glucose level of 400 mg/dL? A. 400 mg/dL of excreted glucose in the urine B. 220 mg/dL of excreted glucose in the urine C. 180 mg/dL of glucose is excreted in the urine D. No excreted glucose in the urine

ANS: C Blood glucose is freely filtered at the glomerulus. Therefore, if a client has a blood sugar level of 400 mg/dl, the filtrate in the proximal convoluted tubule will have a glucose concentration of 400 mg/dL. With a renal threshold of 220 mg/dl, a total of 220 mg/dL of the 400 mg/dL will be reabsorbed back into the systemic circulation, and the final urine will have a glucose concentration of 180 mg/dL.

The client is going home after urography. Which instruction or precaution should the nurse teach this client? A. "Avoid direct contact with the urine for 24 hours until the radioisotope clears." B. "You are likely to experience some dribbling of urine for several weeks after this procedure." C. "Be sure to drink at least 3 L of fluids today to help eliminate the dye faster." D. "Your skin may become slightly yellow-tinged from the dye used in this procedure."

ANS: C Dyes used in urography are potentially nephrotoxic.

6. What is the result of stimulation of erythropoietin production in the kidney tissue? A. Increased blood flow to the kidney B. Inhibition of vitamin D and loss of bone density C. Increased bone marrow production of red blood cells D. Inhibition of the active transport of sodium, leading to hyponatremi

ANS: C Erythropoietin is produced in the kidney and released in response to decreased oxygen tension in the renal blood supply. Erythropoietin stimulates red blood cell (RBC) production in the bone marrow.

Confirmed by palpation and x-ray study, the client's right kidney is lower than the left kidney. What is the nurse's interpretation of this finding? A. The client has a problem involving the right kidney. B. The client has a problem involving the left kidney. C. The client has both kidneys in the normal position. D. The client is at increased risk for kidney impairment.

ANS: C Normally, the right kidney is positioned somewhat lower than the left kidney. This anatomic difference in otherwise symmetric organs is caused by liver displacement. The significance of this difference is that the right kidney is easier to palpate in an adult than is the left kidney.

The client has an elevated blood urea nitrogen (BUN) level and an increased ratio of blood urea nitrogen to creatinine. What is the nurse's interpretation of these laboratory results? A. The client probably has a urinary tract infection. B. The client may be overhydrated. C. The kidney may be hypoperfused. D. The kidney may be damaged.

ANS: C When dehydration or renal hypoperfusion exist, the BUN level rises more rapidly than the serum creatinine level, causing the ratio to be increased, even when no renal dysfunction is present.

The nurse is caring for an older adult client who reports being "afraid to get hooked" on opioid pain medication after surgery. What is the appropriate nursing response? Select all that apply. a. "No one ever gets hooked on these drugs." b. "Don't worry, I won't give you any opioid medications." c. "Have you had concerns with drug dependence in the past?" d. "Tell me what makes you most fearful about taking opioid medication." e. "There are ways we can keep you from becoming dependent on these drugs." f. "Older adults are much less likely to rely on pain medications than younger people."

ANS: C, D Asking the client a direct question about possible prior drug abuse allows the client to identify to the nurse whether he or she has had a problem in the past. Although a prior social history has been taken, it is not unthinkable that the client is just now ready to disclose his or her full history, particularly if he or she is embarrassed about historical drug dependence. Providing an open-ended statement about what makes the client fearful about taking opioids allows the client to express feelings, which will guide the nurse about the underlying reason for the client's concerns.

3. A nurse reviews laboratory results for a client with glomerulonephritis. The client's glomerular filtration rate (GFR) is 40 mL/min as measured by a 24-hour creatinine clearance. How should the nurse interpret this finding? (Select all that apply.) a. Excessive GFR b. Normal GFR c. Reduced GFR d. Potential for fluid overload e. Potential for dehydration

ANS: C, D The GFR refers to the initial amount of urine that the kidneys filter from the blood. In the healthy adult, the normal GFR ranges between 100 and 120 mL/min, most of which is reabsorbed in the kidney tubules. A GFR of 40 mL/min is drastically reduced, with the client experiencing fluid retention and risks for hypertension and pulmonary edema as a result of excess vascular fluid.

The nurse is caring for a client who has inhalation anthrax. What nursing actions are of the highest priority? (Select all that apply.) a. Placing the client in an isolation room b. Teaching the client how to use a mask c. Teaching the client about long-term antibiotic therapy d. Using handwashing and other Standard Precautions e. Reporting suspected cases to the proper authorities

ANS: C, D, E The client should not stop the drug merely because he or she has no manifestations. The client will need to be on the drug for longer than 1 month. The nurse should teach the client about long-term antibiotic therapy to help with compliance. Inhalation anthrax is not spread by person-to-person contact, so isolation would not be necessary. The client would not need a mask. Health care providers need only use handwashing and Standard Precautions. Always report inhalation anthrax to authorities because it is considered an intentional act of terrorism.

1. A nurse assesses a client with asthma and notes bilateral wheezing, decreased pulse oxygen saturation, and suprasternal retraction on inhalation. Which actions should the nurse take? (Select all that apply.) a. Administer prescribed salmeterol (Serevent) inhaler. b. Assess the client for a tracheal deviation. c. Administer oxygen to keep saturations greater than 94%. d. Perform peak expiratory flow readings. e. Administer prescribed albuterol (Proventil) inhaler.

ANS: C, E Suprasternal retraction caused by inhalation usually indicates that the client is using accessory muscles and is having difficulty moving air into the respiratory passages because of airway narrowing. Wheezing indicates a narrowed airway; a decreased pulse oxygen saturation also supports this finding. The asthma is not responding to the medication, and intervention is needed. Administration of a rescue inhaler is indicated, probably along with administration of oxygen. The nurse would not do a peak flow reading at this time, nor would a code be called. Midline trachea is a normal and expected finding.

A nurse assesses a client with asthma and notes bilateral wheezing, decreased pulse oxygen saturation, and suprasternal retraction on inhalation. Which actions should the nurse take? (Select all that apply.) a. Administer prescribed salmeterol (Serevent) inhaler. b. Assess the client for a tracheal deviation. c. Administer oxygen to keep saturations greater than 94%. d. Perform peak expiratory flow readings. e. Administer prescribed albuterol (Proventil) inhaler.

ANS: C, E Suprasternal retraction caused by inhalation usually indicates that the client is using accessory muscles and is having difficulty moving air into the respiratory passages because of airway narrowing. Wheezing indicates a narrowed airway; a decreased pulse oxygen saturation also supports this finding. The asthma is not responding to the medication, and intervention is needed. Administration of a rescue inhaler is indicated, probably along with administration of oxygen. The nurse would not do a peak flow reading at this time, nor would a code be called. Midline trachea is a normal and expected finding.

23. ___________________ refers to a nurse's behavioral and emotional response to a client. These responses may be related to unresolved feelings toward significant others from the nurse's past.

ANS: Countertransference Rationale: Countertransference refers to a nurse's behavioral and emotional response to a client. These responses may be related to unresolved feelings toward significant others from the nurse's past or they may be generated in response to transference feelings on the part of the client.

It is suspected that a client has bacterial pharyngitis. What is the best intervention? a. Administer a broad-spectrum antibiotic. b. Have the client produce a sputum specimen. c. Obtain samples for culture and sensitivity. d. Assess a rapid antigen test (RAT).

ANS: D A common cause of bacterial pharyngitis is group A streptococcal virus, which can lead to serious complications. Both RATs and culture and sensitivity can diagnose this bacterium; however, with an RAT, the health care provider can obtain results in about 15 minutes, and definitive treatment can begin much sooner. A broad-spectrum antibiotic would not be administered before it was determined whether the infection was bacterial. A sputum specimen is needed for lung infection but not for throat infection.

Which belief will best support a nurses efforts to provide patient advocacy during a multidisciplinary patient care planning session? a. All mental illnesses are culturally determined. b. Schizophrenia and bipolar disorder are cross-cultural disorders. c. Symptoms of mental disorders are unchanged from culture to culture. d. Assessment findings in mental disorders reflect a persons cultural patterns.

ANS: D A nurse who understands that a patients symptoms are influenced by culture will be able to advocate for the patient to a greater degree than a nurse who believes that culture is of little relevance. The distracters are untrue statements.

8. A nurse is caring for a client who is scheduled to undergo a thoracentesis. Which intervention should the nurse complete prior to the procedure? a. Measure oxygen saturation before and after a 12-minute walk. b. Verify that the client understands all possible complications. c. Explain the procedure in detail to the client and the family. d. Validate that informed consent has been given by the client.

ANS: D A thoracentesis is an invasive procedure with many potentially serious complications. Verifying that the client understands complications and explaining the procedure to be performed will be done by the physician or nurse practitioner, not the nurse. Measurement of oxygen saturation before and after a 12-minute walk is not a procedure unique to a thoracentesis.

A nurse is part of a multidisciplinary team working with groups of depressed patients. Half the patients receive supportive interventions and antidepressant medication. The other half receives only medication. The team measures outcomes for each group. Which type of study is evident? a. Incidence b. Prevalence c. Co-morbidity d. Clinical epidemiology

ANS: D Clinical epidemiology is a broad field that addresses studies of the natural history (or what happens if there is no treatment and the problem is left to run its course) of an illness, studies of diagnostic screening tests, and observational and experimental studies of interventions used to treat people with the illness or symptoms. Prevalence refers to numbers of new cases. Co-morbidity refers to having more than one mental disorder at a time.Incidence refers to the number of new cases of mental disorders in a healthy population within a given period. See related audience response question.

A client has pharyngitis. Which symptom helps the nurse determine whether the infection is bacterial versus viral? a. Redness in the back of the throat b. Enlarged lymph glands in the neck c. Nasal discharge d. Skin rash

ANS: D Generally a rash can appear with bacterial pharyngitis, but not with viral. The other symptoms are characteristic of both.

The nurse is caring for a client with recurrent bacterial pharyngitis. Which is the nurse's highest priority intervention? a. Assess for symptoms of human immune deficiency virus (HIV). b. Ask about exposure to allergens. c. Perform nasal cultures. d. Teach the client about antibiotic therapy

ANS: D Management of bacterial pharyngitis involves the use of antibiotics and the same supportive care provided for viral pharyngitis. Stress the importance of completing the entire antibiotic prescription, even when symptoms improve or subside. Failure to take all prescribed antibiotics is often the cause of recurrent infections. Although it is important for overall health that the client know his or her HIV status, it is not the highest priority intervention in the treatment plan. Allergens do not cause bacterial infections. Nasal cultures would not be a high priority unless the client had "failed" treatment with more than one antibiotic and was compliant with treatment.

3. A nurse assesses a clients respiratory status. Which information is of highest priority for the nurse to obtain? a. Average daily fluid intake b. Neck circumference c. Height and weight d. Occupation and hobbies

ANS: D Many respiratory problems occur as a result of chronic exposure to inhalation irritants used in a clients occupation and hobbies. Although it will be important for the nurse to assess the clients fluid intake, height, and weight, these will not be as important as determining his occupation and hobbies. Determining the clients neck circumference will not be an important part of a respiratory assessment.

Which is a priority teaching intervention for the client who is using a nicotine patch? a. "Abruptly discontinuing this patch can cause high blood pressure." b. "Abruptly discontinuing this patch can cause nausea and vomiting." c. "Smoking while using this patch increases the risk for pneumonia." d. "Smoking while using this patch increases the risk for a heart attack."

ANS: D Nicotine constricts blood vessels, increases mean arterial pressure, and increases afterload. Smoking while using a nicotine patch increases afterload to such an extent that the myocardium must work harder (with the coronary arteries constricted) and may cause a myocardial infarction. Abruptly discontinuing the patch will not necessarily cause hypertension or nausea and vomiting. Smoking while using the patch will not increase the risk for pneumonia.

Which individual is demonstrating the highest level of resilience? One who: a. is able to repress stressors. b. becomes depressed after the death of a spouse. c. lives in a shelter for two years after the home is destroyed by fire. d. takes a temporary job to maintain financial stability after loss of a permanent job.

ANS: D Resilience is closely associated with the process of adapting and helps people facing tragedies, loss, trauma, and severe stress. It is the ability and capacity for people to secure the resources they need to support their well-being. Repression and depression are unhealthy. Living in a shelter for two years shows a failure to move forward after a tragedy. See related audience response question.

A citizen at a community health fair asks the nurse, What is the most prevalent mental disorder in the United States? Select the nurses best response. a. Schizophrenia b. Bipolar disorder c. Dissociative fugue d. Alzheimers disease

ANS: D The 12-month prevalence for Alzheimers disease is 10% for persons older than 65 and 50% for persons older than 85. The prevalence of schizophrenia is 1.1% per year. The prevalence of bipolar disorder is 2.6%. Dissociative fugue is a rare disorder. See related audience response question.

The Diagnostic and Statistical Manual of Mental Disorders (DSM-5) classifies: a. deviant behaviors. c. people with mental disorders. b. present disability or distress. d. mental disorders people have.

ANS: D The DSM-5 classifies disorders people have rather than people themselves. The terminology of the tool reflects this distinction by referring to individuals with a disorder rather than as a schizophrenic or alcoholic, for example. Deviant behavior is not generally considered a mental disorder. Present disability or distress is only one aspect of the diagnosis.

The nurse is caring for several clients on a respiratory floor. The nurse should place the client with which condition in isolation? a. Fever and weight loss b. Negative QuantiFERON TB gold test c. Negative acid-fast bacillus (AFB) stain d. Positive nucleic acid amplification test (NAAT)

ANS: D The NAAT is a new rapid test for the diagnosis of tuberculosis (TB). Results are available in less than 2 hours. A positive test is conclusive for TB, and the client should be placed in isolation per facility policy. A client with a negative QuantiFERON gold test would not have tuberculosis. Likewise, a client with a negative AFB would not have tuberculosis. The client with fever and weight loss could have tuberculosis, but diagnostic tests would be needed because these are nonspecific manifestations.

A patients relationships are intense and unstable. The patient initially idealizes the significant other and then devalues him or her, resulting in frequent feelings of emptiness. This patient will benefit from interventions to develop which aspect of mental health? a. Effectiveness in work b. Communication skills c. Productive activities d. Fulfilling relationships

ANS: D The information given centers on relationships with others that are described as intense and unstable. The relationships of mentally healthy individuals are stable, satisfying, and socially integrated. Data are not present to describe work effectiveness, communication skills, or activities.

17. A nursing instructor is teaching about suicide. Which student statement indicates that learning has occurred? A. Suicidal threats and gestures should be considered manipulative and/or attention-seeking. B. Suicide is the act of a psychotic person. C. All suicidal individuals are mentally ill. D. Fifty to eighty percent of all people who kill themselves have a history of a previous attempt.

ANS: D It is a fact that between 50% and 80% of all people who kill themselves have a history of a previous attempt. All other answer choices are myths about suicide.

Select the best response for the nurse who receives a question from another health professional seeking to understand the difference between a Diagnostic and Statistical Manual of Mental Disorders (DSM-5) diagnosis and a nursing diagnosis. a. There is no functional difference between the two. Both identify human disorders. b. The DSM-5 diagnosis disregards culture, whereas the nursing diagnosis takes culture into account. c. The DSM-5 diagnosis describes causes of disorders whereas a nursing diagnosis does not explore etiology. d. The DSM-5 diagnosis guides medical treatment, whereas the nursing diagnosis offers a framework for identifying interventions for issues a patient is experiencing.

ANS: D The medical diagnosis is concerned with the patients disease state, causes, and cures, whereas the nursing diagnosis focuses on the patients response to stress and possible caring interventions. Both tools consider culture. The DSM-5 is multiaxial. Nursing diagnoses also consider potential problems.

. An older client reports having a cold and a "full bladder." What does the nurse obtain for or from the client? a. Order for a Foley catheter b. Order for a one-time catheterization c. Urine specimen d. History focusing on current medications

ANS: D The nurse needs to assess more before intervening. Clients often take antihistamines for a "cold." Antihistamines are often composed of anticholinergic drugs. In older adult clients, these medications can cause or worsen urinary retention.

The PACU nurse caring for a client with a nasogastric (NG) tube notes 300 mL of bright red blood has collected. What is the appropriate nursing action? a. Document as a normal finding. b. Immediately remove the NG tube. c. Place the client in Trendelenburg position. d. Call the client's surgeon to report the drainage.

ANS: D The presence of bright red blood reflects active bleeding. The nurse must report this to the surgeon immediately.

A client has a tuberculin skin test as a pre-employment physical requirement. Which statement by the nurse is best made to the client who has the test result seen in the photograph below? a. "Your PPD is negative. No further follow-up is necessary." b. "You will need to have a second PPD." c. "You will need to have titers drawn." d. "You will need further testing."

ANS: D The tuberculin test (Mantoux test) result is the most commonly used reliable test of TB infection. The photo shows a positive reaction. A positive reaction does not mean that active disease is present but indicates exposure to TB or the presence of inactive (dormant) disease. Conclusive evidence of TB is not provided through an examination of the chest or a chest x-ray. Only a sputum specimen will provide definitive evidence of the disease process.

A client has a peritonsillar abscess. Which priority instruction does the nurse provide to this client? a. "If you notice an enlarged node on the side of your neck where the abscess is, call your health care provider." b. "Stay home from work or school until your temperature has been normal for 24 hours." c. "You may gargle with warm water that has a teaspoon of salt in it as often as you like." d. "Take the antibiotic for the entire time it is prescribed, not just until you feel better."

ANS: D Untreated or ineffectively treated peritonsillar abscesses can extend throughout the pharyngeal area, causing swelling that may jeopardize the client's airway. Therefore, the client should take his antibiotic for the entire time prescribed to maximize the therapeutic effect. Gargling with warm water and refraining from normal activities may provide symptomatic relief for the client but would not be considered priority instructions. Also, swelling, pain, and inflammation could be noted by the client on the same side of the neck as the abscess.

A nurse is assessing a client who is recovering from a lung biopsy. Which assessment finding requires immediate action? a. Increased temperature b. Absent breath sounds c. Productive cough d. Incisional discomfort

Absent breath sounds. Absent breath sounds may indicate that the client has a pneumothorax, a serious complication after a needle biopsy or open lung biopsy. The other manifestations are not life threatening.

A nurse assesses a client with a mechanical bowel obstruction who reports intermittent abdominal pain. An hour later the client reports constant abdominal pain. Which action should the nurse take next? a. Administer intravenous opioid medications. b. Position the client with knees to chest. c. Insert a nasogastric tube for decompression. d. Assess the client's bowel sounds.

ANS: D A change in the nature and timing of abdominal pain in a client with a bowel obstruction can signal peritonitis or perforation. The nurse should immediately check for rebound tenderness and the absence of bowel sounds. The nurse should not medicate the client until the provider has been notified of the change in his or her condition. The nurse may help the client to the knee-chest position for comfort, but this is not the priority action. The nurse need not insert a nasogastric tube for decompression.

13. A nurse cares for a client who had a chest tube placed 6 hours ago and refuses to take deep breaths because of the pain. Which action should the nurse take? a. Ambulate the client in the hallway to promote deep breathing. b. Auscultate the clients anterior and posterior lung fields. c. Encourage the client to take shallow breaths to help with the pain. d. Administer pain medication and encourage the client to take deep breaths.

ANS: D A chest tube is placed in the pleural space and may be uncomfortable for a client. The nurse should provide pain medication to minimize discomfort and encourage the client to take deep breaths. The other responses do not address the clients discomfort and need to take deep breaths to prevent complications.

A nurse cares for a client who had a chest tube placed 6 hours ago and refuses to take deep breaths because of the pain. Which action should the nurse take? a. Ambulate the client in the hallway to promote deep breathing. b. Auscultate the client's anterior and posterior lung fields. c. Encourage the client to take shallow breaths to help with the pain. d. Administer pain medication and encourage the client to take deep breaths.

ANS: D A chest tube is placed in the pleural space and may be uncomfortable for a client. The nurse should provide pain medication to minimize discomfort and encourage the client to take deep breaths. The other responses do not address the client's discomfort and need to take deep breaths to prevent complications.

According to Peplau, which nursing action demonstrates the nurses role as a resource person? A. The nurse balances a safe therapeutic environment to increase the clients sense of belonging. B. The nurse holds a group meeting with the clients on the unit to discuss common feelings about mental illness. C. The nurse monitors the administration of medications and watches for signs of cheeking. D. The nurse explains, in language the client can understand, information related to the clients health care.

ANS: D According to Peplau, a resource person provides specific answers to questions usually formulated with relation to a larger problem.

9. A nurse assesses a client after a thoracentesis. Which assessment finding warrants immediate action? a. The client rates pain as a 5/10 at the site of the procedure. b. A small amount of drainage from the site is noted. c. Pulse oximetry is 93% on 2 liters of oxygen. d. The trachea is deviated toward the opposite side of the neck.

ANS: D A deviated trachea is a manifestation of a tension pneumothorax, which is a medical emergency. The other findings are normal or near normal.

15. According to Peplau, which nursing action demonstrates the nurses role as a resource person? A. The nurse balances a safe therapeutic environment to increase the clients sense of belonging. B. The nurse holds a group meeting with the clients on the unit to discuss common feelings about mental illness. C. The nurse monitors the administration of medications and watches for signs of cheeking. D. The nurse explains, in language the client can understand, information related to the clients health care.

ANS: D According to Peplau, a resource person provides specific answers to questions usually formulated with relation to a larger problem. KEY: Cognitive Level: Application | Integrated Processes: Nursing Process: Implementation | Client Need: Psychosocial Integrity

15. A nursing student is developing a plan of care for a suicidal client. Which documented intervention should the student implement first? A. Communicate therapeutically. B. Observe the client. C. Provide a hazard-free environment. D. Assess suicide risk.

ANS: D Assessment is the first step of the nursing process to gain needed information to determine further appropriate interventions.

8. The family of a suicidal client is very supportive and requests more facts related to caring for their family member after discharge. Which information should the nurse provide? A. Address only serious suicide threats to avoid the possibility of secondary gain. B. Promote trust by verbalizing a promise to keep suicide attempt information within the family. C. Offer a private environment to provide needed time alone at least once a day. D. Be available to actively listen, support, and accept feelings.

ANS: D Being available to actively listen, support, and accept feelings increases the potential that a client would confide suicidal ideations to family members.

23. A nurse is caring for a client who is scheduled for a dose of cefazolin and vitamins at this time. Hemodialysis for this client is also scheduled in 60 minutes. Which action by the nurse is best? a. Administer cefazolin since the level of the antibiotic must be maintained. b. Hold the vitamins but administer the cefazolin. c. Hold the cefazolin but administer the vitamins. d. Hold all medications since both cefazolin and vitamins are dialyzable.

ANS: D Both the cefazolin and the vitamins should be held until after the hemodialysis is completed because they would otherwise be removed by the dialysis process.

2. A nurse cares for a client with autosomal dominant polycystic kidney disease (ADPKD). The client asks, "Will my children develop this disease?" How should the nurse respond? a. "No genetic link is known, so your children are not at increased risk." b. "Your sons will develop this disease because it has a sex-linked gene." c. "Only if both you and your spouse are carriers of this disease." d. "Each of your children has a 50% risk of having ADPKD."

ANS: D Children whose parent has the autosomal dominant form of PKD have a 50% chance of inheriting the gene that causes the disease. ADPKD is transmitted as an autosomal dominant trait and therefore is not gender specific. Both parents do not need to have this disorder.

A nurse assesses a client who reports waking up feeling very tired, even after 8 hours of good sleep. Which action should the nurse take first? a. Contact the provider for a prescription for sleep medication. b. Tell the client not to drink beverages with caffeine before bed. c. Educate the client to sleep upright in a reclining chair. d. Ask the client if he or she has ever been evaluated for sleep apnea.

ANS: D Clients are usually unaware that they have sleep apnea, but it should be suspected in people who have persistent daytime sleepiness and report waking up tired. Causes of the problem should be assessed before the client is offered suggestions for treatment.

11. A nurse teaches a client who is recovering from a nephrectomy secondary to kidney trauma. Which statement should the nurse include in this client's teaching? a. "Since you only have one kidney, a salt and fluid restriction is required." b. "Your therapy will include hemodialysis while you recover." c. "Medication will be prescribed to control your high blood pressure." d. "You need to avoid participating in contact sports like football."

ANS: D Clients with one kidney need to avoid contact sports because the kidneys are easily injured. The client will not be required to restrict salt and fluids, end up on dialysis, or have new hypertension because of the nephrectomy.

A nurse assesses clients at a community health center. Which client is at highest risk for the development of colorectal cancer? a. A 37-year-old who drinks eight cups of coffee daily b. A 44-year-old with irritable bowel syndrome (IBS) c. A 60-year-old lawyer who works 65 hours per week d. A 72-year-old who eats fast food frequently

ANS: D Colon cancer is rare before the age of 40, but its incidence increases rapidly with advancing age. Fast food tends to be high in fat and low in fiber, increasing the risk for colon cancer. Coffee intake, IBS, and a heavy workload do not increase the risk for colon cancer.

16.A client is being discharged on long-term therapy for tuberculosis (TB). What referral by the nurse is most appropriate? a. Community social worker for Meals on Wheels b. Occupational therapy for job retraining c. Physical therapy for homebound therapy services d. Visiting Nurses for directly observed therapy

ANS: D Directly observed therapy is often utilized for managing clients with TB in the community. Meals on Wheels, job retraining, and home therapy may or may not be appropriate.

9.A client has been admitted for suspected inhalation anthrax infection. What question by the nurse is most important? a. Are any family members also ill? b. Have you traveled recently? c. How long have you been ill? d. What is your occupation?

ANS: D Inhalation anthrax is rare and is an occupational hazard among people who work with animal wool, bone meal, hides, and skin, such as taxidermists and veterinarians. Inhalation anthrax seen in someone without an occupational risk is considered a bioterrorism event and must be reported to authorities immediately. The other questions are appropriate for anyone with an infection.

14. A nurse cares for a client who has a chest tube. When would this client be at highest risk for developing a pneumothorax? a. When the insertion site becomes red and warm to the touch b. When the tube drainage decreases and becomes sanguineous c. When the client experiences pain at the insertion site d. When the tube becomes disconnected from the drainage system

ANS: D Intrathoracic pressures are less than atmospheric pressures; therefore, if the chest tube becomes disconnected from the drainage system, air can be sucked into the pleural space and cause a pneumothorax. A red, warm, and painful insertion site does not increase the clients risk for a pneumothorax. Tube drainage should decrease and become serous as the client heals. Sanguineous drainage is a sign of bleeding but does not increase the clients risk for a pneumothorax.

A nurse cares for a client who has a chest tube. When would this client be at highest risk for developing a pneumothorax? a. When the insertion site becomes red and warm to the touch b. When the tube drainage decreases and becomes sanguineous c. When the client experiences pain at the insertion site d. When the tube becomes disconnected from the drainage system

ANS: D Intrathoracic pressures are less than atmospheric pressures; therefore, if the chest tube becomes disconnected from the drainage system, air can be sucked into the pleural space and cause a pneumothorax. A red, warm, and painful insertion site does not increase the client's risk for a pneumothorax. Tube drainage should decrease and become serous as the client heals. Sanguineous drainage is a sign of bleeding but does not increase the client's risk for a pneumothorax.

17. The nurse is caring for a client who is prescribed a long-acting beta2 agonist. The client states, The medication is too expensive to use every day. I only use my inhaler when I have an attack. How should the nurse respond? a. You are using the inhaler incorrectly. This medication should be taken daily. b. If you decrease environmental stimuli, it will be okay for you to use the inhaler only for asthma attacks. c. Tell me more about your fears related to feelings of breathlessness. d. It is important to use this type of inhaler every day. Lets identify potential community services to help you.

ANS: D Long-acting beta2 agonists should be used every day to prevent asthma attacks. This medication should not be taken when an attack starts. Asthma medications can be expensive. Telling the client that he or she is using the inhaler incorrectly does not address the clients financial situation, which is the main issue here. Clients with limited incomes should be provided with community resources. Asking the client about fears related to breathlessness does not address the clients immediate concerns.

The nurse is caring for a client who is prescribed a long-acting beta2 agonist. The client states, "The medication is too expensive to use every day. I only use my inhaler when I have an attack." How should the nurse respond? a. "You are using the inhaler incorrectly. This medication should be taken daily." b. "If you decrease environmental stimuli, it will be okay for you to use the inhaler only for asthma attacks." c. "Tell me more about your fears related to feelings of breathlessness." d. "It is important to use this type of inhaler every day. Let's identify potential community services to help you."

ANS: D Long-acting beta2 agonists should be used every day to prevent asthma attacks. This medication should not be taken when an attack starts. Asthma medications can be expensive. Telling the client that he or she is using the inhaler incorrectly does not address the client's financial situation, which is the main issue here. Clients with limited incomes should be provided with community resources. Asking the client about fears related to breathlessness does not address the client's immediate concerns.

28. A nurse evaluates the following arterial blood gas and vital sign results for a client with chronic obstructive pulmonary disease (COPD): Arterial Blood Gas Results Vital Signs pH = 7.32 PaCO2 = 62 mm Hg PaO2 = 46 mm Hg HCO3 = 28 mEq/L Heart rate = 110 beats/min Respiratory rate = 12 breaths/min Blood pressure = 145/65 mm Hg Oxygen saturation = 76% Which action should the nurse take first? a. Administer a short-acting beta2 agonist inhaler. b. Document the findings as normal for a client with COPD. c. Teach the client diaphragmatic breathing techniques. d. Initiate oxygenation therapy to increase saturation to 92%.

ANS: D Oxygen should be administered to a client who is hypoxic even if the client has COPD and is a carbon dioxide retainer. The other interventions do not address the clients hypoxia, which is the priority.

A nurse evaluates the following arterial blood gas and vital sign results for a client with chronic obstructive pulmonary disease (COPD): pH = 7.32 PaCO2 = 62 mm Hg PaO2 = 46 mm Hg HCO3- = 28 mEq/L Heart rate = 110 beats/min Respiratory rate = 12 breaths/min Blood pressure = 145/65 mm Hg Oxygen saturation = 76% Which action should the nurse take first? a. Administer a short-acting beta2 agonist inhaler. b. Document the findings as normal for a client with COPD. c. Teach the client diaphragmatic breathing techniques. d. Initiate oxygenation therapy to increase saturation to 92%.

ANS: D Oxygen should be administered to a client who is hypoxic even if the client has COPD and is a carbon dioxide retainer. The other interventions do not address the client's hypoxia, which is the priority.

20. A nurse cares for a client who has a pleural chest tube. Which action should the nurse take to ensure safe use of this equipment? a. Strip the tubing to minimize clot formation and ensure patency. b. Secure tubing junctions with clamps to prevent accidental disconnections. c. Connect the chest tube to wall suction at the level prescribed by the provider. d. Keep padded clamps at the bedside for use if the drainage system is interrupted.

ANS: D Padded clamps should be kept at the bedside for use if the drainage system becomes dislodged or is interrupted. The nurse should never strip the tubing. Tubing junctions should be taped, not clamped. Wall suction should be set at the level indicated by the devices manufacturer, not the provider.

A nurse cares for a client who has a pleural chest tube. Which action should the nurse take to ensure safe use of this equipment? a. Strip the tubing to minimize clot formation and ensure patency. b. Secure tubing junctions with clamps to prevent accidental disconnections. c. Connect the chest tube to wall suction at the level prescribed by the provider. d. Keep padded clamps at the bedside for use if the drainage system is interrupted.

ANS: D Padded clamps should be kept at the bedside for use if the drainage system becomes dislodged or is interrupted. The nurse should never strip the tubing. Tubing junctions should be taped, not clamped. Wall suction should be set at the level indicated by the device's manufacturer, not the provider.

7. The treatment team is making a discharge decision regarding a previously suicidal client. Which client assessment information should a nurse recognize as contributing to the teams decision? A. No previous admissions for major depressive disorder B. Vital signs stable; no psychosis noted C. Able to comply with medication regimen; able to problem-solve life issues D. Able to participate in a plan for safety; family agrees to constant observation

ANS: D Participation in a plan of safety and constant family observation will decrease the risk for self-harm. All other answer choices are not directly focused on suicide prevention and safety.

22. When is self-disclosure by the nurse appropriate in a therapeutic nurseclient relationship? A. When it is judged that the information may benefit the nurse and client B. When the nurse has a duty to warn C. When the nurse feels emotionally indebted toward the client D. When it is judged that the information may benefit the client

ANS: D Self-disclosure on the part of the nurse may be appropriate when it is judged that the information may therapeutically benefit the client. It should never be undertaken for the purpose of meeting the nurses needs. KEY: Cognitive Level: Knowledge | Integrated Processes: Communication and Documentation | Client Need: Psychosocial Integrity Multiple Response

When is self-disclosure by the nurse appropriate in a therapeutic nurseclient relationship? A. When it is judged that the information may benefit the nurse and client B. When the nurse has a duty to warn C. When the nurse feels emotionally indebted toward the client D. When it is judged that the information may benefit the client

ANS: D Self-disclosure on the part of the nurse may be appropriate when it is judged that the information may therapeutically benefit the client. It should never be undertaken for the purpose of meeting the nurses needs.

A client is in the family practice clinic reporting a severe "cold" that started 4 days ago. On examination, the nurse notes the client also has a severe headache and muscle aches. What action by the nurse is best? a. Educate the client on oseltamivir (Tamiflu). b. Facilitate admission to the hospital. c. Instruct the client to have a flu vaccine. d. Teach the client to sneeze in the upper sleeve.

ANS: D Sneezing and coughing into one's sleeve helps prevent the spread of upper respiratory infections. The client does have manifestations of the flu (influenza), but it is too late to start antiviral medications; to be effective, they must be started within 24 to 48 hours of symptom onset. The client does not need hospital admission. The client should be instructed to have a flu vaccination, but now that he or she has the flu, vaccination will have to wait until next year.

4.A client is in the family practice clinic reporting a severe cold that started 4 days ago. On examination, the nurse notes the client also has a severe headache and muscle aches. What action by the nurse is best? a. Educate the client on oseltamivir (Tamiflu). b. Facilitate admission to the hospital. c. Instruct the client to have a flu vaccine. d. Teach the client to sneeze in the upper sleeve.

ANS: D Sneezing and coughing into ones sleeve helps prevent the spread of upper respiratory infections. The client does have manifestations of the flu (influenza), but it is too late to start antiviral medications; to be effective, they must be started within 24 to 48 hours of symptom onset. The client does not need hospital admission. The client should be instructed to have a flu vaccination, but now that he or she has the flu, vaccination will have to wait until next year.

1. A nurse assesses several clients who have a history of asthma. Which client should the nurse assess first? a. A 66-year-old client with a barrel chest and clubbed fingernails b. A 48-year-old client with an oxygen saturation level of 92% at rest c. A 35-year-old client who has a longer expiratory phase than inspiratory phase d. A 27-year-old client with a heart rate of 120 beats/min

ANS: D Tachycardia can indicate hypoxemia as the body tries to circulate the oxygen that is available. A barrel chest is not an emergency finding. Likewise, a pulse oximetry level of 92% is not considered an acute finding. The expiratory phase is expected to be longer than the inspiratory phase in someone with airflow limitation.

A nurse assesses several clients who have a history of asthma. Which client should the nurse assess first? a. A 66-year-old client with a barrel chest and clubbed fingernails b. A 48-year-old client with an oxygen saturation level of 92% at rest c. A 35-year-old client who has a longer expiratory phase than inspiratory phase d. A 27-year-old client with a heart rate of 120 beats/min

ANS: D Tachycardia can indicate hypoxemia as the body tries to circulate the oxygen that is available. A barrel chest is not an emergency finding. Likewise, a pulse oximetry level of 92% is not considered an acute finding. The expiratory phase is expected to be longer than the inspiratory phase in someone with airflow limitation.

7. A client has just had a central line catheter placed that is specific for hemodialysis. What is the most appropriate action by the nurse? a. Use the catheter for the next laboratory blood draw. b. Monitor the central venous pressure through this line. c. Access the line for the next intravenous medication. d. Place a heparin or heparin/saline dwell after hemodialysis.

ANS: D The central line should have a heparin or heparin/saline dwell after hemodialysis treatment. The central line catheter used for dialysis should not be used for blood sampling, monitoring central venous pressures, or giving drugs or fluids.

6. During a one-to-one session with a client, the client states, Nothing will ever get better, and Nobody can help me. Which nursing diagnosis is most appropriate for a nurse to assign to this client at this time? A. Powerlessness R/T altered mood AEB client statements B. Risk for injury R/T altered mood AEB client statements C. Risk for suicide R/T altered mood AEB client statements D. Hopelessness R/T altered mood AEB client statements

ANS: D The clients statements indicate the problem of hopelessness. Prior to assigning either risk for injury or risk for suicide, a further evaluation of the clients suicidal ideations and intent would be necessary.

16. A newly admitted client diagnosed with obsessive-compulsive disorder (OCD) washes hands continually. This behavior prevents unit activity attendance. Which nursing statement best addresses this situation? A. Everyone diagnosed with OCD needs to control their ritualistic behaviors. B. It is important for you to discontinue these ritualistic behaviors. C. Why are you asking for help if you wont participate in unit therapy? D. Lets figure out a way for you to attend unit activities and still wash your hands.

ANS: D The most appropriate statement by the nurse is, Lets figure out a way for you to attend unit activities and still wash your hands. This statement reflects the therapeutic communication technique of formulating a plan of action. The nurse attempts to work with the client to develop a plan without damaging the therapeutic relationship or increasing the clients anxiety. KEY: Cognitive Level: Application | Integrated Processes: Nursing Process: Implementation | Client Need: Psychosocial Integrity

10. A nurse cares for a client who is recovering after a nephrostomy tube was placed 6 hours ago. The nurse notes drainage in the tube has decreased from 40 mL/hr to 12 mL over the last hour. Which action should the nurse take? a. Document the finding in the client's record. b. Evaluate the tube as working in the hand-off report. c. Clamp the tube in preparation for removing it. d. Assess the client's abdomen and vital signs.

ANS: D The nephrostomy tube should continue to have a consistent amount of drainage. If the drainage slows or stops, it may be obstructed. The nurse must notify the provider, but first should carefully assess the client's abdomen for pain and distention and check vital signs so that this information can be reported as well. The other interventions are not appropriate.

12. A client who frequently exhibits angry outbursts is diagnosed with antisocial personality disorder. Which appropriate feedback should a nurse provide when this client experiences an angry outburst? A. Why do you continue to alienate your peers by your angry outbursts? B. You accomplish nothing when you lose your temper like that. C. Showing your anger in that manner is very childish and insensitive. D. During group, you raised your voice, yelled at a peer, and slammed the door.

ANS: D The nurse is providing appropriate feedback when stating, During group, you raised your voice, yelled at a peer, and slammed the door. Giving appropriate feedback involves helping the client consider a modification of behavior. Feedback should give information to the client about how he or she is perceived by others. Feedback should not be evaluative or be used to give advice. KEY: Cognitive Level: Application | Integrated Process: Nursing Process: Implementation | Client Need: Psychosocial Integrity

A client who frequently exhibits angry outbursts is diagnosed with antisocial personality disorder. Which appropriate feedback should a nurse provide when this client experiences an angry outburst? A. Why do you continue to alienate your peers by your angry outbursts? B. You accomplish nothing when you lose your temper like that. C. Showing your anger in that manner is very childish and insensitive. D. During group, you raised your voice, yelled at a peer, and slammed the door.

ANS: D The nurse is providing appropriate feedback when stating, During group, you raised your voice, yelled at a peer, and slammed the door. Giving appropriate feedback involves helping the client consider a modification of behavior. Feedback should give information to the client about how he or she is perceived by others. Feedback should not be evaluative or be used to give advice.

Chapter 08. Therapeutic Communication Multiple Choice 1. Which therapeutic communication technique is being used in this nurseclient interaction? Client: When I get angry, I get into a fistfight with my wife or I take it out on the kids. Nurse: I notice that you are smiling as you talk about this physical violence. A. Encouraging comparison B. Exploring C. Formulating a plan of action D. Making observations

ANS: D The nurse is using the therapeutic communication technique of making observations when noting that the client smiles when talking about physical violence. The technique of making observations encourages the client to compare personal perceptions with those of the nurse. KEY: Cognitive Level: Application | Integrated Processes: Nursing Process: Evaluation | Client Need: Psychosocial Integrity

Which therapeutic communication technique is being used in this nurse client interaction? Client: When I get angry, I get into a fistfight with my wife or I take it out on the kids. Nurse: I notice that you are smiling as you talk about this physical violence. A. Encouraging comparison B. Exploring C. Formulating a plan of action D. Making observations

ANS: D The nurse is using the therapeutic communication technique of making observations when noting that the client smiles when talking about physical violence. The technique of making observations encourages the client to compare personal perceptions with those of the nurse.

5. A client with acute kidney injury has a blood pressure of 76/55 mm Hg. The health care provider ordered 1000 mL of normal saline to be infused over 1 hour to maintain perfusion. The client is starting to develop shortness of breath. What is the nurse's priority action? a. Calculate the mean arterial pressure (MAP). b. Ask for insertion of a pulmonary artery catheter. c. Take the client's pulse. d. Slow down the normal saline infusion.

ANS: D The nurse should assess that the client could be developing fluid overload and respiratory distress and slow down the normal saline infusion. The calculation of the MAP also reflects perfusion. The insertion of a pulmonary artery catheter would evaluate the client's hemodynamic status, but this should not be the initial action by the nurse. Vital signs are also important after adjusting the intravenous infusion.

7. A nurse assesses a client who is recovering from a radical nephrectomy for renal cell carcinoma. The nurse notes that the client's blood pressure has decreased from 134/90 to 100/56 mm Hg and urine output is 20 mL for this past hour. Which action should the nurse take? a. Position the client to lay on the surgical incision. b. Measure the specific gravity of the client's urine. c. Administer intravenous pain medications. d. Assess the rate and quality of the client's pulse.

ANS: D The nurse should first fully assess the client for signs of volume depletion and shock, and then notify the provider. The radical nature of the surgery and the proximity of the surgery to the adrenal gland put the client at risk for hemorrhage and adrenal insufficiency. Hypotension is a clinical manifestation associated with both hemorrhage and adrenal insufficiency. Hypotension is particularly dangerous for the remaining kidney, which must receive adequate perfusion to function effectively. Re-positioning the client, measuring specific gravity, and administering pain medication would not provide data necessary to make an appropriate clinical decision, nor are they appropriate interventions at this time.

12. On which task should a nurse place priority during the working phase of relationship development? A. Establishing a contract for intervention B. Examining feelings about working with a particular client C. Establishing a plan for continuing aftercare D. Promoting the clients insight and perception of reality

ANS: D The nurse should place priority on promoting the clients insight and perception of reality during the working phase of relationship development. Establishing a contract for intervention would occur in the orientation phase. Examining feelings about working with a client should occur in the preinteraction phase. Establishing a plan for aftercare would occur in the termination phase. KEY: Cognitive Level: Analysis | Integrated Processes: Nursing Process: Implementation | Client Need: Psychosocial Integrity

3. Which is the best nursing action when a client demonstrates transference toward a nurse? A. Promoting safety and immediately terminating the relationship with the client B. Encouraging the client to ignore these thoughts and feelings C. Immediately reassigning the client to another staff member D. Helping the client to clarify the meaning of the current nurseclient relationship

ANS: D The nurse should respond to a clients transference by clarifying the meaning of the nurseclient relationship, based on the current situation. Transference occurs when the client unconsciously displaces feelings toward the nurse about a person from the past. The nurse should assist the client in separating the past from the present. KEY: Cognitive Level: Application | Integrated Processes: Nursing Process: Implementation | Client Need: Psychosocial Integrity

2. During the planning of care for a suicidal client, which correctly written outcome should be a nurses first priority? A. The client will not physically harm self. B. The client will express hope for the future by day 3. C. The client will establish a trusting relationship with the nurse. D. The client will remain safe during the hospital stay.

ANS: D The nurses priority should be that the client will remain safe during the hospital stay. Client safety should always be the nurses priority. The A answer choice is incorrectly written. Correctly written outcomes must be client focused, measurable, and realistic and contain a time frame. Without a time frame, an outcome cannot be correctly evaluated.

10. A mother who has learned that her child was killed in a tragic car accident states, I cant bear to go on with my life. Which nursing statement conveys empathy? A. This situation is very sad, but time is a great healer. B. You are sad, but you must be strong for your other children. C. Once you cry it all out, things will seem so much better. D. It must be horrible to lose a child; Ill stay with you until your husband arrives.

ANS: D The nurses response, It must be horrible to lose a child; Ill stay with you until your husband arrives, conveys empathy to the client. Empathy is the ability to see the situation from the clients point of view. Empathy is considered to be one of the most important characteristics of the therapeutic relationship. KEY: Cognitive Level: Application | Integrated Processes: Nursing Process: Implementation | Client Need: Psychosocial Integrity

5. A nurse states to a client, Things will look better tomorrow after a good nights sleep. This is an example of which communication technique? A. The therapeutic technique of giving advice B. The therapeutic technique of defending C. The nontherapeutic technique of presenting reality D. The nontherapeutic technique of giving false reassurance

ANS: D The nurses statement, Things will look better tomorrow after a good nights sleep, is an example of the nontherapeutic technique of giving false reassurance. Giving false reassurance indicates to the client that there is no cause for anxiety, thereby devaluing the clients feelings. KEY: Cognitive Level: Application | Integrated Processes: Nursing Process: Implementation | Client Need: Psychosocial Integrity

11. What is the purpose of a nurse providing appropriate feedback? A. To give the client good advice B. To advise the client on appropriate behaviors C. To evaluate the clients behavior D. To give the client critical information

ANS: D The purpose of providing appropriate feedback is to give the client critical information. Feedback should not be used to give advice or evaluate behaviors. KEY: Cognitive Level: Application | Integrated Processes: Nursing Process: Implementation | Client Need: Psychosocial Integrity

9. A nurse is assessing a client who has a tracheostomy. The nurse notes that the tracheostomy tube is pulsing with the heartbeat as the clients pulse is being taken. No other abnormal findings are noted. What action by the nurse is most appropriate? a. Call the operating room to inform them of a pending emergency case. b. No action is needed at this time; this is a normal finding in some clients. c. Remove the tracheostomy tube; ventilate the client with a bag-valve-mask. d. Stay with the client and have someone else call the provider immediately.

ANS: D This client may have a tracheainnominate artery fistula, which can be a life-threatening emergency if the artery is breached and the client begins to hemorrhage. Since no bleeding is yet present, the nurse stays with the client and asks someone else to notify the provider. If the client begins hemorrhaging, the nurse removes the tracheostomy and applies pressure at the bleeding site. The client will need to be prepared for surgery.

30. A client states, You wont believe what my husband said to me during visiting hours. He has no right treating me that way. Which nursing response would best assess the situation that occurred? A. Does your husband treat you like this very often? B. What do you think is your role in this relationship? C. Why do you think he behaved like that? D. Describe what happened during your time with your husband.

ANS: D This is an example of the therapeutic communication technique of exploring. The purpose of using exploring is to delve further into the subject, idea, experience, or relationship. This technique is especially helpful with clients who tend to remain on a superficial level of communication. KEY: Cognitive Level: Application | Integrated Processes: Nursing Process: Implementation | Client Need: Psychosocial Integrity

A client states, You wont believe what my husband said to me during visiting hours. He has no right treating me that way. Which nursing response would best assess the situation that occurred? A. Does your husband treat you like this very often? B. What do you think is your role in this relationship? C. Why do you think he behaved like that? D. Describe what happened during your time with your husband.

ANS: D This is an example of the therapeutic communication technique of exploring. The purpose of using exploring is to delve further into the subject, idea, experience, or relationship. This technique is especially helpful with clients who tend to remain on a superficial level of communication.

25. Which nursing statement is a good example of the therapeutic communication technique of focusing? A. Describe one of the best things that happened to you this week. B. Im having a difficult time understanding what you mean. C. Your counseling session is in 30 minutes. Ill stay with you until then. D. You mentioned your relationship with your father. Lets discuss that further.

ANS: D This is an example of the therapeutic communication technique of focusing. Focusing takes notice of a single idea or even a single word and works especially well with a client who is moving rapidly from one thought to another. KEY: Cognitive Level: Application | Integrated Processes: Nursing Process: Implementation | Client Need: Psychosocial Integrity

Which nursing statement is a good example of the therapeutic communication technique of focusing? A. Describe one of the best things that happened to you this week. B. Im having a difficult time understanding what you mean. C. Your counseling session is in 30 minutes. Ill stay with you until then. D. You mentioned your relationship with your father. Lets discuss that further.

ANS: D This is an example of the therapeutic communication technique of focusing. Focusing takes notice of a single idea or even a single word and works especially well with a client who is moving rapidly from one thought to another.

18. A client tells the nurse, I feel bad because my mother does not want me to return home after I leave the hospital. Which nursing response is therapeutic? A. Its quite common for clients to feel that way after a lengthy hospitalization. B. Why dont you talk to your mother? You may find out she doesnt feel that way. C. Your mother seems like an understanding person. Ill help you approach her. D. You feel that your mother does not want you to come back home?

ANS: D This is an example of the therapeutic communication technique of restatement. Restatement is the repeating of the main idea that the client has verbalized. This lets the client know whether or not an expressed statement has been understood and gives him or her the chance to continue, or clarify if necessary. KEY: Cognitive Level: Application | Integrated Processes: Nursing Process: Implementation | Client Need: Psychosocial Integrity

A client tells the nurse, I feel bad because my mother does not want me to return home after I leave the hospital. Which nursing response is therapeutic? A. It's quite common for clients to feel that way after a lengthy hospitalization. B. Why don't you talk to your mother? You may find out she doesn't feel that way. C. Your mother seems like an understanding person. I'll help you approach her. D. You feel that your mother does not want you to come back home?

ANS: D This is an example of the therapeutic communication technique of restatement. Restatement is the repeating of the main idea that the client has verbalized. This lets the client know whether or not an expressed statement has been understood and gives him or her the chance to continue, or clarify if necessary.

Select the most helpful response for a nurse to make when a patient being treated as an outpatient states, I am considering suicide. a. Im glad you shared this. Please do not worry. We will handle it together. b. I think you should admit yourself to the hospital to get help. c. We need to talk about the good things you have to live for. d. Bringing this up is a very positive action on your part.

ANS: D This response gives the patient reinforcement and validation for making a positive response rather than acting out the suicidal impulse. It gives neither advice nor false reassurance, and it does not imply stereotypes such as, "You have a lot to live for." It uses the patient's ambivalence and sets the stage for more realistic problem-solving strategies.

13. A suicidal client says to a nurse, There's nothing to live for anymore. Which is the most appropriate nursing reply? A. Why dont you consider doing volunteer work in a homeless shelter? B. Lets discuss the negative aspects of your life. C. Things will look better in the morning. D. It sounds like you are feeling pretty hopeless.

ANS: D This statement verbalizes the clients implied feelings and allows him or her to validate and explore them.

18.A client has the diagnosis of valley fever accompanied by myalgias and arthralgias. What treatment should the nurse educate the client on? a. Intravenous amphotericin B b. Long-term antiinflammatories c. No specific treatment d. Oral fluconazole (Diflucan)

ANS: D Valley fever, or coccidioidomycosis, is a fungal infection. Many people do not need treatment and the disease resolves on its own. However, the presence of joint and muscle pain indicates a moderate infection that needs treatment with antifungal medications. IV amphotericin is reserved for pregnant women and those with severe infection. Anti-inflammatory medications may be used to treat muscle aches and pain but are not used long term.

A nurse observes that the client's left flank region is larger than the right flank region. What is the nurse's best action? A. Ask the client if he or she participates in contact sports and has been recently injured. B. Document the finding as the only action on the appropriate flowsheet. C. Apply a heating pad to the left flank after inspecting the site for signs of infection. D. Anticipate further diagnostic testing after sharing informing the physician of this finding

ANS: D Asymmetry of the flank or a unilateral protrusion may indicate an enlargement of a kidney. The enlargement may be benign or may be associated with a hydronephrosis or mass on the kidney.

Which change in renal or urinary functioning as a result of the normal aging process increases the older client's risk for infection? A. Decreased glomerular filtration B. Decreased filtrate reabsorption C. Weakened sphincter muscles D. Urinary retention

ANS: D Incomplete bladder emptying for whatever reason increases the client's risk for urinary tract infections as a result of urine stasis providing an excellent culture medium that promotes the growth of microorganisms.

The client reports the regular use of all the following medications. Which one alerts the nurse to the possibility of renal impairment when used consistently? A. Antacids B. Penicillin C. Antihistamine nasal sprays D. Nonsteroidal anti-inflammatory drug

ANS: D NSAIDs inhibit prostaglandin production and decrease blood flow to the nephrons. They can cause an interstitial nephritis and renal impairment.

The client is scheduled to have a renogram (kidney scan). She is concerned about discomfort during the procedure. What is the nurse's best response? A. "Before the test you will be given a sedative to reduce any pain." B. "A local anesthetic agent will be used, so you might feel a little pressure but no pain." C. "Although this test is very sensitive, there is no more discomfort than you would have with an ordinary x-ray." D. "The only pain associated with this procedure is a small needle stick when you are given the radioisotope

ANS: D The test involves an intravenous injection of the radioisotope and the subsequent recording of the emission by a scintillator.

A college student said, Most of the time Im happy and feel good about myself. I have learned that what I get out of something is proportional to the effort I put into it. Which number on this mental health continuum should the nurse select? Mental Illness 1, 2 Mental Health 3, 4, 5 a. 1 b. 2 c. 3 d. 4 e. 5

ANS: E The student is happy and has an adequate self-concept. The student is reality-oriented, works effectively, and has control over own behavior. Mental health does not mean that a person is always happy.

What is your response to the patient's disclosure that she has consumed a can of soda on the morning of the scheduled surgery?

ANS: Gently remind the patient that intake of food and drink is not permitted after midnight, due to the risk of aspiration

22. The term ________________________ implies special feelings on the part of both the client and the nurse, based on acceptance, warmth, friendliness, common interest, a sense of trust, and a nonjudgmental attitude.

ANS: rapport Rationale: Rapport implies special feelings on the part of both the client and the nurse, based on acceptance, warmth, friendliness, common interest, a sense of trust, and a nonjudgmental attitude. Establishing rapport may be accomplished by discussing non-health-related topics.

A client with a history of suicide attempt has been discharged & is being followed in an outpatient clinic. At this time, which is the most appropriate nursing intervention for this client? A. Provide the client with a safe & structure environment B. Isolate the client from all stressful situations that may precipitate a suicide attempt C. Observe the client continuously to prevent self-harm D. Assist the client to develop more effective coping mechanisms

ANSWER D

Which is a misconception about suicide? A. 8 out of 10 individuals who commit suicide give warnings about their intentions B. Most suicidal individuals are very ambivalent about their feelings about suicide C. Most individuals commit suicide by taking an overdose of drugs D. Initial mood improvement can precipitate suicide

ANSWER: C Gunshot wounds are the leading cause of death among suicide victims

Based on the nurses knowledge of the normal function of the kidney, which large particles are not found in the urine because they are too large to filter through the glomerular capillary walls? (SATA) a. Blood cells b. Albumin c. Other proteins d. Electrolytes e. Water

Abc

The nurse is assessing a client admitted with status asthmaticus. The nurse finds a sudden absence of wheezing in the lung fields and sets which of these as the priority action? Education to prevent future exacerbations Administration of a bronchodilator Measures to reduce anxiety Activation of the rapid response team to secure an airway

Activation of the rapid response team to secure an airway Sudden absence of wheezing in a client having an asthma attack indicates complete airway obstruction and requires immediate action; a tracheotomy may be required.This is an emergency and educating the client is not appropriate. A bronchodilator is given when breath sounds are present and the client can inhale. Reducing anxiety is not a consideration in an emergency situation.

A client is experiencing an attack of acute pancreatitis. Which nursing intervention is the highest priority for this client? A. Measure intake and output every shift. B. Do not administer food or fluids by mouth. C. Administer opioid analgesic medication. D. Assist the client to assume a position of comfort.

Administer opioid analgesic medication. For the client with acute pancreatitis, pain relief is the highest priority. Although measuring intake and output, NPO status, and positioning for comfort are all important, they are not the highest priority.

A client with acute exacerbation of asthma has been admitted to the medical surgical unit for treatment. The client is reporting increased shortness of breath with inspiratory and expiratory wheezes. When planning care for this client, which medication will the nurse administer first? Albuterol-2 inhalations Fluticasone-2 inhalations Ipratropium-2 inhalations Salmeterol-2 inhalations

Albuterol-2 inhalations The nurse first needs to administer Albuterol, which is a rescue medication, to treat the client with increased shortness of breath with inspiratory and expiratory wheezes. Albuterol is a rapidly acting beta2 agonist that promotes bronchodilation.Fluticasone is a corticosteroid and needs to be given after a bronchodilator is given to open the airways. It is used to prevent asthma attacks by decreasing airway inflammation, and is not used as a rescue medication. Ipratropium is an anticholinergic drug that allows the sympathetic system to dominate and cause bronchodilation. It is not immediately effective like a short acting a beta2 agonist, so it is not a first-line drug. Salmeterol is a long-acting beta2 agonist that must be used regularly over time and is not used as a rescue medication.

Aside from chemotherapeutic agents, what other medications does the nurse expect to administer to a client with advanced colorectal cancer for relief of symptoms? A. Analgesics and antiemetics B. Analgesics and benzodiazepines C. Steroids and analgesics D. Steroids and anti-inflammatory medications

Analgesics and antiemetics Clients with advanced colorectal cancer and metastasis also receive drugs such as analgesics and antiemetics for relief of symptoms, specifically pain and nausea. Benzodiazepines, steroids, and anti-inflammatory medications are not routinely requested for these clients.

Predisposing factors: Fraud

Anger is turned inward on self, hopelessness, history of aggression and violence, and shame/humiliation

The feeling experienced by a patient that should be assessed by the nurse as most predict of elevated suicide risk is: A. Hopelessness B. Sadness C. Elation D. Anger

Ans: A Of the feelings listed, hopelessness is most closely associated with increased suicide risk. Depression, aggression, impulsivity and shame are other feelings noted as risk factors for suicide.

A 64-year-old obese man is admitted to the hospital for treatment of heart failure secondary to alcoholism. For which of the following negative consequences should the nurse assess? A) Chronic pain B) Obstructive sleep apnea C) Parkinson disease D) RLS

Ans: B Factors associated with increased risk for obstructive sleep apnea include obesity, diabetes, stroke, Parkinson disease, congestive heart failure, genetic predisposition, craniofacial anatomic features, and the use of alcohol or medications that depress the respiratory center

A patient recently hospitalized for two weeks committed suicide during the night. Which initial measure will be most helpful for staff members and other patients regarding this event? A. Request the public information officer to make an announcement to the local media. B. Hold a staff meeting to express feeling and plan of care for other patients C. Ask the patients roommate not to discuss the event with other patients D. Quickly discharge as many patients as possible to prevent panic

Ans: B Interventions should be aimed at helping the staff and patients come to terms with the loss and to grow because of the incident. Then, a community meeting should be scheduled to allow other patients to express their feelings and request help. Staff members should be prepared to provide additional support and reassurance to patients and should seek opportunities for peer support. The incorrect options will not control information or may result in unsafe care.

A 45-year-old obese man arrives in a clinic with complaints of daytime sleepiness, difficulty going to sleep at night, and snoring. What is the patient exhibiting manifestation of? A) Angina B) Diabetes C) Obstructive sleep apnea D) Depression

Ans: C Feedback: Obstructive sleep apnea occurs in men, especially those who are older and overweight. Symptoms include excessive daytime sleepiness, insomnia, and snoring. Daytime sleepiness and difficulty going to sleep at night are not indications of angina or diabetes. Snoring is not an indication of depression.

A 45-year-old obese man arrives in a clinic with complaints of daytime sleepiness, difficulty going to sleep at night, and snoring. The nurse should recognize the manifestations of what health problem? A) Adenoiditis B) Chronic tonsillitis C) Obstructive sleep apnea D) Laryngeal cancer

Ans: C Feedback: Obstructive sleep apnea occurs in men, especially those who are older and overweight. Symptoms include excessive daytime sleepiness, insomnia, and snoring. Daytime sleepiness and difficulty going to sleep at night are not indications of tonsillitis or adenoiditis. This patient's symptoms are not suggestive of laryngeal cancer.

When assessing a patients plan for suicide, what aspect has priority? A. Patients financial and educational status B. Patients insight into suicidal motivation. C. Availability of means and lethality of method D. Quality and availability of patients social supports

Ans: C If the person has definite plans that include choosing a method of suicide readily available and if the method is one that is lethal (ie will cause the person to die with little probability for intervention) the suicide risk is considered high. These areas provide a better indication of risk than the areas mention in the other options.

A nurse and patient construct a no-suicide contract. Select the preferable wording for the contract. A. I will not try to harm myself during the next 24 hours B. I will not make a suicide attempt while I am hospitalized C. For the next 24 hours I will not kill or harm myself in anyway. D. I will not Jill myself until I call my primary nurse or a member of the staff.

Ans: C The correct answer leaves no loopholes. The wording about not harming oneself and not making an attempt leaves loopholes or can be ignored by the patient who thinks, I am not going to harm myself, I am going to kill myself, or I am not going to attempt suicide I am going to commit suicide. A patient may call a therapist and leave the telephone to carry out the suicidal plan.

Which individual in the emergency department should be considered at the highest risk for complying suicide? A. An adolescent Asian-American girl with superior athletic and academic skills who has asthma B. A 38-year old single African American female church member with fibrocystic breast disease C. A 60 year old married Hispanic man with 12 grandchildren who has type 2 diabetes D. A 79 year old single white man with cancer of the prostate gland

Ans: D High risk factors include being an older adult, single and male and having a co-occurring medical illness. Cancer is one of the somatic conditions associated with increased suicidal risk. Protective factors for African American women and Hispanic individuals include strong religious and family ties. Asian Americans have a suicide rate that increases with age.

A nurse answers a suicide crisis line. A caller says I live alone in a home several miles from my nearest neighbors. I have been considering suicide for 2 months. I have had several drinks and now my gum is loaded. I'm going to shoot myself in the heart. How would the nurse assess the lethality of this plan? A. No risk B. Low level C. Moderate level D. High level

Ans: D The patient has a highly detailed plan a highly lethal method the means to carry it out, lowered impulse control because of alcohol ingestion and low potential for rescue.

A nurse counsels a patient with recent suicidal ideation. Which is the nurses most therapeutic comment? A. Let's make a list of all your problems and think of solutions for each one. B. I'm happy you're taking control of your problems and trying to find solutions C. When you have bad feelings try to focus on positive experiences from your life D. Let's consider which problems are most important and which are less important.

Ans:D The nurse helps the patient develop effective coping skills. He or she assists the patient to reduce the overwhelming approaches to problem solving.

Which changes is brain biochemical function is most associated with suicidal behavior? A. Dopamine excess B. Serotonin deficiency C. Acetylcholine excess D. Gamma-aminobutyric acid deficiency

Answer B: research shows that low levels of serotonin may play a role in the decision to attempt suicide. The other neurotransmitter alterations have not been implicated in suicidal crises.

A college student failed 2 tests. Afterward, the student cried for hours a s then tried to telephone a parent but got no answer. The student then gave several expensive sweaters to a roommate. Which behavior provides the strongest clue of an impending suicide attempt? A. Calling parents B. Excessive crying C. Giving away sweaters D. Staying alone in dorm room

Answer C: giving away prized possessions may signal that the individual thinks he or she will have no further need for the items, such as when a suicide plan has been formulated. Calling parents and crying do not provide clues to suicide, in and of themselves. Remaining in the dormitory would be an expected behavior because the student has nowhere else to go.

A nurse uses the SAD PERSONS scale to interview a patient. This tool provides data relevant to: a. current stress level. b. mood disturbance. c. suicide potential. d. level of anxiety.

Answer C: the SAD PERSONS tool evaluates 10 major risk factors in suicide potential: sex, age, depression, previous attempt, ethanol use, rational thinking loss, social supports lacking, organized plan, no spouse, and sickness. The tool does not have appropriate categories to provide information on the other options listed.

A person intentionally overdoses on antidepressant drugs. Which nursing diagnosis has the highest priority? A. Powerlessness B. Social isolation C. Risk for suicide D. Ineffective management of the therapeutic regimen

Answer C: this diagnosis is the only one with life-or-death ramifications and is therefore higher in priority than the other options.

A nurse assesses a client who has diabetes mellitus and notes the client is awake and alert, but shaky, diaphoretic, and weak. Five minutes after administering a half-cup of orange juice, the client's clinical manifestations have not changed. Which action should the nurse take next? A. Administer another half-cup of orange juice B. Administer a half-ampule of dextrose 50% intravenously C. Administer 10 units of regular insulin subcutaneously D. Administer 1 mg of glucagon intramuscularly

Answer: A

A nurse assesses a client with diabetes mellitus and notes the client only responds to a sternal rub by moaning, has capillary blood glucose of 33 g/dL, and has an intravenous line that is infiltrated with 0.45% normal saline. Which action should the nurse take first? A. Administer 1 mg of intramuscular glucagon B. Encourage the client to drink orange juice C. Insert a new intravenous access line D. Administer 25 mL dextrose 50% (D50) IV push

Answer: A

A nurse cares for a client who has a family history of diabetes mellitus. The client states, "My father has type 1 diabetes mellitus. Will I develop this disease as well?" How should the nurse respond? A. "Your risk of diabetes is higher than the general population, but it may not occur." B. "No genetic risk is associated with the development of type 1 diabetes mellitus." C. "The risk for becoming a diabetic is 50% because of how it is inherited." D. "Female children do not inherit diabetes mellitus, but male children will."

Answer: A

A nurse cares for a client with diabetes mellitus who is visually impaired. The client asks, "Can I ask my niece to prefill my syringes and then store them for later use when I need them?" How should the nurse respond? A. "Yes. Prefilled syringes can be stored for 3 weeks in the refrigerator in a vertical position with the needle pointing up." B. "Yes. Syringes can be filled with insulin and stored for a month in a location that is protected from light." C. "Insulin reacts with plastic, so prefilled syringes are okay, but you will need to use glass syringes." D. "No. Insulin syringes cannot be prefilled and stored for any length of time outside of the container."

Answer: A

A nurse prepares to administer insulin to a client at 1800. The client's medication administration record contains the following information: -Insulin glargine: 12 units daily at 1800 -Regular insulin: 6 units QID at 0600, 1200, 1800, 2400 Based on the client's medication administration record, which action should the nurse take? A. Draw up and inject the insulin glargine first, and then draw up and inject the regular insulin B. Draw up and inject the insulin glargine first, wait 20 minutes, and then draw up and inject the regular insulin C. First draw up the dose of regular insulin, then draw up the dose of insulin glargine in the same syringe, mix, and inject the two insulins together D. First draw up the dose of insulin glargine, then draw up the dose of regular insulin in the same syringe, mix and inject the two insulins together

Answer: A

A nurse teaches a client who is diagnosed with diabetes mellitus. Which statement should the nurse include in this client's plan of care to delay the onset of microvascular and macrovascular complications? A. "Maintain tight glycemic control and prevent hyperglycemia." B. "Restrict your fluid intake to no more than 2 liters a day." C. "Prevent hypoglycemia by eating a bedtime snack." D. "Limit your intake of protein to prevent ketoacidosis."

Answer: A

A preoperative nurse assesses a client who has type 1 diabetes mellitus prior to a surgical procedure. The client's blood glucose level is 160 mg/dL. Which action should the nurse take? A. Document the finding in the client's chart B. Administer a bolus of regular insulin IV C. Call the surgeon to cancel the procedure D. Draw blood gases to assess the metabolic state

Answer: A

After teaching a client who is recovering from pancreas transplantation, the nurse assesses the client's understanding. Which statement made by the client indicates a need for additional education? A. "If I develop an infection, I should stop taking my corticosteroid." B. "If I have pain over the transplant site, I will call the surgeon immediately." C. "I should avoid people who are ill or who have an infection." D. "I should take my cyclosporine exactly the way I was taught."

Answer: A

After teaching a client with diabetes mellitus to inject insulin, the nurse assesses the client's understanding. Which statement made by the client indicates a need for additional teaching? A. "The lower abdomen is the best location because it is closest to the pancreas." B. "I can reach my thigh the best, so I will use the different areas of my thighs." C. "By rotating the sites in one area, my chance of having a reaction is decreased." D. "Changing injection sites from the thigh to the arm will change absorption rates."

Answer: A

A nurse provides diabetic education at a public health fair. Which disorders should the nurse include as complications of diabetes mellitus? (Select all that apply.) a.Stroke b.Kidney failure c.Blindness d.Respiratory failure e.Cirrhosis

Answer: A, B, C

A nurse collaborates with the interdisciplinary team to develop a plan of care for a client who is newly diagnosed with diabetes mellitus. Which team members should the nurse include in this interdisciplinary team meeting? (Select all that apply.) a.Registered dietitian b.Clinical pharmacist c.Occupational therapist d.Health care provider e.Speech-language pathologist

Answer: A, B, D

A nurse teaches a client with diabetes mellitus about foot care. Which statements should the nurse include in this client's teaching? (Select all that apply.) a."Do not walk around barefoot." b."Soak your feet in a tub each evening." c."Trim toenails straight across with a nail clipper." d."Treat any blisters or sores with Epsom salts." e."Wash your feet every other day."

Answer: A, C

A nurse assesses clients at a health fair. Which clients should the nurse counsel to be tested for diabetes? (Select all that apply.) a.56-year-old African-American male b.Female with a 30-pound weight gain during pregnancy c.Male with a history of pancreatic trauma d.48-year-old woman with a sedentary lifestyle e.Male with a body mass index greater than 25 kg/m2 f.28-year-old female who gave birth to a baby weighing 9.2 pounds

Answer: A, D, E, F

A nurse assesses a client who has diabetes mellitus. Which arterial blood gas values should the nurse identify as potential ketoacidosis in this client? a. pH 7.38, HCO3- 22 mEq/L, PCO2 38 mm Hg, PO2 98 mm Hg b. pH 7.28, HCO3- 18 mEq/L, PCO2 28 mm Hg, PO2 98 mm Hg c. pH 7.48, HCO3- 28 mEq/L, PCO2 38 mm Hg, PO2 98 mm Hg d. pH 7.32, HCO3- 22 mEq/L, PCO2 58 mm Hg, PO2 88 mm Hg

Answer: B

A nurse assesses a client with diabetes mellitus. Which clinical manifestation should alert the nurse to decreased kidney function in this client? A. Urine specific gravity of 1.033 B. Presence of protein in the urine C. Elevated capillary blood glucose level D. Presence of ketone bodies in the urine

Answer: B

A nurse cares for a client who has diabetes mellitus. The nurse administers 6 units of regular insulin and 10 units of NPH insulin at 0700. At which time should the nurse assess the client for potential problems related to the NPH insulin? A. 0800 B. 1600 C. 2000 D. 2300

Answer: B

A nurse cares for a client who is diagnosed with acute rejection 2 months after receiving a simultaneous pancreas-kidney transplant. The client states, "I was doing so well with my new organs, and the thought of having to go back to living on hemodialysis and taking insulin is so depressing." How should the nurse respond? A. "Following the drug regimen more closely would have prevented this." B. "One acute rejection episode does not mean that you will lose the new organs." C. "Dialysis is a viable treatment option for you and may save your life." D. "Since you are on the national registry, you can receive a second transplantation."

Answer: B

A nurse cares for a client who is prescribed pioglitazone (Actos). After 6 months of therapy, the client reports that his urine has become darker since starting the medication. Which action should the nurse take? A. Assess for pain or burning with urination B. Review the client's liver function study results C. Instruct the client to increase water intake D. Test a sample of urine for occult blood

Answer: B

A nurse cares for a client with diabetes mellitus who asks, "Why do I need to administer more than one injection of insulin each day?" How should the nurse respond? A. "You need to start with multiple injections until you become more proficient at self-injection." B. "A single dose of insulin each day would not match your blood insulin levels and your food intake patterns." C. "A regimen of a single dose of insulin injected each day would require that you eat fewer carbohydrates." D. "A single dose of insulin would be too large to be absorbed, predictably putting you at risk for insulin shock."

Answer: B

A nurse develops a dietary plan for a client with diabetes mellitus and new-onset microalbuminuria. Which component of the client's diet should the nurse decrease? A. Carbohydrates B. Proteins C. Fats D. Total Calories

Answer: B

A nurse is teaching a client with diabetes mellitus who asks, "Why is it necessary to maintain my blood glucose levels no lower than about 60 mg/dL?" How should the nurse respond? A. "Glucose is the only fuel used by the body to produce the energy that it needs." B. "Your brain needs a constant supply of glucose because it cannot store it." C. "Without a minimum level of glucose, your body does not make red blood cells." D. "Glucose in the blood prevents the formation of lactic acid and prevents acidosis."

Answer: B

A nurse prepares to administer prescribed regular and NPH insulin. Place the nurse's actions in the correct order to administer these medications 1. Inspect the bottle for expiration dates 2. Gently roll the bottle of NPH between the hands 3. Wash your hands 4. Inject air into the regular insulin 5. Withdraw the NPH insulin 6. Withdraw the regular insulin 7. Inject air into the NPH bottle 8. Clean rubber stoppers with an alcohol swab A. 1, 3, 8, 2, 4, 6, 7, 5 B. 3, 1, 2, 8, 7, 4, 6, 5 C. 8, 1, 3, 2, 4, 6, 7, 5 D. 2, 3, 1, 8, 7, 5, 4, 6

Answer: B

A nurse reviews laboratory results for a client with diabetes mellitus who is prescribed an intensified insulin regimen: Fasting blood glucose: 75 mg/dL Postprandial blood glucose: 200 mg/dL Hemoglobin A1C level: 5.5% How should the nurse interpret these laboratory findings? A. Increased risk for developing ketoacidosis B. Good control of blood glucose C. Increased risk for developing hyperglycemia D. Signs of insulin resistance

Answer: B

A nurse reviews the chart and new prescriptions for a client with diabetic ketoacidosis: Vital Signs and Assessment: Blood pressure: 90/62 mm Hg Pulse: 120 beats/min Respiratory rate: 28 breaths/min Urine output: 20 mL/hr via catheter Laboratory Results: Serum potassium: 2.6 mEq/L Medications: Potassium chloride 40 mEq IV bolus STAT Increase IV fluid to 100 mL/hr Which action should the nurse take? a.Administer the potassium and then consult with the provider about the fluid order. b.Increase the intravenous rate and then consult with the provider about the potassium prescription. c.Administer the potassium first before increasing the infusion flow rate. d.Increase the intravenous flow rate before administering the potassium.

Answer: B

A nurse reviews the medication list of a client with a 20-year history of diabetes mellitus. The client holds up the bottle of prescribed duloxetine (Cymbalta) and states, "My cousin has depressed and is taking this drug. Do you think I'm depressed?" How should the nurse respond? A. "Many people with long-term diabetes become depressed after a while." B. "It's for peripheral neuropathy. Do you have burning pain in your feet or hands?" C. "This antidepressant also has anti-inflammatory properties for diabetic pain." D. "No. Many medications can be used for several different disorders."

Answer: B

A nurse teaches a client about self-monitoring of blood glucose levels. Which statement should the nurse include in this client's teaching to prevent bloodborne infections? A. "Wash your hands after completing each test." B. "Do not share your monitoring equipment." C. "Blot excess blood from the strip with a cotton ball." D. "Use gloves when monitoring your blood glucose."

Answer: B

A nurse teaches a client with diabetes mellitus about sick day management. Which statement should the nurse include in this client's teaching? A. "When ill, avoid eating or drinking to reduce vomiting and diarrhea." B. "Monitor your blood glucose levels at least every 4 hours while sick." C. "If vomiting, do not use insulin or take your oral antidiabetic agent." D. "Try to continue your prescribed exercise regimen even if you are sick."

Answer: B

A nurse teaches a client with type 2 diabetes mellitus who is prescribed glipizide (Glucotrol). Which statement should the nurse include in this client's teaching? A. "Change positions slowly when you get out of bed." B. "Avoid taking nonsteroidal anti-inflammatory drugs (NSAIDs)." C. "If you miss a dose of this drug, you can double the next dose." D. "Discontinue the medication if you develop a urinary infection."

Answer: B

After teaching a client with type 2 diabetes mellitus, the nurse assesses the client's understanding. Which statement made by the client indicates a need for additional teaching? A. "I need to have an annual appointment even if my glucose levels are in good control." B. "Since my diabetes is controlled with diet and exercise, I must be seen only if I am sick." C. "I can still develop complications even though I do not have to take insulin at this time." D. "If I have surgery or get very ill, I may have to receive insulin injections for a short time."

Answer: B

After teaching a client who is newly diagnosed with type 2 diabetes mellitus, the nurse assesses the client's understanding. Which statement made by the client indicates a need for additional teaching? A. "I should increase my intake of vegetables with higher amounts of dietary fiber." B. "My intake of saturated fats should be no more than 10% of my total calorie intake." C. "I should decrease my intake of protein and eliminate carbohydrates from my diet." D. "My intake of water is not restricted by my treatment plan or medication regimen."

Answer: C

At 4:45 p.m., a nurse assesses a client with diabetes mellitus who is recovering from an abdominal hysterectomy 2 days ago. The nurse notes that the client is confused and diaphoretic. The nurse reviews the assessment data provided in the chart below: Capillary Blood Glucose Testing (ACHS) At 0630: 95 At 1130: 70 At 1630: 47 Dietary Intake Breakfast: 10% eaten - client states she is not hungry Lunch: 5% eaten - client is nauseous; vomits once After reviewing the client's assessment data, which action is appropriate at this time? A. Assess the client's oxygen saturation level and administer oxygen B. Reorient the client and apply a cool washcloth to the client's forehead C. Administer dextrose 50% intravenously and reassess the client D. Provide a glass of orange juice and encourage the client to eat dinner

Answer: C

The nurse is caring for an actively suicidal client on the psychiatric unit. What is the nurse's priority intervention? A. Discuss strategies for the management of anxiety, anger & frustration B. Provide opportunities for increasing the client's self-worth, morale and control C. Place client on suicide precautions with one-to-one observation D. Explore experiences that affirm self-worth and self-efficacy

Answer: C

A nurse assesses a client with diabetes melltius who self-administers subcutaneous insulin. The nurse notes a spongy, swelling area at the site the client uses most frequently for insulin injection. Which action should the nurse take? A. Apply ice to the site to reduce inflammation B. Consult the provider for a new administration route C. Assess the client for other signs of cellulitis D. Instruct the client to rotate sites for insulin injection

Answer: D

A nurse assesses clients who are at risk for diabetes mellitus. Which client is at greatest risk? A. A 29-year-old Caucasian B. A 32-year-old African American C. A 44-year-old Asian D. A 48-year-old American Indian

Answer: D

A nurse cares for a client experiencing diabetic ketoacidosis who presents with Kussmaul respirations. Which action should the nurse take? A. Administration of oxygen via face mask B. Intravenous administration of 10% glucose C. Implementation of seizure precautions D. Administration of intravenous insulin

Answer: D

A nurse cares for a client who has type 1 diabetes mellitus. The client asks, "Is it okay for me to have an occasional glass of wine?" How should the nurse respond? A. "Drinking any wine or alcohol will increase your insulin requirements." B. "Because of poor kidney function, people with diabetes should avoid alcohol." C. "You should not drink alcohol because it will make you hungry and overeat." D. "One glass of wine is okay with a meal and is counted as two fat exchanges."

Answer: D

A nurse reviews laboratory results for a client with diabetes mellitus who presents with polyuria, lethargy, and a blood glucose of 560 mg/dL. Which laboratory result should the nurse correlate with the client's polyuria? A. Serum sodium: 163 mEq/L B. Serum creatinine: 1.6 mg/dL C. Presence of urine ketone bodies D. Serum osmolarity: 375 mOsm/kg

Answer: D

A nurse reviews the laboratory results of a client who is receiving intravenous insulin. Which should alert the nurse to intervene immediately? A. Serum chloride level of 98 mmol/L B. Serum calcium level of 8.8 mg/dL C. Serum sodium level of 132 mmol/L D. Serum potassium level of 2.5 mmol/L

Answer: D

A nurse reviews the medication list of a client recovering from a computed tomography (CT) scan with IV contrast to rule out small bowel obstruction. Which medication should alert the nurse to contact the provider and withhold the prescribed dose? A. Pioglitazone (Actos) B. Glimepiride (Amaryl) C. Glipizide (Glucotrol) D. Metformin (Glucophage)

Answer: D

A nurse teaches a client with diabetes mellitus who is experiencing numbness and reduced sensation. Which statement should the nurse include in this client's teaching to prevent injury? A. "Examine your feet using a mirror every day." B. "Rotate your insulin injection sites every week." C. "Check your blood glucose level before each meal." D. "Use a bath thermometer to test the water temperature."

Answer: D

A nurse teaches a client with type 1 diabetes mellitus. Which statement should the nurse include in this client's teaching to decrease the client's insulin needs? A. "Limit your fluid intake to 2 liters a day." B. "Animal organ meat is high in insulin." C. "Limit your carbohydrate intake to 80 grams a day." D. "Walk at a moderate pace for 1 mile daily."

Answer: D

After teaching a client who has diabetes mellitus and proliferative retinopathy, nephropathy, and peripheral neuropathy, the nurse assesses the client's understanding. Which statement made by the client indicates a correct understanding of the teaching? A. "I have so many complications; exercising is not recommended." B. "I will exercise more frequently because I have so many complications." C. "I used to run for exercise; I will start training for a marathon." D. "I should look into swimming or water aerobics to get my exercise."

Answer: D

After teaching a client with type 2 diabetes mellitus who is prescribed nateglinide (Starlix), the nurse assesses the client's understanding. Which statement made by the client indicates a correct understanding of the prescribed therapy? A. "I'll take this medicine during each of my meals." B. "I must take this medicine in the morning when I wake." C. "I will take this medicine before I go to bed." D. "I will take this medicine before I eat."

Answer: D

After teaching a young adult client who is newly diagnosed with type 1 diabetes mellitus, the nurse assesses the client's understanding. Which statement made by the client indicates a correct understanding of the need for eye examinations? A. "At my age, I should continue seeing the ophthalmologist as I usually do." B. "I will see the eye doctor when I have a vision problem and yearly after age 40." C. "My vision will change quickly. I should see the ophthalmologist twice a year." D. "Diabetes can cause blindness, so I should see the ophthalmologist yearly."

Answer: D

When teaching a client recently diagnosed with type 1 diabetes mellitus, the client states, "I will never be able to stick myself with a needle." How should the nurse respond? A. "I can give your injections to your while you are here in the hospital." B. "Everyone gets used to giving themselves injections. It really does not hurt." C. "Your disease will not be managed properly if you refuse to administer the shots." D. "Tell me what it is about the injections that are concerning you."

Answer: D

When caring for the client returning from thoracotomy and placement of a chest tube, the client reports severe pain. What does the nurse do first? Assess location and quality of pain. Call for the Rapid Response Team (RRT). Check the patency of the chest tubes. Call the health care provider.

Assess location and quality of pain. The nurse would assess the location, quality, radiation, severity of the pain, and the last time the client received pain medication before other actions are taken. Taking medication before pain becomes severe needs to be emphasized.The professional nurse is qualified to assess pain and provide pain medication when indicated. There is no information that suggests the client is unstable requiring the RRT to be called. The nurse will assess the chest drainage system at intervals, but pain is not typical when a chest tubes is blocked. The nurse would not call the health care provider before assessing the client's pain.

A nurse is caring for an older patient who has pulmonary infection. Which action should the nurse take first? a. encourage the client to increase fluid intake b. assess the clients level of consciousness c. raise the head of the bed to at least 45 degrees d. provide the client with humidified oxygen

Assess the client's LOC is the most important because it will show how the client is responding to the presence of infection. Although it will be important for the nurse to encourage the client to turn, cough and frequently breathe deeply; raise the head of the bed, increase oral fluid and humidify the oxygen administered, none of these actions will be as important as assessing the level of consciousness. A client with a pulmonary infection may not be able to cough productively if there is an area of abscess present

A nurse cares for a client who had a bronchoscopy 2 hours ago. The client asks for a drink of water. Which action should the nurse take next? a. Call the physician and request a prescription for food and water. b. Provide the client with ice chips instead of a drink of water. c. Assess the clients gag reflex before giving any food or water. d. Let the client have a small sip to see whether he or she can swallow.

Assess the clients gag reflex before giving any food or water. The topical anesthetic used during the procedure will have affected the clients gag reflex. Before allowing the client anything to eat or drink, the nurse must check for the return of this reflex.

Which action has the highest priority for the nurse to take when a client with type 1 diabetes arrives in the emergency department breathing deeply and stating, "I can't catch my breath." and has vital signs of: T 98.4° F (36.9° C), P 112 beats/min, R 38 breaths/min, BP 91/54 mm Hg, and O2 saturation 99% on room air? Administering oxygen Connecting a cardiac monitor Assessing arterial blood gas (ABG) values Assessing blood glucose level

Assessing blood glucose level The nurse would first obtain the client's glucose level. Breathing deeply and stating, "I can't catch my breath" is indicative of Kussmaul respirations which is a sign of diabetic ketoacidosis (DKA).Based on the oxygen saturation, oxygen administration is not indicated. The diagnosis of DKA does not require ABGs. Cardiac monitoring may be implemented, but the first action would be to obtain the glucose level.

A nurse plans care for a client who is experiencing dyspnea and must stop multiple times when climbing a flight of stairs. Which intervention should the nurse include in this clients plan of care? a. Assistance with activities of daily living b. Physical therapy activities every day c. Oxygen therapy at 2 liters per nasal cannula d. Complete bedrest with frequent repositioning

Assistance with activities of daily living. A client with dyspnea and difficulty completing activities such as climbing a flight of stairs has class III dyspnea. The nurse should provide assistance with activities of daily living. These clients should be encouraged to participate in activities as tolerated. They should not be on complete bedrest, may not be able to tolerate daily physical therapy, and only need oxygen if hypoxia is present.

A male client in a long-term care facility is 2 days postoperative after an open repair of an indirect inguinal hernia. Which nursing action does the RN delegate to unlicensed assistive personnel (UAP)? A. Assessing the client's incision for signs of infection B. Assisting the client to stand to void C. Instructing the client in how to deep-breathe D. Monitoring the client's pain level

Assisting the client to stand to void Assisting the client with activities is part of the UAP role. Assessment of the client's incision and pain level requires broader education and scope of practice and should be done by licensed nursing personnel. Client teaching—even about something as fundamental as taking "deep breaths"—likewise requires broader education and scope of practice and should be done by licensed nursing personnel.

A client is admitted to the emergency department several hours after a motor vehicle crash. The car's driver-side airbag was activated during the accident. Which assessment requires the nurse's immediate intervention? a.Disorientation b.Hemoptysis c.Pulse oximetry reading of 94% d.Chest pain with movement ANS: B The nurse should be concerned about possible pulmonary contusion. Interstitial hemorrhage accompanies pulmonary contusion. Bleeding may not be evident at the initial injury, but the client develops hemoptysis and decreased breath sounds up to several hours after injury as bleeding into the alveoli or airways occurs. The pulse oximetry reading is within normal limits and chest pain is expected with movement after chest trauma. Disorientation needs to be investigated, but does not take priority over a breathing problem.

B

A client with severe respiratory insufficiency becomes short of breath during activities of daily living. Which nursing intervention is best? a.Call the Rapid Response Team. b.Decrease involvement in care until the episode is past. c.Cluster morning activities to provide long rest periods. d.Space out interventions to provide for periods of rest. Clients with shortness of breath and decreased oxygen saturation must be monitored closely. Minimal involvement in activities is required if the client is severely short of breath. The nurse should continue to assess the client and can increase involvement in activities if shortness of breath subsides. The Rapid Response Team is not required. Clustering or spacing of activities does nothing to decrease the client's involvement, which is the cause of shortness of breath.

B

A nurse cares for a client who had a partial laryngectomy 10 days ago. The client states that all food tastes bland. How should the nurse respond? a. I will consult the speech therapist to ensure you are swallowing properly. b. This is normal after surgery. What types of food do you like to eat? c. I will ask the dietitian to change the consistency of the food in your diet. d. Replacement of protein, calories, and water is very important after surgery.

B

A nurse is preparing to admit a client on mechanical ventilation from the emergency department. What action by the nurse takes priority? a. Assessing that the ventilator settings are correct b. Ensuring there is a bag-valve-mask in the room c. Obtaining personal protective equipment d. Planning to suction the client upon arrival to the room Having a bag-valve-mask device is critical in case the client needs manual breathing. The respiratory therapist is usually primarily responsible for setting up the ventilator, although the nurse should know and check the settings. Personal protective equipment is important, but ensuring client safety takes priority. The client may or may not need suctioning on arrival.

B

A nurse teaches a client to use a room humidifier after a laryngectomy. Which statement should the nurse include in this clients teaching? a. Add peppermint oil to the humidifier to relax the airway. b. Make sure you clean the humidifier to prevent infection. c. Keep the humidifier filled with water at all times. d. Use the humidifier when you sleep, even during daytime naps.

B

A nurse teaches a client who has open vocal cord paralysis. Which technique should the nurse teach the client to prevent aspiration? a. Tilt the head back as far as possible when swallowing. b. Tuck the chin down when swallowing. c. Breathe slowly and deeply while swallowing. d. Keep the head very still and straight while swallowing.

B

After teaching a client who is prescribed voice rest therapy for vocal cord polyps, a nurse assesses the clients understanding. Which statement indicates the client needs further teaching? a. I will stay away from smokers to minimize inhalation of secondhand smoke. b. When I speak, I will whisper rather than use a normal tone of voice. c. For the next several weeks, I will not lift more than 10 pounds. d. I will drink at least three quarts of water each day to stay hydrated.

B

The pilot balloon on the endotracheal tube of a client being mechanically ventilated is deflated. What is the nurse's priority action? a.Nothing; this is required during ventilation. b.Inflate the cuff using minimal leak technique. c.Call the Rapid Response Team. d.Increase the tidal volume. The pilot balloon indicates whether the endotracheal tube cuff is inflated or deflated. A deflated balloon means that the cuff is also deflated and a seal is no longer present around the tube to prevent air from escaping. Thus, some of the air being moved into the client's airway by the ventilator is escaping through the client's trachea before it reaches the lower airways and alveoli. The nurse should inflate the cuff. Calling the Rapid Response Team is not necessary, and increasing tidal volume will not improve oxygenation if the cuff is leaking.

B

A student nurse is preparing to administer enoxaparin (Lovenox) to a client. What action by the student requires immediate intervention by the supervising nurse? a. Assessing the client's platelet count b. Choosing an 18-gauge, 2-inch needle c. Not aspirating prior to injection d. Swabbing the injection site with alcohol

B Enoxaparin is given subcutaneously, so the 18-gauge, 2-inch needle is too big. The other actions are appropriate.

Which laboratory test is the best indicator of kidney function? Blood urea nitrogen (BUN) Creatinine Aspartate aminotransferase (AST) Alkaline phosphatase

B Creatinine excretion, the end product of muscle metabolism, remains relatively steady and therefore is the best indicator of renal function. BUN may be affected by protein or fluid intake. AST and alkaline phosphatase are measures of hepatic function.

The charge nurse is making client assignments for the day shift. Which client is best to assign to an LPN/LVN? Client who has just returned from having a kidney artery angioplasty Client with polycystic kidney disease who is having a kidney ultrasound Client who is going for a cystoscopy and cystourethroscopy Client with glomerulonephritis who is having a kidney biopsy

B Kidney ultrasounds are noninvasive procedures without complications; the LPN/LVN can provide this care. A kidney artery angioplasty is an invasive procedure that requires postprocedure monitoring for complications, especially hemorrhage; a registered nurse is needed. Cystoscopy and cystourethroscopy are procedures that are associated with potentially serious complications such as bleeding and infection. These clients should be assigned to RN staff members. Kidney biopsy is associated with potentially serious complications such as bleeding, and this client should be assigned to RN staff members.

The RN is caring for a client who has just had a kidney biopsy. Which action does the nurse perform first? Obtain blood urea nitrogen (BUN) and creatinine. Position the client supine. Administer pain medications. Check urine for hematuria.

B The client is positioned supine for several hours after a kidney biopsy to decrease the risk for hemorrhage. BUN and creatinine would be obtained before the procedure is performed. Only local discomfort should be noted around the procedure site; severe pain would indicate hematoma. Although pink urine may develop, the nurse should position the client to prevent bleeding first; the other actions are appropriate after this procedure, but do not need to be done immediately after the biopsy.

spouse brings pizza d. Tuna salad sandwich and chips

B Warfarin works by inhibiting the synthesis of vitamin K-dependent clotting factors. Foods high in vitamin K thus interfere with its action and need to be eaten in moderate, consistent amounts. The chef's salad most likely has too many leafy green vegetables, which contain high amounts of vitamin K. The other selections, while not particularly healthy, will not interfere with the medication's mechanism of action.

Which percussion technique does the nurse use to assess a client who reports flank pain? Place outstretched fingers over the flank area and percuss with the fingertips. Place one hand with the palm down flat over the flank area and use the other fisted hand to thump the hand on the flank. Place one hand with the palm up over the flank area and cup the other hand to percuss the hand on the flank. Quickly tap the flank area with cupped hands.

B While the client assumes a sitting, side-lying, or supine position, the nurse forms one of the hands into a clenched fist. The other hand is placed flat over the costovertebral (CVA) angle of the client. Then, a firm thump is quickly delivered to the hand over the CVA area. Percussion is not appropriate for flank pain. Placing one hand palm up is not the correct technique. Percussion therapy, not assessment, involves tapping the flank area.

2. A nurse reviews the laboratory findings of a client with a urinary tract infection. The laboratory report notes a shift to the left in a clients white blood cell count. Which action should the nurse take? a. Request that the laboratory perform a differential analysis on the white blood cells. b. Notify the provider and start an intravenous line for parenteral antibiotics. c. Collaborate with the unlicensed assistive personnel (UAP) to strain the clients urine for renal calculi. d. Assess the client for a potential allergic reaction and anaphylactic shock.

B An increase in band cells creates a shift to the left. A left shift most commonly occurs with urosepsis and is seen rarely with uncomplicated urinary tract infections. The nurse will be administering antibiotics, most likely via IV, so he or she should notify the provider and prepare to give the antibiotics. The shift to the left is part of a differential white blood cell count. The nurse would not need to strain urine for stones. Allergic reactions are associated with elevated eosinophil cells, not band cells.

22. A nurse plans care for clients with urinary incontinence. Which client is correctly paired with the appropriate intervention? a. A 29-year-old client after a difficult vaginal delivery Habit training b. A 58-year-old postmenopausal client who is not taking estrogen therapy Electrical stimulation c. A 64-year-old female with Alzheimers-type senile dementia Bladder training d. A 77-year-old female who has difficulty ambulating Exercise therapy

B Exercise therapy and electrical stimulation are used for clients with stress incontinence related to childbirth or low levels of estrogen after menopause. Exercise therapy increases pelvic wall strength; it does not improve ambulation. Physical therapy and a bedside commode would be appropriate interventions for the client who has difficulty ambulating. Habit training is the type of bladder training that will be most effective with cognitively impaired clients. Bladder training can be used only with a client who is alert, aware, and able to resist the urge to urinate.

A client is on intravenous heparin to treat a pulmonary embolism. The client's most recent partial thromboplastin time (PTT) was 25 seconds. What order should the nurse anticipate? a. Decrease the heparin rate. b. Increase the heparin rate. c. No change to the heparin rate. d. Stop heparin, start warfarin (Coumadin).

B For clients on heparin, a PTT of 1.5 to 2.5 times the normal value is needed to demonstrate the heparin is working. A normal PTT is 25 to 35 seconds, so this client's PTT value is too low. The heparin rate needs to be increased. Warfarin is not indicated in this situation.

7. After delegating care to an unlicensed assistive personnel (UAP) for a client who is prescribed habit training to manage incontinence, a nurse evaluates the UAPs understanding. Which action indicates the UAP needs additional teaching? a. Toileting the client after breakfast b. Changing the clients incontinence brief when wet c. Encouraging the client to drink fluids d. Recording the clients incontinence episodes

B Habit training is undermined by the use of absorbent incontinence briefs or pads. The nurse should re-educate the UAP on the technique of habit training. The UAP should continue to toilet the client after meals, encourage the client to drink fluids, and record incontinent episodes.

A nurse is preparing to admit a client on mechanical ventilation from the emergency department. What action by the nurse takes priority? a. Assessing that the ventilator settings are correct b. Ensuring there is a bag-valve-mask in the room c. Obtaining personal protective equipment d. Planning to suction the client upon arrival to the room

B Having a bag-valve-mask device is critical in case the client needs manual breathing. The respiratory therapist is usually primarily responsible for setting up the ventilator, although the nurse should know and check the settings. Personal protective equipment is important, but ensuring client safety takes priority. The client may or may not need suctioning on arrival.

3. A nurse cares for a postmenopausal client who has had two episodes of bacterial urethritis in the last 6 months. The client asks, I never have urinary tract infections. Why is this happening now? How should the nurse respond? a. Your immune system becomes less effective as you age. b. Low estrogen levels can make the tissue more susceptible to infection. c. You should be more careful with your personal hygiene in this area. d. It is likely that you have an untreated sexually transmitted disease.

B Low estrogen levels decrease moisture and secretions in the perineal area and cause other tissue changes, predisposing it to the development of infection. Urethritis is most common in postmenopausal women for this reason. Although immune function does decrease with aging and sexually transmitted diseases are a known cause of urethritis, the most likely reason in this client is low estrogen levels. Personal hygiene usually does not contribute to this disease process.

The nurse is caring for a middle-aged man who complains of excessive daytime sleepiness. The nurse's examination reveals that the patient has large tonsils. He jokingly tells the nurse that his wife complains that he snores. The nurse hypothesizes that the patient has which of the following disorders? a. deviated septum b. obstructive sleep apnea c. occlusion of the nostrils d. upper respiratory infection

B Obstructive sleep apnea occurs when muscles in the nasopharynx and pharynx relax during sleep, resulting in pauses in breathing. Typically patients are overweight, middle-aged men who complain of excessive daytime sleepiness. Logged

A client is hospitalized with a second episode of pulmonary embolism (PE). Recent genetic testing reveals the client has an alteration in the gene CYP2C19. What action by the nurse is best? a. Instruct the client to eliminate all vitamin K from the diet. b. Prepare preoperative teaching for an inferior vena cava (IVC) filter. c. Refer the client to a chronic illness support group. d. Teach the client to use a soft-bristled toothbrush.

B Often clients are discharged from the hospital on warfarin (Coumadin) after a PE. However, clients with a variation in the CYP2C19 gene do not metabolize warfarin well and have higher blood levels and more side effects. This client is a poor candidate for warfarin therapy, and the prescriber will most likely order an IVC filter device to be implanted. The nurse should prepare to do preoperative teaching on this procedure. It would be impossible to eliminate all vitamin K from the diet. A chronic illness support group may be needed, but this is not the best intervention as it is not as specific to the client as the IVC filter. A soft-bristled toothbrush is a safety measure for clients on anticoagulation therapy.

20. An emergency department nurse assesses a client with a history of urinary incontinence who presents with extreme dry mouth, constipation, and an inability to void. Which question should the nurse ask first? a. Are you drinking plenty of water? b. What medications are you taking? c. Have you tried laxatives or enemas? d. Has this type of thing ever happened before?

B Some types of incontinence are treated with anticholinergic medications such as propantheline (Pro-Banthine). Anticholinergic side effects include dry mouth, constipation, and urinary retention. The nurse needs to assess the clients medication list to determine whether the client is taking an anticholinergic medication. If he or she is taking anticholinergics, the nurse should further assess the clients manifestations to determine if they are related to a simple side effect or an overdose. The other questions are not as helpful to understanding the current situation.

21. A nurse teaches a client who is starting urinary bladder training. Which statement should the nurse include in this clients teaching? a. Use the toilet when you first feel the urge, rather than at specific intervals. b. Try to consciously hold your urine until the scheduled toileting time. c. Initially try to use the toilet at least every half hour for the first 24 hours. d. The toileting interval can be increased once you have been continent for a week.

B The client should try to hold the urine consciously until the next scheduled toileting time. Toileting should occur at specific intervals during the training. The toileting interval should be no less than every hour. The interval can be increased once the client becomes comfortable with the interval.

24. A nurse assesses a male client who is recovering from a urologic procedure. Which assessment finding indicates an obstruction of urine flow? a. Severe pain b. Overflow incontinence c. Hypotension d. Blood-tinged urine

B The most common manifestation of urethral stricture after a urologic procedure is obstruction of urine flow. This rarely causes pain and has no impact on blood pressure. The client may experience overflow incontinence with the involuntary loss of urine when the bladder is distended. Blood in the urine is not a manifestation of the obstruction of urine flow.

12. A nurse assesses a client who is recovering from extracorporeal shock wave lithotripsy for renal calculi. The nurse notes an ecchymotic area on the clients right lower back. Which action should the nurse take? a. Administer fresh-frozen plasma. b. Apply an ice pack to the site. c. Place the client in the prone position. d. Obtain serum coagulation test results.

B The shock waves from lithotripsy can cause bleeding into the tissues through which the waves pass. Application of ice can reduce the extent and discomfort of the bruising. Although coagulation test results and fresh-frozen plasma are used to assess and treat bleeding disorders, ecchymosis after this procedure is not unusual and does not warrant a higher level of intervention. Changing the clients position will not decrease bleeding.

A nurse answers a call light and finds a client anxious, short of breath, reporting chest pain, and having a blood pressure of 88/52 mm Hg on the cardiac monitor. What action by the nurse takes priority? a. Assess the client's lung sounds. b. Notify the Rapid Response Team. c. Provide reassurance to the client. d. Take a full set of vital signs.

B This client has manifestations of a pulmonary embolism, and the most critical action is to notify the Rapid Response Team for speedy diagnosis and treatment. The other actions are appropriate also but are not the priority.

A nurse is caring for a client on the medical stepdown unit. The following data are related to this client: Subjective Information Laboratory Analysis Physical Assessment Shortness of breath for 20 minutes Feels frightened "Can't catch my breath" pH: 7.12 PaCO2: 28 mm Hg PaO2: 58 mm Hg SaO2: 88% Pulse: 120 beats/min Respiratory rate: 34 breaths/min Blood pressure 158/92 mm Hg Lungs have crackles What action by the nurse is most appropriate? a. Call respiratory therapy for a breathing treatment. b. Facilitate a STAT pulmonary angiography. c. Prepare for immediate endotracheal intubation. d. Prepare to administer intravenous anticoagulants.

B This client has manifestations of pulmonary embolism (PE). however, many conditions can cause the client's presentation. The gold standard for diagnosing a PE is pulmonary angiography. The nurse should facilitate this test as soon as possible. The client does not have wheezing, so a respiratory treatment is not needed. The client is not unstable enough to need intubation and mechanical ventilation. IV anticoagulants are not given without a diagnosis of PE.

A nurse is caring for four clients on intravenous heparin therapy. Which laboratory value possibly indicates that a serious side effect has occurred? a. Hemoglobin: 14.2 g/dL b. Platelet count: 82,000/L c. Red blood cell count: 4.8/mm3 d. White blood cell count: 8.7/mm3

B This platelet count is low and could indicate heparin-induced thrombocytopenia. The other values are normal for either gender.

A nurse is teaching a client about warfarin (Coumadin). What assessment finding by the nurse indicates a possible barrier to self-management? a. Poor visual acuity b. Strict vegetarian c. Refusal to stop smoking d. Wants weight loss surgery

B Warfarin works by inhibiting the synthesis of vitamin K-dependent clotting factors. Foods high in vitamin K thus interfere with its action and need to be eaten in moderate, consistent amounts. A vegetarian may have trouble maintaining this diet. The nurse should explore this possibility with the client. The other options are not related.

A male college student comes to the clinic reporting burning or difficulty with urination and a discharge from the urethral meatus. Based on the patient's chief complaint, what is the most logical question for the nurse to ask about the patient's past medical history? a. "Do you have a history of a narrow urethra or a stricture?" b. "Could you have been exposed to a sexually transmitted disease (STD)?" c. "Do you have a history of kidney stones?" d. "Have you been drinking an adequate amount of fluids?"

B - "Could you have been exposed to a sexually transmitted disease (STD)?"

Which patient has the highest risk for developing a complicated UTI? a. 26-year-old woman who is sexually active, but not currently pregnant b. 22-year-old man who has a neurogenic bladder due to spinal cord injury c. 35-year-old woman who had three full-term pregnancies and a miscarriage d. 53-year-old woman who is having some menstrual irregularities

B - 22-year-old man who has a neurogenic bladder due to spinal cord injury

For a patient who needs an indwelling catheter for at least 2 weeks, which intervention would help reduce the bacterial colonization along the catheter? a. Secure the catheter to the female patient's thigh. b. Consider the use of a coated catheter. c. Wash the urine bag and outflow tube every day. d. Apply antiseptic ointment to the catheter tubing.

B - Consider the use of a coated catheter.

The nurse is teaching a patient with urge incontinence about dietary modifications. What is the best information the nurse gives to the patient about fluid intake? a. Drink at least 2000 mL per day unless contraindicated. b. Drink 120 mL every hour or 240 mL every 2 hours and limit fluids after dinner. c. Drink fluid freely in the morning hours, but limit intake before going to bed. d. Drinking water is especially good for bladder health.

B - Drink 120 mL every hour or 240 mL every 2 hours and limit fluids after dinner.

The nurse hears in report that the patient is being treated for a fungal UTI. In addition to performing routine care and assessments, the nurse is extra-vigilant for signs/symptoms of which systemic disorder that may underlie the fungal UTI? a. Chronic cardiac disease b. Immune system compromise c. Respiratory system dysfunction d. Connective tissue disorder

B - Immune system compromise

A patient with urinary incontinence is prescribed oxybutynin (Ditropan). What precautions or instructions does the nurse provide related to this therapy? a. Avoid aspirin or aspirin-containing products. b. Increase fluids and dietary fiber intake. c. Report any unusual vaginal bleeding. d. Change positions slowly, especially in the morning.

B - Increase fluids and dietary fiber intake.

The nursing student sees an order for a urinalysis for a patient with frequency, urgency, and dysuria. In order to collect the specimen, what does the student do? a. Use sterile technique to insert a small-diameter (6 Fr) catheter. b. Instruct the patient on how to collect a clean-catch specimen. c. Tell the patient to urinate approximately 10 mL into a specimen cup. d. Take the urine from a bedpan and transfer it into a specimen cup.

B - Instruct the patient on how to collect a clean-catch specimen.

A patient is admitted for an elective orthopedic surgical procedure. The patient also has a personal and family history for urolithiasis. Which circumstance creates the greatest risk for recurrent urolithiasis? a. Giving the patient milk with every meal tray b. Keeping the patient NPO for extended periods c. Giving the patient an opioid narcotic for pain d. Inserting an indwelling catheter for the procedure

B - Keeping the patient NPO for extended periods

Which group has the highest prevalence of urinary tract infections (UTIs)? a. Young men b. Older women c. Older men d. School-aged girls

B - Older women

Which clinical manifestation indicates to the nurse that interventions for the patient's renal colic are effective? a. Urine is pink-tinged. b. Patient reports that pain is relieved. c. Urine output is 50 mL/min. d. Bladder scan shows no residual urine.

B - Patient reports that pain is relieved.

Which patient with incontinence is most likely to benefit from a surgical intervention? a. Patient with vaginal atrophy and altered urethral competency b. Patient with reflex (overflow) incontinence caused by obstruction c. Patient with functional incontinence related to musculoskeletal weakness d. Patient with urge incontinence or overactive bladder

B - Patient with reflex (overflow) incontinence caused by obstruction

A patient received an antibiotic prescription several hours ago and has started the medication, but requests "some relief from the burning." What comfort measures does the nurse suggest to the patient? a. Take over-the-counter acetaminophen. b. Sit in a sitz bath and urinate into the warm water. c. Place a cold pack over the perineal area. d. Rest in a recumbent position with legs elevated.

B - Sit in a sitz bath and urinate into the warm water.

A patient has agreed to try a bladder training program. What is the priority nursing intervention in starting this therapy? a. Start a schedule for voiding (e.g., every 2-3 hours). b. Teach the patient how to be alert, aware, and able to resist the urge to urinate. c. Convince the patient that the bladder issues are controlling his/her lifestyle. d. Give a thorough explanation of the problem of stress incontinence.

B - Teach the patient how to be alert, aware, and able to resist the urge to urinate.

A patient has been performing Kegel exercises for 2 months. How does the nurse know whether the exercises are working? a. Incontinence is still present, but the patient states that it is less. b. The patient is able to stop the urinary stream. c. There are no complaints of urgency from the patient. d. The patient is using absorbent undergarments for protection.

B - The patient is able to stop the urinary stream.

The nurse is reviewing the laboratory results for an older adult patient with an indwelling catheter. The urine culture is pending, but the urinalysis shows greater than 105 colony-forming units, and the differential WBC count shows a "left shift." How does the nurse interpret these findings? a. Interstitial cystitis b. Urosepsis c. Complicated cystitis d. Radiation-induced cystitis

B - Urosepsis

The employee health nurse is conducting a presentation for employees who work in a paint manufacturing plant. In order to protect against bladder cancer, the nurse advises that everyone who works with chemicals should do what? a. Shower with mild soap and rinse well before they come to work. b. Use personal protective equipment such as gloves and masks. c. Limit their exposure to chemicals and fumes at all times. d. Avoid hobbies such as furniture refinishing that further expose to chemicals.

B - Use personal protective equipment such as gloves and masks.

A client is having a peritoneal dialysis treatment. The nurse notes an opaque color to the effluent. What is the priority action by the nurse? a. Warm the dialysate solution in a microwave before instillation. b. Take a sample of the effluent and send to the laboratory. c. Flush the tubing with normal saline to maintain patency of the catheter. d. Check the peritoneal catheter for kinking and curling.

B ~ An opaque or cloudy effluent is the first sign of peritonitis. A sample of the effluent would need to be sent to the laboratory for culture and sensitivity in order to administer the correct antibiotic. Warming the dialysate in a microwave and flushing the tubing are not safe actions by the nurse. Checking the catheter for obstruction is a viable option but will not treat the peritonitis.

A client is recovering from a kidney transplant. The client's urine output was 1500 mL over the last 12-hour period since transplantation. What is the priority assessment by the nurse? a. Checking skin turgor b. Taking blood pressure c. Assessing lung sounds d. Weighing the client

B ~ By taking blood pressure, the nurse is assessing for hypotension that could compromise perfusion to the new kidney. The nurse then should notify the provider immediately. Skin turgor, lung sounds, and weight could give information about the fluid status of the client, but they are not the priority assessment.

The nurse is teaching the main principles of hemodialysis to a client with chronic kidney disease. Which statement by the client indicates a need for further teaching by the nurse? a. My sodium level changes by movement from the blood into the dialysate. b. Dialysis works by movement of wastes from lower to higher concentration. c. Extra fluid can be pulled from the blood by osmosis. d. The dialysate is similar to blood but without any toxins.

B ~ Dialysis works using the passive transfer of toxins by diffusion. Diffusion is the movement of molecules from an area of higher concentration to an area of lower concentration. The other statements show a correct understanding about hemodialysis.

The nurse is caring for four clients with chronic kidney disease. Which client should the nurse assess first upon initial rounding? a. Woman with a blood pressure of 158/90 mm Hg b. Client with Kussmaul respirations c. Man with skin itching from head to toe d. Client with halitosis and stomatitis

B ~ Kussmaul respirations indicate a worsening of chronic kidney disease (CKD). The client is increasing the rate and depth of breathing to excrete carbon dioxide through the lungs. Hypertension is common in most clients with CKD, and skin itching increases with calcium-phosphate imbalances, another common finding in CKD. Uremia from CKD causes ammonia to be formed, resulting in the common findings of halitosis and stomatitis.

The nurse is assessing a client with a diagnosis of pre-renal acute kidney injury (AKI). Which condition would the nurse expect to find in the client's recent history? a. Pyelonephritis b. Myocardial infarction c. Bladder cancer d. Kidney stones

B ~ Pre-renal causes of AKI are related to a decrease in perfusion, such as with a myocardial infarction. Pyelonephritis is an intrinsic or intrarenal cause of AKI related to kidney damage. Bladder cancer and kidney stones are post-renal causes of AKI related to urine flow obstruction.

A client is admitted with acute kidney injury (AKI) and a urine output of 2000 mL/day. What is the major concern of the nurse regarding this client's care? a. Edema and pain b. Electrolyte and fluid imbalance c. Cardiac and respiratory status d. Mental health status

B ~ This client may have an inflammatory cause of AKI with proteins entering the glomerulus and holding the fluid in the filtrate, causing polyuria. Electrolyte loss and fluid balance is essential. Edema and pain are not usually a problem with fluid loss. There could be changes in the client's cardiac, respiratory, and mental health status if the electrolyte imbalance is not treated.

A nurse assesses a client who has developed epistaxis. Which conditions in the clients history should the nurse identify as potential contributors to this problem? (Select all that apply.) a. Diabetes mellitus b. Hypertension c. Leukemia d. Cocaine use e. Migraine f. Elevated platelets

B, C, D

A nurse is giving discharge instructions to a client recently diagnosed with chronic kidney disease (CKD). Which statements made by the client indicate a correct understanding of the teaching? (SATA) a. I can continue to take antacids to relieve heartburn. b. I need to ask for an antibiotic when scheduling a dental appointment. c. I'll need to check my blood sugar often to prevent hypoglycemia. d. The dose of my pain medication may have to be adjusted. e. I should watch for bleeding when taking my anticoagulants.

B, C, D, E ~ In discharge teaching, the nurse must emphasize that the client needs to have an antibiotic prophylactically before dental procedures to prevent infection. There may be a need for dose reduction in medications if the kidney is not excreting them properly (antacids with magnesium, antibiotics, antidiabetic drugs, insulin, opioids, and anticoagulants).

For which adverse drug effects does the nurse assess in a client who is hospitalized for an acute problem and is also prescribed an anticholinergic drug to manage incontinence? Select all that apply. A. Insomnia B. Blurred vision C. Constipation D. Dry mouth E. Loss of sphincter control F. Increased sweating G. Worsening mental function H. Hypotension

B, C, D, G

A nurse is caring for five clients. For which clients would the nurse assess a high risk for developing a pulmonary embolism (PE)? (Select all that apply.) a. Client who had a reaction to contrast dye yesterday b. Client with a new spinal cord injury on a rotating bed c. Middle-aged man with an exacerbation of asthma d. Older client who is 1-day post hip replacement surgery e. Young obese client with a fractured femur

B, D, E Conditions that place clients at higher risk of developing PE include prolonged immobility, central venous catheters, surgery, obesity, advancing age, conditions that increase blood clotting, history of thromboembolism, smoking, pregnancy, estrogen therapy, heart failure, stroke, cancer (particularly lung or prostate), and trauma. A contrast dye reaction and asthma pose no risk for PE.

The nurse is teaching a client with diabetes mellitus how to prevent or delay chronic kidney disease (CKD). Which client statements indicate a lack of understanding of the teaching? (SATA) a. I need to decrease sodium, cholesterol, and protein in my diet. b. My weight should be maintained at a body mass index of 30. c. Smoking should be stopped as soon as I possibly can. d. I can continue to take an aspirin every 4 to 8 hours for my pain. e. I really only need to drink a couple of glasses of water each day.

B, D, E ~ Weight should be maintained at a body mass index (BMI) of 22 to 25. A BMI of 30 indicates obesity. The use of nonsteroidal anti-inflammatory drugs such as aspirin should be limited to the lowest time at the lowest dose due to interference with kidney blood flow. The client should drink at least 2 liters of water daily. Diet adjustments should be made by restricting sodium, cholesterol, and protein. Smoking causes constriction of blood vessels and decreases kidney perfusion, so the client should stop smoking.

A registered nurse (RN) cares for clients on a surgical unit. Which clients should the RN delegate to a licensed practical nurse (LPN)? (Select all that apply.) a. A 32-year-old who had a radical neck dissection 6 hours ago b. A 43-year-old diagnosed with cancer after a lung biopsy 2 days ago c. A 55-year-old who needs discharge teaching after a laryngectomy d. A 67-year-old who is awaiting preoperative teaching for laryngeal cancer e. An 88-year-old with esophageal cancer who is awaiting gastric tube placement

B, E

4. A nurse assesses a client with a fungal urinary tract infection (UTI). Which assessments should the nurse complete? (Select all that apply.) a. Palpate the kidneys and bladder. b. Assess the medical history and current medical problems. c. Perform a bladder scan to assess post-void residual. d. Inquire about recent travel to foreign countries. e. Obtain a current list of medications.

B, E Clients who are severely immunocompromised or who have diabetes mellitus are more prone to fungal UTIs. The nurse should assess for these factors by asking about medical history, current medical problems, and the current medication list. A physical examination and a post-void residual may be needed, but not until further information is obtained indicating that these examinations are necessary. Travel to foreign countries probably would not be important because, even if exposed, the client needs some degree of compromised immunity to develop a fungal UTI.

3. A nurse teaches clients about the difference between urge incontinence and stress incontinence. Which statements should the nurse include in this education? (Select all that apply.) a. Urge incontinence involves a post-void residual volume less than 50 mL. b. Stress incontinence occurs due to weak pelvic floor muscles. c. Stress incontinence usually occurs in people with dementia. d. Urge incontinence can be managed by increasing fluid intake. e. Urge incontinence occurs due to abnormal bladder contractions.

B, E Clients who suffer from stress incontinence have weak pelvic floor muscles or urethral sphincter and cannot tighten their urethra sufficiently to overcome the increased detrusor pressure. Stress incontinence is common after childbirth, when the pelvic muscles are stretched and weakened from pregnancy and delivery. Urge incontinence occurs in people who cannot suppress the contraction signal from the detrusor muscle. Abnormal detrusor contractions may be a result of neurologic abnormalities including dementia, or may occur with no known abnormality. Post-void residual is associated with reflex incontinence, not with urge incontinence or stress incontinence. Management of urge incontinence includes decreasing fluid intake, especially in the evening hours.

A patient with obstructive sleep apnea tells the nurse, "I just hate using this continuous positive airway pressure (CPAP) thing, but I know I need it. Is there anything I can do so that I don't need to use it?" Which of these would be an appropriate suggestion from the nurse? A Taking a nap during the day. B Referral to a weight loss program. C Trying a mild sedative at bedtime. D Drinking a glass of wine just before bedtime.

B. Because excessive weight worsens obstructive sleep apnea (OSA), referral to a weight loss program may be indicated. Weight loss and bariatric surgery reduce OSA. Daytime napping does not help this condition. Instruct the patient to avoid taking sedatives or consuming alcoholic beverages for three to four hours before sleep. Sleep medications often make OSA worse.

Which health promotion activity(ies) will the nurse recommend to prevent harm in a client with type 2 diabetes? (SATA) A. "Avoid all dietary carbohydrate and fat." B. "Have your eyes and vision assessed by an ophthalmologist every year." C. "Reduce your intake of animal fat and increase your intake of plant sterols." D. "Be sure to take your anti diabetes drug right before your engage in any type of exercise." E. "Keep your feet warm in cold weather by using either a hot water bottle or a heating pad." F. "Avoid foot damage from shoe-rubbing by going barefoot or wearing flip-flops when you are at home."

B. "Have your eyes and vision assessed by an ophthalmologist every year." C. "Reduce your intake of animal fat and increase your intake of plant sterols."

A client recently diagnosed with asthma has a prescription to use an inhaled medication with a spacer. The nurse evaluates the client has correct understanding of the use of an inhaler with a spacer when the client states which of these? (Select all that apply) A. "I don't have to wait a minute between the two puffs if I use a spacer." B. "If the spacer makes a whistling sound, I am breathing in too rapidly." C. "I should rinse my mouth and then swallow the water to get all of the medicine." D. "I should shake the canister when I want to see whether it is empty." E. "I should hold my breath for at least ten seconds after inhaling the medication."

B. "If the spacer makes a whistling sounds, I am breathing in too rapidly." E. "I should hold my breath for at least ten seconds after inhaling the medication

A client with diabetes who now has chronic albuminuria asks the nurse how this change will affect his health. How will the nurse answer this question? A. "You will ned to limit your intake of dietary albumin and other proteins to reduce the albuminuria." B. "This change indicates beginning kidney problems and requires good blood glucose control to prevent more damage." C. "Your risk for developing urinary tract infections is greatly increased, requiring the need to take daily antibiotics for prevention." D. "From now on you will need to limit your fluid intake to just 1 L daily and completely avoid caffeine to protect your kidneys."

B. "This change indicates beginning kidney problems and requires good blood glucose control to prevent more damage."

The client says, "I hate this stupid COPD." What is the best response by the nurse? A. "Stopping smoking will help your lungs heal." B. "You sound fed up with managing your illness." C. "Does anyone in your family have COPD?" D. "Most clients get used to it after a few months."

B. "You sound fed up with managing your illness"

Which instruction/precaution does the nurse teach a client to prevent harm during a 24-hour urine specimen collection? A. Be sure to keep the specimen cool for the entire collection period B. Avoid splashing urine in the container when a preservative is present C. Add the preservative to the collection container before adding any urine D. Discard the first specimen that marks the beginning of the 24 hour test period

B. Avoid splashing urine in the container when a preservative is present

A client is diagnosed with irritable bowel syndrome (IBS). What factors does the nurse suspect as possible causes of the client's problem? (Select all that apply.) A. Antihistamines B. Caffeinated drinks C. Stress D. Sleeping pills E. Anxiety

B. Caffeinated drinks C. Stress E. Anxiety Factors such as ingestion of coffee or other gastric stimulants, stress, anxiety, and milk allergy are being investigated as possible causes of IBS. Antihistamines and sleeping pills are not suspected as causing IBS.

The change-of-shift report has just been completed on the medical-surgical unit. Which client will the oncoming nurse plan to assess first? A. Client with chronic obstructive pulmonary disease (COPD) who is ready for discharge, but is unable to afford prescribed medications. B. Client with cystic fibrosis (CF) who has an elevated temperature and a respiratory rate of 38 breaths/min. C. Hospice client with end-stage pulmonary fibrosis and an oxygen saturation level of 89%. D. Client with lung cancer who needs an IV antibiotic administered before going to surgery.

B. Client with cystic fibrosis (CF) who has an elevated temperature and a respiratory rate of 38 breaths/min

When caring for the client with chronic bronchitis, which of these interventions will assist the client in mobilizing secretions? A. Elevate the head of the bed 45 degrees B. consume at least 2 liters of fluid daily C. avoid triggers which cause coughing D. assume the tripod position

B. Consume at least 2 liters of fluid daily

Which hormones help prevent hypoglycemia? (SATA) A. Aldosterone B. Cortisol C. Epinephrine D. Growth hormone E. Glucagon F. Insulin G. Norepinephrine H. Proinsulin

B. Cortisol C. Epinephrine D. Growth hormone E. Glucagon G. Norepinephrine

The nurse is providing teaching for a client who has been newly diagnosed with lung cancer and will be undergoing radiation therapy. Which of these points would be covered in the teaching session? (Select all that apply) A. Hair loss will occur. B. Do not expose the site to sun. C. Loss of appetite may develop. D. Pain in the area is expected. E. Fatigue may occur. F. Changes in taste may occur.

B. Do not expose the site to sun E. Fatigue may occur F. Changes in taste may occur

While making rounds the nurse finds a client with type 1 diabetes mellitus pale, sweaty, and slightly confused; the client can swallow. The client's blood glucose level check is 48 mg/dL. What is the nurse's best first action to prevent harm? A. Call the pharmacy and order a STAT dose of glucagon B. Immediately give the client 30 g of glucose orally C. Start an IV and administer a small amount of a concentrated dextrose solution D. Recheck the blood glucose level and call the Rapid Response team

B. Immediately give the client 30 g of glucose orally

The nurse is educating the client with COPD who requires home oxygen therapy for discharge. Which of these teaching points takes the highest priority? A. Correct performance when setting up the oxygen delivery system B. Removing combustion hazards present in the home C. Understanding the signs and symptoms of hypoxemia D. Demonstrating how to use a pulse oximetry device

B. Removing combustion hazards present in the home

A client with pulmonary artery hypertension on a continuous IV epoprostenol infusion is in the emergency department with symptoms of possible sepsis. The health care provider prescribes a broad-spectrum antibiotic to be administered IV immediately. What is the nurses best action? a. Request a prescription for an oral antibiotic. b. Start a peripheral IV line and administer the antibiotic. c. Administer the IV antibiotic through the continuous infusion's side port. d. Stop the epoprostenol infusion for 15 minutes to administer the IV antibiotic.

B. Start a peripheral IV line and administer the antibiotic

Which assessment finding in a 40 year old client is most relevant for the nurse to assess further for a possible endocrine problem? A. He has lost 10 lb in the past month following a low-carbohydrate eating plan B. The client reports now needing to shave only once weekly instead of daily C. His new prescription for eyeglasses is for a higher strength D. The client's father died of a stroke at age 70

B. The client reports now needing to shave only once weekly instead of daily

How will the nurse modify insulin injection technique for a client who is 5 feet 10 inches tall and weighs 106 lb? A. Use a 6 mm needle and inject at a 90 degree angle B. Use a 6 mm needle and inject at a 45 degree angle C. Use a 12 mm needle and inject at a 90 degree angle D. Use a 12 mm needle and inject at a 45 degree angle

B. Use a 6 mm needle and inject at a 45 degree angle

An adolescent tells the school nurse, my friend threatened to take an overdose of pills. The nurse talks to the friend who verbalized the suicidal threat. The most critical question for the nurse to ask would be: A. Why do you want to kill yourself? B. Do you have access to medications? C. Have you been taking drugs and alcohol? D. Did something happen with your parents?

B: do you have access to medications? The nurse must assess the patients access to the means to carry out the plan and if there is access, alert the parents to remove them from the home. The other questions may be important to ask but are not the most critical.

What does the nurse advise a client diagnosed with irritable bowel syndrome (IBS) to take during periods of constipation? A. Bulk-forming laxatives B. Saline laxatives C. Stimulant laxatives D. Stool-softening agents

Bulk-forming laxatives For treatment of constipation-predominant IBS, bulk-forming laxatives are generally taken at mealtimes with a glass of water. Saline and stimulant laxatives are not used for the treatment of constipation-predominant IBS. Stool-softening agents are not effective.

5. A nurse assesses clients on the medical-surgical unit. Which client is at greatest risk for development of obstructive sleep apnea? a. A 26-year-old woman who is 8 months pregnant b. A 42-year-old man with gastroesophageal reflux disease c. A 55-year-old woman who is 50 pounds overweight d. A 73-year-old man with type 2 diabetes mellitus

C

A 70-year-old client is seeing his primary care provider for an annual examination. Which assessment finding alerts the nurse to an increased risk for bladder cancer? A. A five-pack year history of smoking 45 years ago B. Difficulty starting and stopping the urine stream C. A 30-year occupation as a long-distance truck driver D. A recent colon cancer diagnosis in his 72-year-old brother

C

A client has been brought to the emergency department with a life-threatening chest injury. What action by the nurse takes priority? a. Apply oxygen at 100%. b. Assess the respiratory rate. c. Ensure a patent airway. d. Start two large-bore IV lines. The priority for any chest trauma client is airway, breathing, circulation. The nurse first ensures the client has a patent airway. Assessing respiratory rate and applying oxygen are next, followed by inserting IVs.

C

A client is on a ventilator and is sedated. What care may the nurse delegate to the unlicensed assistive personnel (UAP)? a. Assess the client for sedation needs. b. Get family permission for restraints. c. Provide frequent oral care per protocol. d. Use nonverbal pain assessment tools. The client on mechanical ventilation needs frequent oral care, which can be delegated to the UAP. The other actions fall within the scope of practice of the nurse.

C

A nurse cares for a client who is scheduled for a total laryngectomy. Which action should the nurse take prior to surgery? a. Assess airway patency, breathing, and circulation. b. Administer prescribed intravenous pain medication. c. Assist the client to choose a communication method. d. Ambulate the client in the hallway to assess gait.

C

A nurse is assisting the health care provider who is intubating a client. The provider has been attempting to intubate for 40 seconds. What action by the nurse takes priority? a. Ensure the client has adequate sedation. b. Find another provider to intubate. c. Interrupt the procedure to give oxygen. d. Monitor the client's oxygen saturation. Each intubation attempt should not exceed 30 seconds (15 is preferable) as it causes hypoxia. The nurse should interrupt the intubation attempt and give the client oxygen. The nurse should also have adequate sedation during the procedure and monitor the client's oxygen saturation, but these do not take priority. Finding another provider is not appropriate at this time.

C

A nurse is caring for a client who has sleep apnea and is prescribed modafinil (Provigil). The client asks, How will this medication help me? How should the nurse respond? a. This medication will treat your sleep apnea. b. This sedative will help you to sleep at night. c. This medication will promote daytime wakefulness. d. This analgesic will increase comfort while you sleep.

C

An intubated client's oxygen saturation has dropped to 88%. What action by the nurse takes priority? a. Determine if the tube is kinked. b. Ensure all connections are patent. c. Listen to the client's lung sounds. d. Suction the endotracheal tube. When an intubated client shows signs of hypoxia, check for DOPE: displaced tube (most common cause), obstruction (often by secretions), pneumothorax, and equipment problems. The nurse listens for equal, bilateral breath sounds first to determine if the endotracheal tube is still correctly placed. If this assessment is normal, the nurse would follow the mnemonic and assess the patency of the tube and connections and perform suction.

C

The nurse is caring for a client with acute respiratory distress syndrome (ARDS) who is receiving mechanical ventilation and positive end-expiratory pressure (PEEP). The alarm sounds, indicating decreased pressure in the system. What is the nurse's best action? a.Change the client's position. b.Suction the client. c.Assess lung sounds. d.Turn off the pressure alarm. One of the biggest risks in the client with ARDS on mechanical ventilation with PEEP is tension pneumothorax. The nurse needs to assess lung sounds hourly. The alarms on a ventilator should never be turned off. If the client needed to be suctioned, the high-pressure alarm would sound. Changing the client's position would not change the pressure needed to administer a breath.

C

The nurse is caring for several clients on the respiratory floor. Which client does the nurse assess most carefully for the development of acute respiratory distress syndrome (ARDS)? a.Older adult with COPD b.Middle-aged client receiving a blood transfusion c.Older adult who has aspirated his tube feeding d.Young adult with a broken leg from a motorcycle accident The older adult who has aspirated a tube feeding is at high risk and should be assessed closely for the possibility of ARDS. A client with COPD and a middle-aged client with no other risk factors are not at as high a risk for ARDS. The client who has a broken leg from an accident is not at high risk.

C

The pressure reading during inspiration on the ventilator of a client receiving mechanical ventilation is fluctuating widely. What is the nurse's first action? a.Determine whether an air leak is present in the client's endotracheal tube cuff. b.Have the respiratory therapist check the pressure settings. c.Assess the client's oxygenation. d.Manually ventilate the client with a resuscitation bag. A widely fluctuating pressure reading is one indication of inadequate airflow and oxygenation. The nurse's priority is to check the client's oxygenation status. If oxygenation is inadequate, the nurse would assess for a cause while manually ventilating the client and calling for assistance.

C

Which instruction does the nurse give a client who needs a clean-catch urine specimen? "Save all urine for 24 hours." "Use the sponges to cleanse the urethra, and then initiate voiding directly into the cup." "Do not touch the inside of the container." "You will receive an isotope injection, then I will collect your urine."

C A clean-catch specimen is used to obtain urine for culture and sensitivity of organisms present; contamination by the client's hands will render the specimen invalid and alter results. Saving urine for 24 hours is not necessary for a midstream clean-catch urine specimen. After cleaning, the client should initiate voiding into the commode, then stop and resume voiding into the container. Only 1 ounce (30 mL) is needed; the remainder of the urine may be discarded into the commode. A midstream collection further removes secretions and bacteria because urine flushes the distal portion of the internal urethra. A clean-catch specimen for culture does not require an injection of isotope, simply cleansing of the perineum.

The nurse is reviewing the medical record for a client with polycystic kidney disease who is scheduled for computed tomographic angiography with contrast: History and Physical Assessment Medications Diagnostic Findings Polycystic kidney disease Diabetes Hysterectomy Abdomen distended Negative edema Glyburide Metformin Synthroid BUN 26 mg/dL Creatinine 1.0 mg/dL HbA1c 6.9% Glucose 132 mg/dL Which intervention is essential for the nurse to perform? Obtain a thyroid-stimulating hormone (TSH) level. Report the blood urea nitrogen (BUN) and creatinine. Hold the metformin 24 hours before and on the day of the procedure. Notify the provider regarding blood glucose and glycosylated hemoglobin (HbA1c) values.

C Before studies with contrast media are performed, the nurse must withhold metformin, which may cause lactic acidosis. The focus of this admission is the polycystic kidneys; a TSH level is not essential at this time. BUN and creatinine are normal. The glucose is only mildly elevated (if fasting), and the HbA1c is in an appropriate range.

The nurse visualizes blood clots in a client's urinary catheter after a cystoscopy. What nursing intervention does the nurse perform first? Administer heparin intravenously. Remove the urinary catheter. Notify the health care provider. Irrigate the catheter with sterile saline.

C Bleeding and/or blood clots are potential complications of cystoscopy and may obstruct the catheter and decrease urine output. The nurse should monitor urine output and notify the health care provider of obvious blood clots or a decreased or absent urine output. Heparin will not be administered due to bleeding. The urinary catheter is allowing close monitoring of the urinary system and should not be removed at this time. The Foley catheter may be irrigated with sterile saline, as ordered.

A nurse is assisting the health care provider who is intubating a client. The provider has been attempting to intubate for 40 seconds. What action by the nurse takes priority? a. Ensure the client has adequate sedation. b. Find another provider to intubate. c. Interrupt the procedure to give oxygen. d. Monitor the client's oxygen saturation.

C Each intubation attempt should not exceed 30 seconds (15 is preferable) as it causes hypoxia. The nurse should interrupt the intubation attempt and give the client oxygen. The nurse should also have adequate sedation during the procedure and monitor the client's oxygen saturation, but these do not take priority. Finding another provider is not appropriate at this time.

Which assessment finding alarms the nurse immediately after a client returns from the operating room for cystoscopy performed under conscious sedation? Pink-tinged urine Urinary frequency Temperature of 100.8° F Lethargy

C Fever, chills, or an elevated white blood cell count after cystoscopy suggest infection after an invasive procedure; the provider must be notified immediately. Pink-tinged urine is expected after a cystoscopy; gross hematuria would require notification of the surgeon. Frequency may be noted as a result of irritation of the bladder. If sedation or anesthesia was used, lethargy is an expected effect.

When a client with diabetes returns to the medical unit after a computed tomography (CT) scan with contrast dye, all of these interventions are prescribed. Which intervention does the nurse implement first? Give lispro (Humalog) insulin, 12 units subcutaneously. Request a breakfast tray for the client. Infuse 0.45% normal saline at 125 mL/hr. Administer captopril (Capoten).

C Fluids are needed because the dye used in a CT scan with contrast has an osmotic effect, causing dehydration and potential kidney failure. Lispro is not administered until the breakfast tray arrives. A breakfast tray will be requested, but preventing complications of the procedure is done first. Because the client may be hypovolemic, the nurse should monitor blood pressure and administer IV fluids before deciding whether administration of captopril is appropriate.

Which statement by a patient indicates effective coping with a Kock's pouch? a. "I don't have any discomfort, but the pouch frequently overflows." b. "My wife has been irrigating the pouch daily. She likes to do it." c. "I check the pouch every 2 to 3 hours depending on my fluid and diet." d. "I never undress in front of anyone anymore, but I guess that is okay."

C - "I check the pouch every 2 to 3 hours depending on my fluid and diet."

A client is in the emergency department for an inability to void and for bladder distention. What is most important for the nurse to provide to the client? Increased oral fluids IV fluids Privacy Health history forms

C The nurse should provide privacy, assistance, and voiding stimulants, such as warm water over the perineum, as needed, for the client with urinary problems. Increased oral fluids and IV fluids would exacerbate the client's problem. Obtaining a health history is not the priority for this client's care.

The nurse is teaching a client how to provide a clean-catch urine specimen. Which statement by the client indicates that teaching was effective? "I must clean with the wipes and then urinate directly into the cup." "I will have to drink 2 liters of fluid before providing the sample." "I'll start to urinate in the toilet, stop, and then urinate into the cup." "It is best to provide the sample while I am bathing."

C To provide a clean-catch urine sample, the client should initiate voiding, then stop, then resume voiding into the container. A midstream collection further removes secretions and bacteria because urine flushes the distal portion of the internal urethra. Although cleaning with wipes before providing a clean-catch urine sample is proper procedure, a step is missing. It is not necessary to drink 2 liters of fluid before providing a clean-catch urine sample. Providing a clean-catch urine sample does not involve bathing.

A client has a pulmonary embolism and is started on oxygen. The student nurse asks why the client's oxygen saturation has not significantly improved. What response by the nurse is best? a. "Breathing so rapidly interferes with oxygenation." b. "Maybe the client has respiratory distress syndrome." c. "The blood clot interferes with perfusion in the lungs." d. "The client needs immediate intubation and mechanical ventilation."

C A large blood clot in the lungs will significantly impair gas exchange and oxygenation. Unless the clot is dissolved, this process will continue unabated. Hyperventilation can interfere with oxygenation by shallow breathing, but there is no evidence that the client is hyperventilating, and this is also not the most precise physiologic answer. Respiratory distress syndrome can occur, but this is not as likely. The client may need to be mechanically ventilated, but without concrete data on FiO2 and SaO2, the nurse cannot make that judgment.

15. A nurse assesses a client with bladder cancer who is recovering from a complete cystectomy with ileal conduit. Which assessment finding should alert the nurse to urgently contact the health care provider? a. The ileostomy is draining blood-tinged urine. b. There is serous sanguineous drainage present on the surgical dressing. c. The ileostomy stoma is pale and cyanotic in appearance. d. Oxygen saturations are 92% on room air.

C A pale or cyanotic stoma indicates impaired circulation to the stoma and must be treated to prevent necrosis. Blood-tinged urine and serous sanguineous drainage are expected after this type of surgery. Oxygen saturation of 92% on room air is at the low limit of normal.

9. A confused client with pneumonia is admitted with an indwelling catheter in place. During interdisciplinary rounds the following day, which question should the nurse ask the primary health care provider? a. Do you want daily weights on this client? b. Will the client be able to return home? c. Can we discontinue the indwelling catheter? d. Should we get another chest x-ray today?

C An indwelling catheter dramatically increases the risks of urinary tract infection and urosepsis. Nursing staff should ensure that catheters are left in place only as long as they are medically needed. The nurse should inquire about removing the catheter. All other questions might be appropriate, but because of client safety, this question takes priority.

A client is admitted with a pulmonary embolism (PE). The client is young, healthy, and active and has no known risk factors for PE. What action by the nurse is most appropriate? a. Encourage the client to walk 5 minutes each hour. b. Refer the client to smoking cessation classes. c. Teach the client about factor V Leiden testing. d. Tell the client that sometimes no cause for disease is found.

C Factor V Leiden is an inherited thrombophilia that can lead to abnormal clotting events, including PE. A client with no known risk factors for this disorder should be referred for testing. Encouraging the client to walk is healthy, but is not related to the development of a PE in this case, nor is smoking. Although there are cases of disease where no cause is ever found, this assumption is premature.

1. A nurse assesses clients on the medical-surgical unit. Which client is at greatest risk for the development of bacterial cystitis? a. A 36-year-old female who has never been pregnant b. A 42-year-old male who is prescribed cyclophosphamide c. A 58-year-old female who is not taking estrogen replacement d. A 77-year-old male with mild congestive heart failure

C Females at any age are more susceptible to cystitis than men because of the shorter urethra in women. Postmenopausal women who are not on hormone replacement therapy are at increased risk for bacterial cystitis because of changes in the cells of the urethra and vagina. The middle-aged woman who has never been pregnant would not have a risk potential as high as the older woman who is not using hormone replacement therapy.

The nurse is evaluating outcome criteria for a patient being treated for urge incontinence. Which statement indicates the treatment has been successful? a. "I'm following the prescribed therapy, but I think surgery is my best choice." b. "I still lose a little urine when I sneeze, but I have been wearing a thin pad." c. "I had trouble at first, but now I go to the toilet every 3 hours." d. "I have been using the bladder compression technique and it works."

C - "I had trouble at first, but now I go to the toilet every 3 hours."

4. After teaching a client with bacterial cystitis who is prescribed phenazopyridine (Pyridium), the nurse assesses the clients understanding. Which statement made by the client indicates a correct understanding of the teaching? a. I will not take this drug with food or milk. b. If I think I am pregnant, I will stop the drug. c. An orange color in my urine should not alarm me. d. I will drink two glasses of cranberry juice daily.

C Phenazopyridine discolors urine, most commonly to a deep reddish orange. Many clients think they have blood in their urine when they see this. In addition, the urine can permanently stain clothing. Phenazopyridine is safe to take if the client is pregnant. There are no dietary restrictions or needs while taking this medication.

26. A nurse provides phone triage to a pregnant client. The client states, I am experiencing a burning pain when I urinate. How should the nurse respond? a. This means labor will start soon. Prepare to go to the hospital. b. You probably have a urinary tract infection. Drink more cranberry juice. c. Make an appointment with your provider to have your infection treated. d. Your pelvic wall is weakening. Pelvic muscle exercises should help.

C Pregnant clients with a urinary tract infection require prompt and aggressive treatment because cystitis can lead to acute pyelonephritis during pregnancy. The nurse should encourage the client to make an appointment and have the infection treated. Burning pain when urinating does not indicate the start of labor or weakening of pelvic muscles.

A client is on a ventilator and is sedated. What care may the nurse delegate to the unlicensed assistive personnel (UAP)? a. Assess the client for sedation needs. b. Get family permission for restraints. c. Provide frequent oral care per protocol. d. Use nonverbal pain assessment tools.

C The client on mechanical ventilation needs frequent oral care, which can be delegated to the UAP. The other actions fall within the scope of practice of the nurse.

25. A nurse cares for a client with urinary incontinence. The client states, I am so embarrassed. My bladder leaks like a young childs bladder. How should the nurse respond? a. I understand how you feel. I would be mortified. b. Incontinence pads will minimize leaks in public. c. I can teach you strategies to help control your incontinence. d. More women experience incontinence than you might think.

C The nurse should accept and acknowledge the clients concerns, and assist the client to learn techniques that will allow control of urinary incontinence. The nurse should not diminish the clients concerns with the use of pads or stating statistics about the occurrence of incontinence.

A client has been brought to the emergency department with a life-threatening chest injury. What action by the nurse takes priority? a. Apply oxygen at 100%. b. Assess the respiratory rate. c. Ensure a patent airway. d. Start two large-bore IV lines.

C The priority for any chest trauma client is airway, breathing, circulation. The nurse first ensures the client has a patent airway. Assessing respiratory rate and applying oxygen are next, followed by inserting IVs.

When performing bladder scanning to detect residual urine in a female client, the nurse must first assess which factor? Abdominal girth Presence of urinary infection History of hysterectomy Hematuria

C The scanner must be in the scan mode for female clients to ensure the scanner subtracts the volume of the uterus from the measurement, or in the scan mode for male clients for women who have undergone a hysterectomy. The nurse performs this procedure in response to distention or pressure in the bladder; girth is not a factor. This procedure detects urine retained in the bladder, not infection. The presence of retained urine in the bladder is assessed, regardless of hematuria.

An intubated client's oxygen saturation has dropped to 88%. What action by the nurse takes priority? a. Determine if the tube is kinked. b. Ensure all connections are patent. c. Listen to the client's lung sounds. d. Suction the endotracheal tube.

C When an intubated client shows signs of hypoxia, check for DOPE: displaced tube (most common cause), obstruction (often by secretions), pneumothorax, and equipment problems. The nurse listens for equal, bilateral breath sounds first to determine if the endotracheal tube is still correctly placed. If this assessment is normal, the nurse would follow the mnemonic and assess the patency of the tube and connections and perform suction.

A young female patient reports experiencing burning with urination. What question does the nurse ask to differentiate between a vaginal infection and a urinary infection? a. "Have you noticed any blood in the urine?" b. "Have you had recent sexual intercourse?" c. "Have you noticed any vaginal discharge?" d. "Have you had fever or chills?"

C - "Have you noticed any vaginal discharge?"

Which patient has the highest risk for bladder cancer? a. 60-year-old male patient with malnutrition secondary to chronic alcoholism and self- neglect b. 25-year-old male patient with type 1 diabetes mellitus, who is noncompliant with therapeutic regimen c. 60-year-old female patient who smokes two packs of cigarettes per day and works in a chemical factory d. 25-year-old female patient who has had three episodes of bacterial (Escherichia coli) cystitis in the past year.

C - 60-year-old female patient who smokes two packs of cigarettes per day and works in a chemical factory

The nurse is working in a long-term care facility. Which circumstance is cause for greatest concern, because the facility has a large number of residents who are developing UTIs? a. Residents are not drinking enough fluids with meals. b. Unlicensed personnel are not assisting with toileting in a timely fashion. c. A large percentage of residents have indwelling urinary catheters. d. Many residents have severe dementia and functional incontinence.

C - A large percentage of residents have indwelling urinary catheters.

Which dietary changes does the nurse suggest to a patient with urge incontinence? a. Limit fluid intake to no more than 2 L/day. b. Peel all fruit before consuming. c. Avoid alcohol and caffeine. d. Avoid smoked or salted food

C - Avoid alcohol and caffeine.

The nurse is caring for a patient who has an indwelling catheter and subsequently developed a UTI. The patient has been receiving antibiotics for several days, but develops hypotension, a rapid pulse, and confusion. The nurse suspects urosepsis and alerts the health care provider. Which diagnostic test is the provider most likely to order to confirm urosepsis? a. Culture of the drainage bag b. Culture of the catheter tip c. Blood culture d. Repeat urinalysis

C - Blood culture

The home health nurse reads in the patient's chart that the patient has asymptomatic bacterial urinary tract infection (ABUTI). Which intervention will the nurse perform? a. Obtain an order for urinalysis and urine culture and sensitivity. b. Check the patient's medication list for appropriate antibiotic order. c. Closely monitor for conditions that cause progression to acute infection. d. Ask the patient when the ABUTI first started and when it was diagnosed.

C - Closely monitor for conditions that cause progression to acute infection.

Several patients at the clinic have just been diagnosed with UTIs. Which patients may need longer antibiotic treatment (7 to 21 days) or different agents than the typical first-line medi- cations? (Select all that apply.) a. Postmenopausal patient b. Patient with urethritis c. Diabetic patient d. Immunosuppressed patient e. Pregnant patient

C - Diabetic patient D -Immunosuppressed patient E - Pregnant patient

A patient is diagnosed with a fungal UTI. Which drug does the nurse anticipate the patient will be treated with? a. Trimethoprim/sulfamethoxazole (Bactrim) b. Ciprofloxacin (Cipro) c. Fluconazole (Diflucan) d. Amoxicillin (Amoxil)

C - Fluconazole (Diflucan)

An older adult patient with a cognitive impair-ment is living in an extended-care facility. The patient is incontinent, but as the family points out, "he will urinate in the toilet if somebody helps him." Which type of incontinence does the nurse suspect in this patient? a. Urge b. Overflow c. Functional d. Stress

C - Functional

The nurse is caring for an obese older adult patient with dementia. The patient is alert and ambulatory, but has functional incontinence. Which nursing intervention is best for this patient? a. Help the patient to lose weight. b. Help the patient apply an estrogen cream. c. Offer assistance with toileting every 2 hours. d. Intermittently catheterize the patient.

C - Offer assistance with toileting every 2 hours.

A patient is diagnosed with urethral stricture. What findings does the nurse expect to see documented in the patient's chart for this condition? a. Pain on urination b. Pain on ejaculation c. Overflow incontinence d. Hematuria and pyuria

C - Overflow incontinence

The nurse is talking to a 68-year-old male patient who has lifestyle choices and occupational exposure that put him at high risk for bladder cancer. The nurse is most concerned about which urinary characteristic? a. Frequency b. Nocturia c. Painless hematuria d. Incontinence

C - Painless hematuria

The nurse is caring for an older adult patient with urinary incontinence. The patient is alert and oriented, but refuses to use the call bell and has fallen several times while trying to get to the bathroom. What is the nurse's priority concern for this patient? a. Managing noncompliance b. Accurately measuring urinary output c. Providing fall prevention measures d. Managing urinary incontinence

C - Providing fall prevention measures

A patient reports severe flank pain. The report indicates that urine is turbid, malodorous, and rust-colored; RBCs, WBCs, and bacteria are present; and microscopic analysis shows crystals. What does this data suggest? a. Pyuria and cystitis b. Staghorn calculus with infection c. Urolithiasis and infection d. Dysuria and urinary retention

C - Urolithiasis and infection

The nurse is taking the vital signs of a client after hemodialysis. Blood pressure is 110/58 mm Hg, pulse 66 beats/min, and temperature is 99.8 F (37.6C). What is the most appropriate action by the nurse? a. Administer fluid to increase blood pressure. b. Check the white blood cell count. c. Monitor the client's temperature. d. Connect the client to an electrocardiographic (ECG) monitor.

C ~ During hemodialysis, the dialysate is warmed to increase diffusion and prevent hypothermia. The client's temperature could reflect the temperature of the dialysate. There is no indication to check the white blood cell count or connect the client to an ECG monitor. The other vital signs are within normal limits.

The nurse is teaching a client how to increase the flow of dialysate into the peritoneal cavity during dialysis. Which statement by the client demonstrates a correct understanding of the teaching? a. I should leave the drainage bag above the level of my abdomen. b. I could flush the tubing with normal saline if the flow stops. c. I should take a stool softener every morning to avoid constipation. d. My diet should have low fiber in it to prevent any irritation.

C ~ Inflow and outflow problems of the dialysate are best controlled by preventing constipation. A daily stool softener is the best option for the client. The drainage bag should be below the level of the abdomen. Flushing the tubing will not help with the flow. A diet high in fiber will also help with a constipation problem.

A client is placed on fluid restrictions because of chronic kidney disease (CKD). Which assessment finding would alert the nurse that the client's fluid balance is stable at this time? a. Decreased calcium levels b. Increased phosphorus levels c. No adventitious sounds in the lungs d. Increased edema in the legs

C ~ The absence of adventitious sounds upon auscultation of the lungs indicates a lack of fluid overload and fluid balance in the client's body. Decreased calcium levels and increased phosphorus levels are common findings with CKD. Edema would indicate a fluid imbalance.

The charge nurse of the medical-surgical unit is making staff assignments. Which staff member should be assigned to a client with chronic kidney disease who is exhibiting a low-grade fever and a pericardial friction rub? a. Registered nurse who just floated from the surgical unit b. Registered nurse who just floated from the dialysis unit c. Registered nurse who was assigned the same client yesterday d. Licensed practical nurse with 5 years experience on this floor

C ~ The client is exhibiting symptoms of pericarditis, which can occur with chronic kidney disease. Continuity of care is important to assess subtle differences in clients. Therefore, the registered nurse (RN) who was assigned to this client previously should again give care to this client. The float nurses would not be as knowledgeable about the unit and its clients. The licensed practical nurse may not have the education level of the RN to assess for pericarditis.

A nurse reviews these laboratory values of a client who returned from kidney transplantation 12 hours ago: Sodium 136 mEq/L Potassium 5 mEq/L Blood urea nitrogen (BUN) 44 mg/dL Serum creatinine 2.5 mg/dL What initial intervention would the nurse anticipate? a. Start hemodialysis immediately. b. Discuss the need for peritoneal dialysis. c. Increase the dose of immunosuppression. d. Return the client to surgery for exploration.

C ~ The client may need a higher dose of immunosuppressive medication as evidenced by the elevated BUN and serum creatinine levels. This increased dose may reverse the possible acute rejection of the transplanted kidney. The client does not need hemodialysis, peritoneal dialysis, or further surgery at this point.

The nurse is admitting a school-age child in acute renal failure with reduced glomerular filtration rate. Which urine test is the most useful clinical indication of glomerular filtration rate? a. pH b. Osmolality c. Creatinine d. Protein level

C ~ The most useful clinical indication of glomerular filtration is the clearance of creatinine. It is a substance that is freely filtered by the glomerulus and secreted by the renal tubule cells. The pH and osmolality are not estimates of glomerular filtration. Although protein in the urine demonstrates abnormal glomerular permeability, it is not a measure of filtration rate.

The charge nurse is orienting a float nurse to an assigned client with an arteriovenous (AV) fistula for hemodialysis in her left arm. Which action by the float nurse would be considered unsafe? a. Palpating the access site for a bruit or thrill b. Using the right arm for a blood pressure reading c. Administering intravenous fluids through the AV fistula d. Checking distal pulses in the left arm

C ~ The nurse should not use the arm with the AV fistula for intravenous infusion, blood pressure readings, or venipuncture. Compression and infection can result in the loss of the AV fistula. The AV fistula should be monitored by auscultating or palpating the access site. Checking the distal pulse would be an appropriate assessment.

A client is diagnosed with chronic kidney disease (CKD). What is an ideal goal of treatment set by the nurse in the care plan to reduce the risk of pulmonary edema? a. Maintaining oxygen saturation of 89% b. Minimal crackles and wheezes in lung sounds c. Maintaining a balanced intake and output d. Limited shortness of breath upon exertion

C ~ With an optimal fluid balance, the client will be more able to eject blood from the left ventricle without increased pressure in the left ventricle and pulmonary vessels. Other ideal goals are oxygen saturations greater than 92%, no auscultated crackles or wheezes, and no demonstrated shortness of breath.

When working with women who are taking hormonal birth control, what health promotion measures should the nurse teach to prevent possible pulmonary embolism (PE)? (Select all that apply.) a. Avoid drinking alcohol. b. Eat more omega-3 fatty acids. c. Exercise on a regular basis. d. Maintain a healthy weight. e. Stop smoking cigarettes.

C, D, E Health promotion measures for clients to prevent thromboembolic events such as PE include maintaining a healthy weight, exercising on a regular basis, and not smoking. Avoiding alcohol and eating more foods containing omega-3 fatty acids are heart-healthy actions but do not relate to the prevention of PE.

A 25-year-old sexually active female client diagnosed with cystitis tells the nurse that she doesn't understand why she has these infections yearly because she tries to avoid them by drinking very little at work so she doesn't have to use the "dirty" public toilet. Which suggestions or actions by the nurse are most likely to help this client reduce her risk for cystitis? Select all that apply. A. Reinforce her choice to avoid using a public toilet. B. Teach her to shower immediately after having sexual intercourse. C. Suggest that she drink at least 2-3 L of fluid throughout the day. D. Urge her to change her method of birth control from oral contraceptives to a barrier method. E. Instruct her to always wipe her perineum from front to back after each toilet use. F. Reinforce that she should complete the entire course of antibiotics as prescribed. G. Instruct her to empty her bladder immediately before having intercourse.

C, E, F, G

A patient with sleep apnea asks the nurse, "What can I do to get better sleep?" What is an appropriate nursing response? A "Taking one to two sleeping pills at night will prevent sleep apneic episodes." B "Keeping your hypertension under control is beneficial for general health." C "Being overweight is a contributing factor; losing weight can often resolve apnea." D "High blood glucose levels contribute to the apnea; monitor your sugar carefully."

C. Being overweight is an important risk factor for sleep apnea. Reducing weight often helps in resolving this condition. Taking sleeping pills may habituate a person, and taking it on a regular basis should be avoided. Keeping hypertension and blood glucose in control is beneficial for general health, but is not related to sleep apnea specifically.

The patient is scheduled for a sleep study test to see if the patient has mild sleep apnea. What should the nurse teach the patient to do until the test can be completed? IncorrectA Take sleep medications B Use the spouse's continuous positive airway pressure (CPAP) mask C Sleep in a side-lying position D Do not use pillows when sleeping

C. Conservative treatment for mild obstructive sleep apnea (OSA) begins with sleeping on one's side. Sleep medication often makes OSA worse. CPAP is adjusted for the patient and used with more severe symptoms after diagnosis. Elevating the head of the bed may eliminate OSA.

A patient with severe sleep apnea has been prescribed continuous positive airway pressure (CPAP). A nurse adjusts the blower to maintain adequate positive pressure during inspiration and expiration. The nurse should maintain the pressure in what range? A.1-5 cm of H2O B.5-10 cm of H2O C.5-25 cm of H2O D.1-25 cm of H2O

C. The pressure required to maintain an adequate positive pressure is 5-25 cm of H2O. This range of pressure is essential to avoid collapse of the airway.

A patient with sleep apnea is scheduled for surgery that involves excision of the tonsillar pillars, uvula, and posterior soft palate to remove the obstructing tissue. What is this surgery called? A Mastectomy B Tonsillectomy C Uvulopalatopharyngoplasty D Genioglossal advancement and hyoid myotomy

C. Uvulopalatopharyngoplasty is a surgery for resolving sleep apnea. This surgery involves excision of the tonsillar pillars, uvula, and posterior soft palate to remove the obstructing tissue. Mastectomy is surgery for the excision of breast cancer. Tonsillectomy is surgery for the excision of the tonsils. Genioglossal advancement and hyoid myotomy is another surgery for sleep apnea. It involves advancing the attachment of the muscular part of the tongue on the mandible.

After receiving education on the correct use of emergency drug therapy for asthma, which statement by the client indicates a correct understanding of the nurse/s instructions? A. "All asthma drugs help everybody breathe better." B. "I must carry my emergency inhaler when activity is anticipated." C. "I must have my emergency inhaler with me at all times." D. "Preventive drugs can stop an attack."

C. "I must have my emergency inhaler with me at all times"

The nurse is evaluating understanding of the treatment regimen for a client newly diagnosed with asthma. Which of these statements by the client indicates understanding of the regimen? A. "I will take albuterol when I go to sleep." B. "I will keep the rescue medication readily accessible on the first floor of my home." C. "I will take the long acting beta agonist even when my breathing seems OK." D. "I will immediately take the anti-inflammatory medication for an acute asthma attack."

C. "I will take the long acting beta agonist even when my breathing seems OK."

The school nurse is teaching a group of adolescents about risk factors for lung cancer and lung disease. Which of these would be included in the discussion? A. Alcohol consumption B. Cocaine use C. Cigarette smoking D. Heroin use

C. Cigarette smoking

Which precaution is a priority for the nurse to teach a client prescribed pramlintide to prevent harm? A. Only take this drug once weekly B. Do not drink alcohol when taking this drug C. Do not mix in the sam syringe with insulin D. Report any genital itching to your primary health care provider

C. Do not mix in the sam syringe with insulin

The nurse in the clinic is following up on diagnostic testing for a client recently diagnosed with metastatic lung cancer and back pain. Which of these findings does the nurse expect to uncover? A. Hyperkalemia B. Hyperglycemia C. Hypercalcemia D. Hypernatremia

C. Hypercalcemia

A client newly diagnosed with moderate asthma asks whether he can just take salmeterol instead of salmeterol and albuterol, because he has read that they are both beta agonists. What is the nurses best advice? a. Yes, both of these drugs have the same action, and you only need one. b. Yes, because they both need to be used daily whether you are having symptoms or not, just take a little more of the salmeterol and don't take any of the albuterol. c. No, albuterol is used to relieve the symptoms during an actual asthma attack and salmeterol is used to prevent an attack. Both are needed. d. No, albuterol is taken through the use of an aerosol inhaler and salmeterol is an oral drug (tablet) that is activated in the stomach. Both are needed.

C. No, albuterol is used to relieve the symptoms during an actual asthma attack and salmeterol is used to prevent an attack. Both are needed

Which statement regarding trophic hormones is true? A. All are categorized as catecholamines B. Responses are independent of target tissue receptors C. Their target tissues are always another endocrine gland D. They represent the final hormone secreted in a complex negative feedback pathway

C. Their target tissues are always another endocrine gland

The nurse case manager is discussing community resources with a client who has colorectal cancer and is scheduled for a colostomy. Which referral is of greatest value to this client initially? A. Certified Wound, Ostomy, and Continence Nurse (CWOCN) B. Home health nursing agency C. Hospice D. Hospital chaplain

Certified Wound, Ostomy, and Continence Nurse (CWOCN) A CWOCN (or an enterostomal therapist) will be of greatest value to the client because the client is scheduled to receive a colostomy. The client is newly diagnosed, so it is not yet known whether home health nursing will be needed. A referral to hospice may be helpful for a terminally ill client. Referral to a chaplain may be helpful later in the process of adjusting to the disease.

The school nurse is teaching a group of adolescents about risk factors for lung cancer and lung disease. Which of these would be included in the discussion? Alcohol consumption Cocaine use Cigarette smoking Heroin use

Cigarette smoking Cigarette smoking is highly addictive and is the number-one risk factor for lung cancer and chronic obstructive pulmonary disease.Alcohol can cause some cancers and liver disease and can increase risky behaviors, but it is not a major cause of lung cancer. Cocaine use, while highly addictive, poses a risk for cardiovascular disorders such as ACS, MI, or stroke. Heroin use does not increase one's risk of developing lung disease or lung cancer.

1. What is the most essential task for a nurse to accomplish prior to forming a therapeutic relationship with a client? 1. Clarify personal attitudes, values, and beliefs. 2. Obtain thorough assessment data. 3. Determine the client's length of stay. 4. Establish personal goals for the interaction.

Clarify personal attitudes, values, and beliefs. Rationale: The most essential task for a nurse to accomplish prior to forming a therapeutic relationship with a client is to clarify personal attitudes, values, and beliefs. Understanding one's own attitudes, values, and beliefs is called self-awareness.

A nurse assesses a client after a lug biopsy. Which assessment finding is matched with the correct intervention? a. client states he is dizzy; nurse applies oxygen and pulse ox b. client HR 55 bpm; nurse withholds pain meds c. client has reduced breath sounds; nurse calls physician immediately d. client RR 18bpm; nurse decreases oxygen flow rate

Client has reduced breath sounds; nurse calls physician immediately. A potentially serious complication after biopsy is pneumothorax, which is indicated by decreased or absent breath sounds.

The charge nurse is making assignments for clients cared for on the intensive care stepdown unit. Which client will the charge nurse assign to the RN who has floated from the pediatric unit? Client with acute asthma episode who is receiving oxygen at FiO2 of 60% by non-rebreather mask Client with chronic pleural effusions who is scheduled for a paracentesis in the next hour Client with emphysema who requires instruction about correct use of oxygen at home Client with lung cancer who has just been transferred from the intensive care unit after a left lower lobectomy yesterday

Client with acute asthma episode who is receiving oxygen at FiO2 of 60% by non-rebreather mask The charge nurse would assign the asthma client to the float pediatric nurse. Because asthma is a common pediatric diagnosis, the pediatric nurse would be familiar with the assessment and care needed for a client with this diagnosis.Although chronic pleural effusions can occur in the pediatric population, this diagnosis is more common in the adult population. If this client has not already received teaching for this procedure, he or she may have questions that the pediatric nurse would not be as comfortable answering as a nurse who is regularly assigned to the stepdown unit. Emphysema is a diagnosis associated with an adult population. Although an RN could instruct a client about home oxygen therapy, this client might have questions that would be better answered by an RN with adult experience. The adult client who has just had a lobectomy needs careful assessment from an RN with adult stepdown unit experience.

The change-of-shift report has just been completed on the medical-surgical unit. Which client will the oncoming nurse plan to assess first? Client with chronic obstructive pulmonary disease (COPD) who is ready for discharge, but is unable to afford prescribed medications. Client with cystic fibrosis (CF) who has an elevated temperature and a respiratory rate of 38 breaths/min. Hospice client with end-stage pulmonary fibrosis and an oxygen saturation level of 89%. Client with lung cancer who needs an IV antibiotic administered before going to surgery.

Client with cystic fibrosis (CF) who has an elevated temperature and a respiratory rate of 38 breaths/min. The client with CF with an elevated temperature and respiratory rate of 38 breaths/min is exhibiting signs of an exacerbation/infection and needs to be assessed first.The nurse will need to speak with the client who has COPD to help find a plan that will enable the client to obtain his or her prescribed medications. This may involve contacting case management or social services and discussing the discharge with the discharge health care provider. An oxygen saturation of 89% may be normal and expected for a hospice client with end-stage pulmonary fibrosis. There is no indication that this client is in distress. The nurse can delegate administration of the IV antibiotic to another RN, or it could be administered before the client is brought to the operating room.

A client with a bowel obstruction is ordered a nasogastric (NG) tube. After the nurse inserts the tube, which nursing intervention is the highest priority for this client? A. Attaching the tube to high continuous suction B. Auscultating for bowel sounds and peristalsis while the suction runs C. Connecting the tube to low intermittent suction D. Flushing the tube with 30 mL of normal saline every 24 hours

Connecting the tube to low intermittent suction The NG tube should be attached to intermittent low suction unless otherwise requested by the health care provider. Continuous suction is rarely used because it can injure the gastric mucosa of the client's stomach. Bowel sounds should not be auscultated with suction on and running. The tube should be flushed every 4 hours, minimally.

The nurse observes a patient's respirations during sleep and notes the absence of respirations that lasts from 15 to 45 seconds. What should the nurse consider this patient is experiencing? 1. laryngeal spasm 2. sleep apnea 3. respiratory acidosis 4. renal failure

Correct Answer: 2 Manifestations of obstructive sleep apnea include periods of apnea that last 15 to 120 seconds. No symptoms of laryngeal spasm or renal failure are noted. Respiratory acidosis would be diagnosed from arterial blood gases.

A client is admitted owing to difficulty breathing. The nurse assesses the client's color, lung sounds, and pulse oximetry reading. The pulse oximetry is 90%. What is the nurse's next action? a.Give an intermittent positive-pressure breathing treatment. b.Administer a rescue inhaler. c.Call for a chest x-ray. d.Assess an arterial blood gas. When clients with respiratory problems are assessed, an arterial blood gas is needed for the most accurate assessment of oxygenation. No indications are known for a breathing treatment or an inhaler, nor does the nurse have enough information to know whether a chest x-ray is warranted.

D

A client is brought to the emergency department after sustaining injuries in a severe car crash. The client's chest wall does not appear to be moving normally with respirations, oxygen saturation is 82%, and the client is cyanotic. What action by the nurse is the priority? a. Administer oxygen and reassess. b. Auscultate the client's lung sounds. c. Facilitate a portable chest x-ray. d. Prepare to assist with intubation. This client has manifestations of flail chest and, with the other signs, needs to be intubated and mechanically ventilated immediately. The nurse does not have time to administer oxygen and wait to reassess, or to listen to lung sounds. A chest x-ray will be taken after the client is intubated.

D

A client is on mechanical ventilation and the client's spouse wonders why ranitidine (Zantac) is needed since the client "only has lung problems." What response by the nurse is best? a. "It will increase the motility of the gastrointestinal tract." b. "It will keep the gastrointestinal tract functioning normally." c. "It will prepare the gastrointestinal tract for enteral feedings." d. "It will prevent ulcers from the stress of mechanical ventilation." Stress ulcers occur in many clients who are receiving mechanical ventilation, and often prophylactic medications are used to prevent them. Frequently used medications include antacids, histamine blockers, and proton pump inhibitors. Zantac is a histamine blocking agent.

D

A client with dyspnea is becoming very anxious. An arterial blood gas (ABG) shows a PaO2 of 93 mm Hg. How does the nurse best intervene? a.Increase the oxygen. b.Administer an antianxiety medication. c.Administer a bronchodilator. d.Assist with relaxation techniques. The nurse should assess the client's oxygenation; however, this client's arterial blood gas documents that the client's hypoxia has resolved. At this time it is not necessary to increase the oxygen or administer a bronchodilator; both of these interventions would be appropriate if the client were hypoxic. The client with respiratory problems should not take an antianxiety medication as a first-line intervention, because this may decrease the respiratory rate and/or alertness. The best intervention at this time is to assist with relaxation techniques.

D

A nurse assesses a client who reports waking up feeling very tired, even after 8 hours of good sleep. Which action should the nurse take first? a. Contact the provider for a prescription for sleep medication. b. Tell the client not to drink beverages with caffeine before bed. c. Educate the client to sleep upright in a reclining chair. d. Ask the client if he or she has ever been evaluated for sleep apnea.

D

A nurse cares for a client who has packing inserted for posterior nasal bleeding. Which action should the nurse take first? a. Assess the clients pain level. b. Keep the clients head elevated. c. Teach the client about the causes of nasal bleeding. d. Make sure the string is taped to the clients cheek.

D

A nurse is assessing a client who has suffered a nasal fracture. Which assessment should the nurse perform first? a. Facial pain b. Vital signs c. Bone displacement d. Airway patency

D

For which hospitalized client does the nurse recommend the ongoing use of a urinary catheter? A. 35-year-old woman who was admitted with a splenic laceration and femur fracture (closed repair completed) following a car crash B. 48-year-old man who has established paraplegia and is admitted for pneumonia C. 61-year-old woman who is admitted following a fall at home and has new-onset dysrhythmia D. 74-year-old man who has lung cancer with brain metastasis and is being transitioned to hospice for end-of-life care

D

The client receiving mechanical ventilation has become more restless over the course of the shift. Which is the nurse's first action? a.Sedate the client. b.Call the health care provider. c.Assess the client for pain. d.Assess the client's oxygenation. Increasing restlessness in a client being mechanically ventilated may mean that the client is not receiving sufficient oxygen. It can also be a manifestation of pain. When in doubt, determining the adequacy of ventilation has the highest priority. The nurse would not sedate the client until the cause of the restlessness has been addressed. The nurse would call the provider if the cause could not be determined and addressed, or if the client's status deteriorated.

D

The nurse is caring for a client who has been intubated and placed on a ventilator for treatment of acute respiratory distress syndrome (ARDS). Aside from assessing oxygenation, what is the nurse's priority action? a.Assess hemoglobin. b.Administer ferrous sulfate. c.Assess muscle strength. d.Consult with the registered dietitian. The client who is intubated needs nutrition delivered via enteral tube feeding. If nutrition is ignored, the client's respiratory status can deteriorate, because respiratory muscle function can deteriorate.

D

The nurse is caring for a client who is taken off a ventilator and placed on continuous positive airway pressure (CPAP). What intervention is most appropriate for this client? a.Administering antianxiety medications PRN b.Administering a medication to help the client sleep c.Telling the client to relax and let the ventilator do the work d.Making sure the client is breathing spontaneously A requirement for using CPAP is that the client will be able to breathe spontaneously. Antianxiety and sleep medications should not be administered to the client during weaning. Telling the client to relax may be helpful in some cases but does not take priority over ensuring the client's ability to breathe spontaneously.

D

Which assessment finding of a client requires the nurse's immediate action? a.Being intubated for 4 days b.Uneven breath sounds c.Wheezing on auscultation d.Having the endotracheal (ET) tube taped to the lower jaw The endotracheal tube can be taped to the upper lip but should never be taped to the lower jaw because the lower jaw moves too much. The other clients need to be assessed by the nurse, but the one with the ET tube taped to the jaw requires immediate action.

D

The nurse has these client assignments. Which client does the nurse encourage to consume 2 to 3 liters of fluid each day? Client with chronic kidney disease Client with heart failure Client with complete bowel obstruction Client with hyperparathyroidism

D A major feature of hyperparathyroidism is hypercalcemia, which predisposes a client to kidney stones; this client should remain hydrated. A client with chronic kidney disease should not consume 2 to 3 liters of water because the kidneys are not functioning properly, and this could lead to fluid retention. People with heart failure typically have fluid volume excess. A client with complete bowel obstruction may experience vomiting and should be NPO.

When planning an assessment of the urethra, what does the nurse do first? Examine the meatus. Note any unusual discharge. Record the presence of abnormalities. Don gloves.

D Before examination begins, body fluid precautions (gloves) must be implemented first. Examining the meatus, noting unusual discharge, or recording the presence of abnormalities are things that the nurse should do after putting on gloves.

Which technique does the nurse use to obtain a sterile urine specimen from a client with a Foley catheter? Disconnect the Foley catheter from the drainage tube and collect urine directly from the Foley. Remove the existing catheter and obtain a sample during the process of inserting a new Foley. Use a sterile syringe to withdraw urine from the urine collection bag. Clamp the tubing, attach a syringe to the specimen, and withdraw at least 5 mL of urine.

D Clamping the tubing, attaching a syringe to the specimen, and withdrawing at least 5 mL of urine is the correct technique for obtaining a sterile urine specimen from the client with a Foley catheter. Disconnecting the Foley catheter from the drainage tube and collecting urine directly from the Foley increases the risk for microbe exposure. A Foley catheter should not be removed to get a urine sample. Microbes may be in the urine collection bag from standing urine, so using a sterile syringe to withdraw urine from the urine collection bag is not the proper technique to obtain a urine sample.

Four individuals have given information about their suicide plans. Which plan evidences the highest suicide risk? a. Turning on the oven and letting gas escape into the apartment during the night b. Cutting the wrists in the bathroom while the spouse reads in the next room c. Overdosing on aspirin with codeine while the spouse is out with friends d. Jumping from a railroad bridge located in a deserted area late at night

D This is a highly lethal method with little opportunity for rescue. The other options are lower lethality methods with higher rescue potential.

A client is scheduled for a cystoscopy later this morning. The consent form is not signed, and the client has not had any preoperative medication. The nurse notes that the provider visited the client the day before. What action does the nurse take? Asks the client to sign the informed consent Cancels the procedure Asks the client's spouse to sign the form Notifies the department and the provider

D The client may be asked to sign the consent form in the department; notifying both the provider and the department ensures communication across the continuum of care, with less likelihood of omission of information. The provider gives the client a complete description of and reasons for the procedure and explains complications; the nurse reinforces this information. The procedure should not be cancelled without an attempt to correct the situation. The client has not received sedation, so nothing suggests that the client is not competent to consent.

A client had a computed tomography (CT) scan with contrast dye 8 hours ago. Which nursing intervention is the priority for this client? Maintaining bedrest Medicating for pain Monitoring for hematuria Promoting fluid intake

D The nurse should ensure adequate hydration by urging the client to take oral fluid or by giving IV fluids. Hydration reduces the risk for kidney damage. Bedrest is not indicated for the client who has undergone a CT scan with contrast dye. CT with contrast dye is not a painful procedure, so pain medication is not indicated. The client who has undergone CT with contrast dye is not at risk for hematuria.

One of the nurse's roles is talking to adult clients about urinary and sexual hygiene. Which words does the nurse use when referring to the client's reproductive body parts? Children's terms that are easily understood Slang words and terms that are heard "socially" Technical and medical terminology Words that the client uses

D The nurse should use the terms with which the client is most familiar, so there is no chance for the client to misunderstand information. Using the client's language ensures the comfort level for the client. The use of children's terms is demeaning to adult clients. The use of slang terms is unprofessional. Technical terms should not be used because the client may not know what they mean.

A client is brought to the emergency department after sustaining injuries in a severe car crash. The client's chest wall does not appear to be moving normally with respirations, oxygen saturation is 82%, and the client is cyanotic. What action by the nurse is the priority? a. Administer oxygen and reassess. b. Auscultate the client's lung sounds. c. Facilitate a portable chest x-ray. d. Prepare to assist with intubation.

D This client has manifestations of flail chest and, with the other signs, needs to be intubated and mechanically ventilated immediately. The nurse does not have time to administer oxygen and wait to reassess, or to listen to lung sounds. A chest x-ray will be taken after the client is intubated.

Sleep apnea would be diagnosed in a patient who has which of these findings? 1. More than 15 apnea episodes per hour of sleep 2. More than 5 apnea episodes per hour over an 8-hour period 3. More than 75% of the apneas are obstructive. 4. More than 75% of the apneas and hypopneas are obstructive. a. 1, 4 b. 2, 3 c. 2, 3, 4 d. 1, 3, 4

D During a sleep study, obstructive sleep apnea (OSA) is confirmed when either of the following two conditions exists: 15 or more apneas, hypopneas, or RERAs per hour of sleep (i.e., the AHI or RDI >15 events/hour) in an asymptomatic patient. More than 75% of the apneas and hypopneas must be obstructive. Or, 5 or more apneas, hypopneas, or RERAs per hour of sleep (i.e., the AHI or RDI >5/hour events/hour) in patients with symptoms (e.g., sleepiness, fatigue, and inattention) or signs of disturbed sleep (e.g., snoring, restless sleep, and respiratory pauses). More than 75% of the apneas and hypopneas must be obstructive.

19. A nurse teaches a client with functional urinary incontinence. Which statement should the nurse include in this clients teaching? a. You must clean around your catheter daily with soap and water. b. Wash the vaginal weights with a 10% bleach solution after each use. c. Operations to repair your bladder are available, and you can consider these. d. Buy slacks with elastic waistbands that are easy to pull down.

D Functional urinary incontinence occurs as the result of problems not related to the clients bladder, such as trouble ambulating or difficulty accessing the toilet. One goal is that the client will be able to manage his or her clothing independently. Elastic waistband slacks that are easy to pull down can help the client get on the toilet in time to void. The other instructions do not relate to functional urinary incontinence.

8. A nurse plans care for a client with overflow incontinence. Which intervention should the nurse include in this clients plan of care to assist with elimination? a. Stroke the medial aspect of the thigh. b. Use intermittent catheterization. c. Provide digital anal stimulation. d. Use the Valsalva maneuver.

D In clients with overflow incontinence, the voiding reflex arc is not intact. Mechanical pressure, such as that achieved through the Valsalva maneuver (holding the breath and bearing down as if to defecate), can initiate voiding. Stroking the medial aspect of the thigh or providing digital anal stimulation requires the reflex arc to be intact to initiate elimination. Due to the high risk for infection, intermittent catheterization should only be implemented when other interventions are not successful.

A student nurse asks for an explanation of "refractory hypoxemia." What answer by the nurse instructor is best? a. "It is chronic hypoxemia that accompanies restrictive airway disease." b. "It is hypoxemia from lung damage due to mechanical ventilation." c. "It is hypoxemia that continues even after the client is weaned from oxygen." d. "It is hypoxemia that persists even with 100% oxygen administration."

D Refractory hypoxemia is hypoxemia that persists even with the administration of 100% oxygen. It is a cardinal sign of acute respiratory distress syndrome. It does not accompany restrictive airway disease and is not caused by the use of mechanical ventilation or by being weaned from oxygen.

11. A nurse cares for a client who has kidney stones from secondary hyperoxaluria. Which medication should the nurse anticipate administering? a. Phenazopyridine (Pyridium) b. Propantheline (Pro-Banthine) c. Tolterodine (Detrol LA) d. Allopurinol (Zyloprim)

D Stones caused by secondary hyperoxaluria respond to allopurinol (Zyloprim). Phenazopyridine is given to clients with urinary tract infections. Propantheline is an anticholinergic. Tolterodine is an anticholinergic with smooth muscle relaxant properties.

A client is on mechanical ventilation and the client's spouse wonders why ranitidine (Zantac) is needed since the client "only has lung problems." What response by the nurse is best? a. "It will increase the motility of the gastrointestinal tract." b. "It will keep the gastrointestinal tract functioning normally." c. "It will prepare the gastrointestinal tract for enteral feedings." d. "It will prevent ulcers from the stress of mechanical ventilation."

D Stress ulcers occur in many clients who are receiving mechanical ventilation, and often prophylactic medications are used to prevent them. Frequently used medications include antacids, histamine blockers, and proton pump inhibitors. Zantac is a histamine blocking agent.

17. A nurse cares for a client who is scheduled for the surgical creation of an ileal conduit. The client states, I am anxious about having an ileal conduit. What is it like to have this drainage tube? How should the nurse respond? a. I will ask the provider to prescribe you an antianxiety medication. b. Would you like to discuss the procedure with your doctor once more? c. I think it would be nice to not have to worry about finding a bathroom. d. Would you like to speak with someone who has an ileal conduit?

D The goal for the client who is scheduled to undergo a procedure such as an ileal conduit is to have a positive self-image and a positive attitude about his or her body. Discussing the procedure candidly with someone who has undergone the same procedure will foster such feelings, especially when the current client has an opportunity to ask questions and voice concerns to someone with first-hand knowledge. Medications for anxiety will not promote a positive self-image and a positive attitude, nor will discussing the procedure once more with the physician or hearing the nurses opinion.

14. A nurse assesses clients on the medical-surgical unit. Which client is at greatest risk for bladder cancer? a. A 25-year-old female with a history of sexually transmitted diseases b. A 42-year-old male who has worked in a lumber yard for 10 years c. A 55-year-old female who has had numerous episodes of bacterial cystitis d. An 86-year-old male with a 50pack-year cigarette smoking history

D The greatest risk factor for bladder cancer is a long history of tobacco use. The other factors would not necessarily contribute to the development of this specific type of cancer.

A nurse is caring for a client on mechanical ventilation. When double-checking the ventilator settings with the respiratory therapist, what should the nurse ensure as a priority? a. The client is able to initiate spontaneous breaths. b. The inspired oxygen has adequate humidification. c. The upper peak airway pressure limit alarm is off. d. The upper peak airway pressure limit alarm is on.

D The upper peak airway pressure limit alarm will sound when the airway pressure reaches a preset maximum. This is critical to prevent damage to the lungs. Alarms should never be turned off. Initiating spontaneous breathing is important for some modes of ventilation but not others. Adequate humidification is important but does not take priority over preventing injury.

The nurse is teaching a man about how to prevent UTIs. What information does the nurse include? a. "Have a minimal fluid intake of 5 L daily, unless contraindicated." b. "Empty your bladder before and after sexual intercourse." c. "Make sure that spermicides are used with condoms." d. "Gently wash the genital area before intercourse."

D - "Gently wash the genital area before intercourse."

A patient has had a bladder suspension and a suprapubic catheter is in place. The patient wants to know how long the catheter will remain in place. What is the nurse's best response? a. "Typically it remains for 24 hours postoperatively." b. "It will be removed at your first clinic visit." c. "When you can void on your own, it will be removed." d. "It will be removed when you can void and residual urine is less than 50 mL."

D - "It will be removed when you can void and residual urine is less than 50 mL."

Which patient should not be advised to take cranberry juice? a. 26-year-old pregnant woman with a history of uncomplicated UTI b. 23-year-old man with history of recurrent kidney stones c. 65-year-old man with urinary retention secondary to enlarged prostate d. 33-year-old woman with dysuria associated with interstitial cystitis

D - 33-year-old woman with dysuria associated with interstitial cystitis

The nurse is teaching a patient a behavioral intervention for bladder compression. In order to correctly perform the Credé method, what does the nurse teach the patient to do? a. Insert the fingers into the vagina and gently push against the vaginal wall. b. Breathe in deeply and direct the pressure towards the bladder during exhalation. c. Empty the bladder, wait a few minutes, and attempt a second bladder emptying. d. Apply firm and steady pressure over the bladder area with the palm of the hand.

D - Apply firm and steady pressure over the bladder area with the palm of the hand.

The nurse is performing an assessment on a patient with probable stress incontinence. Which assessment technique does the nurse use to validate stress incontinence? a. Assess the abdomen to estimate bladder fullness. b. Check for residual urine using a portable ultrasound. c. Catheterize the patient immediately after voiding. d. Ask the patient to cough while wearing a perineal pad.

D - Ask the patient to cough while wearing a perineal pad.

The nurse is caring for a patient with functional incontinence. The UAP reports that "the linens have been changed four times within the past 6 hours, but the patient refuses to wear a diaper." What does the nurse do next? a. Thank the UAP for the hard work and advise to continue to change the linens. b. Call the health care provider to obtain an order for an indwelling catheter. c. Instruct the UAP to stop using the word "diaper" and instead use "incontinence pants." d. Assess the patient for any new urinary problems and ask about toileting preferences.

D - Assess the patient for any new urinary problems and ask about toileting preferences.

What role does drug therapy have as an intervention for reflex (overflow) urinary incontinence? a. Captopril (Capoten) is given to lower urine cystine levels. b. Levofloxacin (Levaquin) is given to prevent UTIs with this type of incontinence. c. Midorine (ProAmatine) is given to increase the contractile force of the bladder. d. Bethanechol chloride (Urecholine) may be used short-term after surgery.

D - Bethanechol chloride (Urecholine) may be used short-term after surgery.

A patient has UTI symptoms but there are no bacteria in the urine. The health care provider suspects interstitial cystitis. The nurse prepares patient teaching material for which diagnostic test? a. Urography b. Abdominal sonography c. Computed tomography (CT) d. Cystoscopy

D - Cystoscopy

A patient comes to the clinic and reports severe flank pain, bladder distention, and nausea and vomiting with increasingly smaller amounts of urine with frank blood. The patient states, "I have kidney stones and I just need a prescription for pain medication." What is the nurse's priority concern? a. Controlling the patient's pain b. Checking the quantity of blood in the urine c. Flushing the kidneys with oral fluids d. Determining if there is an obstruction

D - Determining if there is an obstruction

A patient is returning from the postanesthesia care unit after surgery for bladder cancer resulting in a cutaneous ureterostomy. Where does the nurse expect the stoma to be located? a. On the perineum b. At the beltline c. On the posterior flank d. In the midabdominal area

D - In the midabdominal area

A patient has had surgery for bladder cancer. To prevent recurrence of superficial bladder cancer, the nurse anticipates that the health care provider is likely to recommend which treatment? a. No treatment is needed for this benign condition. b. Intravesical instillation of single-agent chemotherapy. c. Radiation therapy to the bladder, ureters, and urethra. d. Intravesical instillation of bacille Calmette- Guérin.

D - Intravesical instillation of bacille Calmette- Guérin.

A patient is considering vaginal cone therapy, but is a little hesitant because she does not understand how it works. What does the nurse tell her about how vaginal cone therapy improves incontinence? a. It mechanically obstructs urine loss from the urethra. b. It repositions the bladder to reduce compression. c. It increases the normal flora of the perineum. d. It strengthens pelvic floor muscles.

D - It strengthens pelvic floor muscles.

The health care provider verbally informs the nurse that the patient needs a fluoroquinolone antibiotic to treat a UTI. The pharmacy delivers gabapentin (Neurontin). What should the nurse do first? a. Administer the medication as ordered. b. Call the pharmacist and ask for a read back of the order. c. Call the health care provider for clarification of the order. d. Look at the written order to clarify the name of the medication.

D - Look at the written order to clarify the name of the medication.

The nurse is caring for a patient with urolithiasis. Which medication is likely to be given in the acute phase to relieve the patient's severe pain? a. Ketorolac (Toradol) b. Oxybutynin chloride (Ditropan) c. Propantheline bromide (Pro-Banthine) d. Morphine sulfate (Astramorph)

D - Morphine sulfate (Astramorph)

In which patient circumstance would the nurse question the order for the insertion of an indwelling catheter? a. Patient is critically ill and at risk for hypovolemic shock. b. Patient has urinary retention with beginnings of hydronephrosis. c. Patient was in a car accident and has a possible spinal cord injury. d. Patient has functional incontinence related to Alzheimer's disease.

D - Patient has functional incontinence related to Alzheimer's disease.

A patient has been started on oxybutynin (Ditropan) for urinary incontinence. What is the major action of this medication? a. Increases blood flow to the urethra b. Blocks acetylcholine receptors c. Causes slight numbing of the bladder d. Relaxes bladder muscles

D - Relaxes bladder muscles

A patient with a history of kidney stones presents with severe flank pain, nausea, vomiting, pallor, and diaphoresis. He reports freely passing urine, but it is bloody. The priority for nursing care is to monitor for which patient problem? a. Possible dehydration b. Impaired tissue perfusion c. Impaired urinary elimination d. Severe pain

D - Severe pain

A patient repots the loss of small amounts of urine during coughing, sneezing, jogging, or lifting. Which type of incontinence do these symptoms describe? a. Urge b. Overflow c. Functional d. Stress

D - Stress

Which urine characteristic suggests that the patient is drinking a sufficient amount of fluid? a. Urine pH is between 6 to 6.5. b. Urine has a high specific gravity. c. Urine has a faint ammonia odor. d. Urine is a pale yellow color.

D - Urine is a pale yellow color.

A client has a long history of hypertension. Which category of medications would the nurse expect to be ordered to avoid chronic kidney disease (CKD)? a. Antibiotic b. Histamine blocker c. Bronchodilator d. Angiotensin-converting enzyme (ACE) inhibitor

D ~ ACE inhibitors stop the conversion of angiotensin I to the vasoconstrictor angiotensin II. This category of medication also blocks bradykinin and prostaglandin, increases renin, and decreases aldosterone, which promotes vasodilation and perfusion to the kidney. Antibiotics fight infection, histamine blockers decrease inflammation, and bronchodilators increase the size of the bronchi; none of these medications helps slow the progression of CKD in clients with hypertension.

A nurse is caring for a client who is scheduled for a dose of cefazolin and vitamins at this time. Hemodialysis for this client is also scheduled in 60 minutes. Which action by the nurse is best? a. Administer cefazolin since the level of the antibiotic must be maintained. b. Hold the vitamins but administer the cefazolin. c. Hold the cefazolin but administer the vitamins. d. Hold all medications since both cefazolin and vitamins are dialyzable.

D ~ Both the cefazolin and the vitamins should be held until after the hemodialysis is completed because they would otherwise be removed by the dialysis process.

A client is assessed by the nurse after a hemodialysis session. The nurse notes bleeding from the client's nose and around the intravenous catheter. What action by the nurse is the priority? a. Hold pressure over the client's nose for 10 minutes. b. Take the client's pulse, blood pressure, and temperature. c. Assess for a bruit or thrill over the arteriovenous fistula. d. Prepare protamine sulfate for administration.

D ~ Heparin is used with hemodialysis treatments. The bleeding alerts the nurse that too much anticoagulant is in the client's system and protamine sulfate should be administered. Pressure, taking vital signs, and assessing for a bruit or thrill are not as important as medication administration.

A client with chronic kidney disease states, "I feel chained to the hemodialysis machine." What is the nurse's best response to the client's statement? a. That feeling will gradually go away as you get used to the treatment. b. You probably need to see a psychiatrist to see if you are depressed. c. Do you need help from social services to discuss financial aid? d. Tell me more about your feelings regarding hemodialysis treatment.

D ~ The nurse needs to explore the client's feelings in order to help the client cope and enter a phase of acceptance or resignation. It is common for clients to be discouraged because of the dependency of the treatment, especially during the first year. Referrals to a mental health provider or social services are possibilities, but only after exploring the client's feelings first. Telling the client his or her feelings will go away is dismissive of the client's concerns.

A client with acute kidney injury has a blood pressure of 76/55 mm Hg. The health care provider ordered 1000 mL of normal saline to be infused over 1 hour to maintain perfusion. The client is starting to develop shortness of breath. What is the nurses priority action? a. Calculate the mean arterial pressure (MAP). b. Ask for insertion of a pulmonary artery catheter. c. Take the client's pulse. d. Slow down the normal saline infusion.

D ~ The nurse should assess that the client could be developing fluid overload and respiratory distress and slow down the normal saline infusion. The calculation of the MAP also reflects perfusion. The insertion of a pulmonary artery catheter would evaluate the client's hemodynamic status, but this should not be the initial action by the nurse. Vital signs are also important after adjusting the intravenous infusion.

The nurse is teaching the importance of a low purine diet to a client admitted with urolithiasis consisting of uric acid. Which statement by the client indicates that teaching was effective? A. "I am so relieved that I can continue eating my fried fish meals every week." B. "I will quit growing rhubarb in my garden since I'm not supposed to eat it anymore." C. "My wife will be happy to know that I can keep enjoying her liver and onions recipe." D. "I will no longer be able to have red wine with my dinner."

D. "I will no longer be able to have red wine with my dinner." Nutrition therapy depends on the type of stone formed. When stones consist of uric acid (urate), the client should decrease intake of purine sources such as organ meats, poultry, fish, gravies, red wines, and sardines. Reduction of urinary purine content may help prevent these stones from forming. Avoiding oxalate sources such as spinach, black tea, and rhubarb is appropriate when the stone consists of calcium oxalate.

The nurse is assessing a client admitted with status asthmaticus. The nurse finds a sudden absence of wheezing in the lung fields and sets which of these as the priority action? A. Education to prevent future exacerbations B. Administration of a bronchodilator C. Measures to reduce anxiety D. Activation of the rapid response team to secure an airway

D. Activation of the rapid response team to secure an airway

The nurse on a medical surgical unit is planning bed assignments for a new admission who has cystic fibrosis (CF) and is infected with Burkholderia cepacia. Which of these room assignments is most appropriate for this client? A. A room with laminar air flow B. A room with a client who has Down syndrome and pneumonia C. A room with another client who has cystic fibrosis D. A private room with a bathroom

D. A private room with a bathroom

When preparing to administer a prescribed subcutaneous dose of NPH insulin from an open vial taken from a medication drawer to a client with diabetes, the nurse notes the solution is cloudy. What action will the nurse perform to ensure client safety? A. Warm the vial in a bowl of warm water until it reaches normal body temperature B. Return the vial to the pharmacy and open a fresh vial of NPH insulin C. Roll the vial between the hands until the insulin is clear D. Check the expiration date and draw up the insulin dose

D. Check the expiration date and draw up the insulin dose

The nurse is providing education to a client with chronic bronchitis who has a new prescription for a mucolytic. Which of these will the nurse teach the client about the purpose of the medication? A. Mucolytics decrease secretion production. B. Mucolytics increase gas exchange in the lower airways. C. Mucolytics provide bronchodilation in clients with chronic obstructive pulmonary disease. D. Mucolytics thin secretions, allowing for easier expectoration.

D. Mucolytics thin secretions, allowing for easier expectoration

The chest tube of a client who is 12 hours postoperative from a lobectomy separates from the drainage system. What is the nurse's best first action? a. Immediately call the surgeon or rapid response team. b. Notify respiratory therapy to set up a new drainage system. c. Cover the insertion site with a sterile occlusive dressing and tape down on three sides. d. Place the end of the disconnected tube into a container of sterile water positioned below the chest.

D. Place the end of the disconnected tube into a container of sterile water positioned below the chest

The nurse reviewing the preadmission testing lab values for a 62 year old client scheduled for a total knee replacement finds an A1C value of 6.2%. How will the nurse interpret this finding? A. The clients A1C is completely normal B. The client has type 1 diabetes C. The client has type 2 diabetes D. The client has prediabetes mellitus

D. The client has prediabetes mellitus

The nurse is preparing to administer oxygen to a client with chronic obstructive pulmonary disease (COPD) who has hypoxemia and hypercarbia. The nurse recognizes that a positive outcome to therapy has been achieved by which of these findings? A. The pCO2 is within normal range. B. The client's face is very pink. C. The client reports decreased distress. D. The oxygen saturation is between 88% and 90%.

D. The oxygen saturation is between 88% and 90%

A nurse observes that a clients anteroposterior (AP) chest diameter is the same as the lateral chest diameter. Which question should the nurse ask the client in response to this finding? a. Are you taking any medications or herbal supplements? b. Do you have any chronic breathing problems? c. How often do you perform aerobic exercise? d. What is your occupation and what are your hobbies?

Do you have any chronic breathing problems? The normal chest has a lateral diameter that is twice as large as the AP diameter. When the AP diameter approaches or exceeds the lateral diameter, the client is said to have a barrel chest. Most commonly, barrel chest occurs as a result of a long-term chronic airflow limitation problem, such as chronic obstructive pulmonary disease or severe chronic asthma. It can also be seen in people who have lived at a high altitude for many years. Therefore, an AP chest diameter that is the same as the lateral chest diameter should be rechecked but is not as indicative of underlying disease processes as an AP diameter that exceeds the lateral diameter. Medications, herbal supplements, and aerobic exercise are not associated with a barrel chest. Although occupation and hobbies may expose a client to irritants that can cause chronic lung disorders and barrel chest, asking about chronic breathing problems is more direct and should be asked first.

When caring for a client who had a lobectomy the nurse notes small bubbles in the water seal chamber of the disposable chest drainage device during coughing. Which of these reflects the appropriate action by the nurse? Document the finding in the medical record. Check the tube for blood clots. Briefly increase the amount of suction. Add additional sterile water to the water seal chamber

Document the finding in the medical record. The nurse recognizes that gentle bubbling in the water seal chamber is normal during the client's exhalation, forceful cough, or position changes. This indicates air is leaving the pleural space which is the intended purpose of the chest drain.Bubbling in the water seal chamber is absent if a kink or a blockage is present because air would not be able to escape from the chest cavity. Increasing the amount of suction without an order could damage lung tissue. There is no indication that the level of fluid in the water seal chamber is low.

A nurse auscultates a harsh hollow sound over a clients trachea and larynx. Which action should the nurse take first? a. Document the findings. b. Administer oxygen therapy. c. Position the client in high-Fowlers position. d. Administer prescribed albuterol.

Document the findings. Bronchial breath sounds, including harsh, hollow, tubular, and blowing sounds, are a normal finding over the trachea and larynx. The nurse should document this finding. There is no need to implement oxygen therapy, administer albuterol, or change the clients position because the finding is normal.

Define the "E" in the RIFLE classification system.

END-STAGE KIDNEY DISEASE = requiring dialysis >3 mos

Which factor is most important for the nurse to assess before providing instruction to a client newly diagnosed with diabetes about the disease and its management? Current energy level and rest patterns Sexual orientation Current lifestyle for diet and exercise Education and literacy levels

Education and literacy levels The most important factor for the nurse to determine before providing instruction to the newly diagnosed client with diabetes is the client's educational level and literacy level. A large amount of information must be synthesized. Written instructions are typically given. The client's ability to learn and read is essential to provide the client with instructions and information about diabetes.Although lifestyle would be taken into account, it is not the priority. Sexual orientation will have no bearing on the ability of the client to provide self-care. Although energy level will influence the ability to exercise, it is not essential.

A client with a family history of colorectal cancer (CRC) regularly sees a health care provider for early detection of any signs of cancer. Which laboratory result may be an indication of CRC in this client? A. Decrease in liver function test results B. Elevated carcinoembryonic antigen C. Elevated hemoglobin levels D. Negative test for occult blood

Elevated carcinoembryonic antigen Carcinoembryonic antigen may be elevated in many clients diagnosed with CRC. Liver involvement may or may not occur in CRC. Hemoglobin will likely be decreased with CRC, not increased. An occult blood test is not reliable to affirm or rule out CRC.

The nurse suspects that a client may have acute pancreatitis as evidenced by which group of laboratory results? A. Deceased calcium, elevated amylase, decreased magnesium B. Elevated bilirubin, elevated alkaline phosphatase C. Elevated lipase, elevated white blood cell count, elevated glucose D. Decreased blood urea nitrogen (BUN), elevated calcium, elevated magnesium

Elevated lipase, elevated white blood cell count, elevated glucose Elevated lipase is more specific to a diagnosis of acute pancreatitis. Many pancreatic and nonpancreatic disorders can cause increased serum amylase levels. Bilirubin and alkaline phosphatase levels will be increased only if pancreatitis is accompanied by biliary dysfunction. Usually, calcium and magnesium will be increased and BUN increased, not decreased, in acute pancreatitis.

A nurse plans care for a client who is at high risk for a pulmonary infection. Which interventions should the nurse include in this clients plan of care? (Select all that apply.) a. Encourage deep breathing and coughing. b. Implement an air mattress overlay. c. Ambulate the client three times each day. d. Provide a diet high in protein and vitamins. e. Administer acetaminophen (Tylenol) twice daily

Encourage deep breathing and coughing, Ambulate the client three times each day & Provide a diet high in protein and vitamins. Regular pulmonary hygiene and activities to maintain health and fitness help to maximize functioning of the respiratory system and prevent infection. A client at high risk for a pulmonary infection may need a specialty bed to help with postural drainage or percussion; this would not include an air mattress overlay, which is used to prevent pressure ulcers. Tylenol would not decrease the risk of a pulmonary infection.

A client with colorectal cancer had colostomy surgery performed yesterday. The client is very anxious about caring for the colostomy and states that the health care provider's instructions "seem overwhelming." What does the nurse do first for this client? A. Encourages the client to look at and touch the colostomy stoma B. Instructs the client about complete care of the colostomy C. Schedules a visit from a client who has a colostomy and is successfully caring for it D. Suggests that the client involve family members in the care of the colostomy

Encourages the client to look at and touch the colostomy stoma The initial intervention is to get the client comfortable looking at and touching the stoma before providing instructions on its care. Instructing the client about colostomy care will be much more effective after the client's anxiety level has stabilized. Talking with someone who has gone through a similar experience may be helpful to the client only after his or her anxiety level has stabilized. The client has begun to express feelings regarding the colostomy and its care; it is too soon to involve others. The client must get comfortable with this body image change first.

20. Which of the following characteristics should be included in a therapeutic nurse-client relationship? (Select all that apply.) 1. Meeting the psychological needs of the nurse and the client 2. Ensuring therapeutic termination 3. Promoting client insight into problematic behavior 4. Collaborating to set appropriate goals 5. Meeting both the physical and psychological needs of the client

Ensuring therapeutic termination Promoting client insight into problematic behavior Collaborating to set appropriate goals Meeting both the physical and psychological needs of the client Rationale: The nurse-client therapeutic relationship should include promoting client insight into problematic behavior, collaboration to set appropriate goals, meeting the physical and psychological needs of the client, and ensuring therapeutic termination. Meeting the nurse's psychological needs should never be addressed within the nurse-client relationship.

3. What should be the priority nursing action during the orientation (introductory) phase of the nurse-client relationship? 1. Acknowledge the client's actions and generate alternative behaviors. 2. Establish rapport and develop treatment goals. 3. Attempt to find alternative placement. 4. Explore how thoughts and feelings about this client may adversely impact nursing care.

Establish rapport and develop treatment goals. Rationale: The priority nursing action during the orientation phase of the nurse-client relationship should be to establish rapport and develop treatment goals. Rapport implies feelings on the part of both the nurse and the client, based on respect, acceptance, a sense of trust, and a nonjudgmental attitude. It is the essential foundation of the nurse-client relationship.

What is the nurse's best action when finding that a client who has had diabetes for 15 years has decreased sensory perception in both feet? Testing the sensory perception of the client's hands Examining both feet for indications of injury Explaining to the client that peripheral neuropathy is now present Documenting the finding as the only action

Examining both feet for indications of injury When reduced peripheral sensory perception is present, the likelihood of injury is high. Any open area or other problem on the foot of a person with diabetes is at great risk for infection and must be managed carefully and quickly. Checking for sensory perception on the hands and other areas is important but can come after a thorough foot examination.

Define the "F" in the RIFLE classification system.

FAILURE STAGE = Serum Creatinine increased by 3.0 or GFR decrease by >75% (or UO <0.3 mL/kg/hr X 24hrs) (or anuria X 12hrs)

The nurse is providing teaching for a client who has been newly diagnosed with lung cancer and will be undergoing radiation therapy. Which of these points would be covered in the teaching session? Select all that apply. Hair loss will occur. Do not expose the site to sun. Loss of appetite may develop. Pain in the area is expected. Fatigue may occur. Changes in taste may occur.

Fatigue may occur Changes in taste may occur Do not expose the site to sun. Skin in the path of radiation is more sensitive to sun damage. Clients must avoid direct skin exposure to the sun during treatment and for at least 1 year after radiation is completed. Side effects also include skin irritation and peeling, fatigue, nausea, and taste changes. Some clients have esophagitis during therapy, making nutrition more difficult.Alopecia, or hair loss, is a side effect of chemotherapy, not of radiation to the chest. Loss of appetite is not specific to radiation therapy. Radiation therapy itself is painless and sensation-free.

A nurse teaches a client who is interested in smoking cessation. Which statements should the nurse include in this clients teaching? (Select all that apply.) a. Find an activity that you enjoy and will keep your hands busy. b. Keep snacks like potato chips on hand to nibble on. c. Identify a punishment for yourself in case you backslide. d. Drink at least eight glasses of water each day. e. Make a list of reasons you want to stop smoking.

Find an activity that you enjoy and will keep your hands busy, Drink at least eight glasses of water each day, Make a list of reasons you want to stop smoking. The nurse should teach a client who is interested in smoking cessation to find an activity that keeps the hands busy, to keep healthy snacks on hand to nibble on, to drink at least 8 glasses of water each day, and to make a list of reasons for quitting smoking. The nurse should also encourage the client not to be upset if he or she backslides and has a cigarette.

10. Which therapeutic communication technique is being used in the following nurse-client interaction? Client: "When I am anxious, the only thing that calms me down is alcohol." Nurse: "Other than drinking, what alternatives have you explored to decrease anxiety?" 1. Reflecting 2. Making observations 3. Formulating a plan of action 4. Giving recognition

Formulating a plan of action Rationale: The nurse is using the therapeutic communication technique of formulating a plan of action to help the client explore alternatives to drinking. The use of this technique may serve to prevent anger or anxiety from escalating.

7. When an individual is "two-faced," which characteristic essential to the development of a therapeutic relationship should a nurse identify as missing? 1. Respect 2. Genuineness 3. Sympathy 4. Rapport

Genuineness Rationale: When an individual is "two-faced," which means double-dealing or deceitful, the nurse should identify that genuineness is missing in the relationship. Genuineness refers to the nurse's ability to be open and honest and maintain congruence between what is felt and what is communicated. When a nurse fails to bring genuineness to the relationship, trust cannot be established.

2. If a client demonstrates transference toward a nurse, how should the nurse respond? 1. Promote safety and immediately terminate the relationship with the client. 2. Encourage the client to ignore these thoughts and feelings. 3. Immediately reassign the client to another staff member. 4. Help the client to clarify the meaning of the relationship, based on the present situation.

Help the client to clarify the meaning of the relationship, based on the present situation. Rationale: The nurse should respond to a client's transference by clarifying the meaning of the nurse-client relationship based on the present situation. Transference occurs when the client unconsciously displaces feelings about a person from the past toward the nurse. The nurse should assist the client in separating the past from the present.

The nurse in the clinic is following up on diagnostic testing for a client recently diagnosed with metastatic lung cancer and back pain. Which of these findings does the nurse expect to uncover? Hyperkalemia Hyperglycemia Hypercalcemia Hypernatremia

Hypercalcemia Hypercalcemia is the result of increasing parathyroid hormone as a paraneoplastic complication of cancer as well as bone metastasis. Bone metastasis should be suspected in the presence of back pain.Paraneoplastic syndromes are manifested by Cushing's syndrome, weight gain and dilution of electrolytes (SIADH) with resulting hyponatremia. Gynecomastia and hypoglycemia may also occur. Hyperkalemia most typically occurs with tumor lysis syndrome where multiple electrolyte imbalances develop impaired renal function and oliguria.

A nurse collaborates with a respiratory therapist to complete pulmonary function tests (PFTs) for a client. Which statements should the nurse include in communications with the respiratory therapist prior to the tests? (Select all that apply.) a. I held the clients morning bronchodilator medication. b. The client is ready to go down to radiology for this examination. c. Physical therapy states the client can run on a treadmill. d. I advised the client not to smoke for 6 hours prior to the test. e. The client is alert and can follow your commands

I held the clients morning bronchodilator medication, I advised the client not to smoke for 6 hours prior to the test. & the client is alert and can follow your commands. To ensure the PFTs are accurate, the therapist needs to know that no bronchodilators have been administered in the past 4 to 6 hours, the client did not smoke within 6 to 8 hours prior to the test, and the client can follow basic commands, including different breathing maneuvers. The respiratory therapist can perform PFTs at the bedside. A treadmill is not used for this test.

Define the "I" in the RIFLE classification system.

INJURY STAGE = Serum Creatinine increased by 2.0 or GFR decrease by >50% (or UO <0.5 mL/kg/hr X 12hrs)

What is the nurse's best response when a client with diabetes who is being treated for hypoglycemic asks why people without diabetes don't become severely hypoglycemic even after fasting for 8 hours? In a person without diabetes, fasting for 8 hours converts proteins into glycose (gluconeogenesis) so that hypergycemia develops rather than hypoglycemia. In a person without diabetes, the secretion of glucagon prevents hypoglycemia by promoting glucose release from liver storage sites (glycogenolysis). Normal metabolism is so slow when a person without diabetes fasts that blood glucose does not enter cells to be used for energy. As a result, hypoglycemia does not occur. Lipolysis (fat breakdown) in fat stores occurs faster in the nondiabetic person, which converts fatty acids into glucose to maintain blood glucose levels.

In a person without diabetes, the secretion of glucagon prevents hypoglycemia by promoting glucose release from liver storage sites (glycogenolysis). Glucagon is a counter-regulatory hormone secreted by pancreatic alpha cells when blood glucose levels are low, as they would be during an 8 hour fast. The body's metabolic rate does decrease during sleep (which is not stated in this question) but not sufficiently to prevent hypoglycemia. Glucagon works on the glycogen stored in the liver, breaking it down to glucose (glycogenolysis) molecules that are then released into the blood to maintain blood glucose levels and prevent hypoglycemia. Although proteins can be broken down and converted to glucose, they are not converted to glycogen. Fat break down through lipolysis can provide fatty acids for fuel but this is not glucose and lipolysis does not occur until all stored glycogen is used.

Which diagnostic results lead the nurse to suspect that a client may have gallbladder disease? A. Increased white blood cell (WBC) count, visualization of calcified gallstones, edema of the gallbladder wall B. Decreased WBC count, visualization of calcified gallstones, increased alkaline phosphatase C. Increased WBC count, visualization of noncalcified gallstones, edema of the gallbladder wall D. Decreased WBC count, visualization of noncalcified gallstones, increased alkaline phosphatase

Increased white blood cell (WBC) count, visualization of calcified gallstones, edema of the gallbladder wall An increased WBC count is evidence of inflammation. Only calcified gallstones will be visualized on abdominal x-ray. Ultrasonography of the right upper quadrant is the best diagnostic test for cholecystitis. Acute cholecystitis is seen as edema of the gallbladder wall and pericholecystic fluid. Alkaline phosphatase will be elevated if liver function is abnormal; this is not common in gallbladder disease.

Which client assessment finding indicates to the nurse the possible presence of diabetic autonomic neuropathy? Loss of sensation in both feet Hyperglycemia Intermittent constipation Increased thirst

Intermittent constipation Autonomic neuropathy can affect the entire GI system. The most common GI problem from diabetic automonic neuropathy is sluggish intestinal movement and chronic intermittent constipation.Loss of sensation in the feet is peripheral neuropathy, not autonomic neuropathy. Hyperglycemia is not related to any type of neuropathy. Increased thirst is related to hyperglycemia and increased blood osmolarity, not neuropathy.

.What is the mechanism of action for the chemotherapeutic drug cetuximab (Erbitux)? A. It destroys the cancer's cell wall, which will kill the cell. B. It decreases blood flow to rapidly dividing cancer cells. C. It stimulates the body's immune system and stunts cancer growth. D. It blocks factors that promote cancer cell growth.

It blocks factors that promote cancer cell growth. Cetuximab, a monoclonal antibody, may be given for advanced disease. This drug works by binding to a protein (epidermal growth factor receptor) to slow cell growth. The medication does not destroy the cancer's cell walls and does not stimulate the body's immune system or stunt cancer growth in that manner. The treatment does not decrease blood flow to rapidly dividing cancer cells.

A client who had been hospitalized with pancreatitis is being discharged with home health services. The client is severely weakened after this illness. Which nursing intervention is the highest priority in conserving the client's strength? A. Limiting the client's activities to one floor of the home B. Instructing the client to take an as-needed (PRN) sleeping medication at night C. Arranging for the client to have a nutritional consult to assess the client's diet D. Asking the health care provider for a request for PRN nasal oxygen

Limiting the client's activities to one floor of the home Limiting the client's activities to one floor of the home will prevent tiring the client with stair climbing. Taking a PRN sleeping medication may not necessarily increase the client's strength level or conserve strength; also, the client may not be experiencing difficulty sleeping. Arranging for a nutritional consult or placing the client on PRN nasal oxygen will not necessarily result in an increase in the client's strength level or conserve strength; no information suggests that the client has any history of breathing difficulties.

Serotonin and suicide

Low serotonin levels are associated with suicide

A nurse obtains the health history of a client who is recently diagnosed with lung cancer and identifies that the client has a 60 pack-year smoking history. Which action is most important for the nurse to take when interviewing this client? a. tell the client that he needs to quit smoking to stop further cancer development b. encourage the client to be completely honest about both tobacco and marijuana use c. maintain a nonjudgmental attitude to avoid causing the client to feel guilty d. avoid giving the client false hope regarding cancer treatment and prognosis.

Maintain a nonjudgmental attitude to avoid causing the client to feel guilty. Smoking history includes the use of cigarettes, cigars, pipe tobacco, marijuana, and other controlled substances. Because the client may have guilt or denial about this habit, assume a nonjudgmental attitude during the interview. This will encourage the client to be honest about the exposure. Pack-years= number of packs smoked daily x number of years the client has smoked. Quitting may not stop cancer development, don't give false hope.

When caring for a client with uremia, the nurse assesses for which symptom? Tenderness at the costovertebral angle (CVA) Cyanosis of the skin Nausea and vomiting Correct Insomnia

Manifestations of uremia include anorexia, nausea, vomiting, weakness, and fatigue. CVA tenderness is a sign of inflammation or infection in the renal pelvis. Cyanosis is related to poor tissue perfusion. Insomnia is nonspecific and may be caused by psychoemotional factors, medications, or other problems.

Suicide risk factors

Marital status (suicide rate is twice than for single people compared to married people) Gender (women attempt more, men succeed more) Age (risk increases with age, particularly in men) Religion (affliction with a religious group decreases the risk of suicide) Socioeconomic status (every low or high social classes) Ethnicity (whites are at the highest risk) Psychiatric illness Severe insomnia Alcohol/barbituates Psychosis with command hallucinations Affliction with chronic, painful, or disabling illness Family history LGBT community Having attempted suicide previously Loss of loved one through death or separation Bullying

What are the normal Creatinine Clearance values?

Men 107-139 mL/min Women 87-107 mL/min

What is the normal Glomerular Filtration Rate (GFR)?

Men 130 mL/min/1.73m2 Women 120 mL/min/1.73m2

The nurse expects that which client will be discharged to the home environment first? A. Older obese adult who has had a laparoscopic cholecystectomy B. Middle-aged thin adult who has had a laparoscopic cholecystectomy C. Middle-aged thin adult with a heart murmur who has had a traditional cholecystectomy D. Older obese adult with chronic obstructive pulmonary disease (COPD) who has had a traditional cholecystectomy

Middle-aged thin adult who has had a laparoscopic cholecystectomy The combination of client age, a thin frame, and the type of procedure performed will determine that the middle-aged thin client who had a laparoscopic cholecystectomy will be discharged first. Although the older obese client who had a laparoscopic cholecystectomy will have a faster discharge time than one with a traditional cholecystectomy, the client's obesity and age probably will require a longer stay. A traditional cholecystectomy will always require a longer recovery time. The older obese client with a history of COPD will likely have a more lengthy recovery because of associated breathing problems.

The nurse is providing education to a client with chronic bronchitis who has a new prescription for a mucolytic. Which of these will the nurse teach the client about the purpose of the medication? Mucolytics decrease secretion production. Mucolytics increase gas exchange in the lower airways. Mucolytics provide bronchodilation in clients with chronic obstructive pulmonary disease. Mucolytics thin secretions, allowing for easier expectoration.

Mucolytics thin secretions, allowing for easier expectoration. Client with chronic bronchitis typically produces large amounts of thick mucus interfering with gas exchange. Mucolytic means "breaking down mucus," resulting in thinner secretions which are easier to expectorate.Mucolytics do not decrease secretion production. Mucolytics may increase gas exchange as secretions are cleared, but this is an indirect property and is not the main function. Mucolytics do not have any bronchodilation properties.

A client diagnosed with irritable bowel syndrome (IBS) is discharged home with a variety of medications for IBS symptoms. Upon returning to the clinic, the client states, "Most of my symptoms have improved, except for the diarrhea." What does the nurse anticipate will be prescribed for this client? A. Antidiarrheal agent B. Muscarinic receptor antagonist C. Serotonin antagonist D. Tricyclic antidepressant

Muscarinic receptor antagonist A muscarinic (M3) receptor antagonist can also inhibit intestinal motility. Antidiarrheal agents and serotonin antagonists are not the most effective choices for this client. A tricyclic antidepressant is not going to be effective for this client's diarrhea.

Which new-onset symptoms will the nurse instruct a client with diabetes who is prescribed to take the sodium-glucose cotransport inhibitor, empagliflozin, to report to the diabetes health care provider to prevent harm? (Select all that apply.) Select all that apply. Muscle weakness and dizziness on standing Redness and tenderness at the injection site Rapid weight gain and shortness of breath Redness and tenderness of the perineum Sensations of hunger, tremors, sweating, and confusion Pain and burning on urination

Muscle weakness and dizziness on standing Redness and tenderness of the perineum Sensations of hunger, tremors, sweating, and confusion Pain and burning on urination Drugs from the lower blood glucose levels by preventing kidney reabsorption of glucose and sodium that was filtered from the blood into the urine. This filtered glucose is excreted in the urine rather than moved back into the blood. Hypoglycemia (symptoms of hunger, tremors, sweating, confusion) is possible as is dehydration with excessive sodium loss (muscle weakness and orthostatic hypotension with dizziness on standing). The excess glucose in the urine increases the risk for urinary tract infections with pain and burning on urination. These drugs increase the risk for Fournier gangrene with perineal fasciitis, which has early symptoms of redness and tenderness of the perineal skin.The drug is taken orally and not by injection. It is not associated with heart failure that may manifest with symptoms of rapid weight gain and shortness of breath.

A nurse is caring for a client who received benzocaine spray prior to a recent bronchoscopy. The client presents with continuous cyanosis even with oxygen therapy. Which action should the nurse take next? a. Administer an albuterol treatment. b. Notify the Rapid Response Team. c. Assess the clients peripheral pulses. d. Obtain blood and sputum cultures.

Notify the Rapid Response Team. Cyanosis unresponsive to oxygen therapy is a manifestation of methemoglobinemia, which is an adverse effect of benzocaine spray. Death can occur if the level of methemoglobin rises and cyanosis occurs. The nurse should notify the Rapid Response Team to provide advanced nursing care. An albuterol treatment would not address the clients oxygenation problem. Assessment of pulses and cultures will not provide data necessary to treat this client.

14. A nurse maintains an uncrossed arm and leg posture when communicating with a client. This nonverbal behavior is reflective of which letter of the SOLER acronym for active listening? 1. S 2. O 3. L 4. E 5. R

O Rationale: The nurse should identify that maintaining an uncrossed arm and leg posture is nonverbal behavior that reflects the O in the active-listening acronym SOLER. The acronym SOLER includes sitting squarely facing the client (S), observing and open posture (O), leaning forward toward the client (L), establishing eye contact (E), and relaxing (R).

Define the "L" in the RIFLE classification system.

OUTCOME LOSS STAGE = Persistent acute kidney injury (AKI) requiring renal replacement therapy for >4 wks

The nurse suspects that which client is at highest risk for developing gallstones? A. Obese male with a history of chronic obstructive pulmonary disease B. Obese female on hormone replacement therapy C. Thin male with a history of coronary artery bypass grafting D. Thin female who has recently given birth

Obese female on hormone replacement therapy Both obesity and altered hormone levels increase a woman's risk for developing gallstones. Men are at lower risk than women for developing gallstones. Although pregnancy increases the risk for a woman to develop gallstones, this woman's thin frame lessens that risk.

A client with acute cholecystitis is admitted to the medical-surgical unit. Which nursing activity associated with the client's care will be best for the nurse to delegate to unlicensed assistive personnel (UAP)? A. Assessing dietary risk factors for cholecystitis B. Checking for bowel sounds and distention C. Determining precipitating factors for abdominal pain D. Obtaining the admission weight, height, and vital signs

Obtaining the admission weight, height, and vital signs Obtaining height, weight, and vital signs is included in the education for UAP and usually is included in the job description for these staff members. Assessment, checking bowel sounds, and determining precipitating factors for abdominal pain require broader education and are within the scope of practice of licensed nursing staff.

A nurse assesses a clients respiratory status. Which information is of highest priority for the nurse to obtain? a. average daily checks b. neck circumference c. height and weight d. occupation and hobbies

Occupation and hobbies. Many respiratory problems occur as a result of chronic exposure to inhalation irritants used in a clients occupation and hobbies. Although it will be important for the nurse to assess in the fluid intake, height and weight, these will not be as important as determining his occupation and hobbies. Determining the clients neck circumference will not be an important part of a respiratory assessment.

Which action is appropriate for the nurse to delegate to the assistive personnel (AP) when caring for clients with diabetes? Monitoring a client who reports palpitations and anxiety Verifying the infusion rate on a continuous infusion insulin pump Performing a blood glucose check on a client who requires insulin Assessing a client who reports tremors and irritability

Performing a blood glucose check on a client who requires insulin Performing bedside glucose monitoring is a task that may be delegated to an AP who has been educated in this technique because it does not require extensive clinical judgment to perform. There is no evidence the client is unstable at this time. The nurse will follow up with the results and insulin administration after assessing the less stable clients.Intravenous therapy and medication administration are not within the scope of practice for AP. The client with tremors and irritability is displaying symptoms of hypoglycemia requiring further assessment and intervention that are not within the scope of practice for AP. The client reporting palpitations and anxiety may have hypoglycemia, requiring further intervention. This client must be assessed by licensed nursing staff.

Which assessment is a priority for the nurse to make when a client with diabetic ketoacidosis (DKA) who is being monitored while receiving an insulin infusion begins to show an irregular heart beat with inverted T-waves? Rate of IV infusion Urine output Potassium level Breath sounds

Potassium level After DKA therapy starts, serum potassium levels drop quickly. An ECG showing an irregular pattern and inverted T-waves is most likely related to low potassium levels (hyperkalemia). Hypokalemia is a common cause of death in the treatment of DKA. Detecting and treating the underlying cause of the cardiac irregularities is essential. The cardiac issues are not associated with changes in urine output even though hyperglycemia will cause osmotic diuresis. The client with DKA is not at risk for hypoventilation or poor gas exchange. Increased fluids treat the symptoms of dehydration secondary to DKA, but do not treat the hypokalemia.

A client with an intestinal obstruction has pain that changes from a "colicky" intermittent type to constant discomfort. What does the nurse do first? A. Administers medication for pain B. Changes the nasogastric suction level from "intermittent" to "constant" C. Positions the client in high-Fowler's position D. Prepares the client for emergency surgery

Prepares the client for emergency surgery The change in pain type could be indicative of perforation or peritonitis and will require immediate surgical intervention. Pain medication may mask the client's symptoms but will not address the root cause. A change in the nasogastric suction rate will not resolve the cause of the client's pain and could be particularly ineffective if a nonvented tube is in use. A high-Fowler's position will have no effect on an intestinal perforation or peritonitis, which this client is likely experiencing.

Which set of assessment findings indicates to the nurse that a client may have acute pancreatitis? A. Absence of jaundice, pain of gradual onset B. Absence of jaundice, pain in right abdominal quadrant C. Presence of jaundice, pain worsening when sitting up D. Presence of jaundice, pain worsening when lying supine

Presence of jaundice, pain worsening when lying supine Pain that worsens when lying supine and the presence of jaundice are the only assessment findings indicative of acute pancreatitis. Pain associated with acute pancreatitis usually has an abrupt onset, is located in the mid-epigastric or upper left quadrant, and lessens with sitting up; also, jaundice is present.

8. On which task should a nurse place priority during the working phase of relationship development? 1. Establishing a contract for intervention 2. Examining feelings about working with a particular client 3. Establishing a plan for continuing aftercare 4. Promoting the client's insight and perception of reality

Promoting the client's insight and perception of reality Rationale: The nurse should place priority on promoting the client's insight and perception of reality during the working phase of relationship development. Establishing a contract for intervention would occur in the orientation phase. Examining feelings about working with a client should occur in the pre-interaction phase. Establishing a plan for aftercare would occur in the termination phase.

A client has been placed on enzyme replacement for treatment of chronic pancreatitis. In teaching the client about this therapy, the nurse advises the client not to mix enzyme preparations with foods containing which element? A. Carbohydrates B. High fat C. High fiber D. Protein

Protein Enzyme preparations should not be mixed with foods containing protein because the enzymes will dissolve the food into a watery substance. No evidence suggests that enzyme preparations should not be mixed with carbohydrates, food with high fat content, and food with high fiber content.

Which assessment finding in a client with diabetes mellitus indicates to the nurse that the disease is damaging the kidneys? White blood cells (WBCs) in the urine during a random urinalysis Ketone bodies in the urine during acidosis Glucose in the urine during hyperglycemia Protein in the urine during a random urinalysis

Protein in the urine during a random urinalysis Urine should not contain protein and the presence of proteinuria in a client with marks the beginning of renal problems known as diabetic nephropathy, that progresses eventually to end-stage renal disease. Chronically elevated blood glucose levels cause renal hypertension and excess kidney perfusion with leakage from the renal vasculature. The excess leakiness allows larger substances, such as proteins, to be filtered into the urine.

Define the "R" in the RIFLE classification system.

RISK STAGE = Serum Creatinine increased by 1.5 or GFR decrease > 25% (or UO < 0.5mL/kg/hr X 6hrs)

After an abdominoperineal resection, a 75-year-old client is referred to a home health agency. Which staff member does the nurse manager assign to perform the initial assessment on this client? A. LPN/LVN who has worked with many home health clients after colostomy surgeries B. LPN/LVN with 20 years of experience in the home health agency C. RN who is new to the agency with 5 years experience in the emergency department D. Social worker who is experienced with case management of older clients

RN who is new to the agency with 5 years experience in the emergency department Clients with medical or surgical diagnoses have complex physiologic needs that should be assessed by an RN. For this reason, Medicare requires that the initial assessment must be done by an RN, although LPN/LVNs and social workers are likely to be part of the health care team.

A 67-year-old male client reports pain in the inguinal area that occurs when he coughs. A bulge that can be pushed back into the abdomen is found in his inguinal area. What type of hernia does he have? A. Femoral B. Reducible C. Strangulated D. Ventral

Reducible The hernia is reducible because its contents can be pushed back into the abdominal cavity. Femoral hernias tend to occur more frequently in obese and pregnant women. A hernia is considered to be strangulated when the blood supply to the herniated segment of the bowel is cut off. It cannot be a ventral hernia because it would have to occur at the site of a previous surgical incision.

The nurse is educating the client with COPD who requires home oxygen therapy for discharge. Which of these teaching points takes the highest priority? Correct performance when setting up the oxygen delivery system Removing combustion hazards present in the home Understanding the signs and symptoms of hypoxemia Demonstrating how to use a pulse oximetry device

Removing combustion hazards present in the home The highest priority of education is that oxygen is highly combustible. The nurse must ensure that no open flames or combustion hazards are present in a room where oxygen is in use.The oxygen delivery system in the home will be different than in the hospital. Therefore, this skill may be verified by the visiting nurse or company providing the oxygen. The client must be able to state signs and symptoms of hypoxemia, although safety is the priority. Pulse oximetry may be useful for monitoring the client's oxygenation status and the visiting nurse or respiratory therapy partner can assess this. The client needs to be able to state the signs and symptoms of hypoxemia and when to notify the health care provider.

9. Which therapeutic communication technique is being used in the following nurse-client interaction? Client: "My father spanked me often." Nurse: "Your father was a harsh disciplinarian." 1. Restatement 2. Offering general leads 3. Focusing 4. Accepting

Restatement Rationale: The nurse is using the therapeutic communication technique of restatement. Restatement involves repeating the main idea of what the client has said. It allows the client to know whether the statement has been understood and provides an opportunity to continue.

Nursing process assessment for suicidal ideas or acts

Seriousness of intent, plan, means of the plan, verbal and behavioral cues

The nurse is attempting to position a client having an acute attack of pancreatitis in the most comfortable position possible. In which position does the nurse place this client? A. Supine, with a pillow supporting the abdomen B. Up in a chair between frequent periods of ambulation C. High-Fowler's position, with pillows used as needed D. Side-lying position, with knees drawn up to the chest

Side-lying position, with knees drawn up to the chest The side-lying position with the knees drawn up has been found to relieve abdominal discomfort related to acute pancreatitis. No evidence suggests that supine position, sitting up in a chair, or high-Fowler's position have any effect on abdominal discomfort related to acute pancreatitis.

After an automobile crash, a client is admitted to the emergency department with possible abdominal trauma. Which health care provider request does the nurse implement first? A. Insert a nasogastric tube and connect it to intermittent suction. B. Obtain a complete blood count and coagulation panel. C. Start an IV line and infuse normal saline at 200 mL/hr. D. Arrange for a computed tomography (CT) scan of the abdomen.

Start an IV line and infuse normal saline at 200 mL/hr. After the initial airway, breathing, and circulation assessment is completed, the most immediate concerns are the high risks for hemorrhage and shock. To rapidly treat for these possible complications, IV access and infusion of fluids are necessary as the priority intervention. Inserting a nasogastric tube, laboratory studies, and arranging a CT scan are secondary to establishing IV access and instilling fluids.

A client asks the nurse, "Can you tell me some foods to include in my diet so that I can reduce my chances of getting colorectal cancer?" Which dietary selection does the nurse suggest? A. Steak with pasta B. Spaghetti with tomato sauce C. Steamed broccoli with turkey D. Tuna salad with wheat crackers

Steamed broccoli with turkey Steamed broccoli with turkey contains low-fat meat and no refined carbohydrates. Animal fat from red meats is carcinogenic, and pasta is high in refined carbohydrates, which are known to contribute to colon cancer. Spaghetti and wheat crackers also contain large amounts of refined carbohydrates.

A 21-year-old with a stab wound to the abdomen has come to the emergency department. Once stabilized, the client is admitted to the medical-surgical unit. What does the admitting nurse do first for this client? A. Administer pain medication. B. Assess skin temperature and color. C. Check on the amount of urine output. D. Take vital signs.

Take vital signs. Assessment of vital signs should be done first to determine the adequacy of the airway and circulation. Vital signs initially reveal the most about the client's condition. The client should not be medicated for pain until his or her alertness level is determined. Skin temperature and color are not specifically indicative of the client's overall condition. If the client is in shock, urine output will be scant and will not be an accurate assessment variable.

Which action will the nurse recommend to a client with type 1 diabetes on insulin therapy who has been having a morning fasting blood glucose (FBG) level of 160 mg/dL (8.9 mmol/L) and is diagnosed with "dawn phenomenon" to achieve better control? Eat a bedtime snack containing equal amounts of protein and carbohydrates." Avoid eating any carbohydrate with your evening meal." Take your evening insulin dose right before going to bed instead of at supper time." Inject the insulin into your arm rather than into the abdomen around the navel."

Take your evening insulin dose right before going to bed instead of at supper time." A client with "dawn phenomenon," diagnosed by checking blood glucose levels during the night, has morning hyperglycemia that results from a nighttime release of adrenal hormones causing blood glucose elevations at about 5 to 6 a.m. It is managed by providing more insulin for the overnight period (e.g., giving the evening dose of intermediate-acting insulin at 10 p.m. instead of with the evening meal).Bedtime snacks are needed for "Somogyi phenomenon" that is morning hyperglycemia caused by the counterregulatory response to nighttime hypoglycemia. Changing the injection site would not prevent morning hyperglycemia. Not eating any carbohydrate with a meal is more likely to cause severe hypoglycemia during the night and is dangerous.

Suicide

The act of taking one's own life It is not a diagnosis or a disorder, it is a behavior "Irrational" behavior

4. Which client action should a nurse expect during the working phase of the nurse-client relationship? 1. The client gains insight and incorporates alternative behaviors. 2. The client establishes rapport with the nurse and mutually develops treatment goals. 3. The client explores feelings related to reentering the community. 4. The client explores personal strengths and weaknesses that impact behavioral choices.

The client gains insight and incorporates alternative behaviors. Rationale: The nurse should expect that that the client will gain insight and incorporate alternative behaviors during the working phase of the nurse-client relationship. The client may also overcome resistance, problem-solve, and continually evaluate progress toward goals.

12. A nurse says to a client, "Things will look better tomorrow after a good night's sleep." This is an example of which communication technique? 1. The therapeutic technique of giving advice 2. The therapeutic technique of defending 3. The nontherapeutic technique of presenting reality 4. The nontherapeutic technique of giving reassurance

The nontherapeutic technique of giving reassurance Rationale: The nurse's statement, "Things will look better tomorrow after a good night's sleep," is an example of the nontherapeutic communication technique of giving reassurance. Giving reassurance indicates to the client that there is no cause for anxiety, thereby devaluing the client's feelings.

A nurse assesses a client after a thoracentesis. Which assessment finding warrants immediate action? a. The client rates pain as a 5/10 at the site of the procedure. b. A small amount of drainage from the site is noted. c. Pulse oximetry is 93% on 2 liters of oxygen. d. The trachea is deviated toward the opposite side of the neck.

The trachea is deviated toward the opposite side of the neck. A deviated trachea is a manifestation of a tension pneumothorax, which is a medical emergency. The other findings are normal or near normal.

How will the nurse evaluate the level of glycemic control for a client with diabetes whose laboratory values include a fasting blood glucose level of 82 mg/dL (mmol/L) and an A1c of 5.9%? The values indicate that the client has poorly managed his or her disease. The values indicate that the client has managed his or her disease well. The client's glucose control for the past 24 hours has been good but the overall control is poor. The client's glucose control for the past 24 hours has been poor but the overall control is good.

The values indicate that the client has managed his or her disease well. Fasting blood glucose levels provide an indication of the client's adherence to drug and nutrition therapy for DM has been for the previous 24 hours. This client's FBG is well within the normal range.A1c provides an indication of general blood glucose control for the past several months because when glucose attaches to hemoglobin, the attachment is permanent for as long as those hemoglobin molecules are present within red blood cells. Normal red blood cell life span is about 120 days. This client's A1c level is within the desirable range, indicating good long-term glucose control as well as short-term control.

Predisposing factors: Durkheim

Three social categories: egoistic, altruistic, and anomic

A client has just been admitted to the intensive care unit after having a left lower lobectomy via video-assisted thorascopic surgery. Which of these prescriptions will the nurse implement first? Titrate oxygen flow rate to keep O2 saturation at or greater than 93%. Administer 2 g of cephazolin IV now. Give morphine sulfate 4 to 6 mg IV for pain. Transfuse 1 unit of packed red blood cells (PRBCs) over 2 hours.

Titrate oxygen flow rate to keep O2 saturation at or greater than 93%. Airway and oxygenation are main priorities in the immediate postoperative period. The client will likely be intubated, so coordination of care with respiratory therapy will be important.Although antibiotic therapy may be ordered, this is not a priority at this time. Pain management in the postoperative period is important, but is secondary to airway, breathing, and circulation. PRBCs to maintain the oxygen-carrying capacity of the blood will be performed after oxygenation. Pain medication and antibiotic administration will be performed last.

16. What is a nurse's purpose for providing appropriate feedback? 1. To give the client good advice 2. To advise the client on appropriate behaviors 3. To evaluate the client's behavior 4. To give the client critical information

To give the client critical information Rationale: The purpose of providing appropriate feedback is to give the client critical information. Feedback should not be used to give advice or evaluate behaviors.

The nurse is teaching a client with gallbladder disease about diet modification. Which meal does the nurse suggest to the client? A. Steak and French fries B. Fried chicken and mashed potatoes C. Turkey sandwich on wheat bread D. Sausage and scrambled eggs

Turkey sandwich on wheat bread Turkey is an appropriate low-fat selection for this client. Steak, French fries, fried chicken, and sausage are too fatty, and eggs are too high in cholesterol for a client with gallbladder disease.

A client is being evaluated in the emergency department for a possible small bowel obstruction. Which signs and/or symptoms does the nurse expect to assess? A. Cramping intermittently, metabolic acidosis, and minimal vomiting B. Intermittent lower abdominal cramping, obstipation, and metabolic alkalosis C. Metabolic acidosis, upper abdominal distention, and intermittent cramping D. Upper abdominal distention, metabolic alkalosis, and great amount of vomiting

Upper abdominal distention, metabolic alkalosis, and great amount of vomiting A small bowel obstruction is characterized by upper or epigastric abdominal distention, metabolic alkalosis, and a great amount of vomiting. Intermittent lower abdominal cramping, metabolic acidosis, and minimal vomiting are all symptoms of a large bowel obstruction.

A client with COPD calls the pulmonary clinic reporting the last 24 hours the peak flow meter readings have been in the yellow range. Which of these interventions by the nurse is appropriate at this time? Use your prescription for rescue medication and retest yourself. This is a satisfactory reading, continue your present regimen. Go to the nearest emergency department. Increase your controller medication dose.

Use your prescription for rescue medication and retest yourself. The nurse would tell the client to use the rescue medication and then retest. This instruction by the nurse is appropriate. Reliever drugs (also called "rescue" drugs) are used to stop an attack once it has started or when the peak flow meter is in the yellow range or 50%-80% of personal best range.The reading is not satisfactory. Frequent readings in the yellow zone indicate the need to reassess the asthma plan and the need to possibly change controller drugs. Satisfactory readings are in the green zone and are at least 80% of or better than the personal best readings. The client needs to seek care in the ED when the readings are in the red zone or below 50% of the personal best reading. Nurses do not prescribe medications or change dosing.

A nurse is caring for a client who is scheduled to undergo a thoracentesis. Which intervention should the nurse complete prior to the procedure? a. Measure oxygen saturation before and after a 12-minute walk. b. Verify that the client understands all possible complications. c. Explain the procedure in detail to the client and the family. d. Validate that informed consent has been given by the client.

Validate that informed consent has been given by the client. A thoracentesis is an invasive procedure with many potentially serious complications. Verifying that the client understands complications and explaining the procedure to be performed will be done by the physician or nurse practitioner, not the nurse. Measurement of oxygen saturation before and after a 12-minute walk is not a procedure unique to a thoracentesis.

A nurse assesses a client who is prescribed varenicline (Chantix) for smoking cessation. Which manifestations should the nurse identify as adverse effects of this medication? (Select all that apply.) a. Visual hallucinations b. Tachycardia c. Decreased cravings d. Impaired judgment e. Increased thirst

Visual hallucinations & Impaired judgment. Varenicline (Chantix) has a black box warning stating that the drug can cause manic behavior and hallucinations. The nurse should assess for changes in behavior and thought processes, including impaired judgment and visual hallucinations. Tachycardia and increased thirst are not adverse effects of this medication. Decreased cravings is a therapeutic response to this medication.

2. What is the name of the substance secreted by the endocrine glands? a. Vasoactive amines b. Chemotaxins c. Hormones d. Cytotoxins

c Hormones

While obtaining a clients health history, the client states, I am allergic to avocados. Which responses by the nurse are best? (Select all that apply.) a. What response do you have when you eat avocados? b. I will remove any avocados that are on your lunch tray. c. When was the last time you ate foods containing avocados? d. I will document this in your record so all of your providers will know. e. Have you ever been treated for this allergic reaction?

What response do you have when you eat avocados; I will document this in your record so all of your providers will know; Have you ever been treated for this allergic reaction? Nurses should assess clients who have allergies for the specific cause, treatment, and response to treatment. The nurse should also document the allergies in a prominent place in the clients medical record. The nurse should collaborate with food services to ensure no avocados are placed on the clients meal trays. Asking about the last time the client ate avocados does not provide any pertinent information for the clients plan of care.

A client newly diagnosed with type 1 diabetes says she is not ready to learn everything about diabetes control right now. Which information has the greatest priority for the nurse to teach this client and her family for now to prevent harm? (Select all that apply.) Select all that apply. Causes of type 1 diabetes What to do when ill? Symptoms and treatment of hypoglycemia Insulin administration Dietary control of blood glucose Importance of regular exercise

What to do when ill? Symptoms and treatment of hypoglycemia Insulin administration The priority information for safety and preventing harm that the nurse needs to teach the client and family about diabetes are:Symptoms and management of hypoglycemia because it is a life-threatening condition.Proper insulin administration is essential for the management of type 1 diabetes and to prevent death.Knowing what to do when ill is critical information because illness will require changes in the client's day-to-day use of insulin and may need contact with the client's diabetes health care provider to prevent harm.The causes of diabetes, dietary control, and exercise are less important for immediate safety and can be taught at another time.

A nurse is providing care after auscultating clients breath sounds. Which assessment finding is correctly matched to the nurses primary intervention? a. Hollow sounds are heard over the trachea. The nurse increases the oxygen flow rate. b. Crackles are heard in bases. The nurse encourages the client to cough forcefully. c. Wheezes are heard in central areas. The nurse administers an inhaled bronchodilator. d. Vesicular sounds are heard over the periphery. The nurse has the client breathe deeply.

Wheezes are heard in central areas. The nurse administers an inhaled bronchodilator. Wheezes are indicative of narrowed airways, and bronchodilators help to open the air passages. Hollow sounds are typically heard over the trachea, and no intervention is necessary. If crackles are heard, the client may need a diuretic. Crackles represent a deep interstitial process, and coughing forcefully will not help the client expectorate secretions. Vesicular sounds heard in the periphery are normal and require no intervention.

Which renal change associated with aging does the nurse expect an older adult patient to report a. Nocturanl polyuria b. Micturition c. Hematuria d. Dysuria

a

The nurse is preparing to instruct a client with chronic pancreatitis who is to begin taking pancrelipase (Cotazym). Which instruction does the nurse include when teaching the client about this medication? A. Administer pancrelipase before taking an antacid. B. Chew tablets before swallowing. C. Take pancrelipase before meals. D. Wipe your lips after taking pancrelipase.

Wipe your lips after taking pancrelipase. Pancrelipase is a pancreatic enzyme used for enzyme replacement for clients with chronic pancreatitis. To avoid skin irritation and breakdown from residual enzymes, the lips should be wiped. Pancrelipase should be administered after antacids or histamine2 blockers are taken. It should not be chewed to minimize oral irritation and allow the drug to be released more slowly. It should be taken with meals and snacks and followed with a glass of water.

After receiving change-of-shift report on these clients, which client does the nurse plan to assess first? A. Young adult client with acute pancreatitis who is dyspneic and has a respiratory rate of 34 to 38 breaths/min B. Adult client admitted with cholecystitis who is experiencing severe right upper quadrant abdominal pain C. Middle-aged client who has an elevated temperature after undergoing endoscopic retrograde cholangiopancreatography D. Older adult client who is receiving total parenteral nutrition after a Whipple procedure and has a glucose level of 235 mg/dL

Young adult client with acute pancreatitis who is dyspneic and has a respiratory rate of 34 to 38 breaths/min Acute respiratory distress syndrome is a possible complication of acute pancreatitis. The dyspneic client is at greatest risk for rapid deterioration and requires immediate assessment and intervention. The client with cholecystitis and the client with an elevated temperature will require further assessment and intervention, but these are not medical emergencies requiring the nurse's immediate attention. The older adult client's glucose level will require intervention but, again, is not a medical emergency.

What is the normal Urine Glucose lab value?

ZERO

What is the normal lab value for Urine Nitrates?

ZERO

A nurse assesses a client who presents with renal calculi. Which question would the nurse ask? "Do any of your family members have this problem?" "Do you drink any cranberry juice?" "Do you urinate after sexual intercourse?" "Do you experience burning with urination?"

a

A nurse evaluates a client with acute glomerulonephritis (GN). Which assessment finding would the nurse recognize as a positive response to the prescribed treatment? The client lost 11 lb (5 kg) in the past 10 days. The client's urine specific gravity is 1.048. No blood is observed in the client's urine. The client's blood pressure is 152/88 mm Hg.

a

A nurse obtains the health history of a client with a suspected diagnosis of bladder cancer. Which question would the nurse ask when determining this client's risk factors? "Do you smoke cigarettes?" "Do you use any alcohol?" "Do you use recreational drugs?" "Do you take any prescription drugs?"

a

A nurse teaches a young female client who is prescribed cephalexin for a urinary tract infection. Which statement would the nurse include in this client's teaching? "Use a second form of birth control while on this medication." "You will experience increased menstrual bleeding while on this drug." "You may experience an irregular heartbeat while on this drug." "Watch for blood in your urine while taking this medication."

a

A patient had a cystoscopy. After the procedure, what does the nurse expect to see in this patient? a. Pink-tinged urine b. Blood urine c. Very dilute urine d. Decreased urine output

a

A patient is scheduled for a CT with iodinated contrast medium. Which medication is discontinued 24 hours before the procedure and for at least 48 hours until kidney function has been reevaluated? a. Glucophage (Metformin) b. Morphine (MS Contin) c. Furosemide (Lasix) d. Oral acetylcysteine (Mucomyst)

a

Which patient is most likely to produce urine with a specific gravity of less than 1.005? a. Takes diuretic medication everyday b. Has dehydration secondary to vomiting c. Is hypovolemic due to blood loss d. Has syndrome of inappropriate antidiuretic horome

a

After teaching a client who has stress incontinence, the nurse assesses the client's understanding. Which statement made by the client indicates a need for further teaching? "I will limit my total intake of fluids." "I must avoid drinking alcoholic beverages." "I must avoid drinking caffeinated beverages." "I shall try to lose about 10% of my body weight."

a

After teaching a client with a history of renal calculi, the nurse assesses the client's understanding. Which statement made by the client indicates a correct understanding of the teaching? "I should drink at least 3 L of fluid every day." "I will eliminate all dairy or sources of calcium from my diet." "Aspirin and aspirin-containing products can lead to stones." "The doctor can give me antibiotics at the first sign of a stone."

a

Impairment in the thirst mechanisms associated with aging makes an older adult patient more vulnerable to which disorder? a. Hypernatremia b. Hypocalcemia c. Hyperkalemia d. Hypoglycemia

a

Limiting fluid intake would have what effect on urine? a. Increases the concentration of urine b. Makes the urine less irritating c. Decreases the risk for urine infection d. decreases the pH of urine

a

Mastering voluntary micturtition is a normal developmental task for which person? a. a healthy 20-month-old toddler b. a 56-year-old women with stress incontinence c. a healthy 8-year-old child d. a 25-year-old with a spinal cord injury

a

The nurse hears in report that the patient is having renal colic pain. Whne performing the physical assessment of this patient during a severe pain episode, what additional sign/symptoms may the nurse expect to observe? a. Diaphoresis b. Redness over the flank c. Jaundice d. Bruit in the renal artery

a

The nurse is assessing a patient for bladder distention. What technique does the nurse use? a. Gently palpate for the outline of the bladder, percuss the lower abdomen, continue toward the umbilicus until dull sounds are no longer produced b. gently palpate for the outline of the bladder, auscultate for sounds in the lower abdomen c. Place one hand under the back and palpate with the other hand over the bladder, percuss the lower abdomen until tympanic sounds are no longer produced. d. Use the hand to depress the bladder as the patient takes a deep breath, then percuss

a

The nurse is reviewing the results of a patient ultrasound of the kidney. The report reveals an enlarged kidney which suggests which possible problem? a. Polycystic kidney b. Kdienyinfection c. Renal carcinoma d. Chronic kidney disease

a

The nurse performs a dipstick urine test for a patient being evaluated for kidney problems. Glucose is present in the urine. How does the nurse interpret this result? a. Blood glucose level is greater than 220 mg/dL b. The kidneys are failing to filter any glucose c. The patient is at risk for hypoglycemia d. The renal threshold has not been exceeded

a

What does the BUN test measure? a. Kidney excretion of urea nitrogen b. Urine osmolality c. Creatinine clearance d. Urine utput

a

Which patient is most likely to have a decreased calcium level? a. Patients with kidney disease b. Patients with cystitis c. Patients with a Foley catheter d. Patients with urinary retention

a

Which urine characteristic listed on a urinalysis report arouses the nurses suspicion of a problem in the urinary tract? a. Cloudiness b. Straw color c. Ammonia odor d. One cast per high-powered field

a

an elderly patient has been in bed for several days after a fall. The nurse encourages ambulation to stimulate the movemnt of urine through the ureter by what phenomenon? a. Peristalsis b. Gravity c. Pelvic pressure d. Back flow

a

the nurse is talking to a group of older women about changes in the urinary system related to aging. what symptoms is likely to be the common concern for this group? a. Incontinence b. Hematuria c. Retention d. Dysuria

a

9. The binding of a hormone to a specific receptor site is an example of which endocrine process? a. "Lock and key" manner b. Negative feedback mechanism c. Neuroendocrine regulation d. "Fight-or-flight" response

a "Lock and key" manner

11. Which hormone is directly suppressed when circulating levels of cortisol are above normal? a. Corticotropin-releasing hormone (CRH) b. ADH c. Adrenocorticotropic hormone (ACTH) d. Growth hormone-releasing hormone (GH-RH)

a Corticotropin-releasing hormone (CRH)

34. In the older adult female, which physiologic changes occur as a result of decreased function of the ovaries? a. Decreased bone density, decreased production of estrogen. b. Decreased sensitivity of peripheral tissues to the effects of insulin c. Decreased urine-concentrating ability of the kidneys d. Decreased metabolic rate

a Decreased bone density, decreased production of estrogen.

35. An older adult reports a lack of energy and not being able to do the usual daily activities without several naps during the day. Which problem may these symptoms indicate that is often seen in the older adult? a. Hypothyroidism b. Hyperparathyroidism c. Overproduction of cortisol d. Underproduction of glucagon

a Hypothyroidism

22. Which hormone responds to a low serum calcium blood level by increasing bone resorption? a. Parathyroid hormone (PTH) b. T4 c. T3 d. Calcitonin

a Parathyroid hormone (PTH)

14. The anterior pituitary gland secretes tropic hormones in response to which hormones from the hypothalamus? a. Releasing hormones b. Target tissue hormones c. Growth hormones d. Demand hormones

a Releasing hormones

According to the dsm-v, which statement made by an adult client supports the criteria for generalized anxiety disorder? (SATA) a. i've been really anxious for at least 2 years now b. my anxiety had to be genetic; my mom was a terrible worrier too c. my marriage is in trouble b/c i'm always so irritable d. i've had a good physical and my hcp says i'm in good health e. it's hard falling asleep and even harder staying asleep - i'm restless all night

a - i've been really anxious for at least 2 years now c - my marriage is in trouble b/c i'm always so irritable d - i've had a good physical and my hcp says i'm in good health e - it's hard falling asleep and even harder staying asleep; i'm restless at night

which statement best describes the diagnostic and statistical manual fifth edition (DSM-5)? a. it is a medical psychiatric assessment system b. it is a compendium of treatment modalities c. it offers a complete list of nursing diagnoses d. it suggests common interventions for mental disorders

a - it is amedical psychiatric assessment system

Which diagnostic test incorpartes contrast dye, but does not place a patient at risk for nephrotoxicity? a. renal scan b. Renal angiogrpahy c. Voiding cystourethrogram d. Computed tomography

c

Which of the following interventions are appropriate for a patient on suicide precautions? (Select all.) a. Remove all sharp objects, belts, and other potentially dangerous articles from the patient's environment b. Accompany the patient to off-unit activities c. Reassess intensity of suicidal thoughts and urges on a regular basis d. Put all of the patient's possessions in storage an explain to her that she may have them back when she is off suicide precautions

a, b, and c

A nurse teaches a client about self-management after experiencing a urinary calculus treated by lithotripsy. Which statements would the nurse include in this client's discharge teaching? (Select all that apply.) "Finish the prescribed antibiotic even if you are feeling better." "Drink at least 3 L of fluid each day." "The bruising on your back may take several weeks to resolve." "Report any blood present in your urine." "It is normal to experience pain and difficulty urinating."

a, b, c

38. Which are diagnostic methods to measure patient hormone levels? (Select all that apply.) a. Stimulation testing b. Suppression testing c. 24-hour urine testing d. Chromatographic assay e. Needle biopsy

a, b, c, d Stimulation testing, Suppression testing, 24-hour urine testing, Chromatographic assay

20. Which assessment findings does the nurse monitor in response to catecholamines released by the adrenal medulla? (Select all that apply.) a. Increased heart rate related to vasoconstriction b. Increased blood pressure related to vasoconstriction c. Increased perspiration d. Constriction of pupils e. Increased blood glucose in response to glycogenolysis

a, b, c, e Increased heart rate related to vasoconstriction. Increased blood pressure related to vasoconstriction. Increased perspiration. Increased blood glucose in response to glycogenolysis.

24. Which statements about T3 and T4 hormones are correct? (Select all that apply.) a. The basal metabolic rate is affected. b. Hypothalamus is stimulated by cold and stress to secret thyrotropin-releasing hormone (TRH). c. These hormones need intake of protein and iodine for production. d. Circulating hormone in the blood directly affects the production of TSH. e. T3 and T4 increase oxygen use in tissue.

a, b, c, e The basal metabolic rate is affected. Hypothalamus is stimulated by cold and stress to secret thyrotropin-releasing hormone (TRH). These hormones need intake of protein and iodine for production. T3 and T4 increase oxygen use in tissue.

The nurse is caring for a client who is diagnosed with urinary tract infection (UTI). What common urinary signs and symptoms does the nurse expect? (Select all that apply.) a. Dysuria b. Frequency c. Burning d. Fever e. Chills f. Hematuria

a, b, c, f

A nurse assesses a client who has had two episodes of bacterial cystitis in the last 6 months. Which question(s) would the nurse ask? (Select all that apply.) "How much water do you drink every day?" "Do you take estrogen replacement therapy?" "Does anyone in your family have a history of cystitis?" "Are you on steroids or other immune-suppressing drugs?" "Do you drink grapefruit juice or orange juice daily?"

a, b, d

A nurse assesses a client with nephrotic syndrome. Which assessment findings would the nurse expect? (Select all that apply.) Proteinuria Hypoalbuminemia Dehydration Lipiduria Dysuria Costovertebral angle (CVA) tenderness

a, b, d

A nurse cares for clients with urinary incontinence. Which types of incontinence are correctly paired with their description? (Select all that apply.) Stress incontinence—urine loss with physical exertion Urge incontinence—loss of urine upon feeling the need to void Functional incontinence—urine loss results from abnormal detrusor contractions Overflow incontinence—constant dribbling of urine Reflex incontinence—leakage of urine without lower urinary tract disorder

a, b, d

A nurse teaches a client with polycystic kidney disease (PKD). Which statements would the nurse include in this client's discharge teaching? (Select all that apply.) "Take your blood pressure every morning." "Weigh yourself at the same time each day." "Adjust your diet to prevent diarrhea." "Contact your provider if you have visual disturbances." "Assess your urine for renal stones."

a, b, d

41. Which are the types of radiographic tests that may be used for an endocrine assessment? (Select all that apply.) a. Ultrasonography b. Skull x-ray c. Chest x-ray d. Magnetic resonance imaging (MRI) e. Computed tomography (CT)

a, b, d, e Ultrasonography, Skull x-ray, Magnetic resonance imaging (MRI), Computed tomography (CT)

13. Which statements about the pituitary glands are correct? (Select all that apply.) a. The main role of the anterior pituitary is to secrete tropic hormones. b. The posterior pituitary gland stores hormones produced by the hypothalamus. c. The anterior pituitary is connected to the thalamus gland. d. The anterior pituitary releases stored hormones produced by the hypothalamus. e. The anterior pituitary gland secretes gonadotropins.

a, b, e The main role of the anterior pituitary is to secrete tropic hormones. The posterior pituitary gland stores hormones produced by the hypothalamus. The anterior pituitary gland secretes gonadotropins.

8. Which statements about hormones and the endocrine system are accurate? (Select all that apply.) a. There are specific normal blood levels of each hormone. b. Hormones exert their effects on specific target tissues. c. Each hormone can bind with multiple receptor sites. d. The endocrine system works independently to regulate homeostasis. e. More than one hormone can be stimulated before the target tissue is affected.

a, b, e There are specific normal blood levels of each hormone. Hormones exert their effects on specific target tissues. More than one hormone can be stimulated before the target tissue is affected.

The nurse is reviewing the results of a client's urinalysis. The client has a diagnosis of acute glomerulonephritis. Which urine findings would the nurse expect? (Select all that apply.) Presence of protein Presence of red blood cells Presence of white blood cells Acidic urine Dilute urine

a, c, d

The nurse in the emergency department encounters a patient, Niko, who is expressing suicide ideation. The nurse recognizes that which of the following considerations are important to good suicide risk assessment? (Select all.) a. Collaborating with the patient b. Asking specific questions about leisure activities c. Establishing trust and open communication with the patient d. Asking the patient specific questions about the strength of intention to die e. Identifying whether the patient has thought about a plan for trying to kill himself

a, c, d, and e

1. Which glands are parts of the endocrine system? (Select all that apply.) a. Thyroid b. Occipital c. Parathyroid d. Adrenal e. Pituitary

a, c, d, e Thyroid, Parathyroid, Adrenal, Pituitary

A client asks the nurse why she has urinary incontinence. What risk factors would the nurse recall in preparing to respond to the client's question? (Select all that apply.) Diuretic therapy Anorexia nervosa Stroke Dementia Arthritis Parkinson disease

a, c, d, e, f

28. Which statements about insulin secretion are correct? (Select all that apply.) a. Insulin levels increase following the ingestion of a meal. b. Insulin is stimulated primarily by fat ingestion. c. Basal levels are secreted continuously. d. Insulin promotes glycogenolysis and gluconeogenesis. e. Carbohydrate intake is the main trigger for insulin secretion.

a, c, e Insulin levels increase following the ingestion of a meal. Basal levels are secreted continuously. Carbohydrate intake is the main trigger for insulin secretion.

The nurse is planning health teaching for a client starting mirabegron for urinary incontinence. What health teaching would the nurse include? (Select all that apply.) "Monitor blood tests carefully if you are prescribed warfarin." "Avoid crowds and individuals with infection." "Report any fever to your primary health care provider." "Take your blood pressure frequently at home." "Report palpitations or chest soreness that may occur."

a, d

5. Which hormones are secreted by the thyroid gland? (Select all that apply.) a. Calcitonin b. Somatostatin c. Glucagon d. Thyroxine (T4) e. Aldosterone f. Triiodothyronine (T3)

a, d, f Calcitonin, Thyroxine (T4), Triiodothyronine (T3)

After treating several young women for urinary tract infections (UTIs), the college nurse plans an educational offering on reducing the risk of getting a UTI. What information does the nurse include? (Select all that apply.) Void before and after each act of intercourse. Consider changing to spermicide from birth control pills. Do not douche or use scented feminine products. Wear loose-fitting nylon panties. Wipe or clean the perineum from front to back.

a, e

17. Which statements about the adrenal glands are correct? (Select all that apply.) a. The cortex secretes androgens in men and women. b. Catecholamines are secreted from the cortex. c. Glucocorticoids are secreted by the medulla. d. The medulla secretes hormones essential for life. e. The cortex secrets aldosterone that maintains extracellular fluid volume.

a, e The cortex secretes androgens in men and women. The cortex secrets aldosterone that maintains extracellular fluid volume.

A depressed patient says, "Nothing matters anymore." What is the most appropriate response by the nurse? a. "Are you having thoughts of suicide?" b. "I am not sure I understand what you are trying to say." c. "Try to stay hopeful. Things have a way of working out." d. "Tell me more about what interested you before you became depressed."

a. "Are you having thoughts of suicide?" The nurse must make overt what is covert; that is, the possibility of suicide must be openly addressed. The patient often feels relieved to be able to talk about suicidal ideation.

16. The nurse is teaching a client with chronic kidney disease (CKD) about the sodium restriction needed in the diet to prevent edema and hypertension. Which statement by the client indicates more teaching is needed? a. "I am thrilled that I can continue to eat fast food." b. "I will cut out bacon with my eggs every morning." c. "My cooking style will change by not adding salt." d. "I will probably lose weight by cutting out potato chips."

a. "I am thrilled that I can continue to eat fast food."

After instructing a client about the correct procedure for a 24-hour urine test, which client statement indicates to the nurse a need for further teaching? a. "I will not eat any fatty foods when I am collecting urine for this test." b. "To end the collection, I must empty my bladder and add this urine to the collection." c. "I need to keep the urine container cool in a separate refrigerator or cooler." d. "I won't save the first urine sample of the day."

a. "I will not eat any fatty foods when I am collecting urine for this test."

a 29-year old patient has been admitted following a suicide attempt. which nursing statement illustrates the concept of patient advocacy? a. "dr. raye, i noticed you ordered fluoxetine for this pt. during the admission interview, the client stated they will refuse the medication b/c of adverse effects they experienced previously." b. "dr. raye, during the admissions interview, the pt stated that there is a family hx of 3 other suicide attempts in the past." c. "i'd like you to tell me more about your depression and your suicide attempt." d. "i will take you on a tour of the unit and orient you to the rules so you can get adjusted to here."

a. "dr. raye, i noticed you ordered fluoxetine for this pt. during the admission interview, the client stated they will refuse the medication b/c of adverse effects they experienced previously."

A patient has undergone a kidney biopsy. what does the nurse monitor for in the patient related to this procedure? a. Nephrotoxicity b. Hemorrhage c. Urinary retention d. Hypertension

b

15. A 70-kg adult with chronic renal failure is on a 40-g protein diet. The client has a reduced glomerular filtration rate and is not undergoing dialysis. Which result would give the nurse the most concern? a. Albumin level of 2.5 g/dL b. Phosphorus level of 5 mg/dL c. Sodium level of 135 mmol/L d. Potassium level of 5.5 mmol/L

a. Albumin level of 2.5 g/dL

Which action immediately after a hypophysectomy will the nurse instruct a client to avoid to prevent harm? (Select all that apply.) a. Bending at the waist b. Talking c. Deep breathing d. Coughing e. Wearing makeup f. Using dental floss

a. Bending at the waist d. Coughing

18. A client with chronic kidney disease (CKD) is experiencing nausea, vomiting, visual changes, and anorexia. Which action by the nurse is best? a. Check the client's digoxin (Lanoxin) level. b. Administer an anti-nausea medication. c. Ask if the client is able to eat crackers. d. Get a referral to a gastrointestinal provider.

a. Check the client's digoxin (Lanoxin) level.

Which action in the plan of care for a client who is hospitalized for pituitary function testing would be most appropriate for the nurse to delegate to an experienced assistive personnel (AP)? a. Checking the client's blood glucose levels every 4 hours b. Monitoring the client's response to the IV insulin given during a stimulation test c. Teaching the client about a hormone suppression test d. Assessing the client for symptoms of hypopituitarism

a. Checking the client's blood glucose levels every 4 hours

For which change reported by a client taking bromocriptine therapy to manage hyperpituitarism will the nurse notify the primary health care provider immediately to prevent harm? a. Chest pain b. Constipation c. Headache d. Increased sleepiness

a. Chest pain

For which symptoms will the nurse instruct the family and client who is being treated for diabetes insipidus (DI) to call 911 or go to the nearest emergency department? (Select all that apply.) a. Decreased urine output b. Hypotension c. Weigh gain of more than 2.2 lb (1 kg) in 24 hours d. Persistent headache e. Hyperglycemia f. Acute confusion

a. Decreased urine output c. Weigh gain of more than 2.2 lb (1 kg) in 24 hours d. Persistent headache f. Acute confusion

What is the nurse's best action when noticing that the phlebotomist, who plans to draw blood from the client with severe hypercortisolism, displays symptoms of a cold? a. Ensuring the phlebotomist wears a facemask while in the client's room b. Asking the phlebotomist to delay the blood draw c. Monitoring the client closely for cold-like symptoms d. Placing a facemask on the client

a. Ensuring the phlebotomist wears a facemask while in the client's room

21. The nurse is caring for a 63-year-old with a possible pituitary tumor who is scheduled for a computed tomography (CT) scan with contrast. Which information about the patient is most important to discuss with the health care provider before the test? a. History of renal insufficiency b. Complains of chronic headache c. Recent bilateral visual field loss d. Blood glucose level of 134 mg/dL

a. History of renal insufficiency Because contrast media may cause acute kidney injury in patients with poor renal function, the health care provider will need to prescribe therapies such as IV fluids to prevent this complication. The other findings are consistent with the patients diagnosis of a pituitary tumor.

Which client assessment finding indicates to the nurse the need to assess further for a possible endocrine problem? a. Increased facial hair and absent menses in a 28-year-old nonpregnant woman b. Increased appetite in a 40-year-old man who started an aerobic exercise program 1 week ago c. Male-pattern baldness in a 32-year-old man d. Dry skin on the shins of a 70-year-old woman

a. Increased facial hair and absent menses in a 28-year-old nonpregnant woman

Which of the following individuals demonstrates the highest number of risk factors for suicide? a. John, who reports that he is in deep emotional plan, feels hopeless, and says "No one is there for me" b. Kelly, who has been seeing a doctor for chronic, intractable pain, verbalizes a deep commitment to her religious faith and is taking pain medications c. Jim, an American Indian, who graduated from high school with honors but does not yet have a job d. Mike, a physician, who reports feeling "burnt out" and is considering retirement

a. John, who reports that he is in deep emotional plan, feels hopeless, and says "No one is there for me"

Signs and symptoms of sleep apnea include which of the following? a. Loud snoring b. Difficulty falling asleep c. Headache in the evening d. Nighttime sleepiness

a. Loud snoring

12. A client is taking furosemide (Lasix) 40 mg/day for management of chronic kidney disease (CKD). To detect the positive effect of the medication, what action of the nurse is best? a. Obtain daily weights of the client. b. Auscultate heart and breath sounds. c. Palpate the client's abdomen. d. Assess the client's diet history.

a. Obtain daily weights of the client.

Which change in serum electrolyte values in the past 12 hours for a client with syndrome of inappropriate antidiuretic hormone (SIADH) being treated with tolvaptan will the nurse report immediately to the health care provider? a. Serum sodium increases from 122 mEq/L to 140 mEq/L. b. Serum potassium decreases from 4.2 mEq/L to 3.8 mEq/L. c. Serum chloride decreases from 109 mEq/L to 99 mEq/L. d. Serum calcium increases from 9.5 mg/dL to 10.2 mg/dL.

a. Serum sodium increases from 122 mEq/L to 140 mEq/L.

The nurse identifies the primary nursing diagnosis for Theresa as "Risk for suicide related to feelings of hopelessness from loss of relationship." Which is the outcome that would be most appropriate for this diagnosis? a. The patient has experienced no physical harm to herself b. The patient sets realistic goals for herself c. The patient expresses some optimism and hope for the future d. The patient has reached a stage of acceptance in the loss of the relationship with her boyfriend

a. The patient has experienced no physical harm to herself

Which assessment finding in a client with diagnosis of diabetes insipidus (DI) indicates to the nurse that desmopressin therapy is effective? a. Urine output of 30 to 50 mL/hr b. Blood glucose level of 110 mg/dL (6.1 mmol/L) c. Respiratory rate of 20 breaths/min d. Potassium level of 3.9 mEq/L (mmol/L)

a. Urine output of 30 to 50 mL/hr

1. Which statements will the nurse include when teaching a patient who is scheduled for oral glucose tolerance testing in the outpatient clinic (select all that apply)? a. You will need to avoid smoking before the test. b. Exercise should be avoided until the testing is complete. c. Several blood samples will be obtained during the testing. d. You should follow a low-calorie diet the day before the test. e. The test requires that you fast for at least 8 hours before testing.

a. You will need to avoid smoking before the test. c. Several blood samples will be obtained during the testing. e. The test requires that you fast for at least 8 hours before testing. Smoking may affect the results of oral glucose tolerance tests. Blood samples are obtained at baseline and at 30, 60, and 120 minutes. Accuracy requires that the patient be fasting before the test. The patient should consume at least 1500 calories/day for 3 days before the test. The patient should be ambulatory and active for accurate test results.

according to current information, what factor is associated w/ the most disabling mental disorders? a. biological influences b. psychological trauma c. learned ways of behaving d. faulty patterns of early nurturance

a. biological influences - (and genetic factors) influence mental health.

the nurse planning care for a mentally ill client bases interventions on which concept? a. every client has a certain degree of resilience b. it is a client right to be treated respectfully c. every client comes w/ experiences that contribute to their problem d. there are universal fears that are shared by all mentally healthy individuals

a. every client has a certain degree of resilience (nurses are expected to evaluate clients w/ mental health issues for their strengths and their areas of high functioning

1. A 22-year-old patient is being seen in the clinic with increased secretion of the anterior pituitary hormones. The nurse would expect the laboratory results to show a. increased urinary cortisol. b. decreased serum thyroxine. c. elevated serum aldosterone levels. d. low urinary catecholamines excretion.

a. increased urinary cortisol. Increased secretion of adrenocorticotropic hormone (ACTH) by the anterior pituitary gland will lead to an increase in serum and urinary cortisol levels. An increase, rather than a decrease, in thyroxine level would be expected with increased secretion of thyroid stimulating hormone (TSH) by the anterior pituitary. Aldosterone and catecholamine levels are not controlled by the anterior pituitary

the nurse is taking a history on a patient with a change in urinary patterns. in additon to medical and surgical history, what does the nurse ask the patient about to complete the assessment? (SATA) a. Occupation exposure to toxins b. Use of illicit substances, such as cocaine c. Financial resources for payment of treatments d. Likelihood of complying with treatment recommendations e. Recent travel to geographic regions that pose infectious disease risks

abe

A patient has undergone a kidney biopsy. In the immediate postprocedural period, the nurse notifies the health care provider about which findings? (SATA) a. Hematuria with blood clots b. Localized pain at the site c. "Tamponade effect" d. Decreasing urine output e. Flank pain f. Decreasing blood pressure

adef

The nurse is assessing a client with acute pyelonephritis. What assessment findings would the nurse expect? (Select all that apply.) Fever Chills Tachycardia Tachypnea Flank or back pain Fatigue

all of these

A nurse assesses a client who is recovering from extracorporeal shock-wave lithotripsy for renal calculi. The nurse notes an ecchymotic area on the client's right lower back. What action would the nurse take? Administer fresh-frozen plasma. Apply an ice pack to the site. Place the client in the prone position. Obtain serum coagulation test results.

b

A nurse assesses a client with polycystic kidney disease (PKD). Which assessment finding would alert the nurse to immediately contact the primary health care provider? Flank pain Periorbital edema Bloody and cloudy urine Enlarged abdomen

b

A nurse cares for a postmenopausal client who has had two episodes of bacterial urethritis in the last 6 months. The client asks, "I never have urinary tract infections. Why is this happening now?" How would the nurse respond? "Your immune system becomes less effective as you age." "Low estrogen levels can make the tissue more susceptible to infection." "You should be more careful with your personal hygiene in this area." "It is likely that you have an untreated sexually transmitted disease."

b

A nurse reviews the laboratory findings of a client with a urinary tract infection (bacterial cystitis). The laboratory report notes a "shift to the left" in the client's white blood cell count. What action would the nurse take? Request that the laboratory perform a differential analysis on the white blood cells. Notify the primary health care provider and start an intravenous line for parenteral antibiotics. Ask assistive personnel (AP) to strain the client's urine for renal calculi. Assess the client for a potential allergic reaction and anaphylactic shock.

b

A patient had a renal scan. What is included in the postprocedural care for this patient? a. Administer laxatives to cleanse the bowel b. Encourage oral fluids to assist excretion of isotope c. Administer captopril (Capoten) to increase blood flow d. Insert a urinary catheter to measure urine output

b

A patient has sustained a minor kidney injury. Which structure must remain functional in order to form urine from blood? a. Medulla b. Nephron C. calyx d. capsule

b

A patient is diagnosed with renal artery stenosis. Which sound does the nurse expect to hear by auscultation when a bruit is present in a renal artery? a. Quiet, pulsating sound b. Swishing sound c. Faint wheezing d. NO sound at all

b

A patient returns to the unit after a renal scan. Which instruction about the patient urine does the nurse give to the UAP caring for the patient? a. It is radioactive, so it should be handled with special biohazard precautions b. It does not place anyone at risk because of the small amount of radioactive material c. its radioactivity is dangerous only to those who are pregnant d. it is potentially dangerous if allowed to sit for prolonged periods in the command

b

After delegating care to assistive personnel (AP) for a client who is prescribed habit training to manage incontinence, a nurse evaluates the AP's understanding. Which action indicates that the AP needs additional teaching? Toileting the client after breakfast Changing the client's incontinence brief when wet Encouraging the client to drink fluids Recording the client's incontinence episodes

b

After teaching a client with early polycystic kidney disease (PKD) about nutritional therapy, the nurse assesses the client's understanding. Which statement made by the client indicates a correct understanding of the teaching? "I will take a laxative every night before going to bed." "I must increase my intake of dietary fiber and fluids." "I shall only use salt when I am cooking my own food." "I'll eat white bread to minimize gastrointestinal gas."

b

After teaching a client with hypertension secondary to renal disease, the nurse assesses the client's understanding. Which statement made by the client indicates a need for additional teaching? "I can prevent more damage to my kidneys by managing my blood pressure." "If I have increased urination at night, I need to drink less fluid during the day." "I need to see the registered dietitian to discuss limiting my protein intake." "It is important that I take my antihypertensive medications as directed."

b

After teaching a client with nephrotic syndrome and a normal glomerular filtration, the nurse assesses the client's understanding. Which statement made by the client indicates a correct understanding of the diet therapy for this condition? "I must decrease my intake of fat." "I will increase my intake of protein." "A decreased intake of carbohydrates will be required." "An increased intake of vitamin C is necessary."

b

An older adult male patient has a history of an enlarged prostate. the patient is most liekly to report which symptoms associated with this condition? a. Inability to sense the urge to void b. Difficulty starting the urine stream c. Excreting large amounts of very dilute urine d. Burning sensation when urinating

b

Damage to which renal structure or tissues can change the acutal production of urine? a. kidney parenchyma b. convulted tubules c. calyces d. ureters

b

During the day, the nursing student is measuring urine output and observing for urine characteristics in a patient. Which abnormal finding is the most urgent, which must be reported to the supervising nurse? a. Specific gravity is decreased b. Output is decreased c. pH is decreased d. Color has changed

b

In which circumstance is the regulatory role of aldosterone most important in order for the person to maintain homeostasis? a. person is having pain related to a kidney stone b. person has been hiking in the desert for several hours c. person experiences stress incontinence when coughing d. person experiences a burning sensation during urination

b

Ketones in the urine may indicate which occurrence or process? a. Increased glomerular membrane permeability b. Chronic kidney infection c. Body's use of fat for cellular energy d. Urianry tract infection

b

The health care provider informs the nurse that there is a change in orders because the patient has a decrease in creatine clearance rate. what change does the nurse anticipate? a. fluid restriction b. reduction of drug dosage c. limitation on activity level d. modification of diet

b

The nurse assesses a client with a history of urinary incontinence who presents with extreme dry mouth, constipation, and an inability to void. Which question would the nurse ask first? "Are you drinking plenty of water?" "What medications are you taking?" "Have you tried laxatives or enemas?" "Has this type of thing ever happened before?"

b

The nurse is caring for a patient with dehydration. Which laboratory test results does the nurse anticipate to see for this patient? a. BUN and creatinine ratio stay the same b. BUN rises faster than creatinine level c. Creatinine rises faster than BUN d. BUN and creatinine have a direct relationship

b

The nurse is interviewing a 35-year-old women who needs evaluation for a potential kidney problem. The woman reports she has been pregant twice and has two healthy children. what would the nurse ask about health problems that occurred during pregnancy? a. "How much weight did you gain during the preganncy? b. "Were you treated for gestational diabetes?" c. "Did both of your pregnancies go to full-term? d. "Did you have a urinary catheter inserted during labor?"

b

The nurse is performing an assessment of the renal system. What is the first step in the assessment process? a. Percuss the lower abdomen; continue toward the umbilicus b. Observe the flank region for asymmetry or discoloration c. Listen for a bruit over each renal artery d. Lightly palpate the abdomen in all quadreants

b

The nurse is preparing to assess a female patients urethra prior to the insertion of a Foley catheter. In addition to gloves, which equipment does the nurse obtain to perform the initial assessment? a. Glass slide b. Good light source c. Speculum d. Cotton ball

b

The nurse is taking a history on a 55-year-old patient who denies any serious chronic health problems Which sudden onset sign/symptoms suggests possible kidney disease in this patient? a. Weakness b. Hypertension c. Confusion d. Dysrhythmia

b

The nurse is teaching a patient scheduled for an ultrasonography. What preprocedural instruction does the nurse give the patient? a. void just before the test begins b. drink water to fill the bladder c. stop routine medications d have nothing to eat or drink after midnight

b

The nurse reads in the assessment note made by the advanced-practice nurse that the "left kidney cannot be palpated." How does the nurse interpret this notation? a. The left kidney is smaller than normal, which indicates CKD b. The left kidney is normally deeper and often cannot be palpated c. The palpation of kidneys should be repeated by another provider d. The patient is too obese for this type of examination

b

The nurse sees that an older patient has a blood osmolarity of 303 mOsm/L. Which additional assessment will the nurse make before notifying the health care provider about the laboratory results? a. Patients mental stauts b. Signs of dehydration c. Patients temeprature d. Odor of the urine

b

The nurse tells the patient that the health care provider recommends a fluid intake of at least 2 liters per day. The nurse then asks the patient to report on fluid intake over the past 24 hours to assess typical intake. The patient reports 15 ounces of coffee and 10 ounces of juice for breakfast; 10 ounces of skim milk for a midmorning snack, 12 ounces of protein shake for lunch, 1/2 liter of sports drink in the afternoon and 3 ounces of wine for dinner. After calculating the 24-hour fluid intake, what does the nurse tell the patient? a. Fluid consumptions should be increased by at least 2 more servings b. Fluid consumption is meeting the 2 liters/day recommendation c. Fluid consumption exceeds recommendation, therefore eliminate the wine d. Fluid consumption only includes liquids such as water, juice, or milk

b

Vitamin D is converted to its active form in the kidney. If this function fails, which electrolyte imbalance will occur? a. Hyperkalemia b. Hypocalcemia c. Hypernatremia d. Hypoglycemia

b

What is an advantage of a renal scan compared to a CT scan for diagnosing the perfusion, function, and structure of the kidneys? a. renal scan is more readily tolerated by elderly patients and small children b. Renal scan is preferred if the patient is allergic to iodine or has impaired kidney function c. renal scans are more likely to detect pathologic changes that CT scans do not detect d. renal scan requires less pre- and postprocedural care than CT scan

b

When patients have problems with kidneys or urinary tract, what is the most common symptoms that prompts them to seek medical attention? a. Change in the frequency or amount of urinartion b. Pain in flank or abdomen or pain when urinating c. Noticing a change in the color or odor of the urine d. Exposure to a nephrotoxic substance

b

Which ethnic group has the highest risk for kidney failure and needs special attention for patient teaching related to hypertension and sodium intake? a. caucasian American b. African Americans c. asian americans d Native Americans

b

Which hormone is released from the posterior pituitary and makes the distal convoluted tubule and the collecting duct permeable to water to maximize reabsorption and produce concentrated urine? a. Aldosterone b. Vasopressin c. Bradykinins d. Natriuretic

b

Which patient is most likely to exceed the renal threshold if there is noncompliance with the prescribed therapeutic regimen? a. Has recurrent kidney stone formation b. has type 2 diabetes mellitus c. has functional urinary incontinence d. has biliary obstruction

b

Which personal action is most likely to cause the kidenys to produce and release erythropoietin? a. person moves to a low desert area where the humidity is very low b. person moves to a high-altitude area where atmospehric oxygen is low c. Person drinks an excessive amount of fluid that resutls in fluid overload d. person eats a large high-protein meal after a rigourous exercise workout

b

Which test is the best indicator of kidney function? a Urine osmolarity b. serum creatinine c. Urine pH d. BUN

b

the nurse is caring for a paitent who sustained major injuries in an automobile accident. Which blood pressure will result in compromised kidney function, in particualr the glomerular filtration rate (GFR)? a. 150/70 mm Hg b. 70/40 mm Hg c. 80/60 mm Hg d. 140/80 mm Hg

b

what is the average urine output of a healthy adult for a 24-hour period? a. 500 to 1000 mL per day b. 1500 to 2000 mL per day c. 3000 to 5000 mL per day d. 5000 to 7000 mL per day

b

6. A patient has a low serum cortisol level. Which hormone would the nurse expect to be secreted to correct this? a. Thyroid-stimulating hormone (TSH) b. Adrenocorticotropic hormone c. Parathyroid hormone d. Antidiuretic hormone

b Adrenocorticotropic hormone

3. Which mechanism is used to transport the substance produced by the endocrine glands to their target tissue? a. Lymph system b. Bloodstream c. Direct seeding d. Gastrointestinal system

b Bloodstream

39. What is the correct nursing action before beginning a 24-hour urine collection for endocrine studies? a. Place each voided specimen in a separate collection container. b. Check whether any preservatives are needed in the collection container. c. Start the collection with the first voided urine. d. Weigh the patient before beginning the collection.

b Check whether any preservatives are needed in the collection container.

18. Which is the major function of the hormones produced by the adrenal cortex? a. "Fight-or-flight" response b. Control of potassium, sodium, and water c. Regulation of cell growth d. Calcium and stress regulation

b Control of potassium, sodium, and water

31. Which disease involves a disorder of the islets of Langerhans? a. Diabetes insipidus b. Diabetes mellitus c. Addison's disease d. Cushing's disease

b Diabetes mellitus

15. Which statement about pituitary hormones is correct? a. ACTH acts on the adrenal medulla. b. Follicle-stimulating hormone (FSH) stimulates sperm production in men. c. Growth hormone promotes protein catabolism. d. Vasopressin decreases systolic blood pressure.

b Follicle-stimulating hormone (FSH) stimulates sperm production in men.

42. A patient is suspected of having a pituitary tumor. Which radiographic test aids in determining this diagnosis? a. Skull x-rays b. MRI/CT c. Angiography d. Ultrasound

b MRI/CT

12. The maintenance of internal body temperature at approximately 98.6F (37C) is an example of which endocrine process? a. "Lock and key" manner b. Neuroendocrine regulation c. Positive feedback mechanism d. Stimulus-response theory

b Nueroendocrine regulation

37. Which statement about performing a physical assessment of the thyroid gland is correct? a. The thyroid gland is easily palpated in all patients. b. The patient is instructed to swallow sips of water to aid palpation. c. The anterior approach is preferred for thyroid palpation. d. The thumbs are used to palpate the thyroid lobes.

b The patient is instructed to swallow sips of water to aid palpation.

29. In addition to the pancreas that secretes insulin, which gland secretes hormones that affect protein, carbohydrate, and fat metabolism? a. Posterior pituitary b. Thyroid c. Ovaries d. Parathyroid

b Thyroid

36. The nurse is performing a physical assessment of a patient's endocrine system. Which gland can be palpated? a. Pancreas b. Thyroid c. Adrenal glands d. Parathyroids

b Thyroid

33. Which statement about age-related changes in older adults and the endocrine system is true? a. All hormone levels are elevated. b. Thyroid hormone levels decrease. c. Adrenal glands enlarge. d. The thyroid gland enlarges.

b Thyroid hormone levels decrease.

Which patients narrative describes the symptoms of dysuria? a. "I have to pee all the time." b. "I have to wait before the pee starts." c. "It burns when I pee." d. "It feels like I am going to pee in my pants."

c

44. A patient is at risk for falling related to the effect of pathologic fractures as a result of bone demineralization. Which endocrine problem is this pertinent to? a. Underproduction of PTH b. Overproductive of PTH c. Underproduction of thyroid hormone d. Overproduction of thyroid hormone

b Overproductive of PTH

how can a nurse best differentiate whether an asian client is demonstrating a mental illness after having attempted suicide? a. ask the client whether he views himself as being depressed b. identify the client's culture's views regarding suicide c. explain that suicide is often regarded as a desperate act d. assess the client for other examples of depressive behaviors

b - identify the client's culture's view regarding suicide

resilience is characteristic of mental health that allows people to adapt to tragedies, trauma, and loss. which client behavior demonstrates this characteristic? a. my mother made decisions about my husband's funeral when i just couldn't do that" b. losing my job was hard, but my skills will help me get another one." c. in spite of all the tx, i know i'll never be really healthy." d. "my kids, happiness is worth any sacrifice i have to make."

b - losing my job was hard, but my skills will help me get another one"

a client tells the mental health nurse, "i am terribly frightened! i hear whispering in my head that someone is going to kill me." which criteria of mental health can the nurse assess as lacking?" a. self-control b. rational thinking c. learning and productivity d. positive self-concept

b - rational thinking

which severe mental illness is recognized across cultures? (SATA) a. antisocial disorder b. schizophrenia c. anorexia nervosa d. social phobia e. bipolar disorder f. borderline personality disorder

b - schizophrenia e - bipolar disorder

Theresa, who has been hospitalized following a suicide attempt, is placed on suicide precautions on the psychiatric unit. She admits that she is still feeling suicidal. Which of the following interventions is the most appropriate in this instance? a. Obtain an order from the physician to place Theresa in restraints to prevent any attempts to harm herself b. Check on Theresa every 15 minutes or assign a staff person to stay with her on a one-to-one basis c. Obtain an order from the physician to give Theresa a sedative to calm her and reduce suicide ideas d. Do not allow Theresa to participate in any unit activities while she is on suicide precautions e. Ask Theresa specific questions about her thoughts, plans, and intentions related to suicide

b and e

25. Which are the target organs of PTH in the regulation of calcium and phosphorus? (Select all that apply.) a. Stomach b. Kidney c. Bone d. Gastrointestinal tract e. Thyroid gland

b, c, d Kidney, Bone, Gastrointestinal tract

21. Which statements about the thyroid gland and its hormones are correct? (Select all that apply.) a. The gland is located in the posterior neck below the cricoid cartilage. b. The gland has two lobes joined by a thin tissue called the isthmus. c. T4 and T3 are two thyroid hormones. d. Thyroid hormones increase red blood cell production. e. Thyroid hormone production depends on dietary intake of iodine and potassium.

b, c, d The gland has two lobes joined by a thin tissue called the isthmus. T4 and T3 are two thyroid hormones. Thyroid hormones increase red blood cell production.

32. Which endocrine tissues are most commonly found to have reduced function as a result of aging? (Select all that apply.) a. Hypothalamus b. Ovaries c. Testes d. Pancreas e. Thyroid gland

b, c, d, e Ovaries, Testes, Pancreas, Thyroid gland

A nurse assesses a client who has a family history of polycystic kidney disease (PKD). Which assessment findings would the nurse expect? (Select all that apply.) Nocturia Flank pain Increased abdominal girth Dysuria Hematuria Diarrhea

b, c, e

19. Which statements about the hormone cortisol being secreted by the adrenal cortex are accurate? (Select all that apply.) a. Cortisol peaks occur late in the day, with lowest points 12 hours after each peak. b. Cortisol has an effect on the body's immune function. c. Stress causes an increase in the production of cortisol. d. Blood levels of cortisol have no effect on its secretion. e. Cortisol affects carbohydrate, protein, and fat metabolism.

b, c, e Cortisol has an effect on body's immune function. Stress causes an increase in the production of cortisol. Cortisol affects carbohydrate, protein, and fat metabolism.

4. Which hormones are secreted by the posterior pituitary gland? (Select all that apply.) a. Testosterone b. Oxytocin c. Growth hormone (GH) d. Antidiuretic hormone (ADH) e. Cortisol

b, d Oxytocin, Antidiuretic hormone (ADH)

A nurse assesses a client who is recovering from a nephrostomy. Which assessment findings would alert the nurse to urgently contact the primary health care provider? (Select all that apply.) Clear drainage Bloody drainage at site Patient reports headache Foul-smelling drainage Urine draining from site

b, d, e

20. The nurse is teaching the main principles of hemodialysis to a client with chronic kidney disease. Which statement by the client indicates a need for further teaching by the nurse? a. "My sodium level changes by movement from the blood into the dialysate." b. "Dialysis works by movement of wastes from lower to higher concentration." c. "Extra fluid can be pulled from the blood by osmosis." d. "The dialysate is similar to blood but without any toxins."

b. "Dialysis works by movement of wastes from lower to higher concentration."

5. A nurse is giving discharge instructions to a client recently diagnosed with chronic kidney disease (CKD). Which statements made by the client indicate a correct understanding of the teaching? (Select all that apply.) a. "I can continue to take antacids to relieve heartburn." b. "I need to ask for an antibiotic when scheduling a dental appointment." c. "I'll need to check my blood sugar often to prevent hypoglycemia." d. "The dose of my pain medication may have to be adjusted." e. "I should watch for bleeding when taking my anticoagulants."

b. "I need to ask for an antibiotic when scheduling a dental appointment." c. "I'll need to check my blood sugar often to prevent hypoglycemia." d. "The dose of my pain medication may have to be adjusted." e. "I should watch for bleeding when taking my anticoagulants."

Which statement made by the client who is going home after a transsphenoidal hypophysectomy indicates to the nurse correct understanding of actions to prevent complications from this treatment? a. "While I am awake, I will be sure to cough and deep breathe at least every 2 hours." b. "I will keep the cat food bowl on my counter so that I do not have to bend over." c. "Whenever I am out-of-doors in the sunshine, I will wear dark glasses." d. "If the dressing gets wet, I will wash the incision line and redress it immediately."

b. "I will keep the cat food bowl on my counter so that I do not have to bend over."

4. The nurse is teaching a client with diabetes mellitus how to prevent or delay chronic kidney disease (CKD). Which client statements indicate a lack of understanding of the teaching? (Select all that apply.) a. "I need to decrease sodium, cholesterol, and protein in my diet." b. "My weight should be maintained at a body mass index of 30." c. "Smoking should be stopped as soon as I possibly can." d. "I can continue to take an aspirin every 4 to 8 hours for my pain." e. "I really only need to drink a couple of glasses of water each day."

b. "My weight should be maintained at a body mass index of 30." d. "I can continue to take an aspirin every 4 to 8 hours for my pain." e. "I really only need to drink a couple of glasses of water each day."

What is the nurse's best response when a client, who has been taking high-dose corticosteroid therapy for a month for a problem that has now resolved, asks you why she needs to continue taking the corticosteroid? a. "Corticosteroids are a type of hormone, and once you have been started on a replacement hormone, you must continue the hormone replacement therapy for the rest of your life." b. "The drug suppressed your own adrenal gland secretion of corticosteroids. Slowly decreasing the dose over time allows your adrenal glands to start adequate secretion again." c. "It is possible for your health problem to recur when corticosteroid therapy is halted suddenly." d. "The drug suppressed your immune system while you were taking it. Slowly decreasing the dose over time prevents your immune system from starting up too quickly and causing allergic reactions."

b. "The drug suppressed your own adrenal gland secretion of corticosteroids. Slowly decreasing the dose over time allows your adrenal glands to start adequate secretion again."

What is the nurse's best response when a client with Cushing syndrome screams at her husband, bursts into tears, throws her water pitcher against the wall, and then says "I feel like I am going crazy"? a. "You must learn to control your behavior. Because you are disturbing others, I am going to keep the door to your room closed and restrict your visitors." b. "You feel this way because of your high hormone levels. Your health care provider can prescribe an antianxiety drug for you." c. "I will tell your primary health care provider order a psychiatric consult for you." d. "You are probably feeling this way because you are frightened about having a chronic disease. Would you like some information about a support group?"

b. "You feel this way because of your high hormone levels. Your health care provider can prescribe an antianxiety drug for you."

a nurse, active in local consumer mental health groups and in local and state mental health associations, keeps aware of state and national legislation affecting mental illness tx. how can this nurse positively affect the climate for effective mental health tx? a. "by becoming active in politics leading to a potential political career." b. "by educating the public on the effects that stigmatizing has on mental health clients." c. "advocating for laws that would make the involuntary long-term commitment process easier and faster for caregivers of mentally ill persons." d "advocating for reduced mental health insurance benefits to discourage abuse of the system by inappropriate psychiatric admissions."

b. "by educating the public on the effects that stigmatizing has on mental health clients."

For which client will the nurse question the prescription for long-term androgen therapy? a. A 40-year-old who also has syndrome of inappropriate antidiuretic hormone (SIADH). b. A 52-year-old with a history of prostate cancer treatment. c. A 30-year-old who is taking antiviral therapy for HIV disease. d. A 66-year-old with impotence that is resistant to standard erectile dysfunction therapy.

b. A 52-year-old with a history of prostate cancer treatment.

Clients who have deficiencies of which hormones will the nurse assess for increased risk of life-threatening consequences? a. Prolactin and prolactin inhibiting hormone (PIH) b. Adrenocorticotrophicn hormone (ACTH) and thyroid-stimulating hormone (TSH) c. Growth hormone (GH) and melanocyte-stimulating hormone (MSH) d. Follicle-stimulating hormone (FSH) and luteinizing hormone (LH)

b. Adrenocorticotrophicn hormone (ACTH) and thyroid-stimulating hormone (TSH)

20. A 35-year-old female patient with a possible pituitary adenoma is scheduled for a computed tomography (CT) scan with contrast media. Which patient information is most important for the nurse to communicate to the health care provider before the test? a. Bilateral poor peripheral vision b. Allergies to iodine and shellfish c. Recent weight loss of 20 pounds d. Complaint of ongoing headaches

b. Allergies to iodine and shellfish Because the usual contrast media is iodine-based, the health care provider will need to know about the allergy before the CT scan. The other findings are common with any mass in the brain such as a pituitary adenoma.

3. A 30-year-old patient seen in the emergency department for severe headache and acute confusion is found to have a serum sodium level of 118 mEq/L. The nurse will anticipate the need for which diagnostic test? a. Urinary 17-ketosteroids b. Antidiuretic hormone level c. Growth hormone stimulation test d. Adrenocorticotropic hormone level

b. Antidiuretic hormone level Elevated levels of antidiuretic hormone will cause water retention and decrease serum sodium levels. The other tests would not be helpful in determining possible causes of the patients hyponatremia.

Which assessment finding in a client with hyperaldosteronism indicates to the nurse that the condition is becoming more severe? a. Urine output for the past 24 hours has increased. b. Client reports numbness and tingling around the mouth. c. Temperature is now elevated. d. pH is now 7.43.

b. Client reports numbness and tingling around the mouth.

In determining degree of suicide risk with a suicidal patient, the nurse assess the following behavioral manifestations: severely depressed, withdrawn, statements of worthlessness, difficulty accomplishing activities of daily living, no close support systems. The nurse identifies the patient's risk for suicide as which of the following? a. Low risk b. High risk c. Imminent risk d. Unable to determine

b. High risk

Which laboratory findings will the nurse use to validate the statement of a client with diabetes that therapy instructions for glucose control "have been followed to the letter" for the past 2 months? a. Random blood glucose level b. Glycosylated hemoglobin (HbA1c) c. Fasting blood insulin level d. Fasting blood glucose level

b. Glycosylated hemoglobin (HbA1c)

What effect on circulating levels of sodium and glucose does the nurse expect in a client who has been taking an oral cortisol preparation for 2 years because of a respiratory problem? a. Decreased sodium; decreased glucose b. Increased sodium; increased glucose c. Increased sodium; decreased glucose d. Decreased sodium; increased glucose

b. Increased sodium; increased glucose

In collaboration with the registered dietitian nutritionist, which dietary alterations will the nurse instruct a client with Cushing disease to make? a. High carbohydrate, low potassium, and fluid restriction b. Low carbohydrate, high calorie, and low sodium c. Low protein, high carbohydrate, and low calcium d. High protein, high carbohydrate, and low potassium

b. Low carbohydrate, high calorie, and low sodium

A patient has a urinalysis ordered. When is the best time for the nurse to collect the specimen? a. In the evening b. After a meal c. In the morning d. After a fluid bolus

c

18. Which action by a new registered nurse (RN) caring for a patient with a goiter and possible hyperthyroidism indicates that the charge nurse needs to do more teaching? a. The RN checks the blood pressure on both arms. b. The RN palpates the neck thoroughly to check thyroid size. c. The RN lowers the thermostat to decrease the temperature in the room. d. The RN orders nonmedicated eye drops to lubricate the patients bulging eyes.

b. The RN palpates the neck thoroughly to check thyroid size. Palpation can cause the release of thyroid hormones in a patient with an enlarged thyroid and should be avoided. The other actions by the new RN are appropriate when caring for a patient with an enlarged thyroid.

19. The nurse is caring for a 45-year-old male patient during a water deprivation test. Which finding is most important for the nurse to communicate to the health care provider? a. The patient complains of intense thirst. b. The patient has a 5-lb (2.3 kg) weight loss. c. The patients urine osmolality does not increase. d. The patient feels dizzy when sitting on the edge of the bed.

b. The patient has a 5-lb (2.3 kg) weight loss. A drop in the weight of more than 2 kg indicates severe dehydration, and the test should be discontinued. The other assessment data are not unusual with this test.

17. Which information about a 30-year-old patient who is scheduled for an oral glucose tolerance test should be reported to the health care provider before starting the test? a. The patient reports having occasional orthostatic dizziness. b. The patient takes oral corticosteroids for rheumatoid arthritis. c. The patient has had a 10-pound weight gain in the last month. d. The patient drank several glasses of water an hour previously.

b. The patient takes oral corticosteroids for rheumatoid arthritis. Corticosteroids can affect blood glucose results. The other information will be provided to the health care provider but will not affect the test results.

Success of long-term psychotherapy with Theresa (who attempted suicide following a break up with her boyfriend) could be measured by which of the following behaviors? a. Theresa has a new boyfriend b. Theresa has an increased sense of self-worth c. Theresa does not take antidepressants anymore d. Theresa told her old boyfriend how angry she was with him for breaking up with her

b. Theresa has an increased sense of self-worth

11. A 40-year-old male patient has been newly diagnosed with type 2 diabetes mellitus. Which information about the patient will be most useful to the nurse who is helping the patient develop strategies for successful adaptation to this disease? a. Ideal weight b. Value system c. Activity level d. Visual changes

b. Value system When dealing with a patient with a chronic condition such as diabetes, identification of the patients values and beliefs can assist the health care team in choosing strategies for successful lifestyle change. The other information also will be useful, but is not as important in developing an individualized plan for the necessary lifestyle changes.

7. During the physical examination of a 36-year-old female, the nurse finds that the patients thyroid gland cannot be palpated. The most appropriate action by the nurse is to a. palpate the patients neck more deeply. b. document that the thyroid was nonpalpable. c. notify the health care provider immediately. d. teach the patient about thyroid hormone testing.

b. document that the thyroid was nonpalpable. The thyroid is frequently nonpalpable. The nurse should simply document the finding. There is no need to notify the health care provider immediately about a normal finding. There is no indication for thyroid- stimulating hormone (TSH) testing unless there is evidence of thyroid dysfunction. Deep palpation of the neck is not appropriate.

14. A nurse will teach a patient who is scheduled to complete a 24-hour urine collection for 17-ketosteroids to a. insert and maintain a retention catheter. b. keep the specimen refrigerated or on ice. c. drink at least 3 L of fluid during the 24 hours. d. void and save that specimen to start the collection.

b. keep the specimen refrigerated or on ice. The specimen must be kept on ice or refrigerated until the collection is finished. Voided or catheterized specimens are acceptable for the test. The initial voided specimen is discarded. There is no fluid intake requirement for the 24-hour collection.

The most effective treatments for sleep apnea include all of the following EXCEPT a. weight loss. b. the use of stimulant drugs. c. the use of a continuous positive airway pressure (CPAP) mask. d. surgery for breathing obstructions.

b. the use of stimulant drugs.

The nurse is assessing a patient with a chronic kidney problem. The nurse notes that the patient has pedal edema and periorbital edema. What additional assessments will the nurse make to assess for fluid overload? (SATA) a. Obtain a urine specimen b. Compare current blood pressure to baseline c. Measure the residual urine with a bladder scanner d. Weigh the patient and compare to baseline e. Auscultate lung fields to determine if fluid is present

bde

A 24-hour urine specimen is required from a patient. Which strategy is best to ensure that all the urine is collected for the full 24-hour period? a. Instruct the UAP to collect all the urine b. Put a bedpan or commode next to the bed as a reminder c. Place a sign in the bathroom reminding everyone to save urine d. Verbally remind the patient about the test

c

A client with pneumonia and dementia is admitted with an indwelling urinary catheter in place. During interprofessional rounds the following day, which question would the nurse ask the primary health care provider? "Do you want daily weights on this client?" "Will the client be able to return home?" "May we discontinue the indwelling catheter?" "Should we get another chest x-ray today?"

c

A nurse cares for a client who has pyelonephritis. The client states, "I am embarrassed to talk about my symptoms." How would the nurse respond? "I am a professional. Your symptoms will be kept in confidence." "I understand. Elimination is a private topic and shouldn't be discussed." "Take your time. It is okay to use words that are familiar to you." "You seem anxious. Would you like a nurse of the same gender to care for you?"

c

A nurse cares for a middle-age female client with diabetes mellitus who is being treated for the third episode of acute pyelonephritis in the past year. The client asks, "What can I do to help prevent these infections?" How would the nurse respond? "Test your urine daily for the presence of ketone bodies and proteins." "Use tampons rather than sanitary napkins during your menstrual period." "Drink more water and empty your bladder more frequently during the day." "Keep your hemoglobin A1C under 9% by keeping your blood sugar controlled."

c

A nurse teaches a client who is starting urinary bladder training. Which statement would the nurse include in this client's teaching? "Use the toilet when you first feel the urge, rather than at specific intervals." "Initially try to use the toilet at least every half hour for the first 24 hours." "Try to consciously hold your urine until the scheduled toileting time." "The toileting interval can be increased once you have been continent for a week."

c

After teaching a client with bacterial cystitis who is prescribed phenazopyridine, the nurse assesses the client's understanding. Which statement made by the client indicates a correct understanding of the teaching? "I will not take this drug with food or milk." "I will have my partners tested for STIs." "An orange color in my urine should not alarm me." "I will drink two glasses of cranberry juice daily."

c

In addition to kidney disease, which patient condition causes the BUN to rise above the noraml range? a. Anemia b. Asthama c. Infection d. Malnutrtion

c

Several patients are scheduled for testing to diagnose potential kidney problems. Which test requires a patient to have a urinary catheter inserted before the test? a. Urine stream testing b. Computed tomography c. Cystography d. Renal scan

c

The community health nurse is talking to a group of African-American adults about renal health. The nurse encourages the participants to have which type of yearly examination to screen for kidney problems a. Kidney ultrasound b. Serum creatinine and blood urea nitrogen c. Urinalysis and microalbuminuria d. 24-hour urine collection

c

The nurse and nutritionist are evaluating the diet and nutritional therapies for a patient with kidney problems. BUN levels for this patient are tracked because of the direct relationship to the intake and metabolism of which substance? a. Lipids b. Carbohydrates c. Protein d. Fluids

c

The nurse assesses a client who has possible bladder cancer. What common assessment finding associated with this type of cancer would the nurse expect? Urinary retention Urinary incontinence Painless hematuria Difficulty urinating

c

The nurse is caring for a client who has chronic pyelonephritis. What assessment finding would the nurse expect? Fever Flank pain Hypertension Nausea and vomiting

c

The nurse is caring for a client with urinary incontinence. The client states, "I am so embarrassed. My bladder leaks like a young child's bladder." How would the nurse respond? "I understand how you feel. I would be mortified." "Incontinence pads will minimize leaks in public." "I can teach you strategies to help control your incontinence." "More people experience incontinence than you might think."

c

The nurse is determining whether a patient has a history of hypertension because of the potential for kidney problems. Which question is best to elicit this information? a. "Do you have high blood pressure?" b. "Do you take any blood pressure medications?" c. "Have you ever been told that your blood pressure was high?" d. "When was the last time you had your blood pressure checked"

c

The nurse is planning the care for several patients who are undergoing diagnostic testing. Which patient is likely to need the most time for postprocedural care? a will have a kidney, ureter, and bladder x-ray b. Needs a kidney ultrasound c. Will have a cystoscopy d. Needs urine for culture and sensitivity

c

The nurse is taking a nutritional history on a patient. The patient states, "I really don't drink as much water as I should." What is the nurses best response? a. "We should probably all drink more water than we do." b. "Its an easy thing to forget; just try to remember to drink more." c. "What would encourage you to drink the recommneded 2 literes per day?" d. "Id like you to read this brochure about kidney health and fluids."

c

What does an increase in the ratio of BUN to serum creatinine indicate? a. Highly suggestive of kidney dysfunction b. definitive for kidney infection c. Suggests kidney factors causing an elevation in BUN d. Suggests nonkidney factors causing an elevation in serum creatinine

c

10. What are tropic hormones? a. Hormones that rigger female and male sex characteristics. b. Hormones that have a direct effect on final target tissues. c. Hormones produced by the anterior pituitary gland that stimulate other endocrine glands. d. Hormones that are synthesized in the hypothalamus and stored in the posterior pituitary gland

c Hormones produced by the anterior pituitary gland that stimulate other endocrine glands.

A nurse's identification badge includes the term, "Psychiatric Mental Health Nurse." A client with a history of paranoia asks, "What does that title mean?" Keeping in mind the diagnosis of the patient, how should the nurse respond to this question? a. "don't be afraid, it means i am here to help, not hurt you." b. "psychiatric mental health nurses care for people w/ mental illnesses" c. "we have the specialized skills needed to care for those w/ mental illnesses" d. "the nurses who work in mental health facilities have that title"

c "we have the specialized skills needed to care for those w/ mental illnesses"

what client assessment data demonstrates parity related to mental health care? a. the client is admitted for a 72 hour mental hygiene evaluation b. advance practice nurse can be certified as psych nurse specialist c. a client's mental health coverage is equal to his/her med-surg coverage d. a client who has attempted suicide is hospitalized for a mental health evaluation

c - a client's mental health coverage is equal to his/her med-surg coverage

which branch of epidemiology is the nurse involved in when seeking outcomes for patients whose depression was treated w/ ECT (electroconvulsive therapy)? a. experimental b. descriptive c. clinical d. analytic

c - clinical epidemiology represents a borad field that addresses what happens to people w/ illnesses who are seen by providers of clinical care.

What term is used to identify the quantitative study of the distribution of mental disorders in human populations? a. mortality b. prevalence c. epidemiology d. clinical epidemiology

c - epidemiology: the quantitative study of the distrubtion of mental disorders in human populations

a nursing dx for a client w/ a psychiatric disorder serves what purpose with considering the plan of care? a. justifying the use of certain psychtropic medication b. providing data essential for insurance reimbursement c. establishing a framework for selecting appropriate interventions d. completing the medical diagnostic statement

c - establishing a framework for selecting appropriate interventions

the mental health status of a particular client can be best assessed by considering which factor? a. the degree of conformity of the individual to society's norms b. the degree to which an individual is logical and rational c. status placement on a continuum from health to illness d. rate of demonstrated intellectual and emotional growth

c - status placement on a continuum from health to illness

a new nurse has accepted a position as staff nurse on a psych unit. which statement made by the new nurse requires additional instructions regarding the therapies provided on the unit? a. you will participate in unit activities and groups daily b. you will be given a schedule daily of the groups we would like you to attend c. you will attend a psychotherapy group that i lead that will help you care for yourself d. you will see your provider daily in a one-to-one session

c - you will attend a psychotherapy group that i lead that will help you care for yourself (Basic level RNs cannot perform psychotherapy. The other options are all appropriate expectations of a patient's schedule on a psychiatric unit.)

40. Which instructions are included when teaching a patient about urine collection for endocrine studies? (Select all that apply.) a. Fast before starting the urine collection. b. Measure the urine in mL rather than ounces. c. Empty the bladder completely, and then start timing. d. Time the test for exactly the instructed number of hours. e. Avoid taking any unnecessary drugs during endocrine testing. f. Empty the bladder at the end of the time period and keep that specimen.

c, d, e, f Empty the bladder completely, and then start timing. Time the test for exactly the instructed number of hours. Avoid taking any unnecessary drugs during endocrine testing. Empty the bladder at the end of the time period and keep that specimen.

What is the nurse's best first response when a client with a suspected endocrine disorder says, "I can't, you know, satisfy my wife anymore."? a. "Don't worry. It happens to everyone occasionally." b. "Do you use any over the counter or recreational drugs?" c. "Can you please tell me more?" d. "Would you like to speak with a counselor?"

c. "Can you please tell me more?"

Theresa, age 27, was admitted to the psychiatric unit from the medical intensive care unit where she was treated for taking a deliberate overdose of her antidepressant medication, trazodone (Desyrel). She says to the nurse, "My boyfriend broke up with me. We had been together for 6 years. I love him so much. I know I'll never get over him." Which is the best response by the nurse? a. "You'll get over him in time, Theresa" b. "Forget him. There are other fish in the sea" c. "You must be feeling very sad about your loss" d. "Why do you think he broke up with you, Theresa?"

c. "You must be feeling very sad about your loss"

21. The charge nurse is orienting a float nurse to an assigned client with an arteriovenous (AV) fistula for hemodialysis in her left arm. Which action by the float nurse would be considered unsafe? a. Palpating the access site for a bruit or thrill b. Using the right arm for a blood pressure reading c. Administering intravenous fluids through the AV fistula d. Checking distal pulses in the left arm

c. Administering intravenous fluids through the AV fistula

5. A 29-year-old patient in the outpatient clinic will be scheduled for blood cortisol testing. Which instruction will the nurse provide? a. Avoid adding any salt to your foods for 24 hours before the test. b. You will need to lie down for 30 minutes before the blood is drawn. c. Come to the laboratory to have the blood drawn early in the morning. d. Do not have anything to eat or drink before the blood test is obtained.

c. Come to the laboratory to have the blood drawn early in the morning. Cortisol levels are usually drawn in the morning, when levels are highest. The other instructions would be given to patients who were having other endocrine testing.

4. Which question will provide the most useful information to a nurse who is interviewing a patient about a possible thyroid disorder? a. What methods do you use to help cope with stress? b. Have you experienced any blurring or double vision? c. Have you had a recent unplanned weight gain or loss? d. Do you have to get up at night to empty your bladder?

c. Have you had a recent unplanned weight gain or loss? Because thyroid function affects metabolic rate, changes in weight may indicate hyperfunction or hypofunction of the thyroid gland. Nocturia, visual difficulty, and changes in stress level are associated with other endocrine disorders.

Which changes in laboratory values will the nurse expect to see in a client who has tumor causing excess secretion of aldosterone? (Select all that apply.) a. Hypoglycemia b. Hyponatremia c. Hypokalemia d. Hypernatremia e. Hyperglycemia f. Hyperkalemia

c. Hypokalemia d. Hypernatremia

16. A 44-year-old patient is admitted with tetany. Which laboratory value should the nurse monitor? a. Total protein b. Blood glucose c. Ionized calcium d. Serum phosphate

c. Ionized calcium Tetany is associated with hypocalcemia. The other values would not be useful for this patient.

13. A client is diagnosed with chronic kidney disease (CKD). What is an ideal goal of treatment set by the nurse in the care plan to reduce the risk of pulmonary edema? a. Maintaining oxygen saturation of 89% b. Minimal crackles and wheezes in lung sounds c. Maintaining a balanced intake and output d. Limited shortness of breath upon exertion

c. Maintaining a balanced intake and output

19. The nurse is taking the vital signs of a client after hemodialysis. Blood pressure is 110/58 mm Hg, pulse 66 beats/min, and temperature is 99.8° F (37.6° C). What is the most appropriate action by the nurse? a. Administer fluid to increase blood pressure. b. Check the white blood cell count. c. Monitor the client's temperature. d. Connect the client to an electrocardiographic (ECG) monitor.

c. Monitor the client's temperature.

What is a misconception about suicide? a. Eight out of ten individuals who commit suicide give warnings about their intentions b. Most suicidal individuals are ambivalent about their feelings regarding suicide c. Most individuals commit suicide by taking an overdose of drugs d. Initial mood improvement can precipitate suicide

c. Most individuals commit suicide by taking an overdose of drugs

17. A client is placed on fluid restrictions because of chronic kidney disease (CKD). Which assessment finding would alert the nurse that the client's fluid balance is stable at this time? a. Decreased calcium levels b. Increased phosphorus levels c. No adventitious sounds in the lungs d. Increased edema in the legs

c. No adventitious sounds in the lungs

Which client symptom appearing after a head injury suffered in a car crash is most relevant for the nurse to consider the possibility of diabetes insipidus (DI)? a. New-onset hypertension. b. The client reports extreme salt craving. c. No change in urine output with minimal fluid intake. d. The client's headache is gradually increasing in intensity.

c. No change in urine output with minimal fluid intake.

The nurse is caring for an actively suicidal client on the psychiatric unit. What is the nurse's priority interventions? a. Discuss strategies for the management of anxiety, anger, and frustration b. Provide opportunities for increasing the client's self-worth, morale, and control c. Place client on suicide precautions with one-to-one observation d. Explore experiences that affirm self-worth and self-efficacy

c. Place client on suicide precautions with one-to-one observation

Which laboratory finding in a client with a possible pituitary disorder will the nurse report to the health care provider immediately? a. Blood glucose 148 mg/dL (7.4 mmol/L) b. Blood urea nitrogen (BUN) 40 mg/dL (14.3 mmol/L) c. Serum sodium 110 mEq/L (110 mmol/L) d. Serum potassium 3.2 mEq/L (3.2 mmol/L)

c. Serum sodium 110 mEq/L (110 mmol/L)

15. Which additional information will the nurse need to consider when reviewing the laboratory results for a patients total calcium level? a. The blood glucose is elevated. b. The phosphate level is normal. c. The serum albumin level is low. d. The magnesium level is normal.

c. The serum albumin level is low. Part of the total calcium is bound to albumin so hypoalbuminemia can lead to misinterpretation of calcium levels. The other laboratory values will not affect total calcium interpretation.

8. Which laboratory value should the nurse review to determine whether a patients hypothyroidism is caused by a problem with the anterior pituitary gland or with the thyroid gland? a. Thyroxine (T4 ) level b. Triiodothyronine (T3 ) level c. Thyroid-stimulating hormone (TSH) level d. Thyrotropin-releasing hormone (TRH) level

c. Thyroid-stimulating hormone (TSH) level A low TSH level indicates that the patients hypothyroidism is caused by decreased anterior pituitary secretion of TSH. Low T3 and T4 levels are not diagnostic of the primary cause of the hypothyroidism. TRH levels indicate the function of the hypothalamus.

Theresa is hospitalized following a suicide attempt after breaking up with her boyfriend. Theresa says to the nurse, "When I get out of here, I'm going to try this again, and next time I'll choose a no-fail method." Which is the best response by the nurse? a. You are safe here. We will make sure nothing happens to you b. You're just lucky your roommate came home when she did c. What exactly do you plan to do? d. I don't understand. You have so much to live for

c. What exactly do you plan to do?

13. The nurse will teach a patient to plan to minimize physical and emotional stress while the patient is undergoing a. a water deprivation test. b. testing for serum T3 and T4 levels. c. a 24-hour urine test for free cortisol. d. a radioactive iodine (I-131) uptake test.

c. a 24-hour urine test for free cortisol. Physical and emotional stress can affect the results of the free cortisol test. The other tests are not impacted by stress.

10. A 60-year-old patient is taking spironolactone (Aldactone), a drug that blocks the action of aldosterone on the kidney, for hypertension. The nurse will monitor for a. increased serum sodium. b. decreased urinary output. c. elevated serum potassium. d. evidence of fluid overload.

c. elevated serum potassium. Because aldosterone increases the excretion of potassium, a medication that blocks aldosterone will tend to cause hyperkalemia. Aldosterone also promotes the reabsorption of sodium and water in the renal tubules, so spironolactone will tend to cause increased urine output, a decreased or normal serum sodium level, and signs of dehydration.

9. The nurse reviews a patients glycosylated hemoglobin (Hb A1C) results to evaluate a. fasting preprandial glucose levels. b. glucose levels 2 hours after a meal. c. glucose control over the past 90 days. d. hypoglycemic episodes in the past 3 months.

c. glucose control over the past 90 days. Glycosylated hemoglobin testing measures glucose control over the last 3 months. Glucose testing before/after a meal or random testing may reveal impaired glucose tolerance and indicate prediabetes, but it is not done on patients who already have a diagnosis of diabetes. There is no test to evaluate for hypoglycemic episodes in the past.

A healthy 34-year-old male with no physical complaints has a BUN of 26 mg/dL. Which questions would the nurse ask to identify nonrenal factors that could be contributing to this laboratory result? (SATA) a. "Did you drink a lot of extra fluid before the blood sample was drawn?" b. "Have you been on a severe protein- or calorie-restricted diet?" c. "Are you taking or have you recently taken any steroid medications?" d. "Have you recently experienced any physical or emotional stress?" e. "Have you noticed any blood in the stool or have you vomited any blood?"

cde

When caring for the client with chronic bronchitis, which of these interventions will assist the client in mobilizing secretions? Elevate the head of the bed 45 degrees consume at least 2 liters of fluid daily avoid triggers which cause coughing assume the tripod position

consume at least 2 liters of fluid daily Clients with chronic bronchitis tend to have thick secretions. Hydration with at least 2 liters of fluid daily thins tenacious (sticky) secretions, making them easier to expectorate. The goal is to consume fluid to thin secretions and perform controlled coughing. If health issues require fluid restriction, the client would attempt to consume the total amount permitted.Head of bed elevation may promote oxygenation and lung expansion, but does not promote secretion mobilization. Clients need to sit with both feet on the floor when performing controlled coughing. The tripod position is assumed during episodes of hypoxemia, but will not facilitate mobilization of fluid.

What is a Hemoptysis?

coughing up of blood

A healthy female patient has no physical symptoms, but urinalysis results reveal a protein level of >0.8 mg/dL and a white blood cell count of 4 per high-powered field. What question would the nurse ask the patient in order to assist the health care provider to correctly interpreting the urinalysis results? a. "Have you ever been treated for a urinary tract infection?" b. "Do you have a family history of cardiac or biliary disease?" c. "Are you sexually active and if so, do you use condoms?" d. "Have you recently performed any strenuous exercise?"

d

A nurse assesses a client who is recovering from a radical nephrectomy for renal cell carcinoma. The nurse notes that the client's blood pressure has decreased from 134/90 to 100/56 mm Hg and urine output is 20 mL for this past hour. What action would the nurse take? Position the client to lay on the surgical incision. Measure the specific gravity of the client's urine. Administer intravenous pain medications. Assess the rate and quality of the client's pulse.

d

A nurse cares for a client who has kidney stones from gout ricemia. Which medication does the nurse anticipate administering? a. Phenazopyridine b. Doxycyline c. Tolterodine d. Allopurinol

d

A nurse plans care for a client with overflow incontinence. Which intervention does the nurse include in this client's plan of care to assist with elimination? Stroke the medial aspect of the thigh. Use intermittent catheterization. Provide digital anal stimulation. Use the Valsalva maneuver.

d

A nurse teaches a client with functional urinary incontinence. Which statement would the nurse include in this client's teaching? "You must clean around your catheter daily with soap and water." "You will need to be on your drug therapy for life." "Operations to repair your bladder are available, and you can consider these." "You might want to get pants with elastic waistbands."

d

A patient appears very uncomfortable with the nurses questions about urinary functions and patterns. what is the best technique for the nurse to use to elixit relevant information and decrease the patients discomfort ? a. Defere the questions until a later time b. Direct the questions toward a family member c. Use anatomic or medical terminology d. Use the patients own terminology

d

A patient is scheduled for retrograde urethrography. Postprocedural care is similar to postprocedural care given for which test? a. Ultrasonography b. Computed tomography c. Renal angiogram d. Cystoscopy

d

A patient reports flank pain and tenderness. What technique does the nurse use to assess for costovertebral angle tenderness? a. Percuss the nontender flank and assess for rebound b. Thump the CVA area with the flat surface of the hand c. Thump the CVA area with a clenched fist d. Place one palm over the CVA area, thump with other fist

d

A patient with chronic kidney disease (CKD) devleops anorexia nausea and vomiting, muscle cramping, and purritus. How does the nurse interepret these findings? a. Oliguria b. Azotemia c. Anuria d. Uremia

d

The nurse is admitting a client who has acute glomerulonephritis caused by beta streptococcus. What drug therapy would the nurse expect to be prescribed for this client? a. Antihypertensives b. Antilipidemics c. Antidepressants d. Antibiotics

d

The nurse is assisting an inexperienced health care provider to assess a patient who has an aneurysm. The nurse would intervene if the provider performed which action? a. Inspected the flank for bruising or redness b. listened for a bruit over the renal artery c. Auscultated the abdomen for bowel sounds d. Palapated deeply to locate masses or tenderness

d

The nurse teaches a client who has stress incontinence methods to regain more urinary continence. Which health teaching is the most important for the nurse to include for this client? a. What type of incontinence pads to use? b. What types of liquids to drink and when? c. Need to perform intermittent catheterizations. d. How to do Kegel exercises to strengthen muscles?

d

Which abnormal finding would be associated with chronic kidney disease? a. Hematuria b. Pus in the urine c. Blood at the urethral meatus d. Decreased urine specific gravity

d

Which event is most likely to trigger renin production? a. patient particpiates in strenuous exercise b. Patient becomes anxious and nervous. c. Patient has urge to urinate during the night d. patient sustains significant blood loss

d

Which hematologic disorder is most likely to occur if the hormonal function of the kidneys is not working properly? a. Leukemia b. Thrombocyopenia c. Neutrpenia d. Anemia

d

Which over-the-counter product used by a patient does the nurse further explore for potential impact on kideny function? a. Mouthwash with alcohol b. Fiber supplement c. Vitamin C d. Acetaminophen

d

23. Which hormone responds to elevated serum calcium blood level by decreasing bone resorption? a. PTH b. T4 c. T3 d. Calcitonin

d Calcitonin

16. Which statement about the gonads is correct? a. Gonads are reproductive glands found in males only. b. The function of the hormones begins at birth in low, undetectable levels. c. The placenta secretes testosterone for the development of male external genitalia. d. External genitalia maturation is stimulated by gonadotropins during puberty.

d External genitalia maturation is stimulated by gonadotropins during puberty.

30. The bloodstream delivers glucose to the cells for energy production. Which hormone controls the cells' use of glucose? a. T4 b. Growth hormone c. Adrenal steroids d. Insulin

d Insulin

27. Which statement about glucagon secretion is correct? a. It is stimulated by an increase in blood glucose levels. b. It is stimulated by a decrease in amino acid levels. c. It exerts its primary effect on the pancreas. d. It acts to increase blood glucose levels.

d It acts to increase blood glucose levels.

7. The target tissue for ADH is which organ? a. Hypothalamus b. Thyroid c. Ovary d. Kidney

d Kidney

43. After an ultrasound of the thyroid gland, which diagnostic test determines the need for surgical intervention for thyroid nodules? a. CT scan b. MRI c. Angiography d. Needle biopsy

d Needle biopsy

26. Which statement about the pancreas is correct? a. Endocrine functions of the pancreas include secretion of digestive enzymes. b. Exocrine functions of the pancreas include secretion of glucagon and insulin. c. The islets of Langerhans are the only source of somatostatin secretion. d. Somatostatin inhibits pancreatic secretion of glucagon and insulin.

d Somatostatin inhibits pancreatic secretion of glucagon and insulin.

an individual is found to consistenly wear only a bathrobe and neglect the cleanliness of his apt. when neighbors ask him to stop his frequent outburts of operatic arias, he acts outrated and tells them he must sing daily and will not promise to be quieter. this behavior suppirts what conclusion about this client? a. the client is demonstrating symptoms of bipolar disorder b. the client is demonstrating socially deviant behavior c. the client is engaging in egocentric behaviors d. the client is not conforming w/ social norms

d - the client is not conforming w/ social norms

Which question asked by a 48-year-old client with sleep apnea whose blood glucose level is elevated suggests to the nurse the possibility of a growth hormone excess? a. "Do you think if I lost weight my sleep apnea would improve?" b. "Why do I feel thirsty all the time?" c. "How can I make my skin less itchy?" d. "Does everyone's feet get bigger during menopause?"

d. "Does everyone's feet get bigger during menopause?"

Which question is most relevant to ask a male client suspected to have a gonadotropin deficiency? a. "Are you experiencing any pain during sexual intercourse?" b. "Do you work with or have hobbies that involve exposure to chemicals?" c. "Have you gained or lost any weight recently?" d. "How often do you need to shave your face?"

d. "How often do you need to shave your face?"

26. A client with chronic kidney disease states, "I feel chained to the hemodialysis machine." What is the nurse's best response to the client's statement? a. "That feeling will gradually go away as you get used to the treatment." b. "You probably need to see a psychiatrist to see if you are depressed." c. "Do you need help from social services to discuss financial aid?" d. "Tell me more about your feelings regarding hemodialysis treatment."

d. "Tell me more about your feelings regarding hemodialysis treatment."

12. An 18-year-old male patient with a small stature is scheduled for a growth hormone stimulation test. In preparation for the test, the nurse will obtain a. ice in a basin. b. glargine insulin. c. a cardiac monitor. d. 50% dextrose solution.

d. 50% dextrose solution. Hypoglycemia is induced during the growth hormone stimulation test, and the nurse should be ready to administer 50% dextrose immediately. Regular insulin is used to induce hypoglycemia (glargine is never given IV). The patient does not require cardiac monitoring during the test. Although blood samples for some tests must be kept on ice, this is not true for the growth hormone stimulation test.

The nurse has just received report on a group of clients. Which client is the nurse's first priority? a. A 42-year-old with diabetes insipidus who has a dose of desmopressin due. b. A 35-year-old with hyperaldosteronism who has a serum potassium of 3.0 mEq/L (3.0 mmol/L). c. A 50-year-old with pituitary adenoma who is reporting a severe headache. d. A 28-year-old with acute adrenal insufficiency who has a blood glucose of 36 mg/dL (2.0 mmol/L).

d. A 28-year-old with acute adrenal insufficiency who has a blood glucose of 36 mg/dL (2.0 mmol/L).

14. A client has a long history of hypertension. Which category of medications would the nurse expect to be ordered to avoid chronic kidney disease (CKD)? a. Antibiotic b. Histamine blocker c. Bronchodilator d. Angiotensin-converting enzyme (ACE) inhibitor

d. Angiotensin-converting enzyme (ACE) inhibitor

A client with a history of a suicide attempt has been discharged and is being followed in an outpatient clinic. At this time, which is the most appropriate nursing intervention for this client? a. Provide the client with a safe and structured environment b. Isolate the client from all stressful situations that may precipitate a suicide attempt c. Observe the client continuously to prevent self-harm d. Assist the client to develop more effective coping mechanisms

d. Assist the client to develop more effective coping mechanisms

For which assessment finding in a client who had a transsphenoidal hypophysectomy yesterday will the nurse notify the primary health care provider immediately? a. Dry lips and oral mucosa on examination b. Nasal drainage that tests negative for glucose c. Urine specific gravity of 1.016 d. Client report of a headache and stiff neck

d. Client report of a headache and stiff neck

Which factor or condition does the nurse expect to result in an increase in a client's production of thyroid hormones (TH)? a. Getting 8 hours of sleep nightly b. Chronic constipation c. Protein-calorie malnutrition d. Cold environmental temperatures

d. Cold environmental temperatures

Theresa is hospitalized following a suicide attempt after breaking up with her boyfriend. Klonsky and May's "Three-Step Theory" suggests that the nurse should assess which three issues to evaluated Theresa's active risk for a suicide attempt? a. Level of education, ethnic background, and current employment b. Relationships with previous boyfriends, coping mechanisms, and intent to have future boyfriends c. Self-esteem, grade point average, and physical attractiveness d. Degree of psychological pain, connectedness with others and suicide ideation in combination with capacity to make an attempt

d. Degree of psychological pain, connectedness with others and suicide ideation in combination with capacity to make an attempt

Which action is most important for the nurse to perform when caring for an older client decreased antidiuretic hormone (ADH) production? a. Inspecting feet and legs for ulcers b. Planning for weight-bearing activities c. Stressing the important of fiber in the diet d. Encouraging fluids every 2 hours

d. Encouraging fluids every 2 hours

2. Which statement by a 50-year-old female patient indicates to the nurse that further assessment of thyroid function may be necessary? a. I notice my breasts are tender lately. b. I am so thirsty that I drink all day long. c. I get up several times at night to urinate. d. I feel a lump in my throat when I swallow.

d. I feel a lump in my throat when I swallow. Difficulty in swallowing can occur with a goiter. Nocturia is associated with diseases such as diabetes mellitus, diabetes insipidus, or chronic kidney disease. Breast tenderness would occur with excessive gonadal hormone levels. Thirst is a sign of disease such as diabetes.

Which primary health care provider order will the nurse perform first for a client with syndrome of inappropriate antidiuretic hormone (SIADH) who has a serum sodium level of 105 mEq/L (105 mmol/L)? a. Administering an infusion of 150 mL hypertonic saline over the next 3 hours b. Drawing blood for hemoglobin and hematocrit levels c. Measuring serial weights at the same daily with the client wearing the same amount of clothing d. Inserting an indwelling catheter and monitoring urine output

d. Inserting an indwelling catheter and monitoring urine output

22. A client is assessed by the nurse after a hemodialysis session. The nurse notes bleeding from the client's nose and around the intravenous catheter. What action by the nurse is the priority? a. Hold pressure over the client's nose for 10 minutes. b. Take the client's pulse, blood pressure, and temperature. c. Assess for a bruit or thrill over the arteriovenous fistula. d. Prepare protamine sulfate for administration.

d. Prepare protamine sulfate for administration

Which client report of changes in appearance indicates to the nurse that a client's adrenal insufficiency is related to direct malfunction of the adrenal glands? a. 5-lb weight loss b. Dry, cracked lips c. Thinning pubic hair d. Skin darkening

d. Skin darkening

A person attempts suicide by overdose, is treated in the emergency department, and then hospitalized. What is the best initial outcome? The patient will: a. verbalize a will to live by the end of the second hospital day. b. describe two new coping mechanisms by the end of the third hospital day. c. accurately delineate personal strengths by the end of first week of hospitalization. d. exercise suicide self-restraint by refraining from gestures or attempts to kill self for 24 hours.

d. exercise suicide self-restraint by refraining from gestures or attempts to kill self for 24 hours.

6. A 61-year-old female patient admitted with pneumonia has a total serum calcium level of 13.3 mg/dL (3.3 mmol/L). The nurse will anticipate the need to teach the patient about testing for _____ levels. a. calcitonin b. catecholamine c. thyroid hormone d. parathyroid hormone

d. parathyroid hormone Parathyroid hormone is the major controller of blood calcium levels. Although calcitonin secretion is a countermechanism to parathyroid hormone, it does not play a major role in calcium balance. Catecholamine and thyroid hormone levels do not affect serum calcium level.

Place the steps of using a bedside bladder scanner in the correct order a. Select the male or female icon the bladder scanner b. Aim the scan head towards the expected location of the bladder c. Place the probe midline bout 1.5 inches above the pubic bone d. explain the purpose and what sensations to expect e. Place the ultrasound probe with gel right above the symphysis pubis f. press and release the scan button

daecbf

The nurse is preparing to administer oxygen to a client with chronic obstructive pulmonary disease (COPD) who has hypoxemia and hypercarbia. The nurse recognizes that a positive outcome to therapy has been achieved by which of these findings? The pCO2 is within normal range. The client's face is very pink. The client reports decreased distress. The oxygen saturation is between 88% and 90%.

the oxygen saturation is between 88% and 90%. Clients with hypoxemia, even those with COPD and hypercarbia, should receive oxygen therapy at rates appropriate to reduce hypoxia and maintain SpO2 levels between 88% and 92%.Gases diffuse independently, therefore applying oxygen will not decrease the carbon dioxide level; hypoxemia may still be present. Flushing of the face can be a symptom of hypercarbia. A report of less distress is appropriate. The nurse, in any case, needs to use an objective measure of oxygenation such as pulse oximetry or blood gas results.


Set pelajaran terkait

Naming Binary Covalent Compounds

View Set

1137 Reduction of Risk Potential

View Set

Chapter 14 - Exploring Social Media and e-Business

View Set

Unit 2 - Meteorology and Weather

View Set

Final Exam Principles of Manufacturing Systems

View Set

biochem exam 3 homework chapter 12

View Set

Unit 1; Assignment 2: A comenzar: Las primeras palabras

View Set